You are on page 1of 155

®

ACP IM KSAP �
Endocrinology and
Metabolism

0
A CPAmerican College of Physicians
1 ®

� Leading Internal Medicine, Improving Lives


Welcome to the Endocrinology and
Metabolism Section of MKSAP 17!
In these pages, you will find updated information on disorders of glucose metabolism, disorders of the pituitary gland, dis­
orders of the adrenal glands, disorders of the thyroid gland, reproductive disorders, and calcium and bone disorders. All of
these topics are uniquely focused on the needs of generalists and subspecialists outside of endocrinology and metabolism.

The publication of the 17111 edition of Medical Knowledge Self-Assessment Program (MKSAP) represents nearly a half-century
of serving as the gold-standard resource for internal medicine education. It also marks its evolution into an innovative learn­
ing system to better meet the changing educational needs and learning styles of all internists.

The core content of MKSAP has been developed as in previous editions-newly generated, essential information in 11 topic
areas of internal medicine created by dozens of leading generalists and subspecialists and guided by certification and recer­
tification requirements, emerging knowledge in the field, and user feedback. MKSAP 17 also contains 1200 all-new, psy­
chometrically validated, and peer-reviewed multiple-choice questions (MCQs) for self-assessment and study, including 84
in Endocrinology and Metabolism. MKSAP 17 continues to include High Value Care (HVC) recommendations, based on the
concept of balancing clinical benefit with costs and harms, with links to MCQs that illustrate these principles. In addition,
HVC Key Points are highlighted in the text. Also highlighted, with blue text, are Hospita/ist-focused content and MCQs that
directly address the learning needs of internists who work in the hospital setting.

MKSAP 17 Digital provides access to additional tools allowing you to customize your learning experience, including regular
text updates with practice-changing, new information and 200 new self-assessment questions; a board-style pretest to help
direct your learning; and enhanced custom-quiz options. And, with MKSAP Complete, learners can access 1200 electronic
flashcards for quick review of important concepts or review the updated and enhanced version of Virtual Dx, an image-based
self-assessment tool.

As before, MKSAP 17 is optimized for use on your mobile devices, with iOS- and Android-based apps allowing you to sync
your work between your apps and online account and submit for CME credits and MOC points online.

Please visit us at the MKSAP Resource Site (mksap.acponline.org) to find out how we can help you study, earn CME credit
and MOC points, and stay up to date.

Whether you prefer to use the traditional print version or take advantage of the features available through the digital version,
we hope you enjoy MKSAP 17 and that it meets and exceeds your personal learning needs.

On behalf of the many internists who have offered their time and expertise to create the content for MKSAP 17 and the
editorial staff who work to bring this material to you in the best possible way, we are honored that you have chosen to use
MKSAP 17 and appreciate any feedback about the program you may have. Please feel free to send us any comments to
mksap_editors@acponline.org.

Sincerely,

Philip A. Masters, MD, FACP


Editor-in-Chief
Senior Physician Educator
Director, Clinical Content Development
Medi.cal Education Division
American College of Physicians

ii
Endocrinology and Metabolism

Committee Jennifer Sipos, MD2


Associate Professor of Medicine
Cynthia A. Burns, MD, FACP, Section Editor1
Director, Benign Thyroid Program
Director, Internal Medicine Clerkship & Acting Internships
Division of Endocrinology, Diabetes, and Metabolism
Associate Professor, Department of Internal Medicine
The Ohio State University School of Medicine
Section on Endocrinology & Metabolism
Columbus, Ohio
Wake Forest University School of Medicine
Winston-Salem, North Carolina
Editor-in-Chief
Howard H. Weitz, MD, MACP, Associate Editor1
Director, Jefferson Heart Institute Philip A. Masters, MD, FACP1
Director, Division of Cardiology Senior Physician Educator
Sidney Kimmel Medical College at Thomas Jefferson Director, Clinical Content Development
University American College of Physicians
Philadelphia, Pennsylvania Philadelphia, Pennsylvania

Jessicah Collins, MD1


Director, Clinical Program Development
Clinical Endocrinologist
Diabetes and Endocrinology Clinic Cynthia D. Smith, MD, FACP2
Augusta Health American College of Physicians
Fishersville, Virginia Philadelphia, Pennsylvania

Kristen G. Hairston, MD, MPH1


Medical Director, Joslin Diabetes Center Endocrinology and Metabolism
Associate Professor Reviewers
Department of Internal Medicine Amindra Singh Arora, MD1
Section on Endocrinology & Metabolism Stewart F. Babbott, MD, FACP 1
Wake Forest University School of Medicine John K. Chamberlain, MD, MACP 1
Winston-Salem, North Carolina Terence Chan, MD2
Pieter A. Cohen, MD2
Erika B. Johnston-MacAnanny, MD 1
Benjamin P. Geisler, MD2
Assistant Professor
Richard Hoffman, MD, MPH 1
Medical Director, Center for Reproductive Medicine
Sowmya Kanikkannan, MD, FACP 1
Associate Faculty, Women's Health Center of Excellence
Mark D. Siegel, MD, FACP2
Wake Forest Center for Reproductive Medicine
Wake Forest School of Medicine
Winston-Salem, North Carolina Endocrinology and Metabolism
Reviewer Representing the American
Wanda C. Lakey, MD, MHS2
Society for Clinical Pharmacology &
Assistant Professor of Medicine
Division of Endocrinology Therapeutics
Duke University Medical Center Linda A. Hershey, MD, PhD2
Durham VA Medical Center
Durham, North Carolina Endocrinology and Metabolism
Sarah Mayson, MD2 ACP Editorial Staff
Assistant Professor Randy Hendrickson 1 , Production Administrator/Editor
Department of Medicine Katie lde!J I , Manager, Clinical Skills and Digital Programs
Division of Endocrinology, Metabolism and Diabetes Margaret Wells1 , Director, Self-Assessment and Educational
University of Colorado School of Medicine Programs
Aurora, Colorado Becky Krumm 1, Managing Editor

iii
ACP Principal Staff Terence Chan, MD
Honoraria
Patrick C. Alguire, MD, FACP 2
AstraZeneca (Canada)
Senior Vice President, Medical Education
Sean McKinney 1 Pieter A. Cohen, MD
Vice President, Medical Education Stock Options/Holdings (spouse)
Bio Reference Labs, Idexx Laboratories, Johnson &
Margaret Wells 1 Johnson, Mettler Toledo International Inc., Stryker
Director, Self-Assessment and Educational Programs Corp., Biota Pharmaceuticals, Pfizer, ResMed Inc., Vertex
Becky Krumm1 Pharmaceuticals
Managing Editor Honoraria
Consumer Union, Wall Street Journal
Katie ldell1
Manager, Clinical Skills Program and Digital Products Benjamin P. Geisler, MD
Valerie A. Dangovetsky 1 Royalties
Administrator UpToDate
Consultantship
Ellen McDonald, PhD 1 Wing Tech Inc., Ardian Medtronic LLC, Medtronic Vascular
Senior Staff Editor Inc., Amgen Inc.
Megan Zborowski1
Linda A. Hershey, MD, PhD
Senior Staff Editor
Research Grants/Contracts
Randy Hendrickson1 Baxter, Novartis, Forum Pharmaceuticals (formerly
Production Administrator/Editor En Vivo)
Linnea Donnarumma 1 Honoraria
Staff Editor Med Link Neurology

Susan Galeone1 Wanda C. Lakey, MD, MHS


Staff Editor Research Grants/Contracts
Janssen, Regeneron, Amarin, Sanofi/Aventis
Jackie T womey1
Staff Editor Sarah Mayson, MD 2
Julia Nawrocki 1 Other (Sub-Investigator for Clinical Studies)
Staff Editor Novartis, Cortendo

Kimberly Kerns1 Mark D. Siegel, MD, FACP


Administrative Coordinator Consultantship
Siemens
Rosemarie Houton1
Administrative Representative Jennifer Sipos, MD
Consultantship
1. Has no relationships with any entity producing, marketing, reselling, or distributing Genzyme
health care goods or services consumed by, or used on, patients.

2. Has disclosed relationship(s) with any entity producing, marketing, reselling, or Cynthia D. Smith, MD, FACP
distributing health care goods or services consumed by, or used on, patients.
Stock Options/Holdings
Merck and Co.; spousal employment at Merck
Disclosure of Relationships with any entity producing,
marketing, reselling, or distributing health care goods or Acknowledgments
services consumed by, or used on, patients.
The American College of Physicians (ACP) gratefully
Patrick C. Alguire, MD, FACP acknowledges the special contributions to the develop­
Consu /tantship ment and production of the 17th edition of the Medical
National Board of Medical Examiners Knowledge Self-Assessment Program' (MKSAP" 17) made
Royalties by the following people:
UpToDate
Graphic Design: Michael Ripca (Graphics Technical
Stock Options/Holdings
Administrator) and WFGD Studio (Graphic Designers).
Amgen Inc, Bristol-Myers Squibb, GlaxoSmithKline, Stryker
Corporation, Zimmer Orthopedics, Teva Pharmaceuticals, Production/Systems: Dan Hoffmann (Director, Web Services
Medtronic, Covidien, Inc., Express Scripts & Systems Development), Neil Kohl (Senior Architect),

iv
Chris Patterson (Senior Architect), and Scott Hurd (Manager, For information on how to apply MKSAP 17 Continuing
Web Projects & CMS Services). Medical Education (CME) credits to the Royal College
MOC Program, visit the MKSAP Resource Site at
MKSAP 17 Digital: Under the direction of Steven Spadt,
mksap.acponline.org.
Vice President, Digital Products & Services, the digital ver­
sion of MKSAP 17 was developed within the ACP's Digital
Product Development Department, led by Brian Sweigard
The Royal Australasian College
(Director). Other members of the team included Dan
of Physicians CPD Program
Barron (Senior Web Application Developer/Architect),
Chris Forrest (Senior Software Developer/Design Lead), In Australia, MKSAP 17 is a Category 3 program that may
Kara Kronenwetter (Senior Web Developer), Brad Lord be used by Fellows ofThe Royal Australasian College
(Senior Web Application Developer), John McKnight of Physicians (RACP) to meet mandatory Continuing
(Senior Web Developer), and Nate Pershall (Senior Web Professional Development (CPD) points.Two CPD cred­
Developer). its are awarded for each of the200 AMA PRA Category 1
Credits rn available in MKSAP 17. More information about
The College also wishes to acknowledge that many other using MKSAP 17 for this purpose is available at the MKSAP
persons, too numerous to mention, have contributed to Resource Site at mksap.acponline.org and at www.racp.
the production of this program. Without their dedicated edu.au. CPD credits earned through MKSAP 17 should be
efforts, this program would not have been possible. reported at the MyCPD site at ww--v.racp.edu.au/mycpd.

MKSAP Resource Site Continuing Medical Education


(mksap.acponline.org)
The American College of Physicians (ACP) is accredited
The MKSAP Resource Site (mksap.acponline.org) is a by the Accreditation Council for Continuing Medical
continually updated site that provides links to MKSAP Education (ACCME) to provide continuing medical educa­
17 online answer sheets for print subscribers; the lat­ tion for physicians.
est details on Continuing Medical Education (CME)
and Maintenance of Certification (MOC) in the United The ACP designates this enduring material, MKSAP 17,
States, Canada, and Australia; errata; and other new for a maximum of200 AMA PRA Category 1 CreditsTM _
information. Physicians should claim only the credit commensurate
with the extent of their participation in the activity.
Up to 14 AMA PRA Category 1 Credits1M are available from
ABIM Maintenance of Certification December 31, 2015, to December 31,2018, for the MKSAP
Check the MKSAP Resource Site (mksap.acponline.org) 17 Endocrinology and Metabolism section.
for the latest information on how MKSAP tests can be
used to apply to the American Board of Internal Medicine
for Maintenance of Certification (MOC) points. Learning Objectives
The learning objectives of MKSAP 17 are to:
Royal College Maintenance • Close gaps between actual care in your practice and
preferred standards of care, based on best evidence
of Certification
• Diagnose disease states that are less common and
In Canada, MKSAP 17 is an Accredited Self-Assessment sometimes overlooked or confusing
Program (Section 3) as defined by the Maintenance of • Improve management of comorbid conditions that can
Certification (MOC) Program of The Royal College of complicate patient care
Physicians and Surgeons of Canada and approved by • Determine when to refer patients for surgery or care by
the Canadian Society of Internal Medicine on December subspecialists
9, 2014. Approval extends from July 31, 2015 until July • Pass the ABIM Certification Examination
31,2018 for the Part A sections. Approval extends from • Pass the ABIM Maintenance of Ce1iification Examination
December 31, 2015 to December 31, 2018 for the Part B
sections.
Fellows of the Royal College may earn three credits per Target Audience
hour for participating in MKSAP 17 under Section 3. • General internists and primary care physicians
MKSAP 17 also meets multiple CanMEDS Roles, includ­ • Subspecialists who need to remain up-to-date in internal
ing that of Medical Expert, Communicator, Collaborator, medicine and in areas outside of their own subspecialty
Manager, Health Advocate, Scholar, and Professional. area

V
• Residents preparing for the certification examination in Accreditation Council for Continuing Medical Education
internal medicine (ACCME), contributors to all ACP continuing medical
• Physicians preparing for maintenance of ce1iification in education activities are required to disclose all relevant
internal medicine (rece1iification) financial relationships with any entity producing, mar­
keting, re-selling, or distributing health care goods or
services consumed by, or used on, patients. Contributors
Earn "Instantaneous" CME are required to use generic names in the discussion of
Credits Online therapeutic options and are required to identify any unap­
Print subscribers can enter their answers online to earn proved, off-label, or investigative use of commercial prod­
instantaneous Continuing Medical Education (CME) cred­ ucts or devices. Where a trade name is used, all available
its. You can submit your answers using online answer trade names for the same product type are also included.
sheets that are provided at mksap.acponline.org, where a If trade-name products manufactured by companies with
record of your MKSAP 17 credits will be available. To earn whom contributors have relationships are discussed, con­
CME credits, you need to answer all of the questions in tributors are asked to provide evidence-based citations
a test and earn a score of at least 50% correct (number of in support of the discussion. The information is reviewed
correct answers divided by the total number of questions). by the committee responsible for producing this text. If
Take any of the following approaches: necessary, adjustments to topics or contributors' roles in
content development are made to balance the discussion.
1. Use the printed answer sheet at the back of this book to
Further, all readers of this text are asked to evaluate the
record your answers. Go to mksap.acponline.org, access
content for evidence of commercial bias and send any rel­
the appropriate online answer sheet, transcribe your
evant comments to mksap_editors@acponline.org so that
answers, and submit your test for instantaneous CME
future decisions about content and contributors can be
credits. There is no additional fee for this service.
made in light of this information.
2. Go to mksap.acponline.org, access the appropriate
online answer sheet, directly enter your answers, and
submit your test for instantaneous CME credits. There is Resolution of Conflicts
no additional fee for this service. To resolve all conflicts of interest and influences of vested
3. Pay a $15 processing fee per answer sheet and submit interests, the American College of Physicians (ACP) pre­
the printed answer sheet at the back of this book by cluded members of the content-creation committee from
mail or fax, as instructed on the answer sheet. Make deciding on any content issues that involved generic or
sure you calculate your score and fax the answer trade-name products associated with proprietary entities
sheet to 215-351-2799 or mail the answer sheet to with which these committee members had relationships.
Member and Customer Service, American College In addition, content was based on best evidence and
of Physicians, 190 N. Independence Mall West, updated clinical care guidelines, when such evidence and
Philadelphia, PA 19106-1572, using the courtesy enve­ guidelines were available. Contributors' disclosure infor­
lope provided in your MKSAP 17 slipcase. You will mation can be found with the list of contributors' names
need your 10-digit order number and 8-digit ACP and those of ACP principal staff listed in the beginning of
ID number, which are printed on your packing slip. this book.
Please allow 4 to 6 weeks for your score report to be
emailed back to you. Be sure to include your email
address for a response. Hospital-Based Medicine
If you do not have a 10-digit order number and 8-digit For the convenience of subscribers who provide care in
ACP ID number or if you need help creating a user hospital settings, content that is specific to the hospital
name and password to access the MKSAP 17 online setting has been highlighted in blue. Hospital icons (C])
answer sheets, go to mksap.acponline.org or email highlight where the hospital-based content begins.
custserv@acponline.org. continues over more than one page. and ends.

Disclosure Policy High Value Care Key Points


It is the policy of the American College of Physicians Key Points in the text that relate to High Value Care con­
(ACP) to ensure balance, independence, objectivity, and cepts (that is, concepts that discuss balancing clinical
scientific rigor in all of its educational activities. To this benefit with costs and harms) are designated by the HVC
end, and consistent with the policies of the ACP and the icon (HVC).

vi
Educational Disclaimer tronic or mechanical, including photocopy, without the
The editors and publisher of MKSAP 17 recognize that the express consent of the American College of Physicians.
development of new material offers many opportunities for MKSAP 17 is for individual use only. Only one account
error. Despite our best efforts, some errors may persist in per subscription will be permitted for the purpose of
print. Drug dosage schedules are, we believe, accurate and earning Continuing Medical Education (CME) credits and
in accordance with current standards. Readers are advised, Maintenance of Certification (MOC) points/credits and for
however, to ensure that the recommended dosages in other authorized uses of MKSAP 17.
MKSAP 17 concur with the information provided in the
product information material. This is especially important
in cases of new, infrequently used, or highly toxic drugs. Unauthorized Use of This Book
Application of the information in MKSAP 17 remains the Is Against the Law
professional responsibility of the practitioner. Unauthorized reproduction of this publication is unlaw­
ful. The American College of Physicians (ACP) prohibits
The primary purpose of MKSAP 17 is educational.
reproduction of this publication or any of its parts in any
Information presented, as well as publications, technol­
form either for individual use or for distribution.
ogies, products, and/or services discussed, is intended
to inform subscribers about the knowledge, techniques, The ACP will consider granting an individual permission
and experiences of the contributors. A diversity of pro­ to reproduce only limited portions of this publication for
fessional opinion exists, and the views of the contribu­ his or her own exclusive use. Send requests in writing to
tors are their own and not those of the American College MKSAP' Permissions, American College of Physicians, 190 N.
of Physicians (ACP). Inclusion of any material in the Independence Mall West, Philadelphia, PA 19106-1572, or
program does not constitute endorsement or recommen­ email your request to mksap_editors@acponline.org.
dation by the ACP. The ACP does not warrant the safety.
MKSAP 17 ISBN: 978-1-938245-18-3
reliability, accuracy, completeness, or usefulness of and
(Endocrinology and Metabolism) ISBN: 978-1-938245-25-1
disclaims any and all liability for damages and claims
that may result from the use of information, publica­ Printed in the United States of America.
tions, technologies, products, and/or services discussed
For order information in the United States or Canada call
in this program.
800-523-1546, extension 2600. All other countries call
215-351-2600, (M-F, 9 AM - 5 PM ET). Fax inquiries to
215-351-2799 or email to custserv@acponline.org.
Publisher's Information
Copyright CG: 2015 American College of Physicians.
All rights reserved. Errata
This publication is protected by copyright. No part of Errata for MKSAP 17 will be available through the MKSAP
this publication may be reproduced, stored in a retrieval Resource Site at mksap.acponline.org as new information
system, or transmitted in any form or by any means, elec- becomes known to the editors.

vii
Table of Contents

Disorders of Glucose Metabolism Gonadotropin-Producing Adenomas ........... 28


Diabetes Mellitus................................. 1 Thyroid-Stimulating Hormone-Secreting
Tum� .................................... �
Screening for Diabetes Mellitus................. 1
Diagnostic Criteria for Diabetes Mellitus ......... 1 Excess Antidiuretic Hormone Secretion......... 28
Classification of Diabetes Mellitus............... 1 Cushing Disease ............................ 28
Management of Diabetes Mellitus ............... 5 Disorders of the Adrenal Glands
Inpatient Management of Hyperglycemia ........... 11
Adrenal Anatomy and Physiology.................. 29
Hospitalized Patients with Diabetes Mellitus ..... 11
Adrenal Hormone Excess ......................... 30
Hospitalized Patients Without Diabetes Mellitus.... 12
Cushing Syndrome .......................... 30
Management of Hypoglycemia .................... 12
Pheochromocytomas and Paragangliomas....... 32
Hypoglycemia in Patients with Diabetes
Primary Hyperaldosteronism ................. 34
Mellitus.................................... 12
Androgen-Producing Adrenal Tumors .......... 36
Hypoglycemia in Patients Without Diabetes
Mellitus.................................... 13 Adrenal Insufficiency ............................ 36
Acute Complications of Diabetes Mellitus ........... 14 Primary Adrenal Failure ...................... 36
Diabetic Ketoacidosis and Hyperglycemic Adrenal Function During Critical lllness ........ 38
Hyperosmolar Syndrome ..................... 14 Adrenal Masses ................................. 39
Chronic Complications of Diabetes Mellitus ......... 16 Incidentally Noted Adrenal Masses ............. 39
Cardiovascular Morbidity..................... 16 Adrenocortical Carcinoma .................... 40
Diabetic Retinopathy ........................ 17
Disorders of the Thyroid Gland
Diabetic Nephropathy........................ 17
Thyroid Anatomy and Physiology .................. 40
Diabetic Neuropathy......................... 17
Evaluation of Thyroid Function.................... 41
Diabetic Foot Ulcers ......................... 18
Functional Thyroid Disorders ..................... 42
Hypoglycemic Unawareness .................. 18
Thyrotoxicosis .............................. 42
Thyroid Hormone Deficiency ................. 45
Disorders of the Pituitary Gland Drug-Induced Thyroid Dysfunction ............46
Hypothalamic and Pituitary Anatomy and Thyroid Function and Disease in Pregnancy ......... 46
Physiology ..................................... 18 Euthyroid Sick Syndrome......................... 48
Pituitary Tumors ................................ 20 Thyroid Emergencies ............................ 48
Approach to a Sellar Mass .................... 20 Thyroid Storm .............................. 48
Mass Effects of Pituitary Tumors............... 21 Myxedema Coma............................ 49
Treatment of Clinically Nonfunctioning Structural Disorders of the Thyroid Gland........... 50
Pituitary Tumors ............................ 21 Thyroid Nodules ............................ 50
Hypopituitarism ................................ 21 Goiters .................................... 51
Adrenocorticotropic Hormone Deficiency Thyroid Cancer ................................. 52
(Secondary Cortisol Deficiency) ............... 22
Thyroid-Stimulating Hormone Deficiency....... 23 Reproductive Disorders
Gonadotropin Deficiency ..................... 24 Physiology of Female Reproduction ................ 53
Growth Hormone Deficiency.................. 24 Amenorrhea.................................... 54
Central Diabetes lnsipidus .................... 24 Clinical Features ............................ 54
Panhypopituitarism ......................... 25 Evaluation of Amenorrhea .................... 55
Pituitary Hormone Excess ........................ 25 Hyperandrogenism Syndromes.................... 55
Hyperprolactinemia and Prolactinoma ......... 25 Hirsutism and Polycystic Ovary Syndrome ...... 55
Acromegaly ................................ 27 Androgen Abuse in Women ................... 56

ix
Female Infertility................................ 56 Diagnosis and Causes of Hypercalcemia ........ 62
Physiology of Male Reproduction .................. 57 Treatment of Hypercalcemia .................. 65
Hypogonadism ................................. 57 Hypocalcemia .................................. 66
Primary Hypogonadism ...................... 57 Clinical Features of Hypocalcemia ............. 66
Secondary Hypogonadism .................... 57 Diagnosis and Causes of Hypocalcemia ......... 66
Androgen Deficiency in the Aging Male......... 57 Treatment of Hypocalcemia................... 66
Evaluation of Male Hypogonadism ............. 58 Metabolic Bone Disease .......................... 66
Testosterone Replacement Therapy................. 58 Osteopenia and Osteoporosis ................. 66
Anabolic Steroid Abuse in Men ....................60 Vitamin D Deficiency ........................ 69
Male Infertility..................................60 Paget Disease of Bone ........................ 70
Gynecomastia ..................................60
Bibliography .................................. 71
Calcium and Bone Disorders
Calcium Homeostasis and Bone Physiology.......... 61 Self-Assessment Test. ........................... 73
Hypercalcemia.................................. 62
Clinical Features of Hypercalcemia ............. 62 Index ........................................ 137

X
Endocrinology and Metabolism High
Value Care Recommendations

The American College of Physicians, in collaboration with • Microprolactinomas in asymptomatic patients do not
multiple other organizations, is engaged in a worldwide require treatment; however, surveillance is recom­
initiative to promote the practice of High Value Care mended (see Item 65).
(HVC). The goals of the HVC initiative are to improve • If the thyroid-stimulating hormone level is frankly
health care outcomes by providing care of proven benefit abnormal, additional evaluation of thyroid function
and reducing costs by avoiding unnecessary and even should be considered to determine the extent of the
harmful interventions. The initiative comprises several dysfunction; measure thyroxine (T,1) when the thyroid­
programs that integrate the important concept of health stimulating hormone is elevated and measure both
care value (balancing clinical benefit with costs and thyroxine (T,il and triiodothyronine (T) when the
harms) for a given intervention into a broad range of edu­ thyroid-stimulating hormone is suppressed.
cational materials to address the needs of trainees, prac­ • There is no clinical indication for serial measurement of
ticing physicians, and patients. thyroid antibody titers to determine the need for or to
guide therapy except to monitor for residual disease in
HVC content has been integrated into MKSAP 17 in several
patients treated for thyroid cancer
important ways. MKSAP 17 now includes HVC-identified
• Radioactive iodine uptake is a measure of iodine uptake
key points in the text, HVC-focused multiple choice
by the thyroid over 24 hours; it is used to evaluate the
questions, and, for subscribers to MKSAP Digital, an HVC
cause of hyperthyroidism and is not indicated in patients
custom quiz. From the text and questions, we have gen­
with normal or elevated thyroid-stimulating hormone
erated the following list of HVC recommendations that
levels.
meet the definition below of high value care and bring us
• In patients with subclinical hyperthyroidism, repeat
closer to our goal of improving patient outcomes while
assessment of thyroid function should be performed
conserving finite resources.
6 to 12 weeks after the initial tests, as the values will
High Value Care Recommendation: A recommendation to normalize in up to 30% of patients.
choose diagnostic and management strategies for patients • An elevated serum thyroid-stimulating hormone level
in specific clinical situations that balance clinical benefit indicates the diagnosis of primary hypothyroidism;
with cost and harms with the goal of improving patient thyroid imaging is not indicated unless there is concern
outcomes. for a nodule on physical examination.
• The typical pattern of euthyroid sick syndrome, nonthy­
Below are the High Value Care Recommendations for the
roidal illness syndrome, or low triiodothyronine (T) syn­
Endocrinology and Metabolism section of MKSAP 17.
drome is a mildly elevated thyroid-stimulating hormone
• Lifestyle modifications are a cost-effective intervention level and slightly low thyroxine (T,1) and triiodothyronine
that has been proven to decrease the risk of patients with (T) levels; this pattern should not prompt further testing
prediabetes developing type 2 diabetes by 41 °lo to 58%. in the hospital.
• The data for the role and cost-effectiveness of self-mon­ • After patients with euthyroid sick syndrome are dis­
itoring of blood glucose levels are less clear for regimens charged from the hospital, thyroid function abnormal­
without multiple daily insulin injections and noninsulin ities may persist for several weeks so follow-up thyroid
regimens; generally this should be avoided. function tests should not be repeated until 6 weeks after
• For noncritically ill patients who are eating, the use of discharge.
basal and prandial subcutaneous insulin is the preferred • A serum thyroid-stimulating hormone measurement
and safest choice for achieving inpatient glycemic con­ is the initial laboratory test in a patient with a thy-
trol; oral agents and noninsulin injectable agents do not roid nodule; if the thyroid-stimulating hormone is
have proven safety or efficacy data in the hospital setting. suppressed, then measurement of thyroxine (T,) and
• Isolated adult-onset growth hormone deficiency is triiodothyronine (T) should be performed, and a radio­
extremely rare, and its clinical significance is debated; nuclide scan should be considered to identify "hot" or
evaluation for growth hormone deficiency should be functioning nodules, which have a very low likelihood
reserved for adults with at least one known pituitary hor­ of malignancy and typically do not require fine-needle
mone deficiency. aspiration.

xi
• If the thyroid-stimulating hormone level is high or vitamin D status, and is the recommended test for
normal, the radionuclide scan is unnecessary as it is vitamin D deficiency.
unlikely to reveal a hot nodule and ultrasonography is • A 25-hydroxyvitamin D level between 30 and 40 ng/mL
an inexpensive and highly effective method for strati­ (75-100 nmol/L) is deemed sufficient for bone health;
fication of malignancy risk for nonfunctioning thyroid most expert groups recommend screening all groups at
nodules. least once for evidence of deficiency since U.S. incidence
• Measurement of testosterone levels is not recommended is 30% to 60% of the population, however, it should not
if a patient is having regular morning erections, does not be a serial, recurring screening test.
have true gynecomastia on examination, and has a nor­ • Treatment for low bone mass in postmenopausal women
mal testicular examination, as it is highly unlikely that he involves lifestyle modification (maximizing weight­
has testosterone deficiency. bearing exercise and avoidance of tobacco or excessive
• Mild, chronic, asymptomatic gynecomastia in the male alcohol) and vitamin D and calcium supplementation; the
patient does not warrant evaluation. need for pharmacologic therapy is based on the 10-year
• 25-Hydroxyvitamin D has a relatively long half-life of estimated fracture risk (�20% for a major osteoporotic
several weeks, is the best indicator of whole body fracture or �3% for hip fracture)(see Item 12).

xii
Endocrinology and Metabolism

Disorders of Glucose does not warrant repeat measurement. The diabetes scree11.lng
tests have several advantages and disadvantages to consider. FPG
Metabolism is cheaper and more readily available in most countries com­
pared with hemoglobin A,c' but the requirement for overnight
Diabetes Mellitus fasting can be problematic. OGTT best reflects the pathophysiol­
Diabetes mellitus is a chronic metabolic disease characterized ogy of diabetes by identifying postprandial hyperglycemia sec­
by elevated plasma glucose levels as a consequence of insulin ondary to pancreatic beta-cell deficiency; however, the test is
deficiency, impaired action of insulin secondary to insulin time-intensive. Hemoglobin A,c testing is more convenient with
resistance, or a combination of both abnormalities. Prediabetes no fasting requirement, is unaffected by acute stress or illness,
is defined as elevated plasma glucose levels below the diagnos­ and provides an accurate reflection of the average plasma glu­
tic criteria for diabetes, but above the normal range. cose over the previous 3 months. By contrast, hemoglobin A,c
testing can miss early glucose abnormalities, such as postpran­
Screening for Diabetes Mellitus dial hyperglycemia. Another disadvantage is its decreased reli­
Patients with diabetes mellitus may exhibit classic symptoms ability in the setting of anemia, hemoglobinopathies, or kidney
(polyuria, polydipsia, polyphagia), or more conm1011.ly, they can or liver disease. Furthermore, conditions that affect the turno­
be asymptomatic. Diabetes screening may detect an early asymp­ ver of erythrocytes, such as anemias/hemoglobinopathies and
tomatic phase. Current guidelines do not recommend routine pregnancy, can affect the reliability of hemoglobin A1c.
screening for type 1 diabetes as there is no consistent evidence
that early treatment during the asymptomatic stage prevents Classification of Diabetes Mellitus
progression of the disease. Similarly, it has not been firmly estab­ Categories for classification of diabetes encompass a range of
lished that screening improves clinical outcomes in type 2 diabe­ insulin abnormalities, including absolute or relative insulin
tes. However, microvascular and macrovascular disease can be deficiency, insulin resistance, or a combination of these abnor­
present at the time of diagnosis of type 2 diabetes, which is malities (Table 3).
indicative of ongoing organ damage during the asymptomatic
phase. Furthermore, there is evidence that the microvascular and Insulin Deficiency
macrovascular disease associated with type 2 diabetes may be Type 1 Diabetes Mellitus
reduced with improved glucose control early in the disease Type 1 diabetes occurs in the setting of insulin deficiency. It
course and that treatment of prediabetes may delay the onset of accounts for 5% of diagnosed diabetes cases. The underlying
type 2 diabetes. In 2008, the U.S. Preventive Services Task Force mechanism is destruction of insulin-producing pancreatic
(USPSTF) recommended screening for type 2 diabetes 011.ly in beta cells, which can be autoimmune-mediated, idiopathic, or
asymptomatic adults with a sustained blood pressure level acquired.
(treated or untreated) greater than 135/80 mm Hg. Updated Autoimmune-mediated type 1 diabetes mellitus can result
USPSTF draft guidelines from 2014 have expanded screening from a combination of genetic, environmental, and autoimmune
recommendations to all adults in primary care settings with risk factors. There is a strong association between type 1 diabetes and
factors for the development of diabetes (Table 1). In contrast, the specific HLA antigens. One or more precipitating events, such as
American Diabetes Association (ADA) recommends screening for viral infections, can trigger the autoimmune process of beta-cell
type 2 diabetes based on BM! (�25) with additional risk factors, destruction in genetically susceptible persons. Autoantibodies
including a history of gestational diabetes, or age (�45 years). (one or more) can be present at the time of diagnosis, including
antibodies to the following: islet cells, glutamic acid decarboxy­
Diagnostic Criteria for Diabetes Mellitus lase (GAD65), tyrosine phosphatases IA-2 and IA-2�, insulin,
Prediabetes and diabetes can be diagnosed based on the ele­ and zinc transporter autoantibodies. Measurements of autoanti­
vated results from one of the following screening tests repeated bodies to GAD65 and lA-2 are recommended for initial confir­
on two separate occasions: fasting plasma glucose (FPG), 2-hour mation as both of these assays are highly automated. In addition,
postprandial glucose during an oral glucose tolerance test autoantibodies to GAD65 persist longer than those to islet cells
(OGTT), or hemoglobin A,c (Table 2). A random plasma glucose after the development of diabetes.
level greater than or equal to 200 mg/dL (11.1 mmol/L) with Autoimmune-mediated type 1 diabetes has classically been
classic hyperglycemic symptoms is diagnostic of diabetes and considered a disease of children and young, thin adults,

1
D i s o r d e r s of Gluco s e Metabolism

TABLE 1. Screening Guidelines for Type 2 Diabetes Mellitus in Asymptomatic Adults


ADA USPSTF

Screening criteria BMI >25• and at least one additional risk factor: 2008 guidelines:

Physical inactivity Sustained BP>135/80 mm Hg treated or


untreated
First-degree relative with diabetes
2014 updated draft:
High-risk race/ethnicity (black, Latino, Native American,
Asian American, Pacific Islander) Screening of adults in primary care settings with at
least one of the following risk factors for IFG, IGT,
Delivery of a baby weighing>4.1 kg (9 lb)
or type 2 diabetes mellitus:
History of GDM
Age 2:45 years
Hypertension (2:140/90 mm Hg or on antihypertensive
Overweight or obese
medication)
First-degree relative with diabetes
HDL cholesterol <35 mg/dL (0.90 mmol/L) and/or
triglyceride level>250 mg/dL (2.82 mmol/L) History of GDM

Polycystic ovary syndrome History of polycystic ovary syndrome


Hemoglobin A 1 c 2:5.7%, IGT, or IFG on previous testing High-risk race/ethnicity (black, American Indian/
Alaska Native, Asian American, Hispanic/Latino,
Other conditions associated with insulin resistance
and Native Hawaiian/Pacific Islander)
(severe obesity, acanthosis nigricans)
History of CVD

Additional screening All patients age 45 years or older Patients age 45 years or younger with any of the
criteria other risk factors in the screening criteria

Additional screening Use of glucocorticoids or antipsychotics


considerations
Screening intervals 3-year intervals if results are normal. Yearly testing if 3-year intervals if low-risk and normal plasma
prediabetes (hemoglobin A1c between 5.7% and 6.5%, glucose values. In high-risk adults or those with
IGT, IFG) is diagnosed. near abnormal test values, yearly testing may be
beneficial.

"At-risk BMI may be lower in some ethnic groups.

ADA= American Diabetes Association; BP= blood pressure; CVD = cardiovascular disease; GDM= gestational diabetes mellitus; IFG= impaired fasting glucose;
!GT= impaired glucose tolerance; USPSTF = U.S. Preventive Services Task Force.

Data from American Diabetes Association. Classification and diagnosis of diabetes. Sec. 2. In Standards of Medical Care in Diabetes-2015. Diabetes Care 2015;38 Suppl 1 :S8-16.
IPMID: 25537714]

TABLE 2. Diagnostic Criteria for Diabetes Mellitus•


Test Normal Range Increased Risk for Diabetes
Diabetes (Prediabetes)

Random plasma glucose Classic hyperglycemic symptoms plus a random


plasma glucose 2:200 mg/dL (11.1 mmol/L)
Fasting plasma glucoseb <100 mg/dL 100-125 mg/dL 2:126 mg/dL
(5.6 mmol/L) (5.6-6.9 mmol/L) (7.0 mmol/L)
Plasma glucose during a <140 mg/dL 140-199 mg/dl 2:200 mg/dl
2-hour 75-g OGTI
(7.8 mmol/L) (7.8-11.0 mmol/L) (11.1 mmol/L)
Hemoglobin A 1c <5.7% 5.7%-6.4% 2'.6.5%

OGTI= oral glucose tolerance test.

din the absence of hyperglycemic symptoms, an abnormal fasting plasma glucose, OGn, or hemoglobin A l e should be confirmed by repeat testing. The same test should be
used when repeating the measurement for confirmation. If two different tests are performed and only one has abnormal results, the American Diabetes Association recommends
repeating the test with the abnormal results.

bFasting for at least 8 hours.

Data from American Diabetes Association. (2) Classification and diagnosis of diabetes. Diabetes Care 2015 Jan;38 Suppl:S8-S16. [PMID: 25537714 J

2
Disorders of Glucose M etabolism

TABLE 3. Classification of Diabetes Mellitus duce some insulin during a "honeymoon" phase lasting several
weeks to many months, although this is not an adequate or
Insulin Deficiency•
sustained effect. Insulin therapy is therefore recommended dur­
Immune-mediated ing the honeymoon phase to reduce the metabolic stress on the
Type 1 diabetes functioning beta cells to preserve any residual function for as
LADA long as possible. Late autoimmune diabetes in adults (LADA)
presents in patients with autoantibodies to pancreatic beta-cell
Rare forms: "stiff man" syndrome, anti-insulin receptor antibodies
antigens and beta-cell destruction who did not require insulin
Idiopathic (seronegative)
initially but eventually progressed to an insulin requirement.
Acquired Patients with autoimmune-mediated type 1 diabetes are at an
Diseases of the exocrine pancreas: pancreatitis, trauma/ increased risk to develop other autoimmune diseases, such as
pancreatectomy, neoplasia, cystic fibrosis, hemochromatosis,
thyroiditis and celiac disease most commonly. Thus, screening
fibrocalculous, pancreatopathy
for associated autoimmune diseases should be considered at the
Infections: congenital rubella
time of diagnosis and/or the development of signs and symp­
Drug-related: Vacor (rat poison), intravenous pentamidine toms. Consensus on the frequency and effectiveness of repeat
Insulin Resistance screening for associated autoimmune diseases is lacking.
Idiopathic type 1 diabetes can present with relative insu­
Type 2 Diabetesb
lin deficiency and episodic DKA without evidence for autoim­
Ketosis pronec munity. There is a strong genetic history of diabetes, and Asian
Other or Rare Types
I and African ancestry appears to be a predisposing factor.
Acquired type 1 diabetes can be caused by diseases affect­
Genetic defects in beta-cell function (including six distinct
MODY syndromes) ing the exocrine pancreas, infections, or drugs. Diffuse dam­
age to the pancreas and beta cells or impaired insulin secretion
Genetic defects in insulin action
with subsequent insulin deficiency occurs in these scenarios.
Endocrinopathies:
KEY POINTS
(acromegaly, Cushing syndrome, glucagonoma,
pheochromocytoma, hyperthyroidism)d • Prediabetes and diabetes mellitus can be diagnosed
(somatostatinoma, aldosteronoma) ° based on the elevated results from one of the following
Drug-related: screening tests repeated on two separate occasions: fast­
ing plasma glucose, 2-hour postprandial glucose during
(glucocorticoids, thiazides, �-blockers, diazoxide, tacrolimus,
cyclosporine, niacin, HIV protease inhibitors, atypical an oral glucose tolerance test, or hemoglobin A 10 .
antipsychotics [clozapine, olanzapine])f
• Measurements of autoantibodies to GAD65 and IA-2 are
Genetic syndromes: recommended for initial confirmation of autoimmune­
Down syndrome9 mediated type 1 diabetes mellitus.
Wolfram syndrome (DIDMOAD)h
(Klinefelter, Turner, and Prader-Willi syndromes; myotonic Insulin Resistance
dystrophy)d
Insulin resistance is characterized by the inability of the
DIDMOAD = diabetes insipidus, diabetes mellitus, optic atrophy, and deafness; LADA: peripheral cells to utilize insulin effectively, with a compensa­
late autoimmune diabetes in adults; MODY= maturity-onset diabetes of the young.

aBeta-cell destruction usually leading to absolute insulin deficiency.


tory increase in the amount of insulin secreted by the pancre­
bJnsulin resistance with progressive relative insulin deficiency.
atic beta cells in response to hyperglycemia. The pancreas
cMore common in nonwhite patients who present with diabetic ketoacidosis but exhibits a relative insulin deficiency when it cannot produce
become non-insulin dependent over time. enough insulin to overcome the hyperglycemia. Obesity pre­
d[mpaired insulin action.
disposes to the development of insulin resistance.
c1mpaired insulin secretion.
11mpaired insulin secretion, impaired insulin action or altered hepatic glucose metabolism.
Metabolic Syndrome
9lnsulin deficiency, immune-mediated.
Metabolic syndrome is the coexistence of a group of risk factors
hlnsulin deficiency.
that increases a person's probability for the development of
type 2 diabetes mellitus and cardiovascular disease (CVD). In
although it can occur at any age or BMI range. The initial pres­ addition to impaired glucose metabolism, these risk factors
entation can range from modest elevations in plasma glucose include central body obesity, hypertension, and hyperlipidernia
levels to diabetic ketoacidosis (DKA). The time course for beta­ (Table 4). Metabolic syndrome increases the relative risk of
cell destruction is also variable, although it is frequently more developing CVD by twofold and diabetes by fivefold, although it
rapid in children compared with adults. Upon initiation of is not clear whether the combination of factors associated with
insulin therapy for type 1 diabetes, the remaining functioning metabolic syndrome imparts a greater risk than that attributa­
pancreatic beta cells may temporarily regain the ability to pro- ble to each individual risk factor present. In the presence of one

3
Disorders of Glucose Metabolism

' TABLE 4. AHA/NHLBI Criteria for the Definition of the Insulin resistance from obesity in the setting of relative
: Metabolic Syndrome insulin deficiency contributes to the development of type 2
Clinical Criteria Qualifying Measurements diabetes in the majority of patients. The degree of hyperglyce­
(Must meet at least mia depends on the extent of beta-cell function, which can
3 of the 5 criteria) decline over time.
Waist circumference• Men 40in (102 cm)b
Type 2 diabetes generally has an insidious onset of pro­
longed asymptomatic hyperglycemia. Most patients do not
Women 35in (89 emf
present with the classic symptoms of polydipsia, polyphagia, or
FastingTG ?:150mg/dl(1.7mmol/L)or
polyuria. Patients with type 2 diabetes may present first with
Drug therapy targeting increased TG macrovascular or microvascular changes. Although type 2 dia­
HDL cholesterol Men <40mg/dl(1.0mmol/L) betes is often considered an adult disease, the incidence is
Women <50mg/dl (1.3mmol/L)or increasing along with obesity rates in children and adolescents.
Residual insulin production from the beta cells is insuf­
Drug therapy targeting decreased
HDL ficient to control glucose adequately, but it is able to suppress
Blood pressure Systolic ?:130mm Hg or
lipolysis for most persons with type 2 diabetes. Under extreme
metabolic stress, such as an illness, some patients with type 2
Diastolic ?:85mm Hg or
diabetes cannot suppress lipolysis and present with DKA.
Fasting glucose Drug therapy for hypertension
Ketosis-prone patients with type 2 diabetes are more likely to
Blood glucose?: 100mg/dl be overweight or obese, middle-aged, male, and of black or
(5.6 mmol/L) or
Latino ethnicity. Insulin use during the time of metabolic
Drug therapy for increased glucose stress can often restore the beta cells from the glucose toxicity
AHA= American Heart Association; HDL= high-density lipoprotein cholesterol; with a return to diet, exercise, and oral hypoglycemic agents
NHLBI = National Heart, Lung, and Blood Institute; TG = triglycerides.
for glucose control. Prior to switching from insulin to oral
asome individuals with minimally elevated waist circumference measurements [e.g.,
(men 37-39 in or 94-99 cm) or(women 31-34 in or 79-86 cm)] may still be at risk for
therapy, pancreatic beta-cell function should be assessed with
type 2 diabetes or cardiovascular disease and will benefit from lifestyle interventions. fasting C-peptide and glucose measurements.
bA lower waist circumference of 35 in {89 cm) should be used for Asian American men. Lifestyle modifications alone or combined with therapeu­
'A lower waist circumference of 31 in (79 cm) should be used for Asian American tic interventions can prevent or delay the development of type
women.
2 diabetes in high-risk persons. Lifestyle modifications are a
Data from Grundy SM, Cleeman JI, Daniels SR, et al. Diagnosis and management
of the metabolic syndrome: An American Heart Association/National Heart, Lung,
cost-effective intervention. Several randomized controlled tri­
and Blood Institute Scientific Statement: Executive Summary. Circulation. 2005 Oct als provide evidence that diet changes, increased daily exer­
25;112(17):2735-52. [PMID: 16157765]
cise, and weight loss targets of 5% to 7% can significantly
decrease the risk of developing type 2 diabetes in persons with
or more risk factors for metabolic syndrome, the Endocrine prediabetes by 41% to 58%. Additionally, metformin has been
Society recommends a 3-year screening interval for the meta­ shown to reduce the risk of diabetes in patients with prediabe­
bolic syndrome components including waist circumference, tes, although this effect was not as robust as lifestyle interven­
fasting lipid profile, fasting plasma glucose, and blood pressure. tions. In the setting of impaired glucose tolerance, impaired
Calculation of the 10-year CVD risk is recommended in persons fasting glucose values, or hemoglobin A1c values between 5.7%
with metabolic syndrome to determine the need for lifestyle and 6.4%, the ADA recommends considering metformin for
modifications and therapeutic interventions to prevent or delay prevention of type 2 diabetes.
progression to type 2 diabetes or CVD. The Framingham Risk Additional therapeutic agents, such as lipase inhibitors,
Score and the new Pooled Cohort Equation from the American a-glucosidase inhibitors, and thiazolidinediones, have been
College of Cardiology/ American Heart Association are fre­ evaluated to delay or prevent type 2 diabetes; however, the
quently used within the United States to assess CVD risk. effectiveness, cost, and potential side effects must be consid­
ered before implementation (Table 5).
Type 2 Diabetes Mellitus
KEY POINTS
Type 2 diabetes mellitus accounts for most (90%-95%) diag­
nosed diabetes cases. It affects 10.9 million (26.9%) of adults in • Lifestyle modifications are a cost-effective intervention HVC
the United States aged 65 years or older. Asian Americans, that has been proven to decrease the risk of patients with
American Indians, Alaska Natives, Hispanics, and non­ prediabetes developing type 2 diabetes by 41% to 58%.
Hispanic black persons are at an increased risk for developing • In high-risk persons, the American Diabetes
diabetes compared with non-Hispanic white persons. The Association recommends considering metformin for
etiology of type 2 diabetes is likely multifactorial. There is often prevention of type 2 diabetes, particularly in patients
a strong family history of type 2 diabetes among first-degree who are younger than 60 years of age, have a BMI
relatives, although the specific genes responsible for the glu­ greater than 35, or have a history of gestational diabetes.
cose abnormalities remain unidentified.

4
Disorders of Glucose Metabolism

TABLE 5. Strategies to Prevent or Delay Onset of Type 2 times daily initially, to include fasting and 1-hour or 2-hour
Diabetes Mellitus postprandial values. Postprandial hyperglycemia in pregnancy
Intervention Effectiveness may predict worse fetal outcomes and complications.
Lifestyle interventions and/or pharmacologic agents
Diet and exercise Sustained weight loss of 7%,
should be implemented when glucose goals for gestational
with at least 150 minutes of
moderate exercise per week, diabetes are not met. Nutrition requirements for gestational
shown to delay onset of diabetes should allow for appropriate maternal weight gain
diabetes by up to 3 years
for normal fetal growth while obtaining goal glucose values. A
Smoking cessation Modestly effective as long as moderate exercise program is recommended for glycemic
it does not cause weight gain,
but is always recommended
control.
Insulin has traditionally been the mainstay of therapy for
Bariatric surgery Effective if used in morbidly
obese persons (BMI >40) gestational diabetes when glycemic goals are not met with diet
and exercise. Off-label use of metformin and glyburide in
Metformin• Shown to delay onset of
diabetes by up to 3 years pregnancy has been studied and there appears to be equiva­
Lipase inhibitors (orlistat) Shown to delay onset of
lence in efficacy with insulin; however, long-term safety data
diabetes by up to 3 years are lacking.
o.-Glucosidase inhibitors Shown to delay onset of Gestational diabetes resolves after pregnancy for most
(acarbose, voglibose) diabetes by up to 3 years women; however, the risk of developing type 2 diabetes is 5%
Thiazolidinediones Shown to delay onset of to 10% after delivery and 35% to 60% in the subsequent 10 to
(troglitazone, rosiglitazone, diabetes by up to 3 years 20 years. The ADA recommends diabetes screening for women
pioglitazone)
with a history of gestational diabetes using standard criteria at
Insulin and insulin Ineffective 6 to 12 weeks postpartum and every 3 years thereafter.
secretagogues (sulfonylureas,
meglitinides) KEY POINTS
ACE inhibitors and Ineffective • Lifestyle interventions should be implemented to meet
angiotensin receptor blockers
glycemic goals in women with gestational diabetes;
Estrogen-progestin Modest effect only however, when these are not met, insulin should be
'Preferred. initiated.
• For women with a history of gestational diabetes, diabe­
tes screening using standard criteria should occur at 6
Gestational Diabetes Mellitus to 12 weeks postpartum and every 3 years thereafter.
Relative insulin deficiency during the increased insulin resist­
ance associated with pregnancy can result in the development Uncommon Types of Diabetes Mellitus
of gestational diabetes mellitus. Pregnant women at high risk Genetic defects in beta-cell function and insulin action cause
for developing gestational diabetes include those from certain some uncommon forms of diabetes (see Table 3). Maturity­
racial or ethnic groups (Hispanic/Latino Americans, blacks, onset diabetes of the young (MODY) is an autosomal dominant
and American Indians), overweight or obese women, women monogenetic defect that affects beta-cell function but not
older than 25 years of age, and women with a strong family insulin action. MODY should be suspected in non-obese
history of type 2 diabetes. An estimated 2% to 10% of pregnant patients with a strong family history for diabetes when the
women have gestational diabetes. Complications related to onset of diabetes occurs before 25 years of age in the absence
gestational diabetes include miscarriage, fetal deformities, of autoantibodies. Genetic defects in insulin action cause insu­
large for gestational age infants, macrosomia, preeclampsia, lin resistance with varying degrees of hyperglycemia, as seen
complications during labor and delivery, increased perinatal with congenital lipodystrophy.
complications, and mortality. Complication risk is on a con­ Several endocrinopathies can impair insulin action or
tinuum with increasing hyperglycemia. secretion as a consequence of excess hormone production.
Disagreement exists among consensus groups regarding Conditions such as Cushing syndrome and pheochromocy­
the definition, screening methods, and diagnostic criteria for toma decrease the action of insulin secondary to excess
gestational diabetes. High-risk pregnant women should be cortisol and epinephrine, respectively. The hypokalemia
screened for overt diabetes at the initial prenatal visit using induced by hyperaldosteronism can inhibit the secretion of
criteria for nonpregnant women, according to the International insulin.
Association of Diabetes and Pregnancy Study Group (IADPSG)
and the ADA. In the absence of overt diabetes at the initial Management of Diabetes Mellitus
office visit, diabetes screening should occur between 24 and The most effective management of diabetes mellitus includes a
28 weeks' gestation. Once a diagnosis of gestational diabetes is multidisciplinary approach, including patient education and
made, glucose monitoring should be performed at least four support, engaging patients in their care and decision making,

5
D i sorde rs of Glucose Metabolism

lifestyle modifications with diet and exercise, reduced caloric Patient Education
intake for overweight and obese patients, and pharmacologic Diabetes self-management education (DSME) and diabetes
therapies when necessary to meet individualized glycemic self-management support (DSMS) are recommended at the
goals (Table 6). time of diagnosis of prediabetes or diabetes and throughout

TABLE 6. American Diabetes Association Recommended Outpatient Glycemic Goals for Adults with Diabetes Mellitus
State of Characteristics of Patients Hemoglobin Preprandial Postprandial Capillary Glucose
Health A,/ Capillary (1-2 hours after meal)•
Glucose•

Healthy Early in disease course <7.0% without 70-130 mg/dl <180 mg/dL(10.0 mmol/L)
severe recurrent (3.9-7.2 mmol/L)
Few comorbidities
hypoglycemia
Preconception
(<6.S% for
Patient preference select patients)b

Life expectancy>10 years

Complex health Significant comorbidities, including <8.0% without


issues advanced atherosclerosis or severe recurrent
microvascular complications hypoglycemia

Longer duration of diabetes


Frequent hypoglycemia
Hypoglycemia unawareness

Life expectancy <10 years

Older adults Few comorbidities <7.0%-7.5% 90-130 mg/dl


without severe (5.0-7.2 mmol/L)
Extended life expectancy
recurrent
No impairment of cognition or hypoglycemia
function

Multiple comorbidities <8.0% without 90-150 mg/dl


severe recurrent (5.0-8.3 mmol/L)
Hypoglycemic risk
hypoglycemia
Fall risk
Mild impairments in cognition and
function
Poor health <8.5% without 100-180 mg/dl
severe recurrent (5.6-10.0 mmol/L)
Chronic comorbidities with end-stage
hypoglycemia
disease
Long-term care placement

Moderate-to-severe impairment in
cognition and function
Limited life expectancy

Pregnant Preexisting type 1 or type 2 diabetes <6.0% without 60-99 mg/dl 100-129 mg/dL(5.6-7.1 mmol/L)
women severe recurrent (3.3-5.5 mmol/L)
hypoglycemia
Gestational diabetes :s;95 mg/dl 1-hour after meal: :s;140 mg/dL
(5.3 mmol/L) (7.8 mmol/L)
2-hours after meal: :s;120 mg/dL
(6.7 mmol/L)

a Recommended if goal can be met without severe recurrent hypoglycemia. lf severe recurrent hypoglycemia is present, there is no recommended hemoglobin A 1 , goal, as
modification of the patient's diabetes mellitus regimen to resolve severe recurrent hypoglycemia should take precedence. When severe recurrent hypoglycemia is resolved, an
hemoglobin A 1 c goal can be chosen, and treatment decisions can again be made based on that individualized hemoglobin A 1 c goal without frequent hypoglycemia.

bThis can be considered for patients with an early diagnosis of diabetes mellitus, no significant cardiovascular disease, or managed with lifestyle modifications or metformin.

Data from American Diabetes Association. Glycemic targets. Sec. 6. In Standards of Medical Care in Diabetes-2015. Diabetes Care. 2015;38(5uppl 1):533-540. [PMID: 25537705[.
(Modikation of Table 6.2 (p. 537).

Data from American Diabetes Association. Older adults. Sec. 10. In Standards of Medical Care in Diabetes-2015. Diabetes Care. 2015;38(5uppl 1):567-569. [PMID: 25537711 ].
(Modification of Table 10.1 (p. 568).

Data from American Diabetes Association. Management of diabetes in pregnancy. Sec. 12. In Standards of Medical Care in Diabetes-2015. Diabetes Care. 2015;38(Suppl 1 ):
577-579. [PMID: 25537713J.

6
Disorders of Glucose Metabolism

the lifetime of the patient. DSMS is an individualized plan that TABLE 7. Comparison of Hemoglobin A 1 c Value and
provides opportunities for educational and motivational Estimated Plasma Glucose Level
support for diabetes self-management. DSME and DSMS Hemoglobin A 1 c (%) Estimated Average Plasma
jointly provide an opportunity for collaboration between the Glucose Level
patient and health care providers to assess educational needs mg/dL(mmol/L)
and abilities, develop personal treatment goals, learn self­
management skills, and provide ongoing psychosocial and 6 126(7.0)
clinical support. Improved outcomes and reduced costs have 7 154(8.6)
been associated with DSME and DSMS. 8 183(10.2)
9 212 (11.8)
Self-Monitoring of Blood Glucose
10 240(13.4)
Blood glucose monitoring can involve a variety of modalities,
including self-monitoring of blood glucose (SMBG), hemo­ 11 269(14.9)
globin A1 e, or continuous glucose monitoring (CGM). 12 298(16.5)
SMBG is recommended for patients on multiple daily Adapted with permission of American Diabetes Association, from Translating the
injection (MDI) insulin therapy or continuous subcutaneous A 1 C assay into estimated average glucose values. Nathan DM, Kuenen J, Borg R,
Zheng H, Shoenfeld D, Heine RJ; A1(-derived average glucose study group.
insulin infusion (CSII) therapy. SMBG should be performed !erratum in Diabetes Care. 2009;32(1 ):2071. Diabetes Care. 2008;31 (8):1476.
IPMID: 185400461
frequently during several critical time periods: preprandial,
bedtime, before and after exercise, periods of symptomatic
hypoglycemia or hyperglycemia, and before important activi­ CGM combined with intensive insulin therapy in adults
ties such as operating dangerous machinery. Monitoring (;;;:25 years of age) with type 1 diabetes as a successful modality
blood glucose levels 1 to 2 hours after food consumption to lower hemoglobin A1e levels. The greatest improvements in
(postprandial) can be useful to assess prandial insulin cover­ glycemic control are associated with longer periods of CGM
age in patients with at-goal preprandial readings but with use. In patients using a CGM system, it is important to note
hemoglobin A 1e not at goal. Overnight blood glucose monitor­ that it does not replace SMBG. Calibration with SMBG is
ing can help detect hypoglycemia or dawn phenomenon. required at least twice daily with CGM systems. All CGM glu­
Success with SMBG requires the physician and patient to act cose values that warrant an immediate intervention should be
upon the information that it provides. This can include insu­ confirmed with SMBG prior to action due to a lag time ranging
lin dose adjustments, changes in meal content, or changes in from 5 to 21 minutes for several CGM brands between capillary
activity level to reach individualized glycemic goals. The data blood glucose and interstitial glucose. Rapid glucose fluctua­
for the role and cost-effectiveness of SMBG are less clear for tions further increase the lag time.
regimens without multiple daily insulin injections and non­ KEY POINTS
insulin regimens.
• Blood glucose monitoring, including self-monitoring of
It is often necessary to combine both SMBG and hemo­
blood glucose levels, hemoglobin A1c levels, or continu­
globin A 1e to determine if adequate control of glucose has been
ous glucose monitoring, is recommended for patients
achieved. There is a strong correlation between hemoglobin
with diabetes mellitus requiring multiple daily insulin
A 1 e and the average 3-month plasma glucose value. Therefore,
injections or continuous intravenous insulin injection
the ADA and the American Association for Clinical Chemistry
therapy.
advocate reporting both the hemoglobin A 1e and the estimated
plasma glucose levels (Table 7). Hemoglobin A1 e monitoring • The data for the role and cost-effectiveness of self- HVC
should be measured at the time of diagnosis and every monitoring of blood glucose levels are less clear for
3 months while making changes to achieve glycemic goals. regimens without multiple daily insulin injections and
Testing intervals can be decreased to twice yearly after glyce­ noninsulin regimens; generally this should be avoided.
mic goals have been met.
CGM technology measures real-time glucose values from Nonpharmacologic Approaches
the interstitial fluid every few seconds through the temporary Nonpharmacologic approaches to diabetes management
placement of a sensor subcutaneously for 3 to 7 days. The sen­ should be implemented throughout the lifespan of the
sor is connected to a transmitter that sends the data through patient. These approaches can be used alone or as adjunct
wireless radiofrequency to a display device. CGM glucose val­ therapy in type 2 diabetes to improve the success rate of
ues average + /- 15% from a laboratory glucose measurement. pharmacologic agents. Medical nutrition therapy and exer­
CGM may be useful in persons with frequent hypoglycemia, cise can be used in conjunction with insulin therapy for
hypoglycemic unawareness, or extreme fluctuations in glu­ patients with type 1 diabetes.
cose levels. CGM systems can rapidly identify hypo- or hyper­ Medical nutrition therapy is an essential component of
glycemia that is not always detected with SMBG or hemoglobin any successful management plan for patients with prediabetes
A 1e measurements. Additionally, the ADA endorses the use of or diabetes. Modest weight loss (2.0-8.0 kg [4.4-17.6 lb] or 7%)

7
Disorders of Glucose Metabol ism

through caloric reduction can benefit some overweight or 42% to 57%. Intensive insulin therapy has risks, including
obese adults with type 2 diabetes. significant increases in hypoglycemia and weight gain.
Consistent exercise provides beneficial effects on glucose Therapy should therefore be individually tailored for each
control, weight, and cardiovascular status. For persons with patient's preferences, lifestyle, education level, financial
diabetes in whom no contraindications exist, aerobic exercise resources, and comorbidities.
should consist of at least 150 minutes/week at a moderate Available insulin preparations and their activity profiles
intensity level, 75 minutes/week at a vigorous activity level, or are indicated in Table 8. Most persons with type 1 diabetes are
a combination of these two. Resistance training should be sensitive to the effects of exogenous insulin therapy, with ini­
incorporated into the exercise routine at least 2 days per week. tial total daily doses of insulin typically ranging from 0.3 to
Hypoglycemia and extreme hyperglycemia can worsen if pre­ 1 U/kg/d. A basal insulin dose should account for half of the
sent at the time of exercise and should be corrected before total daily dose of insulin, while the remaining insulin should
proceeding with increased physical activity. be divided to cover the number of meals consumed during the
Bariatric surgical procedures (restrictive and bypass) can day. Basal insulin coverage can be provided with one to two
be considered in obese patients with type 2 diabetes. Weight daily injections of insulin detemir, glargine, or neutral prota­
loss and diabetes remission rates are significant with these mine Hagedorn (NPH) insulin. CSII can also provide basal
procedures, but the long-term benefits require additional coverage with analogue insulin. For prandial coverage, ana­
studies. See MKSAP 17 Gastroenterology and Hepatology and logue or regular insulin is injected prior to meal consumption
MKSAP 17 General Internal Medicine for more information. or analogue insulin is bolused with CSII prior to meals. Insulin
Depression, anxiety, and diabetes-related stress are com­ dosing immediately after a meal is appropriate in certain situ­
mon among patients with diabetes and may impair their abil­ ations, particularly when food intake is unpredictable.
ity to achieve success with a diabetes management plan. Postprandial insulin dosing allows for a reduction in the insu­
Screening should occur continuously during the course of lin dose that is commensurate with the amount of food
diabetes treatment. ingested to avoid hypoglycemia that could have resulted from
the full insulin dose. For example, the postprandial insulin
Pharmacologic Therapy dose is reduced by 50% if only half of the meal is consumed.
An individualized treatment goal will help guide the selection
of the optimal treatment regimen. For many persons with
diabetes, a reasonable goal for hemoglobin A 1 e is less than 7.0% TABLE 8. Pharmacokinetic Properties of Insulin Products•
(or less than 6.5%, if this can be achieved without significant Insulin Type Onset Peak Duration
hypoglycemia). If severe recurrent hypoglycemia is present,
Rapid-acting or 5-15 min 45-90 min 2-4h
there is no recommended hemoglobin A 1e goal, as modifica­ analogue (lispro,
tion of the patient's diabetes regimen to resolve severe recur­ aspart, glulisine)
rent hypoglycemia should take precedence. The increased Short-acting (regular) 0.5-1h 2-4h 4-8 h
risks of hypoglycemia outweigh the risks of diabetes complica­ NPH insulin 1-3h 4-10h 10-18h
tions in older patients with longer disease duration, which
Detemir 1-2 h Noneb 12-24h<
necessitates consideration of a less-stringent glycemic goal.
The recommended goals from the ADA for blood glucose and Glargine 2-3h Noneb 20-24+h
hemoglobin A 1e levels are located in Table 6. Pre-mixed insulins
70%NPH/30% 0.5-1h 2-10h 10-18h
Therapy for Type 1 Diabetes Mellitus regular
Lifelong insulin therapy is the first-line treatment for type 1 50% NPH/50% 0.5-1 h 2-10 hd 10-18h
diabetes. Physiologic insulin therapy, also known as intensive regular
insulin therapy, is the ideal insulin regimen as it attempts to 75%NPl/25% lispro 10-20 min 1-6 h 10-18 h
mimic the actions of normal pancreatic beta cells. Intensive 50%NPl/50% lispro 10-20 min 1-6hd 10-18 h
insulin therapy includes multiple daily injections (MDI)
70%NPA/30% aspart 10-20 min 1-6h 10-18 h
(<::3 per day) with an intermediate or long-acting insulin for
NPA = neutral protamine aspart; NPH = neutral protamine Hagedorn; NPL = neutral
basal coverage and multiple preprandial injections through­ protamine lispro.
out the day with analogue or regular insulin. Intensive insu­ aThe time course of each insulin varies significantly between persons and within
lin therapy can also include continuous subcutaneous insu­ the same person on different days. Therefore, the time periods listed should be
considered general guidelines only.
lin infusion (CS!!) and meal-time boluses with an insulin
bBoth insulin detemir and insulin glargine can produce a peak effect in some per­
pump. Data support targeting normal glycemic levels with a sons. especially at higher doses.
goal hemoglobin A 1c of less than 7% for most persons with crhe duration of action for insulin detemir varies depending on the dose given.
type 1 diabetes to reduce long-term complications. Long­ dPremixed insulins containing a larger proportion of rapid- or short-acting insulin
term physiologic insulin therapy reduces early microvascular tend to have larger peaks occurring at an earlier time than mixtures containing
smaller proportions of rapid and short-acting insulin.
disease by 34% to 76% and reduces cardiovascular events by

8
Disorders of Glucose Metabolism

Regular insulin requires a longer time interval between KEY POINTS


prandial injection and food consumption compared with ana­
• Lifelong insulin therapy is the first-line treatment for
logue insulin due to its longer onset of action. Classic carbohy­
type 1 diabetes; physiologic insulin therapy reduces
drate counting with prandial analogue insulin allows flexibility
early microvascular disease by 34% to 76% in patients
and variety in the types and sizes of meals consumed by
with type 1 diabetes mellitus compared with nonphysi­
adjusting the dose based on the number of carbohydrates
ologic regimens.
ingested. Typically, 1 U of analogue insulin is used to cover
every 10 to 20 g of carbohydrate in the meal. Modified carbo­ • Continuous subcutaneous insulin infusion is a cost­
hydrate counting for patients who are unwilling or cannot effective treatment modality and should be considered
count carbohydrates includes fixed prandial doses of regular for select patients with type 1 diabetes mellitus if ade­
or analogue insulin that can be adjusted by 50% based upon quate glycemic control is not achieved with adherence
the size of the meal: regular (100% dose), small (50% dose), or to multiple daily injection therapy.
large (150% dose). In the setting of pre-meal hyperglycemia,
the prandial dose of insulin determined by the classic or Therapy for Type 2 Diabetes Mellitus
modified carbohydrate counting methods can be combined Lifestyle modifications must often be combined with oral
with a supplemental insulin dose to correct the hyperglyce­ pharmacologic agents for optimal glycemic control, particu­
mia. There are a variety of methods to determine the supple­ larly as type 2 diabetes progresses with continued loss of pan­
mental insulin dose needed for correction; however, an creatic beta-cell function and insulin production. Multiple oral
additional 1 U of analogue or regular insulin for every SO mg/dL agents may be required or used in conjunction with noninsu­
(2.8 mmol/L) above the target blood glucose at the pre-meal lin injectable agents or insulin as glycemic control worsens.
measurement is a reasonable starting point. For example, an There are many options for oral agents, with major differences
additional 2 U of regular or analogue insulin would be admin­ in cost, timing of administration, mechanism of action, and
istered for a patient with a blood glucose level of 210 mg/dL side-effect profiles (Table 9).
(11.7 mmol/L) if the target blood glucose is 150 mg/dL Metformin is the recommended first-line therapy to be
(8.3 mmol/L) or less. When administering any prandial or sup­ initiated either in conjunction with lifestyle modifications at
plemental insulin doses, the duration of action of previous the time of diagnosis or within 6 weeks of failing to obtain
analogue or regular insulin injections must be considered, as glycemic control with lifestyle changes alone. Metformin has a
the risk of insulin-stacking and subsequent hypoglycemia lower incidence of hypoglycemia and weight gain compared
increases if the dosing is too frequent. Allowing at least 3 to with some of the other oral agents and insulin. Gastrointestinal
4 hours between injections can decrease this risk. Premixed side effects (such as abdominal cramping or diarrhea) are
insulins containing a fixed percentage of a long-acting and common with metformin; initial low doses with gradual
regular or analogue insulin are given twice daily, particularly increases and administration of the tablet following a substan­
in patients who are unable to comply with more frequent daily tial meal can improve tolerance to the medication. Due to the
injections, although greater glycemic variability and hypogly­ potential risk of lactic acidosis, contraindications to met­
cemia are concerns when utilizing a nonphysiologic regimen. formin therapy include serum creatinine greater than 1.5 mg/
CSII should be considered for select patients with type 1 dL (133 µmol/L) in men and 1.4 mg/dL (124 µmol/L) in women,
diabetes if adequate glycemic control is not achieved with symptomatic heart failure or liver disease, and illness with
adherence to MDI therapy. CSII may be beneficial in several hemodynamic instability. Metformin must be withheld for
scenarios, including significant early morning hyperglycemia 48 hours in the setting of intravenous contrast dye. In a non­
("dawn phenomenon"), labile plasma glucose values and fre­ hospitalized patient, metforrnin should be withheld with any
quent DKA, frequent severe hypoglycemia or hypoglycemic illness that may cause dehydration.
unawareness, preconception and pregnancy, or active life­ If lifestyle modifications and maximally tolerated doses
styles/patient preference. If a patient is not adherent with of metformin fail to adequately control glucose, additional
insulin injections and blood glucose monitoring, adherence is agents should be added every 3 months until glycemic goals
unlikely to increase because a pump is prescribed; therefore, have been met. Without strong comparative-effectiveness
pump therapy is not recommended in the nonadherent patient. data to identify the best class of second-line drugs to be
Cost of the insulin regimen chosen should be weighed implemented, several factors must be considered. Patient
against potential benefits. MDI regimens require more insulin preferences and financial resources are key components to
supplies and glucose monitoring. Insulin analogues demon­ developing an individualized treatment plan. Another
strate fewer hypoglycemic events, but cost more than regular important determining factor in selection of the second-line
human insulin. Insulin pens increase both convenience and drug class is the patient's weight. Weight-neutral drug classes
cost when compared with insulin in vials. Insulin pump sup­ include a-glucosidase inhibitors and dipeptidyl peptidase-4
plies are expensive compared with other insulin therapies; (DPP-4) inhibitors . If weight loss is a desired effect, gluca­
however, data from several analyses indicate that overall CSII gon-like peptide 1 (GLP-1) mimetics, pramlintide, and
is a cost-effective treatment modality. sodium-glucose transporter-2 (SGLT2) inhibitors are

9
Disorders of Glucose Metabolism

TABLE 9. Pharmacologic Agents Used to Lower Blood Glucose Levels in Type 2 Diabetes Mellitus
Class Mechanism of Action Effect on Risks and Concerns Long-Term Studies on
Weight Definitive Outcomes

Insulin• Decreases hepatic Increase Hypoglycemia; insulin Decrease in both


glucose production, allergy (rare) microvascular and
increases peripheral macrovascular events
glucose uptake

Sulfonylureas Stimulate insulin Increase Hypoglycemia (especially in Decrease in microvascular


(tolbutamide, secretion drugs with long half-lives or events but possible increase
chlorpropamide, in older populations); in macrovascular events
glipizide, glyburide, weight gain with tolbutamide,
gliclazide, glimepiride)b chlopropamide, glyburide,
and glipizide; not seen with
gliclazide or glimepiride

Biguanides (metformin)b Decrease hepatic Neutral Diarrhea and abdominal Decrease in both
glucose production, discomfort; lactic acidosis microvascular and
increase insulin­ (rare); contraindicated in macrovascular events
mediated uptake of presence of progressive
glucose in muscles liver, kidney or cardiac
failure

a-Glucosidase inhibitors Inhibit polysaccharide Neutral Flatulence; abdominal May reduce CVD events
(acarbose, miglitol, absorption discomfort
voglibose)b

Thiazolidinediones Increase peripheral Increase Fluid retention; heart Unclear whether


(rosiglitazone, uptake of glucose, failure; macular edema; pioglitazone causes net
pioglitazone)b decrease hepatic osteoporosis (possible harm or good
glucose production increased risk of bladder
cancer with pioglitazone)

Meglitinides Stimulate insulin release Increase Hypoglycemia None


(repaglinide,
nateglinide)b

Amylinomimetics Slow gastric emptying, Decrease Nausea; vomiting; None


(pramlintide)• suppress glucagon increased hypoglycemic
secretion, increase risk with insulin
satiety
GLP-1 mimetics Slow gastric emptying, Decrease Hypoglycemia when used None
(exenatide and suppress glucagon in combination with
liraglutide)" secretion, increase sulfonylureas; nausea and
satiety vomiting; possible
increased risk of pancreatitis
and chronic kidney disease

DPP-4 inhibitors Slow gastric emptying, Neutral Hypoglycemia when used No increase in ischemic
(sitagliptin, saxagliptin, suppress glucagon in combination with cardiovascular events;
vildagliptin, linagliptin, secretion sulfonylureas; nausea; increased rate of
alogliptin)b increased risk of infections; hospitalization for heart
possible increased risk of failure with saxagliptin
pancreatitis
SGLT2 inhibitors Increases kidney Decrease Hypoglycemia with insulin None
(dapagliflozin and excretion of glucose secretagogues and insulin;
canagliflozin)b hypotension; kidney
impairment; hypersensitivity
reactions; increased
candidal genital infections
and urinary tract infections

CVD = cardiovascular disease; DPP-4 = dipeptidyl peptidase-4; GLP-1 = glucagon-like peptide-1; SGLT2 = sodium-glucose co-transporter 2.

11lnjection.

bQral.

10
Disorders of Glucose Metabolism

candidates to consider. Weight gain is likely with the use of and severe hyperglycemia that can lead to volume depletion
insulin, sulfonylureas, thiazolidinediones, and meglitinides. and electrolyte abnormalities.
The risk of hypoglycemia must be considered with the selec­
tion of any therapeutic agent, particularly when it is com­ Hospitalized Patients with Diabetes Mellitus
bined with insulin secretagogues or insulin. Gastrointestinal Critically ill patients with type l diabetes mellitus will require
side effects from GLP-1 mimetics and pramlintide may insulin therapy upon admission to the hospital. For critically
decrease tolerability for some patients and should not be ill patients with type 2 diabetes, intravenous insulin infusion
used in patients with gastroparesis. Patients with frequent therapy should be initiated when plasma glucose levels exceed
candidal genital infections would not be ideal candidates for 180 to 200 mg/dL (10-11.l mmol/L). Glucose goals on intrave­
SGLT2 inhibitor therapy. nous insulin are 140 to 200 mg/dL (7.8-11.1 mmol/L) with
Insulin therapy should be strongly considered in the set­ frequent bedside point-of-care (POC) monitoring every 1 to
ting of symptomatic hyperglycemia or markedly elevated 2 hours for insulin adjustments.
hemoglobin A,c (>8.5% to 9%) at the time of diagnosis or In noncritically ill patients, the ADA and AACE advocate a
when lifestyle modifications and/or noninsulin therapies fail premeal glucose goal of less than 140 mg/ell (7.8 mmol/L)
to achieve glycemic goals. The American Association of and random plasma glucose values less than 180 mg/clL
Clinical Endocrinologists (AACE) recommends weight-based (10 mmol/L). Therapy adjustments should be considered when
initiation of basal insulin at initial doses of 0.1 to 0.3 U/kg. plasma glucose levels are less than 100 mg/dL (5.6 mmol/L)
The dose should be increased several units every 2 to 3 days and are necessary when glucose values fall below 70 mg/dL
to reach fasting plasma glucose goals, based on the patient's (3.9 mmol/L) to avoid continued hypoglycemia. In contrast. the
SMBG readings. Reductions of insulin doses by 10% to 40% American College of Physicians (ACP) recommends avoiding
should be made in the setting of hypoglycemia with insulin glucose levels less than 140 mg/dL (7.8 mmol/L) owing to the
titrations. If glycemic goals are not met with basal insulin, increased risk of hypoglycemia with tighter glycemic control.
then prandial insulin should be added to the regimen with Insulin is the preferred therapy and likely the safest choice
frequent titration of doses for optimal glucose control. When for achieving inpatient glycemic control. Use of sliding scale
premeal glucose values are not at a patient-specific goal, the insulin alone is not recommended, as it is not physiologic and
preceding prandial insulin dose should be increased or frequently causes large glucose fluctuations owing to the
decreased by 10% to 20% in the setting of hyper- or hypogly­ inherent reactive nature of its dosing, coupled with the near
cemia, respectively. (Also see section on Therapy for Type 1 universal lag time between measurement of glucose and injec­
Diabetes Mellitus.) tion of insulin that occurs in most hospitals. The recom­
mended insulin regimen should incorporate both basal and
KEY POINTS
prandiaJ coverage. In the setting of preprandial hyperglycemia.
• For patients with type 2 diabetes mellitus, metformin is pranclial coverage can be supplemented with additional insu­
recommended first-line therapy and should be initiated lin (correction factor insulin). Prandial coverage should
in conjunction with lifestyle modifications; it has a account for the carbohydrates consumed at each meal and be
lower incidence of hypoglycemia and weight gain com­ adjusted accordingly. POC glucose monitoring should coincide
pared with some of the other oral agents and insulin. with insulin administration before meals and at bedtime. with
• In older patients with type 2 diabetes mellitus of longer overnight measurements to monitor for hypoglycemia only if
disease duration, treatment of severe recurrent hypo­ fasting readings are elevated or the patient is symptomatic.
glycemia should take precedence over controlling This glucose monitoring regimen will simulate the patient's
hemoglobin A,c values; the increased risks of hypogly­ home routine after discharge. POC monitoring should occur
cemia outweigh the risks of diabetes complications. every 6 hours when a patient is on insulin therapy and receives
nothing by mouth.
Outpatient CSII therapy can be continued if the patient is

CJ Inpatient Management physically and mentally able to safely administer this therapy
under proper supervision from health care providers with CS!I
of Hyperglycemia expertise. POC glucose monitoring, basal rates of insulin, and
Inpatient hyperglycemia, defined as consistently elevated patient-initiated bolus amounts of insulin should be docu­
plasma glucose values above 140 mg/dL (7.8 mmol/L). is asso­ mented in the medical record.
ciated with poor outcomes. Attempts lo decrease morbidity Oral agents and noninsulin injectable agents do not have
and mortality with tight glycemic control (80-110 mg/dL [4.4- safety or efficacy data in the hospital setting. The safest recom­
6.l mrnol/L]) have not consistently demonstrated improve­ mendation is to discontinue these agents upon admission to the
ments in adverse outcomes and, in some settings, have shown hospital. although continuation can be considered in a stable
increased rates of severe hypoglycemic events and mortality. patient with glycemic control at goal and no anticipated changes
As a result, revised inpatient glycemic targets are less stringent in nutrition or hemodynamic status. These agents can be par­
than outpatient glucose targets to avoid both hypoglycemia ticularly dangerous in fasting states or when organ pe1fusion or

11
Disorders of Glucose Metabolism

r,,, !'unction is compromised. Resumption of these medications corrected. cognitive function begins to decline and can be
LIi.i may be considered once a patient is stable with regular activities rapidly followed by loss of consciousness. seizures. and
and nutrition or at the time of hospital discharge. Cl
CONT.
death. Relative hypoglycemia occurs when a patient has
symptoms of hypoglycemia but the plasma glucose level is
KEY POINTS
greater than 70 mg/dL (3.9 mmol/L). This can occur with
• For critically ill patients with type 2 diabetes mellitus, rapid decreases in glucose or with correction of glucose to
intravenous insulin infusion therapy should be initiated near-normal glycemic levels in a patient with a history of
when plasma glucose levels exceed 180 to 200 mg/dL prolonged hyperglycemia (plasma glucose >200 mg/dL
(10-11.1 mmol/L); glucose goals on intravenous insu­ [11.1 mmol/L]). Relative hypoglycemia can be diminished if'
lin are 140 to 200 mg/dL (7.8-11.1 mmol/L) with fre­ glucose levels are maintained closer to normal ranges and if
quent bedside point-of -care monitoring every 1 to treatment to goal glucose level is achieved over a longer
2 hours. period of lime in patients with a history of prolonged uncon­
H'IC • For noncritically ill patients, basal and prandial subcu- trolled diabetes.
taneous insulin is the preferred and safest choice for The etiology of hypoglycemia can be quite variable.
achieving inpatient glycemic; oral agents and noninsu­ Exercise can lead to hypoglycemia if appropriate measures
lin injectable agents do not have proven safety or effi­ are not taken to avoid it. Prior to exercise. consumption of a
cacy data in the hospital setting. snack with 15 to 30 g of carbohydrates can help reduce the
risk of hypoglycemia. or a patient can reduce the close of
Hospitalized Patients Without Diabetes Mellitus prandial insulin given at the meal prior to the planned exer­
cise. if on an MDI regimen. A snack with complex carbohy­
Hyperglycemia as a result of acute stress relalecl to illness.
drates is often required after prolonged exercise lo replenish
concomitant medications. or enteral/parenteral nutrition can
glycogen stores since glucose utilization can be prolonged in
occur in patients without a previous history of glucose abnor­
muscles and the liver. In over weight/obese patients.
malities. The glycemic goals and glucose-management strate­
decreasing the insulin close instead of ingesting snacks
gies in Lhis population should follow those for hospilalized
before exercise can avoid additional weight gain. Poor tim­
patients with diabetes. Hyperglycemia in hospitalized patients
ing or skipping of meals or consumption of smaller amounts
may also indicate the presence of previously undiagnosed
of' food without an adjustment to insulin doses or oral hypo­
diabetes. Measurement of hemoglobin A1 , in hyperglycemic
glycemic agents can cause hypoglycemia. Use of a non­
non-hospilalizecl patients, if feasible, can provide insight into
physiologic sliding scale insulin regimen or use of an
the length of the hyperglycemia. A hemoglobin A1c level
aggressive supplemental insulin correction factor regimen
greater lhan 6.5% suggests long-standing hyperglycemia.
is often the etiology of hypoglycemic events. A reduction in
Follow-up diabetes screening and care should be implemented
kidney function. particularly in elderly patients, can
after discharge from the hospital.
decrease clearance of insulin or insulin secretagogues and
lead to prolonged hypoglycemia. Alcohol consumption can
cause delayed hypoglycemia.
Management of Hypoglycemia Treatment of hypoglycemia is twofold: immediate cor­
Hypoglycemia in Patients with Diabetes Mellitus rection of hypoglycemia and prevention of future events. If' a
Hypoglycemia is a common complication of intensive thera­ patient is conscious. 15 to 20 g of a carbohydrate with glucose
peutic regimens in patients with diabetes mellitus, often limit­ should be consumed. Glucose tablets or glucose gel are ideal
ing the ability to safely reach glycemic goals for many patients. treatment regimens. The blood glucose level should be
Avoidance of hypoglycemia prior to focusing on a patient's checked again after 15 minutes. and consumption of 15 to
hemoglobin A 1 , goal is of utmost importance because of the 20 g of glucose should occur again if the hypoglycemia does
significant morbidity and mortality associated with low not improve to greater than 70 mg/dL (3.9 mmol/L). Since the
plasma glucose levels. eftects of the insulin or oral hypoglycemic agents are likely
Hypoglycemia is defined as a plasma glucose level less still present, a meal or snack should be consumed after the
than 70 mg/ell (3.9 rnrnol/U. Insulin secretion ceases when glucose has been corrected to avoid continued hypoglycemia.
the glucose level falls below 80 mg/ell (4.4 mmol/L). Every patient with diabetes on medications associated with
Hyperaclrenergic symptoms begin to alert the patient to hypoglycemia should receive a prescription for a glucagon kit,
hypoglycemia Lhrough an increase in heart rate. swealing. which should be used when oral consumption of glucose is
lremors. hunger. and anxiety when glucose levels decline. not possible or safe.
Typically. Lhe body responds to hypoglycemia by secreting Relaxing the glycemic targets and hemoglobin A 1 , goals
cou nlerregulatory hormones. such as glucagon, epinephrine, and reducing doses of therapeutic agents will decrease the risk
norepinephrine. cortisol. and growth hormone. in succes­ of future hypoglycemia. A review of a patient's diabetes self-­
sion based on the escalating degree of hypoglycemia. lf the management skills can also help identify recurring risk factors
counterregulatory measures fai I or the hypoglycemia is not for hypoglycemia. Cl

12
Disorders of Gluc o s e Metabolism

KEY POINTS Fasting Hypoglycemia


• Hypoglycemia is defined as a plasma glucose level less Suspected hypoglyccmi,1 that is either spont.:incous or begins
than 70 mg/dL (3. 9 mmol/L) and is associated with sig­ af'rer a [�1st \hould be e\·,1luated by the lcillo\1·ing simultaneous
nificant morbidity and mortality. laboratory measurements: glucose. insulin. C peptide. proin
sulin. � hydroxybulyratc. and insulin secretagogue screen.
• Relaxing the glycemic targets and hemoglobin A,c goals
C-peptide and proinsulin .:ire mc.1,ures of' the endogenous
and reducing doses of therapeutic agents will decrease
production 01· insulin. [3 I lydroxybutyrate is suppressed by
the risk of future hypoglycemia.
endogenous ,rnd exogenous insulin. but \\'CJUld be unsup

Cl Hypoglycemia in Patients Without


pressed in a norm,11 physiologic stale of' hypoglycemia or in a
non insulin mediated condition. If' hypoglyccmi,1 is not pre
Diabetes Mellitus sent at the time or l'\".liu,l!ion. a 72 hour !�1st is indic,1ted.
CONT
Hypoglycemia in patients without diabetes is rare. thus e\·alu­ which is typic,1lly pcrl<irmed in consull,1lio11 \\·ith an endocri
ation for pathologic hypoglycemia should only occur when nologist. This test invol\·cs measurement or the abo\·e
Whipple triad is present: symptomatic hypoglycemia. docu mentioned laboratory 1·.:ilue� every 6 hours until the plasma
mented hypoglycemia al 55 mg dL (3.1 11111101 U or lower. and glucose Je1·cl reaches 60 mg di. (:3.3 mmol L) and subse­
prompt symptomatic relief with correction or hypoglycemia. quently every I tu 2 hours until spccii'ic plasma glucose.
Hypoglycemia should not be conl1rmed 'vYith POC gluco"c svmptom. or time ci'ileria arc met. This test ,1Jso in\'CJlves
monitors. but instead with a 111orc accurate established labor,1 measuring the response to gluc,1go11 ,1clministr.1tion.
tory method. Hypoglyce111ia in p,llients without diabetes is Ev,1lu,11ion l<>r anti insulin ,rntibodiescan detect the rnrc con
usually related to drugs. illness. hormonal dcliciency. 11011 di Lion or· insulin ,iutoimmune hypoglycemi,1 as the underly
islet cell tumor. endogenous hyperinsulinism noninsulinoma. ing etiology !or the hypoglycemia.
pancreatogenous hypoglycemia. depiction of' hep.itic glycogen
stores. or alcohol ingestion. Diagnostic studies should be Postprandial Hypoglycemia
obtained during a spontaneous hypoglycemic episode or dur Postprandial hypoglycemia without a history or a prior b,1riat
ing an attempt to recreate a scenario known to C,lllSL' hypogly ric procedure is rnre. It typically occurs \\'ilhin S hours of'lcmd
cemia. such as prolonged [�isling or arter ,1 mixed mc,11. \Nhich consu111ption. t\ mixed meal tolcr,rnce lest is usually per
consists of' the type or lc>od that induces the hypoglycemia. lormed in consultation with an endocrinologist and measures
typically a simple carbohydrate-rich meal. such as orange the glucose lc\·el as symptollls occur. Clucose. insulin. proin
juice. pancakes. and syrup. Hypoglycemia has classiG1lly been sulin. and C: peptide levels ,ire measured prior to the meal and
categorized as occurring in the l\1sti11g versus postprandial repc,1tcd at :30-minulc inten·.!ls or al the time of'symplomatie
stale. although the etiologies of' each of' l hesc classi 11c,1lions of' hypoglycemia (blood glucose lc\·el <60 mg d1. l:L3 mmol I.I)
hypoglycemia are not mutually exclusive. The dil'lerential within the S hours af'tcr mc,11 consumption. If' symptomatic
diagnoses based on the laboratory test results 111-c found in hypoglyccllli,1 occurs. insulin antibodies arc llleasurcd ,md an
Table 10. J 111aging studies should not occur unless biochemical oral hypoglycclllic ,1genl screening lest is oiJuined. Tre,1lmenl.
evidence of endogenous hyperinsu Iinism is rnn Ii r111ed and is \\·ith or without the detection of' pathologic hypoglycemia on
related to a tumor or pancreatic abnor111ality. the mixed meal test. often invol\·cs slllall. l'requenl complex

TABLE 1 0. Differential Diagnosis of Spontaneous Fasting Hypoglycemia• in a Patient Without Diabetes


Diagnosis Serum Plasma Plasma Serum Serum Insulin Urine or Blood Metabolites
Insulin C-Peptide Proinsulin �-hydroxybutyrate Antibodies of Sulfonylureas or
Meglitinides

lnsulinoma i i i .J, Negative Negative


Surreptitious use of i i i .J, Negative Positive
sulfonylureas
of meglitinides

Surreptitious use of i .J, .J, .J, Negative Negative


insulin
Insulin autoimmune i i i .J, Positive Negative
hypoglycemia

'Symptomatic hypoglycemia, fasting plasma glucose 55 mg/dl (3.1 mmol/L) or lower, and prompt symptomatic relief with correction of hypoglycemia (Whipple triad).

Data from Cryer PE, Axelrod L, Grossman AB, et al. Evaluation and management of adult hypoglycemic disorders: an Endocrine Society Clinical Practice Guideline. J Clin
Endocrinol Metab. 2009 Mar:94(3):709-28. [PMID: 19088155[

13
Disorders of Glucose Metabolism

Cl
COiT.
mc,ils composed olprotcin. l�ll. and ccirbohydr<1te to avoid the
sensation of' hypoglycemia.
ketone laboratory measurements often use the nitroprusside
reaction, which only estimates acetoacetate and acetone lev­
els that may not be elevated initially. Although hyperglyce­
mia is the typical f1nding <11 presentation with DKA. patients
Acute Complications of can present with a range of'plasma glucose values. including
those in the normal range (Figure I). The anion gap is ele­
Diabetes Mellitus vated. Stress-related mild leukocytosis is of'len present.
Diabetic Ketoacidosis and Hyperglycemic Higher levels of leukocytosis may indicate an infectious pro­
Hyperosmolar Syndrome cess as the etiology of' the hyperglycemia. Serum sodium
Diabetic ketoacidosis (OKA) and hyperglycemic hyperosmo levels can be low clue to osmotic shifts of' water f'rom the
IJ r syndromc (I I HS) Jre ,1cute compI ications of' uncontrolled intracellular to extracellular spaces. Normal or elevated
hyperglycemia with life-threatening consequences ii. not serum sodium levels are indicative of severe volume deple­
recognized and treated early. DKA typically occurs in the tion. Serum potassium levels may be elevated due to shifts
setting of' hyperglycemia with relative or absolute insulin from the intracellular to extracellular spaces due to ketoaci­
clef'iciency ,rncl <111 increase i 11 counterregu latory hormones. dosis and the absence of suf'fkienl insulin. Normal or low
Suf'f'icient amounts or insulin are not present to suppress potassium levels on presentation indicate low potassium
lipolysis and oxid,1tion of' f'ree fatty acids. which results in stores in the body with need for correction prior to initiation
ketone body production and subsequent metabolic acidosis. of' insulin therapy to avoid cardiac arrhythmias. Serum amyl­
l)Ki\ occurs more f'requently with type I diabetes. although ase and lipase levels also can be elevated in the absence of
10'':, to 30% of' cases occur in patients with type 2 diabetes. p,rncreatitis.
111 IS occurs in the setting of'parlial insulin def'iciency that is HHS typically presents with normal or small amounts
more typical of' type 2 diabetes. There is sufficient insulin in of'urine or serum ketones. Plasma glucose values in HHS are
patients with HHS to suppress lipolysis and production of' typically greater than in OKA and can exceed 1200 mg/ell
ketone bodies. but inadequate amounts to prevent the (66.6 mmoltU. The serum osmolality is elevated greater
hyperglycemia. dehydration. and hyperosmolality charac­ than 320 mOsm/kg H 2 0. The serum bicarbonate level is
teristic of' HHS. greater than 18 mEq/L (JS mmol/L.). and the pH remains
Se\·eral risk factors Gm precipitate the development or greater than 7.3.
extreme hyperglycemia: infection. intentional or inadvertent Treatment of OKA and HHS requires correction of hyper­
insulin therapy nonadhercnce. myocardial inforction. stress. glycemia with intravenous insulin infusions. f'requent moni­
trauma. <llld confounding medications. such as glucocorti­ toring and replacement of electrolytes. correction of hypov­
coids or ,llypical antipsychotic agents. In addition. DKA may olemia with intravenous fluids. and possible correction of'
be the initial clinical presentation in some patients with previ­ acidosis (Table 11). The ICU is the best place for management
ously undiagnosed type I or type 2 diabetes. An illness or event of severe hyperglycemia because of the frequent monitoring
that leads to dehydration will of'tcn precipitate the hyperglyce­ required with intravenous insulin therapy. the need for mon­
mia associated with HI IS. itoring for potential electrolyte-induced arrhythmias. and the
Symptoms of' extreme hyperglycemia in Dl(A and HHS potential for rapid decompensation. Plasma glucose levels
include polyuria. polydipsia. unintentional weight loss. vomit should be monitored initially every hour while on insulin
ing. weakness. and mentation changes. Dehydration and met­ infusion therapy. Electrolytes should be monitored every 2 to
abolic abnormalities worsen as hyperglycemia progresses. 4 hours. depending on the initial electrolyte deficits and level
which can lead to respiratory failure. lethargy. obtundalion. of' acidosis. Cl
coma. and death. OKA can occur within several hours of the
KEY POINTS
inciting event. The development of HHS is less acute than OKA
and may take dJys to weeks lo develop. HHS typically presents • The development of hyperglycemic hyperosmolar syn­
with more extreme hyperglycemia and mental status changes drome is less acute than that of diabetic ketoacidosis
compared with OKA. and may take days to weeks to develop; however, hyper­
The initi.11 e\·,duation of'severe hyperglycemi;i includes glycemic hyperosmolar syndrome typically presents
�erologic studies (plasma glucose. serum ketones. blood with more extreme hyperglycemia and mental status
urea nitrogen. creatinine. electrolytes. calculated anion changes compared with diabetic ketoacidosis.
gap. arterial blood gases. osmolality. complete blood count • Treatment of diabetic ketoacidosis and hyperglycemic
with differential. blood cultures). urine studies (ketones. hyperosmolar syndrome requires correction of hyper­
urinalysis. urine culture). chest radiograph. and an elec­ glycemia with intravenous insulin infusions, frequent
trocardiogram. monitoring and replacement of electro'lytes, correction
Urine ancl serum ketones are elevated in DKA: however. of hypovolemia with intravenous fluids, and possible
a negative measurement initially does not exclude OKA. correction of acidosis.
�-Hydroxybutyrate is the major ketone body in OKA. but

14
Disorders of Glucose Metabolism

28
Hyperosmolar
26 Normal Nonketotic
Range
24

22

20

18

16

:.c 14
E
12
Ill

10

8
Classic
6 Euglycemic
OKA
4

0
100 200 300 400 500 600 700 800 900 1000
5.6 11.1 16.7 22.2 27.8 33.3 38.9 44.4 50.0 55.5
Plasma glucose (mg/dl [mmol/L))

FIGURE 1. Spectrum of metabolic decompensation that occurs in diabetic ketoacidosis. DKA = diabetic ketoacidosis.

TABLE 11. Management of Hyperglycemic Crisis (DKA and HHS)


Fluids Insulin (Regular) Potassium Correction of Acidosis

Assess for volume status, then Give regular insulin, 0.1 U/kg, as Assess for adequate kidney If pH is <6.9, give sodium
give 0.9% saline at 1 Uh initially an intravenous bolus followed function, with adequate urine bicarbonate, 100 mmol in
in all patients, and continue if by 0.1 U/kg/h as an intravenous output(approximately 50 400 ml of water, and
patient is severely hypovolemic. infusion; if the plasma glucose mUh). If serum potassium is potassium chloride, 20 mEq,
Switch to 0.45% normal saline level does not decrease by <3.3 mEq/L(3.3 mmol/L), do infused over 2 hours. If pH is
at 250-500 mUh if corrected 10% in the first hour, give an not start insulin but instead 6.9 or greater, do not give
serum sodium level becomes additional bolus of 0.14 U/kg give intravenous potassium sodium bicarbonate.
normal or high. When the and resume previous infusion chloride, 20-30 mEq/h, through
plasma glucose level reaches rate; when the plasma glucose a central line catheter until the
200 mg/dl(11.1 mmol/L) in level reaches 200 mg/dl(11.1 serum potassium level is
patients with DKA or 300 mg/ mmol/L) in DKA and 300 mg/dl >3.3 mEq/L(3.3 mmol/L); then
dL(16.7 mmol/L) in HHS, switch (16.7 mmol/L) in HHS, reduce to add 20-30 mEq of potassium
to 5% dextrose with 0.45% 0.02-0.05 U/kg/h, and maintain chloride to each liter of
normal saline at 150-250 mUh. the plasma glucose level intravenous fluids to keep the
between 150-200 mg/dl serum potassium level in the
(8.3-11.1 mmol/L) until anion 4.0-5.0 mEq/L(4.0-5.0 mmol/L)
gap acidosis is resolved in DKA. range. If the serum potassium
level is >5.2 mEq/L(S.2 mmol/L),
do not give potassium chloride
but instead start insulin and
intravenous fluids and check
the serum potassium level
every 2 hours.

DKA = diabetic ketoacidosis; HHS= hyperglycemic hyperosmolar syndrome.

15
Diso rder s of Gluco se Metabolism

Chronic Complications The most recent American College of Cardiology I


American Heart Association guidelines base treatment recom­
of Diabetes Mellitus mendations for patients with diabetes on age, the presence of
Cardiovascular Morbidity atherosclerotic cardiovascular disease (ASCVD), or estimated
Cardiovascular disease (CVD) is a major contributor to mor­ 10-year ASCVD risk using the Pooled Cohort Equations; a spe­
bidity and mortality among patients with diabetes mellitus. cific goal LDL cholesterol level is no longer used in these guide­
Diabetes alone is an independent risk factor for CVD and is lines. Treat patients with diabetes and known cardiovascular
considered a CVD equivalent. Concomitant risk factors in or other vascular disease with high-intensity statin therapy. In
patients with diabetes, such as hypertension, obesity, and dys­ the absence of known cardiovascular or vascular disease, pro­
lipidemia, also contribute to the development of CVD and vide high intensity statin therapy to patients with diabetes
should be identified early through screening (Table 12). if the LDL cholesterol level is greater than 190 mg/dL
The 8 111 Joint National Committee (JNC-8) recently (4.9 mmol/L) or the 10-year ASCVD risk is equal to or greater
revised its recommended blood pressure goals for patients than 7.5%. Provide moderate-intensity statin therapy for
with diabetes to 140/90 mm Hg or less, citing a lack of data patients with diabetes and a 10-year ASCVD risk less than
to support lower targets. In contrast, the American Diabetes 7.5%. Consider withholding statin therapy in patients with
Association (ADA) recommends a blood pressure goal of less diabetes younger than 40 years without additional cardiovas­
than 140/80 mm Hg. The ADA advocates for a lower systolic cular risk factors. In contrast, ADA guidelines continue to
blood pressure (<130 mm Hg) in select patients (young, long recommend an LDL cholesterol goal in patients with diabetes
life expectancy, increased risk of stroke), if this can be accom­ of less than 100 rng/dL (2.6 mmol/L), with the option of LDL
plished safely. Although JNC-8 does not specify use of an ACE cholesterol less than 70 mg/dL (1.8 mmol/L) in patients with
inhibitor or an angiotensin receptor blocker (ARB) as initial clinical ASCVD. Therefore, it is recommended that statin ther­
therapy for patients with diabetes and hypertension in the apy be added to lifestyle modifications in patients with diabe­
absence of chronic kidney disease , the ADA recommends tes who have clinical ASCVD, are older than 40 years of age
preferential use of these agents in treating hypertension in with CVD risk factors, or are younger than 40 years of age with
these patients. LDL cholesterol not at goal.

TABLE 12. Screening Recommendations for Chronic Complications of Diabetes Mellitus


Chronic Clinical Situation When to Start Screening Frequency Preferred Screening Test
Complication Screening

Retinopathy Type 1 diabetes At 5 years after Annually• Dilated and comprehensive


diagnosis eye examination
Type 2 diabetes At diagnosis Annually• Dilated and comprehensive
eye examination
In pregnant women with First trimester Every trimester and then Dilated and comprehensive
either type of diabetes closely for 1 year postpartum eye examination
In women with either During preconception Same as recommendations Dilated and comprehensive
type of diabetes planning for pregnant women once eye examination
planning to conceive conception occurs
Nephropathy Type 1 diabetes At 5 years after Annuallyb Albumin-creatinine ratio on
diagnosis random spot urine
Type 2 diabetes At diagnosis Annuallyb Albumin-creatinine ratio on
random spot urine
Neuropathy Type 1 diabetes At 5 years after Annually 10-g monofilament, 128-Hz
(distal symmetric diagnosis tuning fork, ankle reflexes
polyneuropathy)
Type 2 diabetes At diagnosis Annually 10-g monofilament, 128-Hz
tuning fork, ankle reflexes
Cardiovascular Hypertension At diagnosis Every visit Blood pressure
disease measurement
Dyslipidemia At diagnosis Annuallye Fasting lipid profile
"It is reasonable to screen every 2 years if no diabetic retinopathy is present and to screen more often than annually if diabetic retinopathy is advanced or progressing rapidly.

hThe American Diabetes Association guidelines state that it is reasonable to assess progression of disease and response to therapeutic interventions with continued monitoring of
urine albumin excretion.

cit is reasonable to screen every 2 years if lipid parameters are at goal.

16
Disorders of Glucose Metabolism

KEY POINTS dom spot urine collection or a 24-hour urine collection.


Persistently elevated levels of urine albumin excretion are
• High-intensity statin therapy is indicated for patients
defined as greater than or equal to 30 mg/g in a spot urine
with diabetes and known cardiovascular or vascular
measurement or 30 to 299 mg/24 h and greater than or equal
disease; LDH cholesterol greater than 190 mg/dL
(4.9 mmol/L), or atherosclerotic cardiovascular disease to 300 mg/24 h. Urine albumin levels should be elevated on
multiple samples over 3 to 6 months to diagnose albuminuria,
10-year risk of equal to or greater than 7 .5%.
as false-positive elevations can occur in the setting of illness,
• Moderate intensity statin therapy is indicated for patients menstruation, recent exercise, extreme hyperglycemia or
with diabetes 40 years of age and older and an atheroscle­ hypertension, and heart failure. Screening timelines for urine
rotic cardiovascular disease 10-year risk less than 7.5%. albumin excretion are found in Table 12. Annual measure­
ments of serum creatinine and an estimated glomerular filtra­
Diabetic Retinopathy tion rate (GFR) can be utilized in conjunction with the urine
Among adults aged 20 to 74 years, diabetic retinopathy is the albumin measurement to determine the stage of chronic kidney
leading preventable cause of blindness. Changes associated disease. When the estimated GFR is less than 30 mL/min/
with nonproliferative retinopathy include retinal thickening 1.73 m2, a referral to a nephrologist is recommended.
from macular edema, infarcts (resulting in "cotton wool" spots Diabetic nephropathy can be prevented or delayed with
or soft exudates), hard exudates, and hemorrhages. With pro­ optimal plasma glucose and blood pressure control. In non­
liferative retinopathy, neovascularization occurs secondary to pregnant normotensive patients with persistently elevated
chronic retinal ischemia. These new vessels may rupture, urine albumin excretion, an ACE inhibitor or angiotensin
causing intraocular hemorrhage and subsequent fibrosis and receptor blocker (ARB) is recommended to decrease progres­
retinal detachment. sion of nephropathy. In nonpregnant hypertensive patients
Risk factors for diabetic retinopathy include long-term with persistently elevated urine albumin excretion and hyper­
diabetes, poorly controlled diabetes, hypertension, and tension, the ACE inhibitor or ARB should be titrated to achieve
nephropathy. Retinopathy can be accelerated in pregnant a blood pressure goal of less than 130/80 mm Hg. Measurement
women with type 1 diabetes. Rapid improvements in glycemic of urine albumin annually after initiation of therapy with an
levels for pregnant women and nonpregnant patients can tem­ ACE inhibitor or ARB is reasonable to assess disease progres­
porarily worsen preexisting retinopathy. sion and therapeutic response as evidenced by stabilization or
Screening guidelines vary depending on the type of dia­ reduction of urine albumin excretion. Data are conflicting
betes, time of diagnosis, and pregnancy status (see Table 12). regarding the ability of low-protein diets to slow the progres­
Optimal blood glucose and blood pressure control can sion of kidney disease, but these diets may be considered if
prevent or delay the progression of diabetic retinopathy. Laser nephropathy progresses while using an ACE inhibitor or ARB
photocoagulation is used to treat retinopathy as severity pro­ or after achieving target plasma glucose and blood pressure
gresses. Focal laser photocoagulation of the retina can restore goals. ACE inhibitor/ARB combination treatment is not rec­
some vision and reduce the risk of further vision loss with ommended.
macular edema. Panretinal laser photocoagulation reduces
KEY POINTS
continued vision loss in proliferative diabetic retinopathy and
severe nonproliferative diabetic retinopathy. Laser photocoag­ • Elevated urinary albumin excretion is defined as greater
ulation can also reduce the risk of retinopathy progression than or equal to 30 mg/g in a spot urine measurement;
associated with pregnancy. Intravitreal injections of antiangio­ annual measurements of serum creatinine and an esti­
genic agents, such as vascular endothelial growth factor inhib­ mated glomerular filtration rate can be utilized in con­
itors, may also be included in the management of proliferative junction with the urine albumin measurement to deter­
retinopathy and macular edema. mine the presence of diabetic nephropathy and, if
present, the stage of chronic kidney disease.
KEY POINT
• In nonpregnant normotensive patients with persistently
• Optimal blood glucose and blood pressure control can increased urine albumin excretion, an ACE inhibitor or
prevent or delay the progression of diabetic retinopathy; angiotensin receptor blocker is recommended to
however, laser photocoagulation is used to treat diabetic decrease progression of diabetic nephropathy.
retinopathy as the severity progresses.

Diabetic Nephropathy Diabetic Neuropathy


Diabetic nephropathy not only increases the risk of progres­ There are several categories of diabetic neuropathy, which may
sion to end-stage kidney disease, but is also a risk factor for present separately or in combination. Symptoms of diabetic
CVD. neuropathy depend on the nerve(s) or nerve root that is
Measurement of increased protein excretion can be per­ affected and may present as focal or diffuse disease. Achieving
formed by two methods: albumin-creatinine ratio on a ran- optimal glycemic control early in the course of diabetes can

17
Disorders of the Pituitary Gland

prevent the development of neuropathy, and sustained optimal tions, pain, ankle reflexes, and foot deformities. Patients
glucose levels can delay the progression of neuropathy. should inspect their feet daily for early detection of any abnor­
Distal symmetric polyneuropathy (DPN) is the most mality and wear appropriate footwear. Although patients with
common form of diabetic neuropathy. It is characterized by a diabetes have different footwear needs, the selection of shoes
"stocking-glove" distribution that ascends proximally. DPN should take into account several important factors: intended
frequently presents as a sensation of numbness, tingling, use, plantar protection, shape and fit on the foot, and stability
burning, heaviness, pain, or sensitivity to light touch. The issues (see MKSAP 17 Infectious Disease).
pain may worsen at night and with walking. Muscle weakness
may occur in severe cases. DPN is a risk factor for muscle and Hypoglycemic Unawareness
joint deformities, such as Charcot foot, and foot ulcers. DPN Frequent severe hypoglycemia can diminish the ability to
evaluation includes assessment of ankle reflexes, vibration detect life-threatening hypoglycemia. This unawareness is
sensation with a 128-Hz tuning fork, and touch with a 10-g caused by failure of the release of counterregulatory hormones
monofilament and pinprick. Screening intervals are found in to trigger an autonomic response to decreased glucose levels.
Table 12. Management of DPN symptoms may require one or Continual avoidance of hypoglycemia for several weeks or
more classes of drugs, including antidepressants (amitripty­ longer may help restore the body's ability to detect hypoglyce­
line, venlafaxine, duloxetine, paroxetine), anticonvulsants mia. Plasma glucose levels should be kept greater than 150 mg/
(pregabalin, gabapentin, valproate), or capsaicin cream. dL (8.3 mmol/L) during the time period when restoration of
Autonomic neuropathy can affect a single organ or multi­ hypoglycemic symptoms is the goal to avoid unintended and
ple organs. Symptoms may include gastroparesis, diarrhea, unexpected hypoglycemia. Continuous glucose monitoring
constipation, neurogenic bladder, abnormal hidrosis, and systems can be useful for hypoglycemia management by alert­
erectile dysfunction. Cardiac symptoms include resting sinus ing the patient to rapid decreases in glucose levels to allow
tachycardia, orthostatic or postprandial hypotension, exercise prompt correction and avoidance of hypoglycemia (see Self­
intolerance, and silent myocardial infarction. Cardiovascular Monitoring of Blood Glucose).
autonomic neuropathy is an independent risk factor for mor­
KEY POINT
tality, which underscores the need to reduce other cardiovas­
cular risk factors in these patients. • Distal symmetric polyneuropathy (DPN) is the most com­
Diabetic amyotrophy occurs in older patients or those mon form of diabetic neuropathy, and it presents as a sen­
with type 2 diabetes, and may be due to infarcts in the major sation of numbness or burning pain in a stocking-glove
nerve trunks of the leg. It can present acutely with severe pain distribution; management may require one or more
and asymmetric proximal weakness or pain in the leg, weight classes of drugs, including antidepressants (amitriptyline,
loss, and autonomic neuropathy. Partial remission may occur venlafaxine, duloxetine, paroxetine), anticonvulsants
over many months. Without any approved treatments for dia­ (pregabalin, gabapentin, valproate), or capsaicin cream
betic amyotrophy, management consists of symptomatic ther­
apy for neuropathic pain and ambulatory aids, if necessary.
Mononeuropathies can occur acutely with a cranial or Disorders of the
peripheral distribution. There are no specific treatments for
these mononeuropathies, as the symptoms usual resolve Pituitary Gland
within a few months. Nerve compression syndromes, such as Hypothalamic and Pituitary
carpal tunnel syndrome or peroneal palsy, occur frequently in
patients with diabetes. See MKSAP 17 Neurology for more Anatomy and Physiology
information regarding diabetic neuropathy. Referral to a neurolo­ The anterior pituitary is made up of glandular tissue that
gist for electrodiagnostic testing or evaluation for nondiabetic­ receives its blood supply from the hypothalamus through the
related etiologies should occur with severe, rapidly progressive, hypothalamic-pituitary portal plexus, whereas the posterior
or atypical neuropathies. pituitary consists of direct extension of neurons from the
hypothalamus. Both the portal blood system and the hypo­
Diabetic Foot Ulcers thalamic neurons transverse from the hypothalamus to the
Diabetic foot ulcers increase the risk for amputation and subse­ pituitary by way of the pituitary stalk. The hypothalamus
quent morbidity and disability. The etiology is often multifacto­ regulates anterior pituitary gland function by synthesizing
rial. Loss of peripheral sensation can result in significant injuries specific stimulating and inhibiting hormones, which are
that may be undetected by the patient. Peripheral arterial disease released in the portal blood. Posterior pituitary hormones are
predisposes to the development of lower extremity ischemic synthesized in the hypothalamus and travel through hypotha­
ulcers and impairs healing. Altered leukocyte function from lamic neurons to be secreted by the posterior pituitary gland.
hyperglycemia can impede wound healing of injuries. The anterior and posterior lobes are joined by the Rathke
Clinicians should evaluate the feet at least annually to pouch. Table 13 lists the pituitary hormones and initial testing
assess for pedal pulses, sensation, ulcers, skin or nail infec- for suspected pituitary hormone excess or deficiency.

18
Disorder s of the Pituitar y Gland

TABLE 13. Initial Testing for Pituitary Hormone Deficiency and Excess
Pituitary Hormone Excess
Pituitary Hormone Peripheral Hormone Initial Test(s)

ACTH Cortisol 24 hour urine free cortisol (x2) OR nocturnal salivary


cortisol (x2) OR overnight low dose dexamethasone test
ADH ADH Simultaneous serum, urine sodium, and urine osmolality

GH IGF-1 IGF-1
TSH Thyroxine, TSH, free (or total) thyroxine
triiodothyronine

Pituitary Hormone Deficiency


Pituitary Hormone Peripheral Hormone Initial Test(s) Confirmatory Test•

ACTH Cortisol Simultaneous ACTH, cortisol ACTH stimulation test

ADH ADH Simultaneous serum sodium, urine and serum osmolality Water deprivation test
LH and FSH b
Sex hormones Simultaneous LH, FSH, testosterone (male), estriol (female)
TSH Thyroxine, Simultaneous TSH, free (or total) thyroxine
triiodothyronine

ACTH = adrenocorticotropic hormone; AOH = antidiuretic hormone; FSH "" follicle -stimulating hormone; GH = growth hormone; IGF-1 = insulin-like growth factor 1;
LH = luteinizing hormone; TSH = thyroid-stimulating hormone.

asee Table 15 for additional information on confirmatory testing for pituitary dysfunction.

bRoutine testing for deficiency is not recommended without specific signs of deficiency such as amenorrhea, gynecomastia, or impotence.

The anterior pituitary gland secretes and releases six hor­ gonadotropin-releasing hormone (GnRH). LH and FSH regu­
mones: adrenocorticotropic hormone (ACTH), thyroid-stimu­ late normal male and female reproductive function. GH pro­
lating hormone (TSH), the gonadotropins-luteinizing duction is regulated by somatostatin. Prolactin controls
hormone (LH) and follicle-stimulating hormone (FSH), lactation and is inhibited by dopamine.
growth hormone (GH), and prolactin. ACTH is released in The posterior pituitary gland secretes oxytocin, which is
response to corticotrophin-releasing hormone (CRH) and acts necessary for parturition, and antidiuretic hormone (ADH,
on the adrenal glands to promote the synthesis and secretion also called vasopressin), which regulates water balance.
of cortisol. TSH is released in response to thyrotropin-releas­ The pituitary gland is posterior and superior to the sphe­
ing hormone (TRH) and acts on the thyroid to stimulate thy­ noid sinus, which provides surgical access to the gland, and is
roid hormone production. LH and FSH are differentially adjacent to the optic chiasm, the carotid arteries, and the cav­
released from the pituitary gland in response to pulses of ernous sinuses (Figure 2).

FIGURE 2. A coronal MRI (left) and sagittal MRI (right) showing the pituitary gland (open arrow), pituitary stalk (thin arrow), optic chiasm (arrowhead), sphenoid sinus
(star), and carotid artery (curved arrow).

19
Disorders of the Pituitary Gland

The pituitary gland is best imaged using MRI with gado­ Pituitary tumors are almost always nonmalignant. Two
linium. Because the normal pituitaty is relatively small, a exceptions are metastatic disease and the very rare pituita1y
dedicated pituitary protocol that obtains thin MRI slices carcinoma. Additional kinds of noncancerous pituitary lesions
through the sella is used. include craniopharyngiomas, meningiomas, and Rathke cleft
cysts. Inflammatory and infiltrative disorders, including lym­
phocytic hypophysitis, sarcoidosis, hemochromatosis, amyloi­
Pituitary Tumors dosis, L angerhans cell histiocytosis, lymphoma, and
Pituitaiy adenomas, which are benign, are the most common tuberculosis, can affect the pituitary gland.
tumor of the pituitary gland. A tumor less than 1 cm is defined Lymphocytic hypophysitis is an inflammatmy pituitary
as a microadenoma, and a tumor 1 cm or larger is termed a lymphocytic infiltration that most commonly occurs in preg­
macroadenoma (Figure 3). Pituitary adenomas are common. nant and postpartum women. It may cause transient or per­
Autopsy studies document that 10% of the general population manent pituitary insufficiency. Lymphocytic hypophysitis is
had undiagnosed pituitary adenomas. Frequently, pituitary treated with glucocorticoids.
adenomas are incidental findings on imaging studies completed A sellar mass can compress normal surrounding tissue
for other reasons. When patients undergo brain MRI, 10% to and impair normal neurologic and pituitaty function. Pituitary
38% are found to have incidental pituita1y microadenomas and adenomas may also be functional and secrete excess hormone.
0.2% have incidental pituitaiy macroadenomas. Certain genetic
mutations increase the chance of developing a pituitary tumor. Incidentally Noted Pituitary Masses
When J pituitary tumor is incidentally noted. investigation CJ
Approach to a Sellar Mass must determine (!) whether it is causing a mass effect. (2)
When a sellar mass is noted, pituitary adenomas are most \\'hether it is secreting excess hormones. and (.3) whether it has
likely; however, they need to be distinguished from other a propensity to grow and cause problems in the ruture. After a
pituita1y lesions and nonpathologic pituitary enlargement. thorough history and physical cxa111ination. bioche111ical test­
The pituitary gland is enlarged diffusely in untreated pri­ ing can be undertaken in a targeted fashion based on the
mary hypothyroidism and during pregnancy. When possible, 1x11icn1·s clinical signs and symptoms. Initial tests could include
imaging of the pituitary gland should be avoided or delayed in me;1surement or 8 .\,! cortisol. TSH. f'ree (or tot,11) thyroxine
pregnancy and in untreated primary hypothyroidism because Cl). prolactin. and insulin like growth factor J (IGF \).
gland enlargement on imaging may prompt an expensive and II the tu111or is not causing mass effect and there is no
unnecessa1y evaluation for pituita1y hormone abnormality C\'idence of' hor111onc excess. a pituitary MRI should be
and tumor. rcpc�1tecl in 6 111onths frir a macroadenoma ,rnd 12 months for

FIGURE 3. A coronal MRI (left) and sagittal MRI (right) showing a large pituitary macroadenoma. The normal pituitary gland and the optic chiasm cannot be seen because
of compression from the tumor. The tumor is invasive into the left cavernous sinus. Likely, the tumor appears heterogeneous because of internal necrosis.

20
Disorders of the Pituitary Gland

CJ a microadenoma to assess for growth. If no growth occurs. tumor may cause minimal peripheral vision loss, bilemporal
MR ls should be repeated every I to 2 years for the next 3 years hemianopsia. or complete blindness. Visual field testing is a
CONT.
and then intermittently thereafter. In a patient at risk for can- sensitive measure of optic nerve damage and should be evalu­
cer or with a histo1y of cancer. metastatic disease must be ated by an ophthalmologist in patients who report a change in
excluded. Cl vision. who have a pituitary t·umor that abuts or compresses
the chiasrn on MRI. or who have any evidence of gross periph­
Empty Sella eral vision loss on physical examination. Change in vision due
Empty sella is diagnosed when the normal pituitary gland is lo optic chiasm compression is an indication for treating a
not visualized or is excessively small on MRI; it is a radiologic pituitary tumor.
finding and not a distinct clinical condition. The pituitary sella Pituitary tumors can also invade surrounding brain tissue
is said to be "empty" because normal tissue is not seen. The leading to seizures and neurologic manifestations. Pituitary
finding may be primarily due to increased cerebrospinal fluid tumors can invade the cavernous sinus. causing damage to
entering and enlarging the sella, or it may be secondary to a cranial nerves Ill, IV. and VJ that pass through the sinus caus­
tumor, previous pituitary surgery, radiation, or infarction. ing diplopia and extraocular muscle palsies/paralysis. Cl
Empty sella can also occur as a congenital abnormality when
KEY POINT
the sella is normal size, but the pituitary is small. When empty
sella is found incidentally on imaging, an evaluation should be • Pituitary masses can compress the normal pituitary
completed to determine if there is a known cause for second­ gland, causing hormone deficiencies; a large pituitary
ary empty sella and if the patient has signs or symptoms of mass may cause panhypopituitarism in which there is
pituitary hormone deficiency. A patient without signs or impaired secretion of all pituitary hormones.
symptoms should be screened for cortisol deficiency and
hypothyroidism with 8 AM cortisol, TSH, and free (or total) T4. Treatment of Clinically Nonfunctioning
A patient with signs of pituitary hormone deficiency should Pituitary Tumors
receive a more complete biochemical evaluation of the pitui­ Nonfunctioning pituitary tumors that are growing or causing
tary axes, based on the signs and symptoms found. mass effect are treated with neurosurgery. The most common
Repeat imaging is not necessary unless indicated as sur­ surgical approach is transsphenoidal through the nares or the
veillance for the underlying pathology that resulted in the mouth. A very large or invasive tumor may require craniotomy
empty sella. for decompression. Indications for surgery include mass effect,
particularly a visual field defect; tumor that abuts the optic
KEY POINTS
chiasm; tumor growth; or an invasive tumor (invading the
• Incidentally noted pituitary tumors are common, and brain or cavernous sinus). Surgery should also be considered
biochemical testing is informed by findings on history in a patient with a tumor close to the optic chiasm who plans
and physical examination. to become pregnant (due to the physiologic enlargement of the
• Initial tests for pituitary incidentally noted masses pituitary associated with pregnancy).
include measurement of 8 AM cortisol, thyroid-stimulat­ Functional pituitary tumors will be discussed later in this
ing hormone, free (or total) thyroxine (T4), prolactin, chapter, based on the hormone in excess (see Pituitary
and insulin-like growth factor 1. Hormone Excess).
• Empty sella is diagnosed when the normal pituitary KEY POINT
gland is not visualized or is excessively small on MRI;
• Nonfunctioning pituitary tumors that are growing or
it is a radiologic finding and not a distinct clinical
causing mass effect are treated with neurosurgery.
condition.

Mass Effects of Pituitary Tumors Hypopituitarism


Pituitary tumors may cause headaches in some but nol all Hypopituitarism is caused by one or more pituitary hormone
patients: the size of the tumor does not always correlate with deficiencies, usually resulting from damage to the normal
Lhe presence and severity of headache. pituitary gland by a tumor. Hypopituitarism can also occur as
Pituitary masses can compress the normal pituitary a complication from surgery if the normal gland or the pitui­
gland. causing hormone deficiencies. A large pituitary mass tary stalk is damaged during tumor resection or radiation
may cause panhypopituitarism in which there is impaired therapy. Additional causes of hypopituitarism are listed in
secretion of all pituitary hormones.
CJ
Table 14.
Because the optic chiasm is located superior to the pitui­ Pituitary apoplexy is acute hemorrhage into the pituitary
tary gland, a large pituitary mass may compress the optic gland often al Lhe site of a preexisting pituitary adenoma
chiasm resulting in vision changes. Depending on the size of (typically a macroadenoma). Pituitary apoplexy can cause
the tumor and severity of optic nerve damage. a pituitary acute pituitary hormone deficiency or mass effect from rapid

21
Disorders of the Pituitary Gland

TABLE 14. Causes of Hypopituitarism Adrenocorticotropic Hormone Deficiency


(Secondary Cortisol Deficiency)
Pituitary adenoma
Although secondary cortisol deficiency may result from dam­
Pituitary surgery
age to the pituitary gland or pituitary stalk that impairs ACTH
Pituitary radiation production, it is most commonly iatrogenic due to exogenous
Pituitary apoplexy glucocorticoid use that suppresses pituitary ACTH secretion.
Pituitary infarction Patients with secondary cortisol deficiency have only gluco­
Craniopharyngioma
corticoid deficiency. The remainder of the adrenal gland func­
tions normally and the renin-angiotensin system is intact, so
Metastatic tumor
these patients do not have mineralocorticoid deficiency (see
Meningioma
Disorders of the Adrenal Glands for a discussion of primary
Lymphocytic hypophysitis adrenal failure). Although patients with secondary cortisol
Sarcoidosis deficiency do require stress-close glucocorticoicls, they are at
Langerhans cell histiocytosis less risk for hypotension. hyponatremia. and adrenal crisis
Lymphoma
than those with primary cortisol deficiency (failure of the
adrenal glands) because the production of'mineralocorticoid is
Hemochromatosis
retained. Also. unlike patients with primary cortisol defi­
Congenital deficiencies
ciency. patients with secondary cortisol deficiency do not
Hypothalamic disease develop hyperpigrnentation or bronzing of the skin because
ACTH and its prohormone responsible for these changes. pro­
opiomelanocortin (POMC), are not hypersecretecl.
expansion of the sellar contents due to bleeding. It is an endo­
Oral. injectable (including joint injections). and even top­
crine and neurosurgical emergency. Acute ACTH deficiency is
CONT. ical glucocorticoids are able to suppress ACTH secretion.
common and can be life-threatening. If suspected, stress-dose
Glucocorticoids prescribed at doses above physiologic replace­
glucocorticoid replacement should be initiated emergently.
ment for longer than 3 weeks should be tapered when discon­
Patients with vision changes or loss associated with apoplexy
tinued to allow recovery of the pituitary-adrenal axis: if ther­
require urgent surgical decompression. Cl
apy has lasted less than 3 weeks . no taper is required for
Hypopituitarism can occur due to postpartum pituitary
pituitary-adrenal axis recovery.
infarction (Sheehan syndrome) because of excessive postpar­
When tapering glucocorticoicls, the patient can be tran­
tum hemorrhage causing hypotension and hypoperfusion.
sitioned to a hydrocortisone dose that is JO% to 20% lower
Patients who may have Sheehan syndrome should be emer­
than the equivalent, current glucocorticoid dose. The dose
gently tested and treated for secondary cortisol deficiency. A
can then be decreased by 2.5 to S mg of hydrocortisone every
patient with Sheehan syndrome will not lactate because of
1 to 2 weeks. When tapering with prednisone. taper large
prolactin deficiency; no treatment is available to induce lacta­
closes by 25% to SO% weekly until the patient is on a dose of
tion. Other hormone deficiencies can be evaluated 6 weeks
after delivery.
s mg daily and then taper by l mg every 1 to 2 weeks. It is dif­
ficult to taper dexamethasone clue to the limited mg tablets
GH and gonadotropin deficiencies often occur early when
available. The taper can be slower if symptoms such as light­
the pituitary gland is damaged by tumor, radiation, surgery, or
headedness persist.
hemorrhage because the cell lines that synthesize GH (soma­
After prolonged glucoco11icoicl use, recovery of the pituitary­
totrophs) and LH and FSH (gonadotrophs) are most sensitive
adrenal axis should be tested prior to discontinuing glucocor­
to injury. Secondary hypothyroidism (TSH deficiency) and
ticoid replacement. Specifically. morning serum cortisol
secondary cortisol deficiency (ACTH deficiency) often occur
should normalize to greater than 11 µg/dL (303.6 nmol/L)
later in the disease process. Pituitary adenomas can cause
when glucocorticoids are withheld for 36 to 48 hours follow­
elevation in prolactin due to stalk compression, leading to a
ing the taper. Even after endogenous ACTH production has
decrease in dopaminergic inhibition of prolactin secretion.
returned. patients may require more time to mount an ade­
KEY POINTS quate ACTH response to stress. After discontinuing the gluco­
• Pituitary tumors and surgery for pituitary tumors are corticoid taper . the patient can undergo an ACTH stimulation
the most common causes of hypopituitarism. test (Table IS) to document adequate glucoco11icoicl response
to stress. The diagnosis relies on demonstrating a low basal
• Stress-dose glucocorticoid replacement should be initi­
serum cortisol level that does not increase appropriately after
ated emergently in patients with pituitary apoplexy or
stimulation with the ACTH analogue cosyntropin. This is done
infarction, as well as emergent neurosurgical interven­
tion; patients with vision loss associated with apoplexy by measuring early morning (8 AM) serum cortisol. A serum
require urgent surgical decompression. cortisol level less than 3 µg/clL (82.8 nmol/L) is consistent with
cortisol deficiency. A normal response is a peak serum cortisol

22
Disorders of the Pituitary Gland

TABLE 15. Dyn�micTesting for Pituitary Dysfunction


Indication Test Technique Interpretation

ACTH (cortisol) ACTH stimulation Measure baseline serum cortisol level. Serum cortisol level >18 µg/dl(496.8 nmol/L)
deficiency test Administer 250 µg of synthetic ACTH. indicates a normal response.
Measure cortisol levels at 30 and 60 minutes.
ADH deficiency Water deprivation Patient empties bladder, and baseline Water deprivation test interpretation:
(DI) test, followed by weight is measured. Measure urine volume
desmopressin and osmolality hourly. Measure serum Urine osmolality >600 mOsm/kg H 2 0 is a
challenge if sodium, osmolality, and weight every normal response to water deprivation,
indicated 2 hours. indicating ADH production and peripheral
effect are intact.
The test is stopped when one of the Urine osmolality <600 mOsm/kg H 2 0, serum
following occurs: osmolality >295 mOsm/kg H 2 0 and/or serum
sodium >145 mEq/L(145 mmol/L) are
Urine osmolality exceeds 600 mOsm/kg diagnostic of DI.
H 20
Patient has lost 5% of body weight
Desmopressin challenge interpretation:
Urine osmolality is stable for 2-3 h while
serum osmolality rises >100% increase in urine osmolality is
diagnostic of complete central DI.
- Plasma osmolality >295 mOsm/kg H 20
0% increase in urine osmolality is diagnostic of
- Serum sodium >145 mEq/L(145 mmol/L) complete nephrogenic DI.
Desmopressin challenge if final urine >50% increase in urine osmolality is
osmolality <600 mOsm/kg H 2 0, serum diagnostic of partial central DI.
osmolality >295 mOsm/kg H 2 0, or serum
sodium >145 mEq/L(145 mmol/L): <50% increase in urine osmolality is
diagnostic of partial nephrogenic DI.
Give desmopressin 1 µg subcutaneously.
Measure urine osmolality every 30 minutes
for 2 hours.
Growth Glucose 75 g oral glucose tolerance test. Measure GH <0.2 ng/ml(0.2 µg/L)is a normal response.
hormone excess tolerance test glucose and GH at 0, 30, 60, 90, 120, and GH 2'1.0 ng/ml(1.0 µg/L)(or 2'0.3 ng/ml
(acromegaly) 150 minutes. [0.3 µg/L] on an ultrasensitive assay) is
diagnostic of acromegaly.
ACTH = adrenocorticotropic hormone; ADH = antidiuretic hormone; DI = diabetes insipidus; GH = growth hormone.

Cl greater than 20 µg/dL (552 nrnol/L). When the test result is physiologic stress (major surgery, trauma. critical illness, or
normal, patients no longer require daily cortisol replacement. childbirth). hydrocortisone (150-200 mg/d intravenously in
CONT.
but should follow "sick day rules" (increasing cortisol replace- 3-4 divided doses; 100 mg/d the next day; taper to baseline in
ment dose during illness) for up to a year after cessation of 3-5 days) may be used. An alternative would be dexametha­
daily cortisol replacement. sone (6-8 mg/cl intravenously in 2-3 divided closes). If the
Symptoms of secondary cortisol deficiency include patient has pituitary apoplexy and urgent/emergent neuro­
weight loss, nausea. vomiting. lightheadedness, hypoglyce­ surgery is planned with no time for ACTH-stimulation test­
mia. hypotension, and hyponatremia. Secondary cortisol defi­ ing. the patient should empirically be treated with glucocor­
ciency is also diagnosed using an ACTH stimulation test. ticoicls and then receive an ACTH stimulation test 4 to 8 weeks
Secondary cortisol deficiency can be life threatening and must after surgery. Cl
be treated with glucocorticoid replacement. often with hydro­
cortisone. although prednisone or dexamethasone may also be Thyroid-Stimulating Hormone Deficiency
used. Hydrocortisone (15-30 mg/d) should be administered in Thyroid-stimulating hormone (TSH) deficiency leads to sec­
2 to 3 cliviclecl doses. or hydrocortisone should be dosed 10 to ondary or central hypothyroidism. Secondary hypothyroidism
20 mg in the morning and 5 to 10 mg in the early afternoon. is clinically identical to primary hypothyroidism (see Disorders
Patients require stress doses of glucocorticoids when of the Thyroid Gland).
acutely ill. hospitalized, or undergoing the stress of' surgery. Secondary hypothyroidism is diagnosed by demonstrating
For moderate physiologic stress (minor or moderate surgery a simultaneously inappropriately normal or low TSH and low
with general anesthesia), hydrocortisone should used (45- T4 (free or total). Patients are treated with levothyroxine
75 mg/d orally or intravenously in 3-4 divided doses for replacement in the same manner as primary hypothyroidism;
2-3 days). Prednisone (10-20 mg or dexamethasone 2-3 mg/d however, the serum TSH cannot be used to monitor and assess
in 1-2 divided doses) may be used alternatively. For major for adequacy of thyroid hormone replacement dosing. Instead,

23
Disorders of the Pituitary Gland

the levothyroxine dose is adjusted based on free T4 levels with isolated adult-onset GH deficiency is extremely rare, and its
the goal of obtaining a value within the normal reference range. clinical significance in adults is debated. Therefore, evaluation
for GH deficiency is recommended in patients with at least one
KEY POINTS
known pituitary hormone deficiency. Unfortunately, GH ther­
• Patients with secondary cortisol deficiency have iso­ apy has been used inappropriately as an alternative medica­
lated glucocorticoid deficiency without mineralocorti­ tion. GH naturally declines with age and does not require
coid deficiency; in addition, they do not develop hyper­ replacement. The use of GH does not promote longevity and
pigmentation or bronzing of the skin because when used inappropriately can be harmful. Specifically, GH
adrenocorticotropic hormone and pro-opiomelanocortin therapy can encourage cancer growth, worsening the disease
are not hypersecreted. in a patient with cancer, or promoting growth of an occult,
• Secondary or central hypothyroidism is diagnosed by undiagnosed cancer.
demonstrating a simultaneously inappropriately normal Because GH secretion is pulsatile, testing random levels is
or low thyroid-stimulating hormone and low thyroxine not diagnostically useful. Therefore, GH deficiency is diag­
(T) (free or total) level. nosed by measurement of IGF-1. A GH deficiency is confirmed
by measuring the response of serum GH on a stimulatory test,
such as the insulin tolerance test. An insulin tolerance test car­
Gonadotropin Deficiency
ries a high risk of severe hypoglycemia, so referral to an endo­
The pituitary gland normally secretes LH and FSH in response
crinologist for testing is appropriate.
to GnRH from the hypothalamus. LH and FSH stimulate the
A decision regarding replacement therapy should be
secretion of normal male and female sex hormones; LH and
made based on that patient's symptoms, goals, and risks in
FSH deficiency causes hypogonadotropic hypogonadism (see
consultation with the patient's endocrinologist. When clini­
Reproductive Disorders).
cally indicated, GH deficiency is treated with daily subcutane­
Hypogonadotropic hypogonadism may be caused by
ous GH injections. In an otherwise healthy adult, treatment of
GnRH deficiency. The most common cause of GnRH defi­
GH deficiency can improve quality of life and increase the
ciency in women is hypothalamic amenorrhea, which is
percentage of lean muscle mass. Also, it can reduce the risk of
associated with excess exercise, illness, or anorexia.
osteoporosis. However, the risks and benefits of therapy must
Additional causes of GnRH deficiency include congenital
be carefully considered. Replacement of GH is cost prohibitive
GnRH deficiency and Kallmann syndrome, a condition in
for some patients. It is contraindicated in patients with cancer
which hypothalamic neurons responsible for releasing GnRH
and should not be used in patients with an untreated pituitary
fail to migrate into the hypothalamus during embryonic
tumor due to potential stimulation of tumor growth.
development.
Treatment of hypogonadotropic hypogonadism depends KEY POINT
on the goals of therapy and whether the patient desires fertil­ • Isolated adult-onset growth hormone deficiency is HVC
ity. Fertility treatment requires replacement of the gonadotro­ extremely rare, and its clinical significance is debated;
pins in men and women. Premenopausal women who do not evaluation for growth hormone deficiency should be
desire fertility may be treated with estrogen- and progester­ reserved for adults with at least one known pituitary
one-containing oral contraceptives (after assessment of risk of hormone deficiency.
thromboembolic disease). Treatment of premenopausal hypo­
gonadotropic hypogonadism is recommended to avoid loss of Central Diabetes lnsipidus
estrogen-dependent bone at a young age, which could lead to Central diabetes insipidus (DI) results from inadequate pro­ C]
osteoporosis. Treatment of postmenopausal hypogonadotropic duction of antidiuretic hormone (ADI-!) by the posterior pitui-
hypogonadism is not indicated. Men who do not desire fertility tary gland. In the presence of ADI-!. aquaporin water channels
may be treated with testosterone replacement therapy (see are inserted in the collecting tubules and allow water to be
Reproductive Disorders). reabsorbed. In the absence of ADI-!, excessive water is excreted
by the kidneys. Excretion of more than 3 liters of urine per day
Growth Hormone Deficiency is considered polyul"ic.
Growth hormone (GH) is vital for normal linear growth, and The severity of DI varies with the completeness of the
deficiency prior to puberty will lead to short stature. At puberty, deficiency. Patients describe mild to extreme polyuria and cor­
the epiphyses close, halting linear growth. In adulthood, GH responding thirst: partial DI is common.
production is necessary for normal physiology but is not as Frank hypernatremia is unusual because patients develop
important for growth as during childhood. In adults, GH defi­ extreme thirst and polydipsia, and with free access to water.
ciency causes fatigue, loss of muscle mass, an increased ratio of can maintain serum sodium in the high normal range. When
fatty tissue to lean tissue, and increased risk for osteoporosis. patients do not drink enough to replace the water lost in the
GH deficiency is often the first hormone deficiency to urine, due to poor or absent thirst drive or lack of free access
occur when a patient is developing pituitary insufficiency, but to water. they develop hypernatremia.

24
Disorders of the Pituitary Gland

Cl taneous
CONT.
In the patient with polyuria. DI is diagnosed with simul­
laboratory evidence of' inability lo concentrate urine
Pituitary Hormone Excess
Pituitary tumors are called functional when they secrete
in the face or elevated serum sodium and osmolality. with
excessive amounts of hormone. The most common functional
inappropriately low urine osmolality. If' necessary. a water
pituitary tumor is a prolactinoma. GH and ACTH overproduc­
deprivation test can confirm the diagnosis (see Table 15).
tion by pituitary tumors is important to recognize because the
Patients with mild partial DI with an adequate thirst drive
clinical consequences of oversecretion are potentially severe.
and access to water may choose to compensate wilhout hor­
TSH-secreting tumors cause hyperthyroidism but are
mone replacement therapy, but highly symptomatic polyuria
extremely rare.
and nocturia that interferes with restful sleep and daily func­
Occasionally, a pituitary tumor can oversecrete more than
tion necessitate treatment. In those requiring treatment, hor­
one hormone, most commonly GH and prolactin, or less com­
mone replacement is with desmopressin (J-clesamino 8-D­
monly, TSH and GH or prolactin.
arginine vasopressin, or c!DAVP) either intranasally,
subcutaneously, or orally. Desmopressin is not absorbed well in
Hyperprolactinemia and Prolactinoma
the gastrointestinal tract, so oral doses are much higher than
intranasal or subcutaneous doses. Most patients with DI require Causes
either evening dosing to aid in sleep or twice daily closing or Prolactinomas are pituitary tumors that secrete excessive
desmopressin. If ADH is overreplacecl, patients will develop amounts of prolactin; however, they are not the only cause of
water intoxication. volume overload, and hyponatremia. Cl hyperprolactinemia (Table 16).
The most common cause of hyperprolactinemia is phys­
KEY POINTS iologic; prolactin is released during pregnancy and postpar­
• In the patient with polyuria, diabetes insipidus is diag­ tum to cause lactation. Nipple stimulation such as during sex
nosed by clinical symptoms with simultaneous labora­ can cause mild hyperprolactinemia (serum prolactin <40 ng/
tory evidence of inability to concentrate urine mL [40 µg/L]). Physiologic stress, coitus, and exercise can
with elevated semm sodium and osmolality, and inap­ also increase prolactin levels up to 40 ng/mL (40 µg/L).
propriately low urine osmolality; a water deprivation Nipple piercing can raise prolactin levels above 200 ng/mL
test can confirm the diagnosis. (200 µg/L). Clinical breast examination should not raise pro­
• Treatment of central diabetes insipidus is once or twice lactin levels above the reference range, unless evaluation for
daily hormone replacement with desmopressin. milk production is performed, but if desired, palpation of the
breast can be deferred until after a serum prolactin level is

Cl Panhypopituitarism
Panhypopituitarism occurs when patients lack all anterior and
measured.
Medications are a common cause of hyperprolactinemia
(see Table 16). Antipsychotic agents cause hyperprolactinemia
posterior pituitary hormone production. Panhypopituitarism
due to their antidopaminergic effect that interrupts the inhibi­
may be caused by a large or aggressive pituitary tumor or as a
tion of prolactin by dopamine. Specific agents, such as risperi­
complication of surgery. lf'the pituitary stalk is transected dur­
done or metoclopramide, may raise the prolactin level above
ing surgery or as the result of trauma. panhypopituitarism will
200 ng/mL (200 µg/L). Evaluation for pituitary hypersecretion
result acutely. Patients with panhypopituitarism require lite­
when a patient is taking a medication known to raise the pro­
long replacement ofT 1 , cortisol, and ADH because these defi­
lactin level is difficult. When the prolactin level is only mildly
ciencies can be life-threatening. GH and sex hormones are
elevated (<50 ng/mL [50 µg/L]), it may be reasonable to assume
replaced dependent on each patient's preference. coupled
that hyperprolactinemia is a medication side effect. When
with a discussion of the risks and benefits of therapy. In addi­
significantly elevated (>100 ng/mL [100 µg/L]), either the
tion to requiring exogenous gonaclotropins to conceive. a
reproductive-aged woman with panhypopituitarisrn will not
go into spontaneous labor and will not lactate. These pregnan­ TABLE 16. Causes of Hyperprolactinemia
cies are classified as high risk. and obstetric care should be Physiologic Medications Other
provided by a maternal-fetal specialist.
Patients with panhypopituitarism should wear medical Pregnancy Antipsychotic agents• Prolactinoma

alert identification documenting their panhypopituitarisrn. Lactation Metoclopramide Pituitary tumor-stalk


compression
specifically noting the need for stress-dose glucocorticoid Nipple Cimetidine
therapy and desmopressin dosing in emergent situations. Cl stimulation Hypothyroidism
Verapamil
Cirrhosis
KEY POINT Methyl dopa
Chronic kidney
• Patients with panhypopituitarism require lifelong replace­ Opiates
disease
ment of thyroxine (T4), cortisol, and antidiuretic hormone Cocaine
because these deficiencies can be life-threatening. "Including risperidone. olanzapine, haloperidol, chlorpromazine, and clomipramine.

25
Disorders of the Pituitary Gland

medication needs to be withheld to further assess or a pitui­ ceptive pills (if fertility is not desired) or dopamine agonists.
tary MRI obtained to evaluate for prolactinoma. Caution is Postmenopausal women with microadenomas do not require
warranted when discontinuation of an antipsychotic agent is treatment. Patients with hypogonadism from medication­
being considered, and consultation with a psychiatrist is rec­ induced hyperprolactinemia may be treated with hormone
ommended prior to discontinuation. replacement.
Another common cause of hyperprolactinemia is primary Unlike other pituitary tumors, medication rather than
hypothyroidism. Hypothyroidism can cause diffuse swelling of surgery is first-line therapy for prolactinomas. Even patients
the pituitary gland that may resemble enlargement due to a with severe mass effect such as vision loss are treated with
pituitary adenoma on imaging. Therefore, a patient with pri­ medical therapy initially. Rarely, very large tumors or more
mary hypothyroidism and hyperprolactinemia should be invasive prolactinomas do not shrink with medical therapy
treated with thyroid hormone replacement with retesting of and, also rarely, continue to grow. In these patients, surgery
the prolactin level once the TSH has normalized. Further evalu­ should be considered, followed by radiotherapy if growth
ation is indicated if the hyperprolactinemia does not correct recurs or continues. After being debulked, the prolactinoma
when hypothyroidism is treated. If pituitary imaging has noted may respond better to medical therapy.
pituitary enlargement prior to treatment of hypothyroidism, Prolactinomas are treated with dopamine agonists (DA).
repeat MRI should be obtained when the TSH is normal. The two FDA-approved dopamine agonists are bromocriptine
Nonfunctioning pituitary adenomas can also cause and cabergoline. Dopamine agonists typically decrease the size
hyperprolactinemia by compressing the pituitary stalk and and hormone production of prolactinomas rapidly. Response
decreasing dopamine inhibition of prolactin secretion. It is to therapy can be monitored by checking serum prolactin
important to distinguish between prolactinomas and non­ levels 1 month after initiating therapy and then every 3 to
functioning pituitary adenomas as the cause of hyperpro­ 4 months. Decreasing serum prolactin usually correlates with
lactinemia because of different treatment approaches. decreasing size of the tumor. MRI should be repeated in 1 year
for microprolactinomas if the prolactin level normalizes on
Clinical Features and Diagnosis dopamine agonists. After tumor shrinkage is confirmed, addi­
Physiologically, prolactin induces and regulates lactation. tional MR!s are not necessary unless the serum prolactin level
Hence, elevated levels of prolactin cause galactorrhea. Women rises. An MRI should be repeated after 3 months of medical
are more likely to develop galactorrhea than men. therapy for macroprolactinomas, or if prolactin levels are ris­
Hyperprolactinemia also causes hypogonadotropic hypo­ ing on therapy with good medication adherence. MRI should
gonadism because of negative feedback on GnRH, LH, and FSH be repeated every 6 to 12 months until the macroprolactinoma
by high levels of prolactin. Both men and women present with is stable on serial studies and the prolactin level is not rising.
hypogonadism. Women of reproductive age often present ear­ Bromocriptine is dosed 1 to 3 times daily, so adherence
lier than men because of amenorrhea. They may also have early can be challenging. When initiated, it is associated with
menopausal symptoms. Symptoms in men are insidious and orthostasis and lightheadedness, and patients can have dizzi­
may go unrecognized for years. Both men and women with ness, nausea, and headache during treatment. Cabergoline is
hyperprolactinemia are likely to be infertile and are at risk for much better tolerated and more effective at normalizing prol­
osteoporosis. Postmenopausal women are already hypogonadal actin and tumor shrinkage, so it is typically the initial therapy
because of ovarian failure; therefore, hyperprolactinemia may chosen. It is dosed once or twice a week, but typically costs
have minimal clinical implications in this population. However, more than bromocriptine.
the cause of postmenopausal hyperprolactinemia still requires Therapy may be tapered after the prolactin level has been
diagnosis because it may be due to a pituitary tumor. normal for 2 years, and there is no longer a visible tumor on
Diagnostic imaging is indicated in situations in which pituitary MRI. After discontinuing the dopamine agonist, pro­
there is unexplained hyperprolactinemia. lactin levels should be followed once a month for 3 months,
The degree of hyperprolactinemia is useful in differenti­ then every 3 months for the first year, and then annually
ating prolactinomas from nonfunctioning macroadenomas. In thereafter; a pituitary MRI should be repeated if the prolactin
general, large nonfunctioning tumors cause mild serum pro­ level rises above normal.
lactin elevations (<100 ng/mL [100 µg/L]} from stalk compres­
sion. Macroprolactinomas raise serum prolactin levels to Prolactinomas and Pregnancy
greater than 250 ng/mL (250 µg/L). Very large macroprolacti­ Hyperprolactinemia is a frequent cause of infertility because
nomas may raise prolactin levels greater than 10,000 ng/mL of the effect on gonadotropin release. DA therapy lowers pro­
(10,000 µg/L). lactin, normalizing gonadotropin regulation and allowing
normal ovulation. DA therapy should be discontinued when
Therapy the pregnancy is diagnosed. The pituitary increases in size
Patients with microprolactinomas without symptoms of during normal pregnancy, and prolactinomas can increase in
hypogonadism do not require treatment. Symptomatic women size as well. The risk for significant tumor expansion is negli­
with microadenomas may be treated with either oral contra- gible in patients with microprolactinomas.

26
Disorders of the Pituitary Gland

Women with macroprolactinomas are at risk for clinically Clinical Features and Diagnosis in the Adult
significant tumor growth or vision compromise during preg­ Patient with Acromegaly
nancy. If the tumor is very large or abuts the optic chiasm, Patients have changes in facial structure such as a prominent
patients should be counseled on risk of tumor growth during brow and jawline, an enlarged skull, a large nose, facial edema,
pregnancy, as well as the risks and benefits of surgical resec­ excessive spacing between teeth, and macroglossia. The hands
tion of the tumor before pregnancy. DA therapy is sometimes and feet may be disproportionately large. Other manifestations
continued during pregnancy if the patient has a history of may include arthritis, skin tags, diabetes mellitus, hyperten­
visual field defect. sion, colon polyps, thickened skin, and excessive perspiration.
Pregnant women with macroprolactinomas should be Acromegaly can cause severe obstructive sleep apnea because
assessed clinically at least once per trimester and have visual of soft-tissue swelling and macroglossia. Additionally, it can
fields tested every trimester or more frequently for vision result in heart disease, including left ventricular hypertrophy,
change. Changes in visual fields or severe headache are indica­ cardiomyopathy, valvular heart disease, arrhythmia and dias­
tions to proceed with pituitary MRI. If the macroprolactinoma tolic hea1i failure. Increased rates of cancer are observed in
causes mass effect during pregnancy, bromocriptine may be acromegaly, including colon, esophageal, and gastric adeno­
started. If the bromocriptine does not decrease tumor size and carcinomas; thyroid cancer; and melanoma. Acromegaly
reduce symptoms of mass effect, surgical debulking may be increases mortality, likely due to cardiovascular disease, diabe­
necessary. tes, sleep apnea, and cancer. Age-appropriate testing for these
Normal pregnancy causes hyperprolactinemia, so hyper­ conditions should occur for the lifetime of the patient with
prolactinemia from prolactinoma does not require treatment acromegaly.
during pregnancy. Prolactin levels should not be measured Acromegaly is diagnosed biochemically. Because GH is
during pregnancy. Postpartum, prolactin levels return to pulsatile throughout the day, it is not useful for diagnosis, so
normal within a few months, and lactation becomes non­ measurement of serum IGF-1 is used instead. Excess GH is
prolactin mediated. confirmed with an oral glucose tolerance test (see Table 15)
because glucose normally suppresses GH levels to less than
KEY POINTS
1 ng/mL (1 µg/L). GH levels greater than 1 ng/mL (1 µg/L) are
• Prolactinomas, pregnancy and lactation, or medications diagnostic of GH excess. A pituitary MRI should be obtained
such as antipsychotic agents are frequent causes of once GH excess is confirmed biochemically. Consultation with
hy perprolactinemia. an endocrinologist is recommended if IGF-1 is elevated.
• A patient with primary hypothyroidism and hyper­
prolactinemia should be treated with thyroid hor­ Treatment
mone replacement with retesting of the prolactin Treatment of acromegaly is transsphenoidal tumor resection;
level once the thyroid-stimulating hormone level has surgery is the only treatment that is potentially curative. In
normalized. many instances, cure with surgery is not possible and addi­
tional therapy is necessary to treat the residual GH excess and
• Dopamine agonists (bromocriptine and cabergoline) are
tumor.
first-line therapy for symptomatic patients with hyper­
Remission is achieved when IGF-1 levels are within the
prolactinemia and prolactinomas.
normal reference range for age and the response of GH to a
glucose tolerance test is normal. Patients not achieving
Acromegaly remission require medication to decrease GH levels and the
Acromegaly is a rare diagnosis that is often missed for years long-term effects of GH excess. The initial therapy of choice
because of the insidious onset and rare presentation in pri­ is injectable somatostatin analogues to inhibit GH secretion.
mary care; however, it has very serious implications for a If a patient fails to benefit from somatostatin analogue treat­
patient's health and longevity and must be diagnosed and ment, high-dose dopamine agonist therapy is marginally
treated in as timely a manner as possible. effective when the tumor co-secretes prolactin. If IGF-1
remains elevated, pegvisomant, a GH receptor blocker, is
Causes used. Pegvisomant effectively lowers IGF-1 levels, but
Acromegaly is the clinical syndrome that occurs when a pitui­ patients on pegvisomant have risk of tumor growth because
tary tumor secretes excessive amounts of GH in an adult the medication works in the peripheral tissues as an antag­
patient. Prior to puberty, patients with a GH-secreting tumor onist to GH and does not decrease GH production by
develop excessive longitudinal growth and gigantism, a term the tumor.
used to indicate excessive growth and height above normal for Stereotactic radiosurgery (gamma knife) may be offered
age. Because epiphyseal growth plates require sex hormones to to increase the chance of remission or cure. External beam
close, patients with large pituitary tumors causing hypog­ radiation carries a high risk of causing pituitary insuffi­
onadism will not have closure of their growth plates and will ciency, but the risk is decreased when stereotactic radiosur­
continue growth into adulthood. gery is used.

27
Disorders of the Pituitary Gland

When acromegaly is in remission, MRI and hormone test­ Cushing syndrome refers to hypercortisolism rrom any cause.
ing should be completed annually. When the pituitary tumor exogenous or endogenous. ACTH-dependent or not . The mosl
is stable but the IGF-1 level is elevated, MRI should be repeated common cause or endogenous Cushing syndrome is Cushing
annually and treatment should be altered until the IGF-1 disease. When undiagnosed. Cushing disease is associated
declines. with devastating Jong term morbidity such as diabetes. mor
bid obesity. hypertension. infertility. and osteoporosis.
KEY POINTS
The initial step in evaluation for Cushing disease is to seek
• Acromegaly occurs when a pituitary tumor secretes biochemical evidence or hypercorlisolism (see Disorders or Lhe
excessive amounts of growth hormone in an adult Adrenal Glands).
patient resulting in changes in facial structure, an Once ACTH-dependent Cushing syndrome is confirmed
enlarged skull, a large nose, facial edema, excessive biochemically. a pituitary MRI should be obtained. Ir no pitui­
spacing between teeth, macroglossia, and dispropor­ tary tumor or a tumor less than 6 mm is visualized on MRI. an
tionately large hands and feet. 8 mg dexamethasone suppression test is used to dil'Jerentiale
• Treatment of acromegaly is transsphenoidal tumor Cushing disease from an ectopic source of ACTH. Ectopic
resection; however, in some patients, adjuvant radiation ACTH production f'rom a nonpiluitary tumor (most often lung.
therapy or medical therapy, such as injectable somato­ pancreas. or thymus carcinomas) is very uncommon.
statin analogues, is needed for residual disease. Dexamethasone is administered al JI Pill. and cortisol is
lesled al 8 AM. A pituitary source of' ACTH will respond lo
negative feedback rrom high doses or dexamethasone. sup­
Gonadotropin-Producing Adenomas
pressing cortisol to Jess lhan 5 µg/dL (l38 nmol/L). while an
Gonadotropin-producing pituitary adenomas are typically
ectopic source of' ACTH will not have suppressible cortisol.
asymptomatic and are treated similarly to nonfunctioning
J Jowever. this test has low sensitivity (88%) and specificity
adenomas because they either do not secrete functional gon­
(STX,) for Cushing disease. so intrapelrosal sinus sampling
adotropins or do not secrete enough FSH or LH to produce a
(I PSS) is often recommended before exploratory pituitary sur­
clinical syndrome. Often, the diagnosis is made postopera­
gery. In IPSS. a catheter is threaded through the petrosal sinus.
tively, based on histopathologic staining of surgical pathology
and ACTH levels in the sinus are compared with those in the
specimens.
periphery af'ler the administration or corticotropin-releasing
hormone (CRH). A central lo peripheral gradient greater than
Thyroid-Stimulating Hormone-Secreting Tumors
2.0 betore CRH or greater than 3.0 after CRH is diagnostic of
TSH-secreting tumors are extremely rare. These tumors may
Cushing disease (95% sensitivity. 93% specificity). Imaging of'
co--secrete TSH and prolactin or GH. TSH-secreting tumors
the chest and abdomen is indicated in patients with a sus­
cause hyperthyroidism. Patients with TSH-secreting tumors
pected ectopic source of ACfH.
have either an inappropriately normal or a high TSH level
with a simultaneous elevation ofT4 and T 3 levels. They pre­ Treatment
sent with identical symptoms associated with non-TSH­ Cushing disease is treated by Lranssphenoidal pituitary
mediated thyrotoxicosis (see Disorders of the Thyroid Gland). tumor resection. which may be curative. Endogenous ACTH
After biochemical proof of TSH excess is obtained, pituitary production in the remaining normal pituitary gland will be
imaging is recommended to confirm a pituitary mass. suppressed af'ler removal or the tumor due to long-standing
Neurosurgery is first-line therapy, but patients often require hypercortisolism. so patients with successful surgical treat­
additional medical therapy with either somatostatin ana­ ment will have acute ACTH deficiency and require gluco
logues or dopamine agonists. corticoid replacement. It may take up to J year for
endogenous ACTH production to return to normal. and
Excess Antidiuretic Hormone Secretion sometimes the hypothalamic-pituitary adrenal axis does
The syndrome of inappropriate ADH secretion (SlADH) causes not recover. After successful resection. Cushing disease can
water retention and hyponatremia. Central nervous system recur. and patients musl be monitored annually ror several
pathology such as stroke, hemorrhage, trauma, or infection yeus . .:incl then less frequently. or sooner if' symptoms of'
can cause SIADH because of the excessive release of hypotha­ hypercorlisolism recur.
lamic and pituitary ADH. Also, transient SIADH is a common Ir surgical cure is not achieved. patients may be offered
complication of pituitary surgery, occurring in about one third pituitary radiation or medical therapy. Medic.:il options include
of patients approximately 3 to 10 days after surgery (see inhibitors or adrenal enzyme synthesis of cortisol, ketocona­
MKSAP17 Nephrology). zole. or metyrapone: the dopamine agonisl, cabergoline; or the
somalostatin analogue. pasireotide. Medical cure or Cushing

Cl Cushing Disease
Cushing disease is Lhe term used to indicate excess cortisol
disease has a relatively low success rate. but hypercortisolism
symptom control is an achievable goal in all patients with
production clue to an ACTH-secreting pituitary adenoma . endogenous Cushing syndrome.

28
Disorders of the Adrenal Glands

l"l"I In patients who do not benef'it f'rom surgicil treatment Synthesis of adrenal androgens, dehydroepiandrosterone
LI.I and who have an inadequate response Lo medical treat­ (DHEA) and its sulfate (DHEAS), and androstenedione, occurs
CONT.
ment, bilateral aclrenalectomy to remove the target of" ACTH primarily in the zona reticularis and is regulated by ACTH.
stimulation is an option. However. these patients will Although the adrenal androgens themselves have minimal
require lifelong glucocorticoicl and mineralocorticoicl intrinsic androgenic activity, they are converted peripherally
replacement. Cl to testosterone and dihydrotestosterone. Unlike glucocorti­
coids and mineralocorticoids, deficiencies of adrenal andro­
KEY POINTS
gens are not typically recognized clue to parallel production of
• Cushing disease refers to excess cortisol production clue gonadal androgens.
to an adrenocorticotropic hormone (ACTH)-secreting The adrenal medulla and extra-adrenal sites of the sym­
pituitary adenoma; Cushing syndrome refers to hyper­ pathetic nervous system consist of chromaffin cells, which
cortisolism from any cause, exogenous or endogenous, synthesize catecholamine hormones from the amino acid
ACTH-dependent or not. tyrosine (Figure 4). Catecholamines are stored within chro­
• Cushing disease is treated by transsphenoidal pituitary maffin granules, which release their contents in response to
tumor resection; after surgery glucocorticoid replace­ stress. Although catecholamine excess produces disease,
ment therapy will be required at least transiently while hypofunction of the adrenal medulla does not because of
the hypothalamic-pituitary-adrenal axis recovers redundancy of catecholamine production throughout the
endogenous function. sympathetic nervous system. Norepinephrine is synthesized in
the adrenal medulla and the extra-adrenal sites of the sympa­
thetic nervous system. It causes vasoconstriction due to pref­
Disorders of the erential binding to a-receptors. Epinephrine is almost
exclusively produced in the adrenal medulla. It binds pre­
Adrenal Glands dominantly to �-receptors, and thus has positive effects on
cardiac inotropy and chronotropy, produces peripheral vaso­
Adrenal Anatomy and Physiology dilation, and increases plasma glucose levels in response to
Located just superior to each kidney, the paired adrenal glands hypoglycemia.
consist of an outer cortex and an inner medulla that are dis­
tinct in embryologic origin and endocrine function. The adre­
nal cortex is composed of three zones: the zona (outer) glo­ Tyrosine
merulosa, zona (middle) fasciculata, and zona (inner)
reticularis. Within these zones corticosteroid hormones are
synthesized from cholesterol by cytochrome P450 enzymes.

J0
Aldosterone, the principal mineralocorticoid hormone, is pro­ �
duced in the zona glomerulosa. Aldosterone production is
triggered by an increase in the extracellular potassium con­
centration and by activation of aldosterone synthase through Dopamine

the renin-angiotensin-aldosterone pathway. Upon binding to


type 1 mineralocorticoid receptors (MR) in the kidney, aldos­
terone promotes potassium wasting and sodium retention,
- ®
� -�
Norepinephrine Normetanephrines
which leads to an increase in intravascular volume and conse­
quently blood pressure.
�--�®

�-��'---�
The zona fasciculata is the main site of glucocorticoid Epinephrine Metanephrines
synthesis. Production of cortisol, the principal glucocorti­
coid, is stimulated by adrenocorticotropic hormone (ACTH) CD CD
secretion from the anterior pituitary. Cortisol secretion
varies according to the circadian rhythm with relatively ____Cl_ihydroxymandelic acid � �anillylmandelic acid

little secretion overnight, peak levels in the early morning,


FIGURE 4. Catecholamine hormones are produced in the adrenal medulla
and smaller oscillations throughout the day. Cortisol atten­ and sympathetic ganglia. The pathways of synthesis and degradation are shown.
uates inflammatory responses and contributes to glucose Excessive catecholamine secretion can occur with pheochromocytomas and
homeostasis by promoting lipolysis, hepatic gluconeogen­ paraganglionomas.
esis, and insulin resistance. Physical stress (for example, 1 = Tyrosine hydroxylase
2 = DOPA decarboxylase
critical illness) stimulates increased cortisol secretion,
3 = Dopamine �·hydroxylase
which enhances vascular smooth muscle tone and respon­ 4 = Phenylethanolamine N·methyltransferase (PNMT)
siveness to endogenous vasoconstrictors, thereby augment­ 5 = Monoamine oxidase (MAO)
ing blood pressure. 6 = Catechol·O·methyltransferase (COMT)

29
Disord ers of the Adrenal Glands

Adrenal Hormone Excess Clinical manifestations of CS are listed in Table 18. Clinical
findings that are highly specific for CS include centripetal obe­
Cushing Syndrome
sity, facial plethora, abnormal fat deposition in the supraclav­
Cushing syndrome (CS) is a rare disorder affecting two to three icular or dorsocervical ("buffalo hump") areas, and wide (>l
persons per million per year that results from elevated levels of cm) violaceous striae (Figure 5). It is important to initiate
cortisol. Poor suppressibility of cortisol with dexamethasone evaluation for CS in patients who have specific signs and
and loss of normal diurnal variation in cortisol secretion are symptoms of CS, rather than in patients who are diffusely
seen. Without treatment, it is associated with high morbidity obese, have nonpathologic striae, and are having trouble losing
and mortality. weight because endogenous CS is such a rare condition with a
However, iatrogenic hypercortisolism from the admin­ costly evaluation algorithm.
istration of exogenous oral, inhaled, intra-articular, or topi­ Biochemical testing is used to establish the diagnosis of CS.
cal glucocorticoids is often seen in clinical practice and is It is critical that the biochemical diagnosis is firmly established
the most common cause of CS overall. The pharmacokinetics prior to any imaging studies due to the relatively high prevalence
and relative potencies of synthetic oral glucocorticoids are of clinically insignificant pituitary and adrenal nodules. At least
shown in Table 17. The sustained administration of any syn­ two first-line tests should be diagnostically abnormal before the
thetic glucocorticoid above the normal physiologic cortisol diagnosis is confirmed. Initial tests include the overnight low­
requirement can result in iatrogenic CS and hypothalamic­ dose dexamethasone suppression test (LOST), 24-hour urine
pituitary-adrenal (HPA) axis suppression, but is more likely free cortisol (UFC), and late-night (LN) salivary cortisol. All three
to occur the longer the half-life of the drug. Doses equivalent tests have similar diagnostic utility, but the LOST or LN salivary
to prednisone 5 mg/d or less are unlikely to cause clinically cortisol tests are more convenient. The 24-hour UFC and LN
significant HPA axis suppression, while those in excess oflO salivary cortisol tests should be performed at least twice to
to 20 mg/d commonly do after 3 weeks or more of consecu­ ensure reproducibility of results. Because the secretion of corti­
tive use. sol is pulsatile, measurement of random serum cortisol is neither
Endogenous CS can result from ACTH-dependent and sensitive nor specific for the diagnosis of CS. An algorithm to
ACTH-independent causes. Cushing disease, which results establish the diagnosis of CS is shown in Figure 6. Referral to an
from the autonomous secretion of ACTH by a corticotroph endocrinologist is indicated if two initial tests are abnormal.
adenoma of the pituitary gland, is the cause of CS in more than
two thirds of patients (see Disorders of the Pituitary Gland).
Ectopic ACTH secretion by carcinomas and carcinoid tumors
(usually bronchial origin) is less common, accounting for 10%
to 15% of cases, while ectopic corticotropin-releasing hormone
(CRH) production is rare. The most common ACTH­
independent etiologies of CS are adrenal adenomas and carci­
nomas, which collectively account for approximately 20% of
CS cases.
CS must be differentiated from other disorders and clini­
cal states that are associated with physiologic hypercortisolism
(pseudo-Cushing syndrome). Causes of pseudo-Cushing syn­
drome include severe obesity, polycystic ovary syndrome, f I GU RE 5. Wide violaceous striae are seen on the abdomen of a patient with
pregnancy, anorexia nervosa, depression, alcoholism, and Cushing syndrome. Striae larger than 1 cm in width are highly specific for hyper­
extreme physical stress, as in the setting of infection. cortisolism.

TABLE 17. Dose Equiyal!'!C:�.<!_nd -�j�t.i_v� !:'oteric�es <>f Co mmon Syn�h�tic_Oral Glucocorticoids
Synthetic Glucocorticoid Equivalent Biologic Relative Anti- Relative
Replace ment Half-Life Infla mmatory Mineralocorticoid
Dose ( mg)• (hours) Potencyh Potency c

Hydrocortisone 20 8-12 1/125


Prednisolone/prednisone 5 18-36 4 1/156
Methylprednisolone 4 18-36 5 0
Dexamethasone 0.75 36-54 25-50 0
a Denotes common glucocorticoid dosing for primary adrenal failure equivalent to hydrocortisone, 20 mg.

bAnti-inflammatory potency relative to hydrocortisone.

'Mineralocorticoid potency relative to fludrocortisone.

30
Disorders of the A drenal Glands

TABLE 18. Clinical Features of Cushing Syndrome other proteins. Therefore the LDST should not be performed
Specific Findings Less Specific Associated
when CBG is likely to be abnormal, such as with malnutrition,
Findings Conditions• cirrhosis, the nephrotic syndrome, and hyperestrogenemia
(oral contraceptive pills or pregnancy). There is no clear asso­
Centripetal obesity Easy bruising Osteoporosis
ciation between dexamethasone responses and BM! or weight,
Facial plethora Excessive skin Hypertension and therefore the LDST may be used similarly in the obese
fragility
Supraclavicular fat Diabetes mellitus population. The LDST is best avoided in patients taking medi­
pads Proximal muscle
Obesity cations that could accelerate dexamethasone metabolism,
weakness
Dorsocervical fat such as antiepileptic drugs (phenytoin, phenobarbital, and
Depression
pads Impaired memory
carbamazepine), rifampin, or pioglitazone. Concomitant
Hypokalemia
Wide violaceous Temporal balding b
measurement of serum dexamethasone can confirm altered
striae Nephrolithiasis
Hirsutism (in dexamethasone metabolism and patient adherence.
women) b VTE/PE Measuring 24-hour UFC circumvents problems related to
Menstrual cortisol pulsatility and binding protein abnormalities. The test
abnormalitiesb
should be performed at least twice to ensure accuracy. To con­
PE= pulmonary embolism; VTE= venous thromboembolism. firm adequate collection, 24-hour urine creatinine is also
aMedical disorders that may be seen in association with but are not specific for measured (normal range, 20-25 mg/kg/24 h [177-221 mmol/
Cushing syndrome.
kg/24 h] in men; 15-20 mg/kg/24 h [133-177 mmol/L/24 h] in
bFeatures of androgen excess seen with pituitary corticotroph adenoma or adreno­
cortical carcinoma.
women). A test is considered abnormal when UFC exceeds the
upper limit of the normal range of the assay (45 µg/24 h [124
nmol/24 bl), while values greater than 3 times normal are
diagnostic of CS. Less marked elevations are seen with pseudo­
In the standard LOST, dexamethasone (0.5 mg) is admin­ Cushing syndrome and polyuria. A falsely low UFC can occur
istered every 6 hours for 48 hours and serum cortisol is meas­ in chronic kidney disease and when CS is subclinical or mild.
ured at 9 AM. In the overnight LDST, 1 mg of dexamethasone is The LN salivary cortisol test is performed between 11 PM
administered at 11 PM or midnight, and serum cortisol is meas­ and midnight. The normal evening nadir in cortisol secretion
ured the next morning at 8 AM. With either test, serum cortisol is lost in patients with CS, while it is preserved in patients with
will typically be suppressed to less than 2 µg/dL pseudo-Cushing syndrome. Both emotional and physical
(55 nmol/L). Standard assays measure total serum cortisol, or stress (for example, exercise) can cause a physiologic increase
that which is bound to cortisol-binding globulin (CBG) and of salivary cortisol. False-positive results are seen with

Perform initial testing:


Exclude use of
Clinically - 24 hr UFC'
exogenous
suspected CS - 1 mg DST
glucocorticoids
- LN salivary cortisol•

Abnormal: Normal
Exclude physiologic
hypercortisolism

Physiologic Physiologic
hypercortisolism hypercortisol ism 1--- - -
.. 1 CS unlikely I
excluded suspected

Normal:
Abnormal: Additional
Repeat initial abnormal
CS confirmed testing
test; do additional tests

FIGURE 6. Algorithm to confirm or rule out the diagnosis of Cushing syndrome. CS= Cushing syndrome; DST= dexamethasone suppression test; LN salivary
cortisol = late-night salivary cortisol; UFC = urine free cortisol.
AMust be performed at least twice.

31
Disorders of the Adrenal Glands

cigarette smoking or use of chewing tobacco. LN salivary cor­ suppression and contralateral adrenal atrophy from long-stand­
tisol testing should not be performed in patients with erratic ing elevated cortisol levels. All patients should therefore be treated
sleep schedules (for example, shift-workers). with stress-dose glucocorticoids during the petioperative petiod
After CS has been confirmed biochemically, further testing and continued on physiologic replacement until HPA axis recov­
is required to distinguish ACTH-dependent or -independent ery has been confirmed. Following successful surgery, the physi­
causes, and consultation with an endocrinologist is recom­ cal changes associated with CS can take up to 1 year to resolve.
mended. The first step is to measure plasma ACTH on two
KEY POINTS
separate occasions. With adrenal (ACTH-independent) CS,
plasma ACTH is usually less than S pg/mL (1.1 pmol/L), • Cushing syndrome results from endogenous hypercorti­
whereas values greater than 20 pg/mL (4.4 pmol!L) are typi­ solism or exogenous exposure to glucocorticoids; it is
cally seen with ACTH-dependent causes. Plasma ACTH values associated with poor suppressibility of endogenous cor­
of S to 20 pg/mL (1.1-4.4 pmol/L) are nondiagnostic but are tisol production with oral dexamethasone.
more likely to be seen with ACTH-dependent disorders. For a • The most common cause of Cushing syndrome is the
discussion of the evaluation and management of ACTH­ administration of exogenous glucocorticoid therapy for
dependent CS, see Disorders of the Pituitary Gland. another medical condition.
The next step in the evaluation of ACTH-independent CS is • Initial tests for Cushing syndrome include the overnight
with imaging of the adrenal glands, such as dedicated adrenal low-dose dexamethasone suppression test, 24-hour
imaging with thin-section CT or MRI. Both studies have equal urine free cortisol, and late-night salivary cortisol.
sensitivity; however, MRI is more costly. Adrenal adenomas and
carcinomas can usually be distinguished from one another Pheochromocytomas and Paragangliomas
radiographically (Table 19). Surgery is considered first-line Paragangliomas are tumors composed of chromaffin cells.
treatment for adrenal adenomas and nonmetastatic adrenocor­ Approximately 80% are intra-adrenal (pheochromocytomas);
tical carcinomas (ACCs). When surgery is delayed for patients the rest originate from extra-adrenal sympathetic or parasym­
with overt CS, adrenal enzyme inhibitors (metyrapone, keto­ pathetic paraganglia. The most common location for extra­
conazole, and etomidate) can be used to reduce cortisol levels adrenal sympathetic paragangliomas is the abdomen, whereas
and decrease the risk of complications, such as opportunistic parasympathetic paragangliomas are usually found in the head
infections and cardiovascular events. The management of ACC and neck. Pheochromocytomas and extra-adrenal sympathetic
is discussed elsewhere (see Adrenocortical Carcinoma). paragangliomas almost always secrete catecholamines (norepi­
Following adrenalectomy, patients with adrenal CS will nephrine, epinephrine, dopamine); however, head and neck
often develop acute adrenal insufficiency because of HPA axis parasympathetic paragangliomas almost never do.

TABLE 19. Typical Imaging Characteristics of Adrenal Masses


Adrenal Mass Overall CT MRI Signal Intensity•

Adrenal adenoma Diameter <4 cm Density <10 HU lsointense on T2-weighted images


Homogeneous enhancementb Contrast washout >50% (10 min)
Round, clear margins
Adrenocortical Usually>4 cm Density>10 HU Hyperintense on T2-weighted images
carcinoma b
Heterogeneous enhancement Contrast washout <50% (10 min)
Irregular margins
Calcifications, necrosis
Pheochromocytoma Variable size Density>10 HU Hyperintense on T2-weighted images
b
Heterogeneous enhancement , Contrast washout <50% (10 min)
cystic areas
Round, clear margins
Can be bilateral
Metastases Variable margins Density>10 HU Hyperintense on T2-weighted images
Can be bilateral Contrast washout <50% (10 min)

HU = Hounsfield units (measure of radiodensity compared with water).

aSignal intensity as compared with liver.

l>Enhancement following intravenous contrast administration.

32
Disorders of the Adrenal Glands

Although catecholamine-secreting tumors are rare over­ low (adrenal mass without typical radiographic appearance),
all, they are found in 0.5% of patients with hypertension, and while measurement of plasma free metanephrines is preferred
pheochromocytomas account for 5% of adrenal incidentalo­ when clinical suspicion is higher (known hereditary syn­
mas (see Incidentally Noted Adrenal Masses). Most pheochro­ drome). Referral to an endocrinologist is recommended when
mocytomas secrete norepinephrine, resulting in episodic or biochemical testing is abnormal.
sustained hypertension. Orthostatic hypotension can also be Many medications and other substances cause falsely
seen and likely reflects low plasma volume. In addition to the high levels of plasma and urine catecholamines or metane­
classic triad of diaphoresis, headache, and tachycardia, com­ phrines (Table 21); therefore, discontinuation of these agents
mon symptoms include palpitations, tremor, pallor, and anxi­ before testing is recommended. If a catecholamine-secreting
ety. Less common features are papilledema, diabetes mellitus, tumor is strongly suspected in a critically ill hospitalized
and cardiomyopathy. Approximately 10% of pheochromocyto­ patient, CT or MRI of the abdomen is the preferred initial test
mas and 20% to 50% of paragangliomas are malignant. because biochemical testing cannot be interpreted reliably in
One third of pheochromocytomas and paragangliomas this setting.
occur in the context of a genetic disorder. Pheochromocytomas Following the biochemical diagnosis of pheochromocy­
are seen with multiple endocrine neoplasia (MEN) syndromes toma or catecholamine-secreting paraganglioma, radiographic
type 2A and 2B (Table 20), neurofibromatosis type 1, and von localization is needed. Because most catecholamine-secreting
Hippel-Lindau syndrome (VHL). Paragangliomas and, less tumors are located in the abdomen, CT or MRI of the abdomen
frequently, pheochromocytomas can occur with familial para­ and pelvis is the best initial study. If negative, iodine 123
(123 1
ganglioma syndrome mutations, some of which are associated )-metaiodobenzylguanidine (MIBG) scanning can be per­
with high rates of malignancy. formed. Adjunctive diagnostic tests are CT or MRI of the chest
The diagnosis of pheochromocytoma and paraganglioma or head and neck region.
is based on confirmation of the excessive secretion of catecho­ When multiple tumors or malignant pheochromocyto­
lamines or their metabolites, as measured in the plasma or mas or paragangliomas are suspected, MIBG scanning should
urine. Evaluation is recommended, if clinically suspected, in be performed preoperatively. However, for identification of
the evaluation of an incidentally noted adrenal mass or in the metastatic disease, fluorine 18 (18 F)-fluorodeoxyglucose (FOG)
setting of hereditary pheochromocytoma or paraganglioma PET scanning is superior to other diagnostic tests.
syndromes. The sensitivity of plasma free metanephrines is Preoperative pharmacologic treatment is mandatory for
the highest of any screening test (96%-100%); however, its pheochromocytomas and paragangliomas to prevent life­
specificity is relatively low (85%-89%). Therefore, plasma free threatening cardiovascular complications related to the mas­
metanephrines will reliably exclude a pheochromocytoma sive release of catecholamines during surgery. Preoperative
when negative, but further testing is needed to confirm the blockade of a-adrenoceptors, usually with phenoxyben­
diagnosis unless the result is markedly abnormal (above 4 zamine, is first-line medical therapy. The dosage is titrated to
times the upper limit of normal). The sensitivity and specific­ achieve a blood pressure below 130/80 mm Hg seated and
ity of 24-hour urine fractionated metanephrines and catecho­ greater than 90 mm Hg (systolic) standing. Commonly used
lamines are 91% to 98%. Due to the lower frequency of but non-FDA approved alternatives include calcium channel
false-positive results, 24-hour urine measurements are recom­ blockers and selective a1 -blockers (terazosin or doxazosin).
mended when the pre-test probability of disease is relatively �-Adrenoceptor blockers (metoprolol or propranolol) are

TABLE 20. Multiple Endocrine Neoplasm Syndromes


Type Mutation Most Common Associated Features
Feature

MEN1 (inheritance of one mutated Parathyroid adenoma Pancreatic islet cell and enteric tumors (gastrinoma, insulinoma
allele with somatic mutation in (often multiple) most common)
other allele leads to neoplasia)
Pituitary adenoma
Other (carcinoid tumors, adrenocortical adenoma)

2A RET Medullary thyroid Pheochromocytoma (often multifocal)


carcinoma
(exon 11, codon 634 ) °
Parathyroid hyperplasia
2B RET Medullary thyroid Pheochromocytoma (often multifocal)
carcinoma
(exon 16, codon 918 °) Mucosal neuroma
Gastrointestinal ganglioneuroma
Marfanoid body habitus

.,,Most common mutation observed.

33
Disorders of the Adrenal Glands

TABLE 21 . Substances Associated with False-Positive KEY POINTS (continued)


Biochemical Testing for Pheochromocytoma
• When clinical suspicion of pheochromocytoma or para­
Drug Class Medication/Substance ganglioma is low, measurement of 24-hour urine frac­
Analgesics Acetaminophen tionated metanephrines and catecholamines are the
tests of choice because of their hjgh specificity (low
Antiemetics Prochlorperazine
false-positive rates); when clinical suspicion is hlgh,
Antihypertensives Phenoxybenzamine• measuring plasma free metanephrines is preferred due
Psychiatric Antipsychotics to its greater sensitivity.
medications
Buspirone • P reoperative a-adrenergic blockade is first-line medical
Monoamine oxidase inhibitors therapy for pheochromocytomas and paragangliomas;
Tricyclic antidepressants• �-adrenoceptor blockers (metoprolol or propranolol)
Stimulants Amphetamines are added after a-blockade to treat reflex tachycardia.
Cocaine • Pheochromocytomas and paragangliomas require life­
Caffeine long surveillance for recurrence with annual plasma
free metanephrine measurement.
Other agents Levodopa
Decongestants ( pseudoephedrine) Primary Hyperaldosteronism
Reserpine Primary hyperaldosteronism (PA) results f'rom the autono­
Withdrawal Clonidine mous secretion of' excessive aldosterone. PA is relatively com­
Ethanol mon , occurring in approximately 10% of' patients with
hypertension. Additional signs of PA include hypokalemia and
Illicit drugs
metabolic alkalosis. Without treatment. excess cardiovascular
.1Most likely to cause false-positive results.
morbidity and mortality are seen.
Aldosterone-producing adrenocortical adenomas (APA;
aldosteronomas) cause approximately 40% of PA, whereas
added later to treat reflex tachycardia, but should never be nearly all other cases are due to bilateral adrenal hyperplasia.
started before adequate a.-blockade has been achieved due to Unilateral adrenal hyperplasia and aldosterone-secreting
the risk of hypertensive crisis from unopposed a-receptor adrenocorlical carcinomas (ACCs) are rare. Familial hyperal­
stimulation. A heart rate of 60 to 70/min seated and 70 to 80/ dosteronislll is also uncolllmon.
min standing can be targeted in most patients. Testing for PA should be considered in all patients with
Because it is associated with fewer surgical complications dif'ficult to control hypertension. It should also be performed
and shorter postoperative hospital stays, laparoscopic adrenal­ in patients with hypertension and an incidentally noted adre­
ectomy is preferred for pheochromocytoma except in the case nal lllass or spontaneous or diuretic-induced hypokalemia. A
of large or malignant tumors, when open adrenalectomy is nulllber of disorders can lllimic PA clinically; however, the
required. Following the surgical removal of a catecholamine­ results of biochemical testing will differ. Examples include
secreting tumor, large-volume intravenous crystalloid is secondary hyperaldosteronislll (renal artery stenosis or renin­
administered to counter hypotension. Vasopressors (for exam­ secreting tumors), autosolllal dolllinant pseudoaldosteronism
ple, norepinephrine) are sometimes required. Long-term (Liddle syndrollle). CS, certain forms of congenital adrenal
follow-up is needed for pheochromocytomas and paraganglio­ hyperplasia (CAH). and licorice-induced hypermineralocorti­
mas due to difficulty distinguishing benign from malignant coidism.
tumors. Metastases have been reported up to 20 years after Initial screening for PA is with the simultaneous measure­
diagnosis. In addition to routine clinical surveillance, annual ment or midmorning ambulatory plasma renin activity (PRA)
measurement of plasma or urine metanephrines is indicated and plasma aldosterone concentration (PAC). in a volume
to assess for recurrent or metastatic disease. Metastatic disease replete normokalemic patient. Testing is positive if PAC is
is managed with additional surgery, iodine 131 (1 :i1[)-labeled frankly elevated (>15 ng/dL (414 pmol/L]). PRA is suppressed,
MIBG therapy, chemotherapy, and/or radiotherapy. Cure is not and PAC'PRA ratio is greater than 20. Many medications.
possible unless all disease can be surgically resected. including common antihypertensive agents. can affect meas­
urements or PAC. PRA, or both (Table 22). However, because
KEY POINTS
patients undergoing screening often have drug-resistant hyper­
• Pheochromocytomas are seen with multiple endocrine tension. discontinuing all potentially offending medications
neoplasia (MEN) syndromes type 2A and 2B, neurofi­ can be unsafe. Stopping mineraloco1iicoid receptor antagonists
bromatosis type 1, and von Hippel-Lindau syndrome. (spironolactone and eplerenone) for 4 to 6 weeks prior to test­
ing is recommended. Diuretics should also be discontinued
(Continued)
prior to testing to assure euvolemia. Most other medications

34
Disorders of the Adrenal Glands

TABLE 22. The Effect of Commonly Prescribed : lemia with undetectable PRA and PAC greater than 30 ng/dL
Medications on Measurements of Plasma Renin Activity and (828 pmol/L). Confirmatory tests include oral and intravenous
Plasma Aldosterone Concentration ', salt loading and the fludrocortisone suppression and captopril
Effect on Test Medication Class PRA PAC PAC/ challenge tests (Table 23).
Results PRA Once the diagnosis or PA has been confirmed biochemi­
False-Positive a-Adrenoceptor J,J, J, i cally, radiographic localization with abdominal CT is indi­
agonist cated. CT is recommended over MRI in most cases due to
P-Adrenoceptor J,J, J, i similar efficacy and lower cost. Adrenal hyperplasia and
blocker aclenomas can often be visualized and aclrenocortical carci­
Direct renin J, J, i noma can be ruled out. Adrenal vein sampling (AVS) is
inhibitor needed in most patients to determine the source of aldoster­
NSAID J,J, J, i one secretion when imaging is unrevealing and to confirm
False-Negative ACE inhibitor/ARB ii J, J, lateralization when imaging demonstrates an adrenal ade­
noma. AVS is especially important in older patients (40 years
Dihydropyridine i J, J,
CCB and older) because of a higher frequency of nonfunctioning
Diuretic• ii i J,
adrenal incidentalomas. AVS should be performed at experi­
enced centers only.
Mineralocorticoid ii i J,
receptor The goals of treatment include improvement in blood
antagonist pressure (resolution of hypertension is unlikely). normali­
SSRI i J, zation of serum potassium (this is very likely) , and reduc­
tion in plasma aldosterone because hyperaldosteronemia is
ARB= angiotensin receptor antagonist; CCB = calcium channel blocker; PAC =
plasma aldosterone concentration; PRA = plasma renin activity; SSRI = selective associated with a blood pressure-independent increase in
serotonin reuptake inhibitor.
cardiovascular events. The treatment of choice for PA clue
asoth potassium-sparing (amiloride) and potassium-wasting (hydrochlorothiazide)
diuretics.
to APA or unilateral adrenal hyperplasia is Japaroscopic
aclrenalectomy.

Cl
For patients with bilateral adrenal hyperplasia or those
can be continued, but results must be interpreted in context. with unilateral causes of PA who are not surgical candidates.
For example, if PRA is suppressed despite treatment with an medical therapy with a mineralocorticoid antagonist is indi­
CONT.
ACE inhibitor or angiotensin receptor blocker. PA is likely. If cated. Spironolactone is the most commonly used medication
results are difficult to interpret, repeat testing after eliminating due to its proven efficacy and cost-effectiveness. Eplerenone
potential interfering medications is advised. Verapamil. is less likely to cause side effects (gynecomastia in men and
hydralazine, and ex-blockers (doxazosin) can be substituted for menstrual irregularities in women) because of greater miner­
blood pressure control if necessary. Referral to an endocrinolo­ alocorticoid receptor selectivity. Amiloride is a potassium­
gist is recommended when screening tests are abnormal. sparing diuretic that blocks the aldosterone-sensitive sodium
Confirmatory testing is performed except when initial channel. Use of'amiloride in PA is second-line therapy because
testing is diagnostic for PA, as in cases of spontaneous hypoka- of lower efficacy. Cl

TABLE 23. Laboratory Testing Used in the Diagnosis of Hyperaldosteronism


Test Details Positive If...

Captopril Administer: Captopril2 5-50 mg orally after the patient has been PAC remains elevated and PRA suppressed
challenge test seated for 1 hour
(Normal response is suppression of PAC by
Measure: PAC, PRA, and cortisol at O and 1 or2 hours while seated >30%)
Fludrocortisone Administer: Fludrocortisone0. 5 mg orally every6 hours for4 days PAC>6 ng/dl (165.6 pmol/L)
suppression along with sodium and potassium supplementation
PRA <1 ng/mUh (1 µg/Uh)
test
Measure: Serum cortisol at7 and 10 AM, and PAC and PRA at
(Cortisol at 10 AM lower than 7 AM)
10 AM on day4
Oral salt Administer: Sodium chloride 6 g orally daily (in divided doses)for 24-hour urine aldosterone>12 µg
loading test 3 days
(Urine Na>200 mEq [220 mmol/L])
Measure: 24-hour urine aldosterone and urine Na on the third day
Intravenous salt Administer: 2 L0. 9% saline intravenously over4 hours while supine PAC>10 ng/dl (276.0 pmol/L)
loading test
Measure: PAC, PRA, cortisol, and serum Kat O and4 hours

IM = intramuscular; IV= intravenous; K = potassium; Na = sodium; PAC= plasma aldosterone concentration; PRA = plasma renin activity.

35
Disorders of the Adrenal Glands

KEY POINTS have at least one other autoimmune endocrine disorder, and
more than 80% have adrenal autoantibodies (21-hydroxylase
• Testing for primary hyperaldosteronism is with the
antibodies). Infiltration of the adrenal glands by tuberculosis
simultaneous measurement of midmorning ambulatory
(Addison disease) was formerly the most common etiology of
plasma renin activity and plasma aldosterone levels;
primary adrenal failure; now it is responsible for only 7% to
testing is positive if plasma aldosterone concentration is
20%. Replacement of the adrenal glands can also occur with
frankly elevated (>15 ng/dL [414 pmol/L]), plasma renin
metastatic cancer. Genetic causes include autoimmune poly­
activity is suppressed, and a ratio of the former over the
glandular syndromes (APS) type 1 and 2, congenital adrenal
latter is greater than 20.
hyperplasia, and X-linked adrenoleukodystrophy. Adrenal cri­
• The treatment of choice for primary hyperaldosteronism sis resulting from bilateral adrenal hemorrhage can occur with
due to an aldosteronoma or unilateral adrenal hyperpla­ the antiphospholipid syndrome, disseminated intravascular
sia is laparoscopic adrenalectomy; for patients with coagulation, or systemic anticoagulation.
bilateral adrenal hyperplasia or those with unilateral The clinical presentation of primary adrenal failure
causes of primary hyperaldosteronism who are not can­ depends on disease chronicity and the presence of physical
didates for surgery, medical therapy with a mineralocor­ stressors. In autoimmune adrenalitis, the zona glomerulosa is
ticoid antagonist such as spironolactone is indicated. usually affected first, which is manifest by an increase in PRA.
With involvement of the zona fasciculata, a diminished corti­
Androgen-Producing Adrenal Tumors sol response to ACTH is seen, followed by an increase in basal
Pure androgen-secreting adrenal neoplasms are very rare. plasma ACTH, and lastly a decrease in serum cortisol. Patients
These tumors usually secrete DHEA and DHEAS and/or typically do not have symptoms until hypocortisolemia occurs.
androstenedione, which are converted peripherally to testos­ Table 24 shows the clinical and laboratory manifestations of
terone. Approximately half of androgen-producing tumors are primary adrenal failure. Hyperpigmentation is a clinical hall­
benign and half are malignant. Manifestations of androgen­ mark of this disorder that is not seen with secondary cortisol
producing adrenal tumors are usually absent in adult men, deficiency (seeDisorders of the Pituitary Gland for discussion
although decreased testicular volume can occur. In women, of secondary cortisol deficiency).
rapid onset of hirsutism, menstrual irregularities, and viriliza­ Adrenal crisis may occur when onset of adrenal failure is
tion can be seen and, if present, should raise suspicion for abrupt (bilateral adrenal hemorrhage) or when increased
tumoral hyperandrogenism. Signs of virilization are deepen­ stress occurs in the setting of chronic adrenal failure.
ing of the voice, clitoromegaly, and temporal hair loss. The Manifestations of adrenal crisis include shock, hypotension,
diagnosis of an androgen-producing adrenal tumor is based fever, nausea, vomiting, abdominal pain, tachycardia, and
on demonstrating elevated levels ofDHEA and its sulfate (usu­ even death. Aldosterone is critical to the maintenance of intra­
ally greater than 800 µg/dL [21.6 µmol/L]) and/or androsten­ vascular volume and blood pressure, while cortisol contrib­
edione. Although adrenal androgen excess can be seen in 30% utes to augmentation of blood pressure mostly during times of
to 40% of women with polycystic ovary syndrome, mild eleva­ increased physical stress (see Adrenal Anatomy and
tion ofDHEAS (approximately 300 µg/dL [8.1 µmol/L) is typi­ Physiology). Aldosterone deficiency is the major impetus for
cal. Adrenal imaging with CT or MRI is indicated following the development of hypotension and shock in patients with
biochemical diagnosis of disease to locate the tumor. Treatment untreated primary adrenal failure. Adrenal crisis is rare in the
is surgical removal of the tumor. setting of secondary cortisol deficiency because the renin­
angiotensin-aldosterone pathway is intact.

Adrenal Insufficiency Diagnosis


Adrenal insufficiency may be due to failure of the adrenal The diagnosis of primary adrenal failure is based on demon­
glands (primary adrenal failure), or there may be inadequate strating inappropriately low serum cortisol levels. Because
secretion of cortisol from the adrenals due to other causes, most assays measure total cortisol, abnormalities in cortisol­
including critical illness and pituitary ACTH deficiency (sec­ binding protein or albumin can trigger spurious results. An
ondary cortisol deficiency). For a discussion of secondary early morning (8 AM) serum cortisol of less than 3 µg/dL
cortisol deficiency, see Disorders of the Pituitary Gland. (82.8 nmol/L) is consistent with cortisol deficiency, whereas
values greater than 15 to 18 µg/dL (414.0-496.8 nmol/L)
Primary Adrenal Failure exclude the diagnosis when binding protein abnormalities and
Causes and Clinical Features synthetic glucocorticoid exposure are excluded. For patients
Primary adrenal failure is a rare disorder resulting from a fail­ with nondiagnostic basal cortisol values (5-12 µg/dL (138-331.2
ure in production of all the hormones of the adrenal cortex. nmol/L), stimulation testing with synthetic ACTH (cosyntro­
The overall prevalence is 10 to 15 per 100,000 persons. pin) is indicated (seeDisorders of the Pituitary Gland). A nor­
Autoimmune adrenalitis is the most common etiology mal response is a peak serum cortisol level greater than 20 µg/
accounting for 70% to 80% of cases. Up to two thirds of patients dL (552 nmol/L). ACTH stimulation testing should not be used

36
D i s orders of the Adrenal Glands

:TABLE 24. Clinical and Laboratory Manifestations of Primary Adrenal Failure


Hormone Deficiency Symptoms Signs Laboratory
Findings

Cortisol Fatigue Hyperpigmentationb (palmar creases, -1. Serum cortisol


extensor surfaces, buccal mucosa)
Weakness i Plasma ACTH
Decrease in BP
Low-grade fever -1. Serum sodium c
Weight loss -1. Plasma glucosed
Anorexia

Nausea/vomiting

Abdominal pain

Arthralgia
Myalgia
Aldosterone Salt craving Orthostasis iPRA
Dizziness Hypotension -1. Serum sodium
i Serum potassium

DHEAS Reduced libido• Decreased axillary or pubic hair• -1. Serum DHEAS

ACTH= adrenocorticotropic hormone; BP= blood pressure; DHEAS = dehydroepiandrosterone sulfate; PRA = plasma renin activity.

"Women only.

bQccurs exclusively in primary adrenal failure.

ccortisol inhibits the secretion of antidiuretic hormone (ADH), so hypocortisolemia will lead to increased secretion of ADH and hyponatremia.

dRare in adults.

for diagnosis in the critical care setting (see Adrenal Function longer-acting glucocorticoids (prednisone or dexamethasone)
During Critical Illness). is acceptable. All patients with cortisol deficiency need to
Once the diagnosis of cortisol deficiency has been estab­ receive instructions for increasing their cortisol replacement
lished, measurement of 8 AM plasma ACTH will differentiate dose during illness ("sick day rules''). Patients should always
primary and secondary causes. In primary adrenal failure, wear a medical alert identification.
ACTH is typically greater than 200 pg/mL (44 pmol/L), In contrast to patients with secondary cortisol deficiency
whereas it will be low or inappropriately normal in secondary (see Disorders of the Pituitary Gland), those with primary
cortisol deficiency. Although not specific for the diagnosis, adrenal failure also require mineralocorticoid replacement.
hyponatremia and hyperkalemia are characteristic of primary Usual doses are 0.05 to 0.2 mg per day of fludrocortisone.
adrenal failure and principally result from aldosterone defi­ Measurements of serum sodium and potassium help guide
ciency. dosing. Replacement of DH EA is controversial. It is not indi­
cated for men but can be considered for some women with
Treatment primary adrenal failure. However, the objective benefit is min­
Without appropriate treatment, primary adrenal failure is imal, and there are concerns regarding the quality and safety
uniformly fatal. Even when treated. the mortality of patients is of U.S. preparations where DH EA is considered a supplement
twice that of the general population. Normal adrenal physiol­ rather than a pharmaceutical.
ogy cannot be reproduced exactly by the administration of Patients who present emergently with suspected adrenal
exogenous glucocorticoids and mineralocorticoids. Moreover. crisis should be treated empirically prior to confirmation of
the administration of doses of glucocorticoid in excess of the diagnosis. A blood sample should be drawn for serum
physiologic replacement can be associated with decreased cortisol, plasma ACTH. and routine chemistries. The patient
bone mineral density and features ores. with increased risk of should receive immediate treatment with 100 mg of hydro­
metabolic syndrome. type 2 diabetes mellitus. hypertension , cortisone intravenously and aggressive fluid resuscitation.
hyperlipidemia. obesity, and cardiovascular disease. Avoidance Hydrocortisone is continued at 100 to 200 mg per day in
of chronic overreplacement is paramount. divided closes (every 6-8 hours) and then tapered to physio­
Table 25 shows the medical treatment for primary adrenal logic replacement if cortisol deficiency is confirmed with the
failure. Most patients require glucocorticoid doses equivalent above testing. Other synthetic glucocorticoids can also be
to 12.5 to 25 mg of hydrocortisone daily. Hydrocortisone is used for the treatment of adrenal crisis; however. only hydro­
administered 2 to 3 tin1es daily. while once daily dosing of cortisone in supraphysiologic doses has clinically relevant

37
Disord e rs of the Adrenal Glands

'
,TABLE 25. Chronic Medical Treatment of Primary Adrenal Failure , .
Medication Basal Dose Considerations

Glucocorticoid a
Hydrocortisone "Sick Day Rules": patient follows at home
Hydrocortisone Usually 12.5-25 mg/d, divided into For minor physiologic stress (upper respiratory infection, fever, minor
2-3 doses over the day surgery under local anesthesia)
Prednisone
Alternatives to hydrocortisone: 2-3 times basal dose for 2-3 days
Prednisolone
Prednisone 2.5-5 mg once daily Stress Dosing: health care providers follow while patient is in the hospital
Dexamethasone
Dexamethasone 0,25-0.75 mg once For moderate physiologic stress (minor or moderate surgery with general
daily anesthesia)

How to dose: Hydrocortisone 45-75 mg/d orally or IV in 3-4 divided doses for 2-3 days
Titrate to clinical response with goal Alternatives: Prednisone 10-20 mg or dexamethasone 2-3 mg/d in
of no signs or symptoms of cortisol 1-2 divided doses
deficiency or excess (increase dose if
For major physiologic stress (major surgery, trauma, critical illness, or
symptoms of cortisol deficiency
childbirth)
remain; decrease if CS signs and
symptoms are present) Hydrocortisone 150-200 mg/day IV in 3-4 divided doses; 100 mg/day the
next day; taper to baseline in 3-5 days
Alternative: Dexamethasone 6-8 mg/d IV in 2-3 divided doses
Mineralocorticoid 0.05-0.2 mg once daily in the morning Fludrocortisone is not required if hydrocortisone dose >50 mg/d
Fludrocortisone How to dose:
Titrate to
1. Normal BP
2. Normal serum Na, K
Adrenal 25-50 mg once daily Consider DHEA for women with impaired mood or sense of well-being
androgen when glucocorticoid replacement has been optimized.
DHEA

BP= blood pressure; CS= Cushing syndrome; DHEA = dehydroepiandrosterone; IV= intravenous; Na= sodium; K= potassium.

ashorter acting glucocorticoids may be preferred over longer acting agents due to lower risk of glucocorticoid excess. Longer-acting preparations have the advantage of once
daily dosing (see Table 17).

CJ
COH
mineralocorticoid activity. If present. electrolyte abnormali­
ties and hypoglycemia should be treated. and precipitants of
replacements for which there has not been a demonstrated
biochemical need.
adrenal crisis (for example. infection) should be sought and
treated. Adrenal Function During Critical Illness
It is critical that patients with suspected primary or sec­ Glucocorticoid deficiency related to critical illness is an entity
ondary cortisol deficiency and concomitant hypothyroidism that has not been well characterized. It has been postulated that
be treated with glucocorticoids first because correcting thy­ critical illness may lead to transient pri111ary or secondary cor­
roid hormone deficiency will accelerate the clearance of corti­ tisol deficiency (ACTH deficiency) or an increase in tissue resist­
sol and can precipitate acute adrenal crisis. ance to cortisol. The American College of'Critical Care Medicine
In the nonmedical literature. the term ''adrenal Fatigue" recommends considering this diagnosis in patients with hypo­
has been used to describe a constellation of symptoms. tension who have responded insufficiently to fluids and vaso­
including difficulty sleeping. Fatigue. and salt and sugar pressor therapy. A maximum increase in serum cortisol of
craving. hypothetically from long-term emotional or physi­ 9 pg/dL (248.4 nmol/L) or less following the administration of
cal stress having a deleterious effect on the adrenal glands , synthetic ACTH has been associated with increased mortality
resulting in a simultaneous excess and deficiency of cortisol . from septic shock: however, results of testing do predict benefit
However. there is no scientific evidence to support this from glucocorticoid therapy. In the setting of' critical illness,
claim. and the term "adrenal Fatigue" should not be used. both CBG and albumin concentrations decrease resulting in
Proponents of adrenal fatigue prescribe synthetic glucocor­ lower total cortisol . Free cortisol levels. either directly measured
ticoids and supplements containing adrenal, pituitary, or or calculated based on total cortisol and CBC, may be more reli­
hypothalamic extracts that can cause iatrogenic CS. as well able in critically ill patients with hypoalbuminemia. It is not
as mineralocorticoid supplements that can lead to hyperten­ known iffree cortisol levels provide useful prognostic infor111a­
sion. Patients should receive appropriate evaluation for their tion. The ad111inistration ofglucocorticoids has not been shown
symptoms and be educated to avoid taking hormonal to benefit critically ill patients who do not have shock. and the

38
Disorders of the Adrenal Glands

Cl
CONT.
results of placebo-controlled randomized trials in patients vvith
septic shock are conflicting. Further research is needed to clar­
be undertaken promptly. An algorithm for the evaluation and
follow up of an adrenal mass is shown in Figure 7.
ify if there is a population of critically ill patients who can objec­ The risk of malignancy varies according to size. OnJy 2%
tively benefit from glucocorticoid therapy. CJ of adrenal masses smaller than 4 cm are cancerous; however,
25'Yo of those larger than 6 cm are malignant. An adrenaJ
mass·s risk of malignancy can be clarified based on its appear­
Adrenal Masses ance on CT or MRI (see Table 19 for the typical radiographic

Cl Incidentally Noted Adrenal Masses


Adrenal masses are often discovered incidentally when
features of adrenal masses).
Adrenal metastases account for about half of adrenal
abdominal imaging is performed for another reason. These masses in patients with known nonadrenal malignancies.
adrenal incidentalomas are seen on 4% of all CT scans and 7% Cancers that metastasize to the adrenal glands include lym­
of those performed in patients 70 years of age and older. The phomas. carcinomas. and melanomas. Percutaneous biopsy is
differential diagnosis includes benign and malignant neoplasia indicated to confirm the diagnosis of adrenal metastasis: how­
of adrenal cortex or medulla. adrenal cysts. adrenal hyperpla­ ever. this should never be performed prior to ruling out pheo­
sia. metastatic tumors of nonadrenal origin, and infections chrornocytorna biochemically. Biopsy is not recommended
and infiltrative disorders. The most common cause of an adre­ when adrenocortical carcinoma (ACC) is suspected because it
nal mass is an adrenal adenoma. and adrenal metastasis is the cannot reliably distinguish benign from malignant adrenocor­
neJ(t most common. The two main goals in the evaluation of an tical neoplasia. The evaluation and management of ACC are
incidentally noted ad.renal mass are to identify adrenal masses covered in the Adrenocortical Carcinoma section.
that are likely to be malignant and those that are associated One quarter of incidentally noted adrenal masses autono­
wi.th hormonal hypersecretion so that targeted treatment can mously secrete hormones (cortisol 6%-10%; catecholamines

Incidentally Noted - Imaging Phenotypeb


Adrenal Mass

+ Favors Benign:
• Size <4 cm
Suspicious:
• Size >4 cm

Imaging Phenotype • Density <1 O HU' • Density >10 HU'


� • Contrast washout • Contrast washout I-- Indications for Adrenalectomy
>50% (10 min)' <50% (10 min)'
Suspicious imaging
Hormonal Evaluation Growth >1 cm/year

+
Functioning tumors:
Tests for Hormone Excessd
• Pheochromocytoma
• Aldosterone-producing adrenal tumor
Follow Up• Cortisol: Aldosterone:
• Subclinical CS with complication(s)9
• Who: All patients • Who: HTN or -.vK+
• Repeat CT or MRI
• Test: LDST • Test: PRA/PAC
at 3-6 months then
annually for 1-2
years
• Repeat hormonal Catecholamines: Androgens:
evaluation annually • Who: All patients • Who: If suspected1
for 4 years
• Test: Plasma or • Test: DHEAS,
urine metanephrines, testosterone,
urine catecholamines• androstenedione

FIGURE 7. Algorithm for the initial diagnostic evaluation and follow up of an incidentally noted adrenal mass. CS= Cushing syndrome; DHEAS= dehydroepiandroster­
one sulfate; HTN = hypertension; HU= Hounsfield units; K= potassium; LOST= low-dose (1-mg) dexamethasone suppression test; PAC= plasma aldosterone concentra­
tion; PRA = plasma renin activity.
aRepeat imaging c1nd hormone testing are indicated for adrenal masses not meeting crlterio for surgery ot initiol diagnosis.
hRefer to Tobie 19 for more CT and MRI findings. If imaging is suspicious in a patient with known malignancy, biopsy should be considered to confirm adrenal metastasis after screening for pheochromocytoma is completed.

ca scan findings.
dPositive screening tests usually require further biochemical evaluation to confirm the diagnosis {see text).

eMeasure plasma metanephrines if radiographic appearance is typical for a pheochromocytoma; otherwise measure 24·hour urine metanephrines and catecholamines.
1Hormonal evaluation for an androgen.producing adrenal tumor is indicated only if clinically suspected based on the presence of hirsutism, virilization, or menstrual irreguloritles in women.

9Adrenalectomy is considered for confirmed cases of subclinical CS associated with recent onset of diabetes, hypertension, obesity, or low bone mass.

39
Disorders of the Thyroid Gland

[::J mon; however, the majority or patients have subclinical dis­


5%; aldosterone 1%). Excess cortisol secretion is most com­ Adrenocortical Carcinoma
ACC is a rare malignancy affecting 0.5 to 2 persons per million
CrlNT.
ease without classic stigmata of CS. Despite this, important per year that is often associated with the excessive production
complications may be seen. including osteoporosis. hyperten­ of adrenal hormones. Patients with ACC most frequently pre­
sion. diabetes mellitus. and cardiovascular events. The LOST is sent with signs and symptoms related to hormonal excess.
the initial screening test of choice. A serum cortisol value They may also experience symptoms related to local tumor
greater than 5 µg/dL (138 nmol/L) is considered positive; how­ growth (abdominal fullness, nausea, or back pain) or metasta­
ever, some advocate using a cut-off ofl.8 µg/dL (49.7 nmol/L) sis. ACC is sometimes detected incidentally when abdominal
to increase diagnostic sensitivity if CS is suggested by history imaging is performed for another reason (see Incidentally
or physical examination. Because the specificity of the LOST is Noted Adrenal Masses).
only approximately 90% . the diagnosis of subclinical CS Autonomous secretion of adrenal hormones or their bio­
should be confirmed with additional testing. For a review. see logically inactive precursors is seen in more than 80% of
the Cushing Syndrome section. patients with ACC (cortisol 50%; multiple hormones 20%;
Aldosteronomas are usually smaller than 2 cm. Case androgens 5% to 10%; aldosterone rarely). The pathologic
detection for PA is performed in all patients with hypertension diagnosis of ACC is challenging, such that with low-risk
or those on antihypertensive medications. Testing for autono­ pathology but large tumor size or concerning imaging find­
mous secretion of adrenal androgens is performed if clinically ings (see Table 19) close interval radiographic follow up is
suspected following careful history and physical examination. needed after surgery.
All patients with an incidental adrenal mass should be tested The prognosis of ACC is very poor with an overall mortal­
for pheochromocytoma. Measurement of 24-hour urine ity rate of 67% to 94%. Management depends on the extent of
metanephrines and catecholamines is the preferred first test in disease at presentation. Open surgical resection is first-line
most asymptomatic patients. due to the lower incidence of treatment for early disease. Adjuvant radiotherapy to the
false-positive test results. However, if the radiographic appear­ tumor bed is used when resection is incomplete. Adjuvant
ance of the adrenal mass is suspicious for a pheochromocy­ medical therapy with mitotane, an adrenolytic drug, is recom­
toma (see Table 19) or the patient is symptomatic . then plasma mended for patients with known or suspected residual or
free metanephrines should be measured (see Primary metastatic disease. Cytotoxic chemotherapy has poor efficacy.
Hypera ldosteronism. Androgen-Producing AdrenaI Tu rnors. In addition to mitotane, inhibitors of adrenal steroidogenesis
and Pheochromocytomas and Paragangliomas). (metyrapone, ketoconazole, and etomidate) are used to treat
Adrenal masses that are larger than 4 cm. those with wor­ CS, if present. Surgery for metastatic ACC is indicated if symp­
risome radiographic findings. and pheochromocytomas should toms related to hormonal hypersecretion cannot be controlled
be removed surgically. Surgery is also indicated for unilateral with medical therapy alone. Percutaneous radiofrequency
aldosteronomas and is considered for patients with subclinical ablation may also be used to treat unresectable primary
CS associated with the recent onset of diabetes, hypertension. tumors or metastases when needed.
obesity. or low bone mass. For nonfunctioning adrenal masses.
KEY POINT
if imaging favors a benign lesion, repeat radiographic evalua­
tion is recommended in 3 to 6 months, and then annually for 1 • Adrenocortical carcinoma is marked by signs and
to 2 years. Adenomas usually will not grow more than l cm over symptoms related to hormonal excess as well as symp­
12 months. More rapid growth should prompt adrenalectomy. toms related to local tumor growth (abdominal fullness,
Screening for hormonal hypersecretion is repeated annually nausea, or back pain) or metastasis.
for 4 years, as in the rare instance that the mass becomes func­
tional. it is likely to occur in the first 4 years following its dis­
covery. A recent study documented subclinical CS on follow-up
testing in approximately 8% of patients who were thought to
Disorders of the
have nonfunctioning adenomas at initial screening. c:J Thyroid Gland
KEY POINTS Thyroid Anatomy and Physiology
• The two main goals of evaluation of adrenal inciden­ In healthy adults in the United States, each thyroid lobe nor­
talomas are to identify adrenal masses that are likely to mally measures up to 5 cm in length, 2 cm in width, and 2 cm
be malignant and those that are associated with hormo­ in depth; the entire gland weighs 10 to 20 grams. The isthmus,
nal hypersecretion so that targeted treatment can be a thin band of thyroid tissue that connects the two lobes, is 1
undertaken promptly. to 4 mm in thickness and is typically not palpable. Diffuse
• Adrenal masses tl1at are larger than 4 cm, those with thyroid disorders, such as lymphocytic thyroiditis, may result
worrisome radiographic findings, and pheochromocy­ in enlargement of the isthmus to 5 mm or more, which may be
tomas should be removed surgically. palpable and give the clinician the false sense that the entire
thyroid is enlarged.

40
Disorders of the Thyroid Gland

There are two forms of active thyroid hormone: thyrox­ range. During pregnancy, the range shifts lower and varies by
ine (T) and triiodothyronine (T/ Iodine is necessary for the trimester (see Thyroid Function and Disease in Pregnancy).
formation of thyroid hormone. Deficiency may result in The second important exception is in the elderly. With aging,
hypothyroidism. The hypothalamic-pituitary-thyroid axis the reference range shifts higher; the upper limit of normal
responds to the subsequent hormone deficiency by increasing extends to 7 µU/mL (7 mU/L) in patients older than 70 years.
thyroid-stimulating hormone (TSH) secretion, resulting in The third exception is in patients with known pituitary dys­
development of a goiter. Iodine is typically obtained through function or a risk of pituitary dysfunction (history of cranial
diet; it is present in seafood, dairy products, and iodized salt. irradiation, pituitary surgery, or massive head trauma).
Although iodine deficiency is a worldwide health problem, it If the serum TSH level is frankly abnormal, additional
is relatively rare in the United States with the incorporation of evaluation of thyroid function should be considered to deter­
iodine into salt. mine the extent of the dysfunction. This is accomplished by
Thyroid hormone production is controlled by two main measuring T., when the TSH is elevated and by measuring T,1
forces: secretion of TSH (thyrotropin) and regulation of and Ti when the TSH is suppressed.
peripheral conversion of T. 1 to T3 • TSH release from the anterior When indicated by an abnormal TSH, the circulating level
pituitary is stimulated by decreases in concentrations of serum of thyroid hormone should be assessed using total or free T4
T4 and T3 and secretion of thyrotropin-releasing hormone levels. Total T. 1 is a reflection of the bound and unbound frac­
(TRH) from the hypothalamus. The T3 and T." in turn, decrease tions of the hormone and is a reasonable method of assessing
secretion of TSH from the anterior pituitary as part of a nega­ overall thyroid hormone levels in most patients. However, in
tive feedback loop. Additionally, T3 inhibits further secretion of patients with disorders of protein metabolism, such as kidney
TRH from the hypothalamus. or liver disease, measurement of free T,1 ensures a more accu­
Although the thyroid gland produces both T3 and T.I' the rate representation of the hormone concentrations.
ratio of T4 to T3 secretion is nearly 20:1, with most T3 (80%) Additionally, conditions that raise serum total protein level,
resulting from 5'-deiodination of T,1 in peripheral tissues. such as in patients taking estrogen or during pregnancy, may
The vast majority of both hormones are bound to circulat­ result in a higher total T., concentration, and measuring the
ing proteins, including thyroxine-binding globulin, trans­ free T. 1 is indicated. The same rules apply to T3 as to T,1 regard­
thyretin, albumin, and lipoproteins. The function of these ing protein levels, but free T3 has a very short half-life and
proteins is to increase the circulating pool of hormone by levels fluctuate more and are less reliable; therefore, it is con­
delaying clearance and maintaining a reservoir of hormone troversial whether total or free T3 should be measured in
available for use. Only a small percentage of total circulating patients at risk for protein abnormalities.
thyroid hormone is free (unbound); this fraction is readily Measurement of serum T3 is necessary if the patient has a
available for cellular uptake and determines the biologic activ­ suppressed TSH level because, in some patients with thyro­
ity of the hormone. toxicosis, T3 may be preferentially secreted over T,1 (T3 toxico­
sis). In patients with an elevated TSH level, indicating
KEY POINT
hypothyroidism, measurement of serum T 3 is not helpful
• Thyroid hormone production is controlled by two main because it will be maintained in the normal range even in
forces: secretion of thyroid-stimulating hormone (thy­ those with significant disease.
rotropin) and regulation of peripheral conversion of Thyroid autoantibody measurement may be helpful
thyroxine (T4) to triiodothyronine (T). under certain clinical circumstances. In patients with a
personal history of autoimmune disease (such as type 1
diabetes mellitus, systemic lupus erythematosus, or celiac
Evaluation of Thyroid Function disease) or a strong family history of thyroid dysfunction,
In patients with an intact hypothalamic-pituitary axis, the measuring thyroid autoantibodies may indicate the cause of
initial laboratory test of thyroid activity is TSH measurement, the thyroid dysfunction or whether a patient is at risk for
which is exquisitely sensitive for detection of disorders of thy­ developing thyroid autoimmune disease if the TSH is nor­
roid dysfunction. In patients for whom there is a high suspi­ mal. There is no clinical indication for serial measurement
cion of thyroid or pituitary dysfunction, a concomitant thyroid of thyroid antibody titers to determine the need for or to
hormone (T4) level should be assessed with the TSH level to guide therapy. There are three forms of thyroid autoanti­
evaluate for central hypothyroidism. The TSH level may reflect bodies: thyroid peroxidase (TPO), thyrotropin receptor
hypofunction (high TSH). hyperfunction (low TSH), or a nor­ (TRAb), and thyroglobulin (TgAb). Elevated titers of TPO
mal range. antibodies are associated with autoimmune hypothyroid­
The normal reference range for TSH is variable among ism, or Hashimoto disease. Patients with TPO antibodies
laboratories but is generally between 0.5 and 5 µU/mL (0.5- and normal thyroid function tests are at an increased risk of
5 mU/L), and determinations of normal TSH levels should be developing overt thyroid failure (2%-4 "lo per year).
made based on the reference range of the laboratory being Thyrotropin receptor antibodies (TRAb) are divided into
used. There are three very important exceptions to this general three types: blocking (also called thyrotropin-binding

41
Disorders of the Thyroid Gland

inhibitory immunoglobulins), stimulating (also called KEY PO I NT S (continued)


thyroid-stimulating immunoglobulins or TS]), and neutral.
• If central hypothyroidism is suspected, a concomitant
The presence of TS! autoantibodies is responsible for the
thyroid hormone (thyroxine [T4]) level should be
development of Graves disease. TS! autoantibodies should
assessed in conjunction with the thyroid-stimulating
be measured if autoimmune hyperthyroidism is suspected.
hormone level.
Thyroglobulin (Tg) is a glycoprotein located within the col­
• If the thyroid-stimulating hormone level is frankly HVC
loid on which thyroid hormones are synthesized and stored.
Serum Tg and TgAb measurements are used to monitor abnormal, additional evaluation of thyroid function
patients with thyroid cancer; serum Tg is a highly sensitive should be considered to determine the extent of the
dysfunction; measure thyroxine (T4) when the thyroid­
and specific marker of residual thyroid tissue. After total
thyroidectomy and radioactive iodine ablation, the persis­ stimulating hormone is elevated and measure both
thyroxine (T,) and triiodothyronine (T) when the
tence of a detectable serum Tg is a possible indicator of
residual or recurrent disease. Thyroglobulin antibodies are thyroid-stimulating hormone is suppressed.
present in up to 30% of patients. Their presence in the • There is no clinical indication for serial measurement of HVC
serum is only significant in patients with thyroid cancer, as thyroid antibody titers to determine the need for or to
they can falsely lower the serum Tg. Therefore, TgAb titers guide therapy except to monitor for residual disease in
should always be assessed simultaneously with the Tg; if patients treated for thyroid cancer.
antibodies are present. the Tg level may not be reliable. • Radioactive iodine uptake is a measure of iodine uptake HVC
Calcitonin, secreted by the C cells of the thyroid, is most by the thyroid over 24 hours; it is used to evaluate the
frequently used as a tumor marker in patients with a history cause of hyperthyroidism and is not indicated in
of medullary thyroid carcinoma. Serum calcitonin levels can patients with normal or elevated thyroid-stimulating
help increase the sensitivity of detection of medullary thyroid hormone levels.
carcinoma when used in conjunction with tine-needle aspira­
tion (FNA). However, it is not recommended as a screening test
in all patients with thyroid nodules because it lacks the requi­
site specificity. Instead. measurement of calcitonin should be Functional Thyroid Disorders
considered if a patient with thyroid nodular disease has a his­ Thyrotoxicosis
tory of hyperparathyroidism or a family history of medullary Evaluation
thyroid carcinoma or multiple endocrine neoplasia type 2, or Thyrotoxicosis is a term used to describe thyroid hormone
if there is clinical suspicion for these disorders. excess from all sources. whereas hyperthyroidism is the more
Radioactive iodine uptake (RAIU) is a measure of iodine specific term to describe thyroid gland overactivity.
uptake by the thyroid over a pre-specified time frame, typi­ Thyrotoxicosis may result from endogenous thyroid disorders,
cally 24 hours. RAJU is used to evaluate the cause of hyperthy­ pituitary tumors, and exogenous levothyroxine. The most
roidism; it is not indicated in patients with normal or elevated common causes of hyperthyroidism are Graves disease and
TSH levels. The degree of uptake is useful for distinguishing toxic adenoma(s).
the various causes of hyperthyroidism. RAJU percentage is The symptoms of thyrotoxicosis include heat intolerance,
typically very high in patients with Graves disease (diffusely palpitations, dyspnea, tremulousness, menstrual irregulari­
increased uptake) and only moderately elevated in those with ties, hyperdefecation, weight loss, increased appetite, proxi­
toxic multinodular goiter (patchy uptake in areas of nodules mal muscle weakness, fatigue, insomnia, and mood
with relative suppression of normal tissue). In contrast, the disturbances. The severity of symptoms may not correlate with
RAJU is very low in those with thyroiditis or exposure to exog­ the level of thyroid hormone derangement. In older patients,
enous thyroid hormone. The presence of a ·'cold" nodule on many of the classic symptoms of thyroid hormone excess may
isotope scanning is an indication for ultrasonography to help be absent, and the only presenting symptom may be atrial
determine if FNA is indicated. RAIU is contraindicated during fibrillation or heart failure; this is known as apathetic hyper­
pregnancy and while breastfeeding. thyroidism.
KEY POINTS
The initial evaluation based on clinical signs and/or
symptoms of thyrotoxicosis should be measurement of
• The initial laboratory test of thyroid activity is thyroid­ serum TSH alone, followed by measurement of T 4 and T 3
stimulating hormone measurement; in patients with an levels if TSH is suppressed. The typical pattern of hyperthy­
intact anterior pituitary, measurement of thyroid­
roidism is TSH suppression with an elevated T.1 and/or T 3 • A
stimulating hormone is exquisitely sensitive for normal serum TSH in the setting of an elevated T 1 and/or T 3
detection of disorders of thyroid hypofunction (high concentration suggests the presence of a TSH-secreting pitu­
thyroid-stimulating hormone) and hyperfunction (low itary adenoma; these tumors are extremely rare and are
thyroid-stimulating hormone). managed differently from other causes of thyrotoxicosis (see
(Continued)
later discussion).

42
Disorders of the Thyroid Gland

KEY POINT serious illness. they should be assessed promptly for an


adverse reaction to the medication.
• The typical pattern of laboratory studies in hyperthy­
The goal of radioactive iodine ablation is to render the
roidism is thyroid-stimulating hormone suppression
patient hypothyroid, which can typically be accomplished in
with an elevated thyroxine (T4) and/or triiodothyronine
90% of patients with the first treatment. Although a minority
(T3) level.
of patients may develop acute anterior neck pain from radfa­
tion thyroiditis, radioactive iodine ablation is typically well
c:::J Management tolerated. It may take several months for the development of
Although the specific intervention usecl is usually determined hypothyroidism, so it is important to monitor thyroid function
by the unclerlying cause and patient and physician preference. tests monthly after therapy. In a patient with severe thyrotoxi­
control of the thyrotoxic state may be clchievecl by one of three cosis, radioactive iodine may provide additional substrate to
treatment modalities: thionamides. radioactive iodine abla­ the hyperfunctioning gland. resulting in exacerbation of the
tion, or surgery. hyperthyroid state. Consequently. it may be reasonable to initi­
Rapid control of adrenergic symptoms with a �-blocker is ate a thionamide prior to ablation to lower the thyroid hor­
indicated in most patients with thyrotoxicosis. Although pro­ mone levels.
pranolol is frequently used for its added cf'fect of inhibition of Surgery is rarely first-line therapy. given the inherent
peripheral conversion or T, to T,. cardioselective �-blockers. risks with any surgery. Patients in whom control cannot be
such as atenolol, are preferred owing to the additional benefits achievecl with thionamides ancl those who are not comfortable
of clecreasecl central nervous system sicle effects and improved with radioiodine therapy are typically referred for surgery.
adherence with once-daily dosing. Because or the increased intrathyroidal vascularity. the proce­
Methimazole and propylthiouracil (PTU) are the two dure can be technically more difTicult than a typical thyroid­
thionamicles available in the Unitecl States. Methimazole is the ectomy. Aclditionally. restoration of the euthyroid state before
preferred agent because it has a higher intrathyroidal retention surgery with thionamides is important to improve hemody­
(potency). a preferable closing regimen (typically once daily). namics during general anesthesia and decrease the patient's
and a reduced side-effect profile. Antithyroid medications risk of thyroid storm. c:J
reduce T 1 and T, levels within a few clays or initiation. but the
KEY POINT
full effect may take several weeks. Normalization or a previ­
ously suppressed TSH level may take several months. It is • Control of hyperthyroidism may be achieved by one of
critical. therefore. to monitorT 1 and T:1 levels during treatment three treatment modalities: thionamides, radioactive
of hyperthyroidism because the TSH may not be an accurate iodine ablation, or surgery; modality choice depends on
reflection of the thyroidal status. the underlying cause and patient preference.
Thionamides may be used to prepare patients for thy­
roidectomy or radioiodine treatment. or they may be used as Graves Disease
the primary therapy. Thionamicles may be used for 1 to 2 years Graves disease is a multiorgan system autoimmune disorder
in patients with Graves disease in the hope of achieving that can aftect the thyroid, eyes, and skin. It is frequently seen
remission: more definitive therapy with radioactive iodine or in women between the ages of20 and SO years and is the most
surgery may then be sought after that timeframe if hyperthy­ common cause of hyperthyroidism in the United States.
roidism persists. Although thionamides are generally well Antibodies against the TSH receptor (TS! or TRAb) stimulate
tolerated, it is important to be familiar with their side-effect autonomous production of T, 1 and T 3 • Patients frequently
profile. The most common reaction to antithyroid medica­ report a family history of Graves disease, Hashimoto thyroidi­
tions is a rash, seen in up to 10% of patients. Additionally. PTU tis, or other autoimmune conditions.
may cause elevations of aminotransferase levels. Rare cases of On physical examination, patients have elevated sys­
fatal hepatotoxicity have been described with PTU. Therefore. tolic blood pressure with a widened pulse pressure, tachy­
its use is reserved for patients who cannot tolerate methima­ cardia, and a diffusely enlarged thyroid. Further inspection
zole and during the first trimester or pregnancy, when methi­ of the thyroid may reveal a bruit. Careful examination of the
mazole has a possible teratogenic ef'fect. A cholestatic pattern skin may reveal pretibial myxedema, an infiltrative process
of liver test abnormalities may also be seen with methimazole. that is typically patchy with a peau d'orange appearance
but it is typically temporary and milder than that seen with to the skin.
PTU. Both drugs may be associated with reversible agranulo­ Diagnosis of Graves disease is made clinicaLly in most
cytosis in approximately 1 in 500 patients. Baseline liver instances, and measurement of TS! antibodies is reserved for
chemistry studies and complete blood count with differential patients who are not markedly thyrotoxic on examination and
are recommended before initiation of antithyroid medica­ do not have a classic smooth, rubbery, diffuse goiter. In those
tions. with serial monitoring of the complete blood count patients, TS! antibodies may help determine the cause of the
during therapy. If patients taking a thionamicle develop a hyperthyroidism. RA!U and scan will show markedly
fever. rash, severe sore throat, jaundice. or other symptoms of increased uptake with diffuse activity on the scan.

43
Disorders of the Thyroid Gland

If ophthalmopathy is present, the patient may exhibit lid should be withheld for S to 7 days before the administration of
retraction (lid lag), whereby contraction of the levator palpe­ radioactive iodine therapy. If a patient has a particularly large
brae muscles of the eyelids results in immobility of the upper goiter with compressive symptoms or if there is concern for
eyelid with downward rotation of the eye. Additionally, malignancy, surgery is recommended as first-line therapy.
patients may have proptosis, scleral injection, and periorbital KEY POINT
edema.
Because thionamide drugs also have an immunomodula­ • Radioactive iodine ablation or surgery is the most com­
tory effect that reduces autoantibody titers, antithyroid drugs mon treatment for toxic thyroid nodules; indications for
are often first-line treatment for Graves disease. Up to SO% of surgery include a large goiter with compression symp­
patients may go into remission within 24 months, and some toms or concern for malignancy.
may maintain a euthyroid state without further therapy after an Destructive Thyroiditis
initial treatment with thionamides. If the patient does not go Thyroiditis is a selt�Limited inflammatory condition of the thyroid
into remission or if disease recurs, definitive therapy with radi­ resulting in the release of preformed thyroid honnone into the
oactive iodine ablation or surgery is recommended. However, in circulation. The dmation of the thyrotoxic phase is typically 2 to
patients with Graves ophthalmopathy, there is an acute escala­ 6 weeks, during which patients may exhibit classic symptoms of
tion of thyroid autoantibody titers following radioiodine ther­ thyrotoxicosis. Following the release of preformed hormone, the
apy that may exacerbate ocular symptoms. Such patients may
damaged thyroid ceases production ofT3 andT4 dming the recov­
be better treated with thionamjdes and/or surgery. ery phase; consequently, administration of thionamides will not
be effective in treating elevated horn1one levels. The patient may
Toxic Adenoma and Multinodular Goiter
then become clinically hypothyroid, a condition that may require
Activating mutations in the TSH receptor gene are responsible
tempora1y levothyroxine therapy. The length of the hypothyroid
for the autonomous production of thyroid hormone in a toxic
phase can va1y but classically is 6 to 12 weeks.
nodule (adenoma) or in multiple hyperfunctioning nodules in
There are two categories of thyroiditis: painful and painiess.
a toxic multinodular goiter. Because of this loss of normal
The causes of painful thyroiditis are inflammatory (de Quervain
regulation of thyroid hormone production, patients are at risk
or subacute granulomatous thyroiditis), infectious (suppurative),
for developing acute thyrotoxicosis when exposed to iodine
and radiation-induced. The pain, typically only present during
excess, particularly after a contrast load for medical testing
the thyrotoxic phase, can be quite intense. Treatment is aimed at
(Jod-Basedow phenomenon), such as in cardiac catheteriza­
controlling inflammation with NSAIDs or systemic glucocorti­
tion and contrast-enhanced CT scans. Although patients with
coids if severe. Subacute thyroiditis is the most common form
a toxic adenoma or multinodular goiter may exhibit the typi­
and is presumably caused by a postviral inflammatory process;
cal symptoms of thyrotoxicosis, they can be relatively asymp­
many patients report a recent history of upper respiratmy illness
tomatic. On physical examination, a nodule may be palpable
preceding the thyroiditis. Radiation thyroiditis may occur s to
or there may be a diffusely enlarged goiter with a nodular
10 days after treatment with radioactive iodine. This may be
contour but no discrete palpable nodules.
lf a patient is suspected of having a toxic nodule, thyroid associated with transient exacerbation of the hyperthyroidism.
The accompanying pain is usually mild and lasts for up to 1 week.
scintigraphy should be performed to determine if the nodule
Infectious thyroiditis is rare but may be seen in an immunocom­
is autonomous. The thyroid uptake scan will reveal increased
promised patient; the most common causative organisms are
activity in the "hot" nodule with relative suppression of the
Staphylococcus and Streptococcus species.
remaining thyroid tissue. These results should then be corre­
Painless thyroiditis is more commonly seen than painful
lated with the ultrasonographic findings to determine if any
thyroiditis and has several causes, including postpartum thy­
additional nodules exist, which will require further investiga­
tion with FNA. roiditis, silent thyroiditis, and drug-induced thyroiditis.
Radioactive iodine ablation or surgery is the most com­ Postpartum thyroiditis may occur up to 1 year after delivery;
mon treatment for toxic nodules. Thionarnides can be used to the frequency is variably reported but may occur in up to 10%
decrease hormone production in the short term, but unlike of pregnancies. The presence of TPO antibodies is nearly uni­
Graves disease, this condition has no chance of spontaneous versal, and the likelihood of subsequent permanent hypothy­
remission and would require lifelong medical therapy, which roidism is ve1y !ugh. Thyroiditis is also likely to recur in later
is not recommended. Radioiodine therapy will ideally ablate pregnancies.
only the autonomous areas. In elderly patients, those with KEY POINT
coronary disease, those who are highly symptomatic, and • Thyroiditis is a self-limited inflammatory condition of
those with severe thyrotoxicosis, thionamides are recom­ the thyroid resulting in the release of preformed thyroid
mended to normalize thyroid hormone levels prior to radioac­ hormone into the circulation; the duration of the thyro­
tive iodine; this is done to avoid exacerbation of the toxic phase is typically 2 to 6 weeks, which is followed
thyrotoxicosis due to release of preformed hormone from the by a hypothyroid phase typically lasting 6 to 12 weeks.
gland acutely after radioactive iodine ablation. Thionamides

44
Disorders of the Thyroid Gland

Central Hyperthyroidism trauma, pituitary surgery, or cranial radiation can cause cen­
TSH-secreting pituitary adenomas are extremely rare. In this tral hypothyroidism.
condition, serumTSH is detectable or normal in the setting of The clinical manifestations of hypothyroidism include
an elevatedT4 and/or T3 concentration. A dedicated pituitary fatigue, cold intolerance, constipation, heavy menses, weight
MRI will demonstrate an adenoma.Treatment should focus on gain, impaired concentration, dry skin, edema, depression,
removal of the pituitary tumor; thyroid-targeted therapy is mood changes, muscle cramps, myalgia, and reduced fertility.
ineffective (see Disorders of the Pituitary Gland). The physical examination findings may include reduction in
basal temperature, diastolic hypertension, bradycardia, dry
Subclinical Hyperthyroidism skin, brittle hair, hoarseness, delayed relaxation phase of the
Subclinical hyperthyroidism is a laboratory-based diagnosis, deep tendon reflexes, and an enlarged thyroid.
defined as the presence of a suppressedTSH level with normal An elevated serum TSH level indicates the diagnosis of
T 3 andT4 levels. Repeat assessment of thyroid function should primary hypothyroidism. ln patients with an elevated TSH
be performed 6 to 12 weeks after the initial tests, as the values that is less than 10 µU/mL (10 mU/L), a low serumT 4 measure­
will normalize in up to 30% of patients. Symptoms of thyro­ ment is helpful, as a frankly low value indicates that thyroid
toxicosis are typically mild; most patients are asymptomatic. hormone replacement is necessary.The presence ofTPO anti­
Which patients will benefit most from normalization of bodies suggests that Hashimoto thyroiditis is the underlying
theTSH level is not universally agreed on, but consensus opin­ cause.Thyroid imaging is not indicated unless there is concern
ion recommends treatment for patients with aTSH level below for a nodule on physical examination.
0.1 µU/mL (0.1 mU/L).The benefits of treatment for asympto­ KEY POINTS
matic patients with aTSH level between 0.1 µU/mL (0.1 mU/L)
• An elevated serum thyroid-stimulating hormone level HVC
and the lower limit of the normal reference range are less clear.
Emerging data suggest that chronic subclinical hyperthyroid­ indicates the diagnosis of primary hypothyroidism; thy-
ism has a negative effect on cardiac function, the central nerv­ roid imaging is not indicated unless there is concern for
ous system, and bone mass. The risk of atrial fibrillation is a nodule on physical examination.
significantly increased when theTSH level is below 0.3 µU/mL • The most frequent cause of primary hypothyroidism is
(0.3 mU/L), so patients over the age of 65 years and those with Hashimoto thyroiditis (chronic lymphocytic thyroidi­
a history of coronary artery disease or tachyarrhythmias, as tis); the presence ofTPO antibodies is suggestive of
well as patients with osteoporosis, may benefit from normali­ Hashimoto thyroiditis.
zation of theTSH level. Radioiodine is the preferred treatment,
but often the gland does not have sufficient iodine avidity and Management
methimazole must be used. In patients with aTSH level greater than 10 µU/mL (10 mU/L),
daily thyroid hormone replacement is recommended.
KEY POINTS
Thyroid hormone replacement with levothyroxine alone
HVC • In patients with subclinical hyperthyroidism, repeat is recommended. The goal of thyroid hormone replacement
assessment of thyroid function should be performed 6 therapy is normalization of the TSH. The starting dose can be
to 12 weeks after the initial tests, as the values will nor­ weight-based at 1.67 µg/kg/d, using ideal body weight. In
maUze in up to 30% of patients. patients with prevalent cardiac disease, tachyarrhythmias, or
• Treatment for subclinical hyperthyroidism is recom­ multiple comorbidities, or in those who are older than
mended when the thyroid-stimulating hormone level is 65 years, the dose should not be based on weight but rather
less than 0.1 µU/mL (0.1 mU/L). should be 25 to SO µg/d.The dose should be titrated based on
TSH levels measured 6 to 8 weeks after any dose change. To
improve gastrointestinal absorption, levothyroxine should be
Thyroid Hormone Deficiency taken on an empty stomach, 1 hour before or 2 to 3 hours after
Hypothyroidism ingestion of food or medications that would interfere with
Evaluation absorption, such as calcium- or iron-containing supplements.
Hypothyroidism refers to low circulating thyroid hormone Patients with celiac disease may require higher levothyroxine
levels. Hypothyroidism is more prevalent in women than men doses because of impaired absorption.
(2% versus 0.2%) and in those with other autoimmune dis­ There has been significant debate regarding the need for
eases. The most frequent cause is Hashimoto thyroiditis, also supplementation of T3 Oiothyronine) in patients with hypo­
known as chronic lymphocytic thyroiditis. Iatrogenic causes thyroidism. The short half-life of T3 triggers acute spikes in
include surgery, radioiodine therapy, and external beam radia­ serum T 3 levels, which are of significant concern for elderly
tion therapy to the neck. Hypothyroidism may also be medica­ patients or those with preexisting cardiac issues. Additionally,
tion induced; the most common causative agents include numerous studies have failed to show a clear benefit of aT/T3
lithium, amiodarone, interferons, interleukin-2, and tyrosine combination over T4 alone; therefore, this is not generally
kinase inhibitors. Rarely, pituitary tumors, severe head recommended.

45
Disorders of the Thyroid Gland

KEY POINT effect). The result is a temporary reduction in circulating T3


and T4 levels with a minor rise in the TSH level; these changes
• In patients with a thyroid-stimulating hormone level
typically reverse within the first 3 months of treatment and
greater than 10 µU/mL (10 mUIL), daily thyroid hor­
require no intervention.
mone replacement is recommended and should be
Facultative effects, seen in up to 15% of patients, may
taken on an empty stomach; the dose should be titrated
cause either hypo- or hyperthyroidism. In areas of iodine suf­
based on thyroid-stimulating hormone levels measures
ficiency, hypothyroidism is the more common toxicity. Those
6 to 8 weeks after any dose change.
at highest risk are women with preexisting TPO antibody
positivity. Amiodarone-induced thyrotoxicosis (AIT) is more
Subclinical Hypothyroidism commonly seen in males and in those living in iodine-defi­
Subclinical hypothyroidism is defined as an elevated serum cient areas. Type 1 AIT is the result of exposure to excess iodine
TSH level with a normal T4 level. The potential causes are the and occurs in those with preexisting thyroid conditions, such
same as for overt hypothyroidism. Repeat measurement of as latent Graves disease or nodular goiter, in which the iodine
the TSH level is recommended, particularly in an asympto­ increases unregulated thyroid hormone production. Type 2
matic patient, as it will normalize in up to 30% of patients by AIT is the result of the cytotoxic effects of amiodarone on thy­
6 weeks. roid tissue, producing a clinical picture of painless thyroiditis,
Patients typically have mild or no symptoms of hypothy­ with abnormal release of thyroid hormone. The treatments
roidism. Subclinical hypothyroidism may be associated with differ with each type, but distinguishing between the two
several laboratory abnormalities including elevated total cho­ forms of AIT often can be challenging and may require the aid
lesterol, LDL cholesterol, and C-reactive protein levels. Large of an endocrinologist. The time to recovery of normal thyroid
studies suggest that these laboratory abnormalities translate function may be several months, even with prompt diagnosis
into an increased risk of atherosclerosis and cardiac events. and treatment. Discontinuation of amiodarone is typically
However, supplementation with levothyroxine has not been necessary, particularly in those patients with type 1 AIT.
shown to mitigate this risk. Treatment is generally recom­
mended for those with a TSH level greater than 10 µU/mL KEY POINTS

(10 mU/L), but levothyroxine treatment should be considered • In the majority of patients, amiodarone causes a tempo­
in patients who have positive TPO antibodies or a large goiter, rary reduction in circulating triiodothyronine (T) and
as these patients are at risk for progression to overt hypothy­ thyroxine (T) and levels with a minor rise in the thy­
roidism at rates of 3% to 8% per year. A goal TSH level less than roid-stimulating hormone that reverses within first
or equal to 2.5 µU/mL (2.5 mU/L) is recommended for women 3 months of treatment and requires no intervention.
with subclinical hypothyroidism and positive TPO antibody • In 15% of patients, amiodarone may cause either
status who are planning to conceive. hypo- or hyperthyroidism; those at highest risk for
KEY POINT amiodarone-induced hypothyroidism are women with
preexisting thyroid peroxidase antibody positivity.
H If( • In patients with suspected subclinical hypothyroidism,
repeat measurement of the thyroid-stimulating hor­
mone level is recommended, as it will normalize in up
to 30% of patients by 6 weeks. Thyroid Function and Disease
in Pregnancy
Drug-Induced Thyroid Dysfunction Significant changes in thyroid function occur during preg­
Various medications can affect thyroid function and are listed nancy; understanding the normal physiology during gestation
in Table 26 based on their mechanism of action. is critical for a correct interpretation of thyroid laboratory
Amiodarone may have a potentially toxic effect on the studies. Abnormalities of thyroid function can have a dramatic
thyroid. The iodine content of amiodarone is 37% by weight. It effect on the health of the mother and the fetus. A diagram of
is stored in fat, myocardium, liver, lung, and thyroid tissues, the physiologic changes in thyroid function during each tri­
with a half-life exceeding SO days. This long half-life, coupled mester is shown in Figure 8.
with the high iodine content, renders it a potentially toxic Increased estrogen levels cause a rise in thyroxine-bind­
compound to the thyroid. The two types of amiodarone thy­ ing globulin. To maintain a stable free T4 and T3 , thyroid hor­
roid toxicity are changes in thyroid function studies seen in all mone production is increased and TSH remains within the
patients (obligatory effects), and those seen in only a subset of normal reference range for the patient's trimester (see later
patients (facultative effects). discussion for trimester-specific ranges). Routine screening of
The obligatory eflects result from the increased circulat­ TSH is not indicated for every pregnant woman. TSH screening
ing iodine after initiation of the drug. Adaptation to the acute is indicated in women with a risk of thyroid gland dysfunc­
iodine excess causes a reduction in organification of iodine tion, including those already on thyroid hormone replacement
f
and reduced production of thyroid hormone (Wolf -Chaikoff therapy; those with autoimmune disorders, goiter, previous

46
Disorders of the Thyroid Gland

TABLE 26. Medications that Affect Thyroid Function


Mechanism of Action Drugs Comments

Decreased absorption or Calcium It is recommended that levothyroxine ingestion be separated from


enterohepatic circulation these medications by several hours
Proton pump inhibitors

Iron

Cholestyrarnine

Aluminum hydroxide
Soybean oil
Sucralfate
Psyllium

Increased metabolism of Phenytoin Higher levothyroxine doses may be required to maintain


levothyroxine levothyroxine in the normal range
Carbamazepine
Rifampin

Phenobarbital

Sertraline
T hyroiditis Amiodarone May cause hypo- or hyperthyroidism
Lithium

Interferon alfa

lnterleukin-2
Tyrosine kinase inhibitors

De novo development of Interferon alfa May develop Hashimoto thyroiditis, Graves disease, or painless
antithyroid antibodies thyroiditis
Inhibition ofTSH synthesis or Glucocorticoids
release
Dopamine
Dobutamine
Octreotide
Increased thyroxine-binding Estrogen False elevation of total T3, total T 4 levels; free T3,T 4 may be more
globulin accurate reflection of hormone levels
Tamoxifen

Methadone

Decreased thyroxine-binding Androgen therapy False lowering of total T 3, total T 4 levels; free T 3, T4 may be more
globulin accurate reflection of hormone levels
Glucocorticoids
Niacin

T3 = triiodothyronine; T4 = thyroxine; TSH = thyroid-stimulating hormone.

head/neck irradiation, previous thyroid surgery, known posi­ pregnancy, with adjustments in thyroid hormone replacement
tive TPO antibodies or positive TS! antibodies, or a strong fam­ closing as neeclecl to maintain the TSH within the trimester­
ily history of thyroid dysfunction; those who live in sµecific normal range. The largest dose escalations typically
iodine-deficient areas; or those older than 30 years. In patients occur in the first trimester, with more dose stability later in
on Ievothyroxine replacement, the dose of the medication may pregnancy.
need to be increased, on average by 30% to 50%, and µatients Diagnosing possible hyperthyroidism during pregnancy
should have their TSH level checked as soon as a pregnancy may be challenging because some physiologic changes during
test is µositive. gestation may overlap with symptoms of thyrotoxicosis, such
Fetal thyroid tissue is not functional until 10 to 12 weeks' as tachycardia, fatigue, and heat intolerance. Serum TSH and
gestation, necessitating maternal thyroid hormone transfer human chorionic gonadotropin have a common a-subunit,
through the placenta. Thyroid hormone deficiency can nega­ allowing cross-reactivity at the TSH receptor. Consequently,
tively affect fetal neurocognitive development. It is critical to TSH declines during the first trimester; the reference range
maintain a euthyroid state during pregnancy in these patients. shifts down to 0.03 to 2.5 µU/mL (0.03-2.5 mU/L). During the
TSH testing should be performed every 6 weeks throughout second and third trimesters the upper limit of TSH rises to

47
Disorders of the Thyroid Gland

perturbations of the hypothalamic-pituitary-thyroid (HPT) axis


that occur in up to 75'Y., of hospitalized patients.The underlying
cause or the critical illness may influence the pattern of thyroid
!'unction abnormalities. Drugs that are frequently used in criti­
cally ill patients can have a significant effect on the 1-1 PT axis (see
Table 26). The typical pattern is initially a lowT I level. followed
by a decline in theT 1 level. As the patient becomes more criti­
cally ill, theTSH level may also decline.creating a clinical pic­
ture that is difficult lo discern from central hypothyroidism.
Rarely. TSI-I can be elevated in ESS.
Two general guidelines are important in evaluating a criti­
cally ill patient. First. measurement of TSH alone should be
obtained only if' there is a high clinical suspicion of thyroid dys­
function. If' TSI-I is abnormal. the previously described recom­
mendations lor additional laboratory studies should be lollowed.
Ir theTSI-I is greater than 20µU/111L(20111 U/L)or is undetectable,
ESS is less likely to be the cause and overt thyroid dysrunction
should be strongly considered. If theTSI-I falls between these two
values.historical clues and examination f1ndings are very impor­
Week 10 20 30 40
tant for identifying patients with true thyroid dysfunction.
FIGURE 8. Thyroid function in pregnancy. HCG = human chorionic gonadotro­ Arter discharge from the hospital. thyroid function abnor-
pin; T4 = thyroxine; TBG = thyroid-binding globulin; TSH = thyroid-stimulating 111alities may persist for several weeks. The typical pattern is a
hormone.
111ildly elevated TSl-l level and slightly lowT1 andT i levels. In a
clinically euthyroid patient. thyroid function tests should be
3.0 µU/mL (3.0 mU/L). An additional complicating factor is repeated 6 weeks af'ler hospitalization to confirm overt thyroid
that radioiodine scanning is contraindicated during preg­ dysfunction with persistent TSI-I abnormality or confirm ESS
nancy because of the risk for fetal thyroid exposure to radia­ with normalization ofTSI-I . c:J
tion. Instead, several clinical clues may be used to help
KEY POINTS
determine if the patient has thyrotoxicosis, including the pres­
ence of a goiter, ophthalmopathy, or TS! antibodies, all of • Critical illness can cause changes in thyroid function
which are suggestive of Graves disease. tests in up to 75% of hospitalized patients, known as
The use of thionamides is considered safe during preg­ euthyroid sick syndrome; measurement ofTSH should
nancy, but PTU is preferred during the first trimester because only be obtained in the hospital when there is a high
of potential teratogenic effects from methimazole during clinical suspicion of thyroid dysfunction.
organogenesis. Although rarely indicated, surgery may be per­ • The typical pattern of euthyroid sick syndrome, nonthy- HVC
formed during the second trimester. It should be reserved for roidal illness syndrome, or low triiodothyronine (T3)
those who are unable to tolerate thionamides or who have syndrome is a mildly elevated thyroid-stimulating hor-
inadequate control on medical therapy. Radioiodine therapy is mone level and slightly low thyroxine (T) and triiodo­
contraindicated during pregnancy and while breastfeeding. thyronine (T) levels.
Treatment goals with thionamides are a detectable TSH in the • After patients with euthyroid sick syndrome are discharged HVC
lower end of the pregnancy reference range and a free T.1 in the from the hospital, thyroid ftmction abnormalities may per-
upper end of the reference range. sist for several weeks so follow-up thyroid function tests
KEY POINT should not be repeated unti.16 weeks after discharge.
• In pregnant patients on levothyroxine replacement, the
dose may need to be increased, on average by 30% to 50%;
patients should have their thyroid-stimulating hormone Thyroid Emergencies CJ
level checked as soon as a pregnancy test is positive. Although most thyroid conditions are not urgent. thyroid
storm and myxedema coma represent true medical emergen­

Cl Euthyroid Sick Syndrome cies requiring critical care. Failure to make a timely diagnosis
and institute treatment is associated with a high mortality rate.
l:uthyroid sick syndrome (ESS). nonthyroidal illness synclmme.
ur lowT, syndrome arc ,·arious 11c1mcs that have been assigned Thyroid Storm
lO the chc1ngcs seen in thyroid function test results during criti­ Thyroid storm is a severe manifestation or thyrotoxicosis with
cJl illness. Although not a true syndrome. there are signilkant life threatening secondary systemic decompensation (shock).

48
Disorders of the Thyroid Gland

Cl The cardinal features for diagnosis include elevated tempera bleeding. or metabolic derangements. Cold exposure appears
ture, significant tachycardia. heart failure. gastrointestinal dys­ to be a risk factor. as this condition is more commonly seen in
CONT. . . .
f unction
· (nausea, vom1tmg, ct·1arrhea. and/ or Jaunct·ice). and the winter months.
neurologic disturbances. The range of central nervous system Mental status changes and hypothermia are the most
manifestations includes increasing agitation. emotional !ability, common clinical manifestations. The spectrum of mental sta­
confusion, paranoia. psychosis, or coma. Although thyroid tus changes includes lethargy. stupor. coma. depression. or
storm has been reported with many causes of thyrotoxicosis. it even psychosis. Hypothermia (temperature less than 34 .4 °C
occurs most commonly with Graves disease. Thyroid storm [94.0 °F]) is present in nearly .111 patients: lower temperntures
may be precipitated by another event such as infection. surgery. are associated with a worse prognosis. Ventilatory drive is
myocardial infarction. trauma, or part1.1rition. Administration decreased. resulting in hypoxemia ancl hypercapnia. Additional
of radioactive iodine therapy to a patient with untreated or signs include bradycardia. hypoglycemia. hyponatremia. and
uncontrolled hyperthyroidism can trigger thyroid storm. or hypotension. A significant percentage of patients experi­
The diagnosis is based on clinical presentation but can ence seizures. which may be related to the coexisting meta­
generally be ruled out if T I and T3 levels are within normal bolic derangements.
limits. If myxeclema coma is suspected. the serum TSH and T 1
Treatment of thyroid storm should be directed toward levels should be tested immediately. Diagnosis is macle based
reduction of thyroid hormone production. decreasing peripheral on the clinical presentation and the coexisting metabolic
conversion of T4 to T1 • addressing adrenergic symptoms and abnormalities. The serum cortisol level should be checked as
thermoregulatory changes, searching for and treating precipilat soon as possible to evaluate for concomitant adrenal insum­
ing factors, and reversing systemic decompensation. ciency prior to initiation or thyroid hormone replacement.
Thionamides and �-blockers are the mainstay or treatment to While awaiting the results of the serum cortisol measurement.
reduce thyroid hormone production and control adrenergic it is generally advisable to empirically initiate high-dose gluco­
symptoms. PTU and propranolol are the preferred agents corticoid therapy. This therapy may be discontinued if the
because they have the added benent or blocking peripheral con­ serum cortisol level is found to be normal or high.
version of T.1 to T3 • Additionally, high-dose glucocorticoids The lre.1tment or111yxedem.1 coma is aimed at restoration
reduce T.1 conversion to bioactive T3 • At least 1 hour after the first of the euthyroid state with thyroid hor111one therapy. support­
dose of a thionamide. iodine drops should be administered to ive care (mechanical ventilation. vasopressors. and glucocorti­
inhibit further release of thyroid hormone from the gland. coids). war111ecl intravenous nuids. warming blankets. and
Acetaminophen and cooling blankets may be used to control the management or the underlying precipitating event. The exact
hyperthermia. However. even vvith aggressive therapy and sup­ close and preparation or thyroid hormone to administer is
portive measures, mo11ality rates are as high as 15% to 20'7.,. Cl controversial: minimal clinical trial information is available to
ascertain the optimal treatment regi111en. It is important to
KEY POINTS
balance the need for rapid reinstate111ent or a euthyroicl state
• Thyroid storm is a severe manifestation of thyrotoxicosis with the risk of precipitating .1 fatal cardiac event due to
with life-threatening secondary systerruc decornpensation increased cardiac work with administration of thyroid hor­
(shock); it occurs most commonly with underlying mone. Generally. intravenous levothyroxine therapy is admin­
Graves disease coupled with a precipitating factor such as istered. initially as an intravenous bolus or 200 to 500 µg,
infection, surgery, myocardial infarction, or parturition followed by claily closes of SO to 100 µg intravenously until
and mortality is 15% to 20%. transition to an oral formulation is feasible. Treatment with T 1
• In addition to supportive ca.re and treating the partici­ is not recommended.
pating cause, thionamides and �-blockers are the main­ Even with aggressive therapy. the mortality rate for 111yx­
stay of treatment to reduce thyroid hormone production edema coma is 20'Y., to 25%. Cl
and are often combined with iodine drops and high­
KEY POINTS
dose glucocorticoids to treat thyroid storm.
• Myxedema coma is an extreme but rare manifestation

Cl Myxedema Coma
Myxedema coma is an extreme but rare manifestation orhypo­
of hypothyroidism, resulting in life-threatening second­
ary systerruc decompensation and a mortality rate of
thyroidism, resulting in life-threatening secondary systemic 20% to 25%.
decompensation. Without a frankly low T I level. myxedema • The treatment of myxeclema coma is aimed at restoration
coma is unlikely. regardless of the degree of TSH elevation. 11 of the euthyroicl state with thyroid hormone therapy,
has a very high mortality rate if there is a delay in treatment. supportive care (mechanical ventilation, vasopressors,
Myxedema coma is more common in elderly women: it may and glucocorticoids), warmed intravenous fluids,
occur in those with a history of hypothyroidism or no ante­ warrrung blankets, and management of the underlying
cedent illness. Precipitating events are frequent, such as myo­ precipitating event.
cardial infarction, infection. stroke. trauma. gastrointestinal

49
Disorders of the Thyroid Gland

Structural Disorders a neck ultrasound that includes evaluation of the thyroid and
cervical lymph nodes.
of the Thyroid Gland Nodules are frequently detected incidentally on imaging
Thyroid Nodules sh1dies performed for other reasons. The diagnostic evaluation
Nodularity of the thyroid is extremely common; large popula­ of incidentally discovered thyroid nodules is identical to those
tion studies suggest that up to 5% of women and 1% of men that are clinically detected, with the same rate of malignancy.
have a clinically evident nodule. The prevalence increases with Nodules incidentally identified on fluorodeoxyglucose-PET
age. In autopsy series and screening ultrasound studies , nod­ (FOG-PET) scanning, however, have a malignancy rate of30%
ules may be seen in up to 60%. to 50%. Consequently, FOG-avid nodules found on PET scans
The differential diagnosis for a nodule in the thyroid is require heightened suspicion and a lower threshold for inter­
varied and includes both primary thyroid disorders and meta­ vention or diagnostic evaluation.
static spread from other primary malignancies (Table 27). Most A careful history should be performed in patients with a
thyroid nodules are benign, with only approximately 10% thyroid nodule. Increased risk of malignancy is found in
harboring a malignancy. Ultrasonography is an inexpensive patients with history of radiation exposure to the head or
and highly effective method for stratification of malignancy neck, a family history of thyroid cancer, or a personal history
risk. All patients with a suspected thyroid nodule should have of thyroid cancer. Additional factors that increase the risk for
malignancy in a nodule include male sex, extremes of age (<20
or >60 years), rapid nodule growth, and hoarseness. On phys­
TABLE 27. Types ofThyroid Nodules ical examination, the nodule should be assessed for texture,
Benign Malignant mobility, and associated lymphadenopathy. If the nodule is
hard, fixed to surrounding tissue (nonmobile with swallow­
Multinodular goiter Papillary thyroid cancer
(colloid adenoma)
ing), and/or there is associated cervical lymphadenopathy, the
risk of malignancy is greater. Pain is an uncommon finding
Hashimoto (chronic Follicular thyroid cancer
lymphocytic) thyroiditis
with thyroid nodules, but when present it is usually associated
with benign conditions.
Colloid cyst Medullary thyroid cancer
A serum TSH measurement is the initial laboratory test in
Hemorrhagic cyst Anaplastic thyroid cancer
a patient with a thyroid nodule. If the TSH is suppressed,
Follicular adenoma Primary thyroid lymphoma measurement ofT4 and T3 should be performed, and a radio­
Hurthle cell adenoma Metastatic cancer nuclide scan should be considered (Figure 9). The objective of
Breast the scan is to identify "hot" or functioning nodules, which
Melanoma
have a very low likelihood of malignancy and typically do not
require FNA. In contrast, if the TSH is high or normal, the
Renal cell
radionuclide scan is unnecessary as it is unlikely to reveal a hot

History, physical, TSH

High or normal TSH Low TSH

.----1�
Ultrasound Thyroid scan, FT,, FT,

>1 cm <1 cm Functioning Nonfunctioning


Evaluate for US-guidedFNA Repeat US in 6-24 months "Hot" "Cold/warm"

NoFNA Evaluate
Treat hyperthyroidism for US-guided FNA

FIGURE 9. Initial evaluation of a thyroid nodule. There are size thresholds for FNA based on US appearance. A less suspicious lesion may not need FNA until it is larger
than 2 cm, suspicious nodules if larger than 1 cm. FNA= fine-needle aspiration; FT 3 = free triiodothyronine; FT 4 = free thyroxine; TSH = thyroid-stimulating hormone; US=
ultrasound.

50
Disorders of the Thyroid Gland

nodule; the evaluation should proceed with an ultrasound and Nodules that are benign by FNA should be followed with
possible FNA. As thyroid nodular disease can be altered by repeat ultrasound examination in 6 to 18 months to assess for
normalization of the TSH, ultrasound andFNA should be post­ significant changes. lf the nodule is stable on repeat imaging
poned in patients with elevated TSH until TSH is normal, and lacks suspicious features, clinical examination and repeat
unless there is marked concern for malignancy. One-time ultrasound can be extended to longer intervals, such as 3 to
measurement of thyroid antibodies may be appropriate if S years. Greater than SO% change in nodule volume or interval
autoimmune thyroid disease is suspected or if muitinoduiar development of concerning ultrasound characteristics should
goiter is identified by ultrasound to stratify the patient's future prompt a repeat FNA to evaluate for a false-negative initial
risk of developing overt thyroid failure. Serum thyroglobulin biopsy.
measurement is not useful and is not recommended. Malignant nodules and those that are suspicious for
FNA, performed under ultrasound guidance, is the opti­ malignancy require prompt excision; this is typically done
mal test to determine whether a nodule is malignant. When with total thyroidectomy, but hemithyroidectomy may be
performed by an experienced clinician, FNA is safe and rela­ preferable for patients younger than 45 years of age with a
tively simple to perform. The sensitivity ofFNA cytology is 90% tumor smaller than 4 cm. A nondiagnostic FNA warrants a
to 95%, and the false-negative rate is 3% to 5%. FNA of a nodule repeat attempt. ln a solid nodule with two unsatisfactory biop­
is generally recommended for those nodules larger than 1 cm sies, diagnostic hemithyroidectomy is indicated. Surgical
that are solid and hypoechoic. The threshold for FNA of nod­ complications include hypoparathyroidism and recurrent
ules that are partially cystic and lacking suspicious ultrasound laryngeal nerve paresis; although typically temporary, either
features is 2 cm in size or greater. Aspirating a nodule of S mm complication may be permanent in up to 3% of patients.
or more may be considered if a patient has risk factors such as
KEY POINTS
a personal or family history of thyroid cancer or prior radia­
tion exposure. • Thyroid nodules are found in 1 % to 5% of the popula­
The sonographic appearance of a nodule may be used to tion; most thyroid nodules are benign, with only
assess the risk of malignancy and thereby guide the decision of approximately 10% harboring a malignancy.
which nodules require biopsy. Features concerning for malig­ • A serum thyroid-stimulating hormone measurement is HVC
nancy include microcalcifications, marked hypoechogenicity, the initial laboratory test in a patient with a thyroid
irregular borders, and taller-than-wide shape. Such findings nodule; if the thyroid-stimulating hormone is sup-
are nearly 70% specific for cancer, but their poor sensitivity pressed, then measurement of thyroxine (T,1) and triio­
cannot exclude the presence of malignancy. dothyronine (T3) should be performed, and a radionu-
The various diagnoses obtained on FNA and the associ­ clide scan should be considered to identify "hot" or
ated risks of malignancy are listed in Table 28. functioning nodules, which have a very low likelihood
of malignancy and typically do not require fine-needle
aspiration.
TABLE 28. Diagnoses Obtained by Fine-Needle • lf the thyroid-stimulating hormone level is high or nor- HVC
Aspiration of Thyroid Nodules and Risk for Malignancy
ma!, the radionuclide scan is unnecessary as it is
FNA Diagnosis Risk for Management unlikely to reveal a hot nodule and ultrasonography is
Malignancy
an inexpensive and highly effective method for stratifi-
Benign <3% Serial ultrasound cation of malignancy risk for nonfunctioning thyroid
examinations for nodules.
growth
• Fine-needle aspiration of a nodule is generally recom­
Atypia of uncertain 5%-10% Repeat FNA
significance/follicular mended for those nodules larger than 1 cm (0.4 in) that
lesion of uncertain are solid and hypoechoic and is the optimal test to
significance determine whether a nodule is malignant.
Suspicious for 20%-30% Hemithyroidectomy
follicular lesion
Goiters
Suspicious for 50%-75% Hemithyroidectomy
malignancy or total thyroidectomy Multinodular Goiter
Malignant 97%-100% Total thyroidectomy
Multinodular goiters occur more frequently with advancing
age, low iodine intake, or Hashjmoto disease. The risk for
Nondiagnostic 0%-50% Repeat FNA; if two
nondiagnostic FNAs, malignancy is the same for multiple nodules as it is for a soli­
surgery tary nodule; therefore, the evaluation and management are
Modified from: Cibas ES. Ali SZ; NCI Thyroid FNA State of the Science Conference.
identical. Biopsy should be performed on the three or four
The Bethesda System For Reporting Thyroid Cytopathology. Am J Clin Pathol. nodules (larger than 1 cm) with the most suspicious ultra­
2009 Nov; 132(5):658-65. [PMID: 19846805]
sound features. In the absence of suspicious features, the larg­
FNA = fine.needle aspiration.
est nodules should be chosen for aspiration.

51
Disorders of the Thyroid Gland

A large multinodular goiter may be associated with com­ the trend, there is evidence that larger tumors are increasingly
pressive symptoms such as dysphagia, hoarseness, or posi­ being discovered.
tional dyspnea. To assess the extent of mass effect, additional The vast majority of patients with thyroid cancer have
testing and imaging, including noncontrast CT of the neck/ well-differentiated thyroid cancer, with excellent long-term
chest, barium swallow, direct Jaryngoscopy, and/or spirometry survival. The major forms and their relative frequency are
with flow-volume loops, may be indicated. Levothyroxine listed in Figure 10. The most common well-differentiated thy­
therapy to suppress TSH secretion and reduce goiter size is roid cancers are papillary, papillary-follicular variant, and
generally not helpful, poses a risk of thyrotoxicosis, and is not follicular. There are rare, less well-differentiated variants of
recommended. Radioactive iodine ablation is not an option for papillary thyroid cancer (columnar, tall cell, insular, oxyphilic,
euthyroid and hypothyroid patients. Surgical removal is the clear cell, diffuse sclerosing) that are more aggressive and
treatment of choice if the compressive symptoms are signifi­ carry a worse prognosis. Anaplastic thyroid cancer is undif­
cant, if malignancy is suspected, or if the patient desires cos­ ferentiated and is the most aggressive form of thyroid cancer;
metic intervention. 1-year survival rates range from 20% to 30%.
Staging and prognosis of well-differentiated thyroid can­
KEY POINTS
cers (papillary and follicular) are based on the American Joint
• In patients with a multinodular goiter, the risk for Committee on Cancer criteria, which include age (<45 or 2:45
malignancy is the same for multiple nodules as it is for years), primary tumor size, local and distant metastases, and
a solitary nodule; therefore, the evaluation and manage­ capsular and Jymphovascular invasion. However, because the
ment are identical. majority of patients have excellent survival, T (tumor) N (node)
• Surgical removal of a large multinodular goiter is the M (metastasis) staging plays a minimal role in the management
treatment of choice if the compressive symptoms are of thyroid cancers. Instead, decisions regarding treatment are
significant, if malignancy is suspected, or if the patient aimed at lowering the likelihood of recurrent disease.
desires cosmetic intervention. Treatment of well-differentiated thyroid cancer includes
a combination of surgery, radioactive iodine, and levothyrox­
Simple Goiter ine suppression. The extent of surgery is largely based on the
A simple goiter is defined as an enlargement of the thyroid tumor size; solitary tumors smaller than 1 cm may be suffi­
gland without the presence of nodules. It may be seen in ciently managed with lobectomy alone. Patients with larger
conditions of dyshormonogenesis, autoimmune thyroid dis­ tumors, multifocal disease, nodal metastases, or a history of
ease, or primary thyroid lymphoma. Primary thyroid lym­ irradiation are best treated with total or near-total thyroidec­
phoma is a rare condition that typically occurs in elderly tomy. For patients younger than 45 years of age with a tumor
women with a history of Hashimoto thyroiditis. The clinical smaller than 4 cm and no evidence of nodal or distant metas­
presentation is a symptomatic, rapidly enlarging goiter with tases, hemithyroidectomy may be a reasonable alternative to
a very firm texture. Patients may also have systemic lym­ total thyroidectomy. The decision to administer radioactive
phoma symptoms and lateral cervical lymphadenopathy. The iodine is based on two factors: improvement in mortality rates
diagnosis can be made by FNA. Treatment typically involves and/or reduction in recurrence risk. Patients with distant
chemotherapy and/or radiation therapy. Surgery generally is metastases have improved survival with successful radioiodine
not indicated, but it can be used to aid in diagnosis when
FNA is not informative.

Thyroid Cancer
The incidence of thyroid cancer is rising at a faster rate than
any other type of malignancy; the incidence has more than
doubled in the last 30 years. This increase is due solely to pap­ • Papillary
illary cancers, with the highest rate of rise occurring in tumors
• Follicular
measuring less than 2 cm. Meanwhile, the survival rate for
• Medullary
thyroid cancer has remained stable or slightly improved.
• Anaplastic
Autopsy series reveal that occult thyroid cancers measuring
Jess than 1 cm may be identified in as many as 20% of dissected Other (thyroid lymphoma
and metastases from other
specimens. This finding, coupled with the improving survival cancers)
rate, has Jed some investigators to conclude that the change in
incidence of thyroid cancer is due solely to increased inciden­
tal detection of indolent tumors because of greater use of FIGURE 1 0. Relative frequency of the types of thyroid cancer.
imaging modalities. Although there is little doubt that esca­
Data from Hundahl SA, Fleming ID, Fremgen AM, Menck HR. A National Cancer Data Base report on 53,856 cases
lated detection of otherwise occult tumors has contributed to of thyroid carcinoma treated in the U.S., 1985-1995. Cancer. 1998 Dec 15;83(12):2638-48. IPMID: 9874472]

52
Reproductive Disorders

therapy, whereas administration of radioactive iodine may 8


-�
decrease the likelihood of recurrent disease in those patients
with nodal metastases. Suppression ofTSH with levothyroxine

� 9
therapy may also be used to improve morbidity and reduce �
mortality, particularly in patients with persistent disease or 8 8
distant metastases. The necessary degree of TSH suppression
varies according to the risk of cancer progression and comor­
bidities of the patient. Patients with persistent disease typi­ Theca cells , - - - ->- Granulosa cells
cally require lowering of theirTSH level to less than 0.1 µU/mL
(0.1 mU/L), whereas those who are free of disease but have a
high risk for recurrence should have a targetTSH level ofO.l to Androstenedione Estradiol Ovulation I
0.5 µU/mL (0.1-0.5 mU/L) for 5 to 10 years. Those patients
who are disease-free with a low risk of recurrence should FIGURE 11 . Female reproductive axis. Pulses of GnRH drive LH and FSH pro·
maintain aTSH level of 0.3 to 2.0 µU/mL (0.3-2.0 mU/L). duction. LH acts on theca cells to stimulate androgen (principally androstenedione)
Medullary thyroid cancer represents less than 10% of all production. Androstenedione is metabolized to estradiol in granulosa cells. FSH
acts on granulosa cells to enhance follicle maturation. Granulosa cells produce
thyroid cancers. Approximately 25% of medullary thyroid can­ inhibin Bas a feedback regulator of FSH production. FSH = follicle-stimulating
cers are hereditary; all patients with medullary thyroid cancer hormone; GnRH = gonadotropin-releasing hormone; LH = luteinizing hormone;
should be screened with RET proto-oncogene sequencing. - (circled)= negative feedback.
Medullary thyroid cancer may be associated with several syn­
dromes, including multiple endocrine neoplasia type 2A
(MEN2A) (which may include pheochromocytoma and hyper­ FSH, under control of pulsatile GnRH secretion, rises in the
parathyroidism), MEN2B (marfanoid habitus and mucosa! early menstrual cycle to promote recruitment and growth of a
ganglioneuromas), or familial medullary thyroid cancer (med­ follicle containing a microscopic oocyte (follicular phase).
ullary thyroid cancer alone). Biochemical screening for pheo­ Granulosa cells lining the follicle secrete estradiol, which contrib­
chromocytoma with measurement of plasma fractionated utes to negative feedback inhibition ofFSH secretion and result­
metanephrine levels should be done in all patients with an ant monofollicular development in the majority of women.
RET mutation prior to thyroidectomy. Estradiol also stimulates endometrial proliferation. Further into
the follicular phase, estradiol levels peak and exert acute positive
KEY POINTS feedback on the pituitary gland, which elicits an LH surge.This
• The vast majority of patients with well-differentiated LH surge results in ovulation and initiates the luteal phase of the
thyroid cancer have excellent long-term survival. menstrual cycle. LH stimulates androgen production by the theca
• Treatment of well-differentiated thyroid cancer includes cells, which also line the follicle; androgen is subsequently aro­
a combination of surgery, radioactive iodine, and levo­ matized to estrogen in the granulosa cells via aromatase enzyme.
thyroxine suppression of thyroid-stimulating hormone After the LH surge and ovulation, the follicle develops into the
for patients with persistent disease or high risk of corpus luteurn, which secretes both estradiol and progesterone
recurrence. and causes the secretory phase of the endometriurn in prepara­
tion for implantation of a fertilized oocyte. With implantation, the
early embryo produces human chorionic gonadotropin, which
maintains the corpus luteurn. However, when a fertilized embryo
Reproductive Disorders is not present, progesterone levels decline, leading to the men­
strual phase of the endometriurn and menstrual bleeding.
Physiology of Female An average menstrual cycle ranges from 25 to 35 days in
Reproduction length. The follicular phase may vary in each woman but is
A regular, predictable menstrual cycle requires coordination typically from 14 to 21 days. Variability in a menstrual cycle is
of inhibition and stimulation between the hypothalamus typically the result of a shortened or lengthened follicular phase,
(secreting gonadotropin-releasing hormone [GnRH]), the more commonly seen during the first 5 years of menstruation.
pituitary (secreting follicle-stimulating hormone [FSH] and A decrease in follicular phase length occurs commonly in peri­
luteinizing hormone [LH]), and the ovaries (secreting estra­ menopause.The luteal phase is usually 14 days and is constant.
diol and progesterone). The coordination of these signals is In women younger than 40 years of age, menstrual cycles less
referred to as the hypothalamic-pituitary-ovarian axis. The than 25 days or greater than 35 days are likely anovulatory.
GnRH pulse frequency varies throughout the menstrual KEY POINT
cycle to promote follicular development and ovulation
• In women younger than 40 years of age, menstrual
(Figure 11). The phases of the menstrual cycle are referred to
cycles less than 25 days or greater than 35 days are likely
in reference to the activity of the ovary (follicular and luteal
anovulatory.
phases).

53
Reproductive Disorders
------ - ---- ---- - ----- ---- -------- -
-

Amenorrhea small functional pockets of active endometrium with out­


flow obstruction by synechia closer to the cervix, resulting
Clinical Features
in cyclic pain and hematometrium identifiable on ultra­
Primary Amenorrhea
sound.
Primary amenorrhea is the absence of menses by age 16 years
After structural causes and pregnancy are excluded, the
accompanied by normal sexual hair pattern and normal breast
hormonal status should be assessed. Low estradiol and inap­
development. Primary amenorrhea with absence of thelarche propriately normal FSH and LH levels indicate hypogonado­
(breast development at the beginning of puberty) and/or tropic hypogonadism and point to a central cause
adrenarche (androgen production increase that typically (hypothalamic-pituitary). Low estradiol in the setting of
occurs at age 8 or 9 years) prior to 14 years of age should be elevated FSH and LH levels indicates hypergonadotropic
evaluated. Pregnancy must be ruled out in any patient with hypogonadism and points to ovarian insufficiency.
primary amenorrhea before additional evaluation occurs. The most common hormonal cause of secondary amen­
Causes of primary amenorrhea may be genetic, hormonal, or orrhea is polycystic ovary syndrome (PCOS) (see Hirsutism
structural. Fifty percent of patients with primary amenorrhea and Polycystic Ovary Syndrome), which accounts for 40% of
have a chromosomal abnormality, such as Turner syndrome cases. Additional hormonal causes of secondary amenorrhea
(45,XO) (gonadal dysgenesis), although some patients with include hypothalamic amenorrhea (hypogonadotropic hypog­
Turner mosaicism may have secondary amenorrhea (see onadism), hyperprolactinemia, thyroid disease, and prema­
Secondary Amenorrhea). ture ovarian insufficiency (POI) (hypergonadotropic
Turner syndrome is commonly characterized by other hypogonadism).
clinical manifestations such as short stature, neck webbing, Hypogonadotropic hypogonadism caused by hypotha­
recurrent otitis media with hearing loss, aortic coarctation, lamic amenorrhea (HA) or functional hypothalamic amenor­
and bicuspid aortic valve. The diagnosis of Turner syndrome rhea (FHA) affects 3 % of women between the ages of 18 and
may be made with a karyotype. 40 years. Risk factors include low BM! and low body fat per­
Approximately 15% of patients presenting with primary centage, rapid and substantial weight loss or weight gain,
amenorrhea may have an anatomic abnormality of the uterus, eating disorders, excessive exercise, severe emotional stress,
cervix, or vagina such as mullerian agenesis, transverse vaginal or acute and chronic illness. FSH and LH levels are inappro­
septum, or imperforate hymen. Digital vaginal examination, priately low in HA but may be inappropriately normal in
transvaginal ultrasound, or MRI may help to identify outflow FHA. Estradiol levels are typically low, and patients may
tract anomalies. experience vasomotor symptoms and sleep disturbance. If
KEY POINT left untreated, patients are at increased risk for osteoporosis
• Fifty percent of patients with primary amenorrhea have owing to this low-estrogen state. Recovery of menses may
a chromosomal abnormality, such as Turner syndrome occur if BM! returns to normal. Cognitive-behavioral therapy
(45,XO) (gonadal dysgenesis); 15% of patients presenting for cases caused by emotional stress has been shown to be
with primary amenorrhea may have an anatomic effective.
abnormality of the uterus, cervix, or vagina. Hyperprolactinemia causes secondary amenorrhea
through direct inhibition of GnRH secretion. Treatment of the
cause of hyperprolactinemia typically results in restoration of
Secondary Amenorrhea menses. Hypothyroidism may cause secondary amenorrhea
Secondary amenorrhea is the absence of a menstrual cycle through increased thyrotropin-releasing hormone levels,
for three cycles or 6 months in previously menstruating which causes stimulation of prolactin secretion.
women. The most common cause of secondary amenorrhea Hypergonadotropic hypogonadism as a result of POI is
is pregnancy. A potential structural cause of secondary defined as amenorrhea before age 40 years in the setting of
amenorrhea, such as Asherman syndrome, should be con­ two elevated FSH levels (>40 mU/mL [40 U/L]) more than
sidered. Asherman syndrome is an uncommon complica­ 1 month apart. Possible causes include Turner mosaicism (in
tion of dilation and curettage, intrauterine device place­ which secondary amenorrhea may occur due to POI), fragile X
ment, or surgical procedures such as hysteroscopic premutation, chemotherapy or radiation, and autoimmune
myomectomy; it is caused by lack of basal endometrium oophoritis. In patients in whom an autoimmune cause is diag­
proliferation and formation of adhesions (synechiae). nosed, evaluation of other endocrine organs (thyroid, parathy­
Diagnosis should be considered in any woman with amen­ roid, pancreas, and adrenal) is recommended at the time of
orrhea and past exposure to uterine instrumentation. The diagnosis and annually thereafter.
classic presentation is amenorrhea or scant bleeding during Estrogen replacement is necessary in patients with hyper­
periods (hypomenorrhea) with ovulatory symptoms (cervi­ gonadotropic hypogonadism to prevent bone mass loss until
cal mucous changes, adnexal tenderness associated with the average age of natural menopause (50-51 years). Estrogen
follicle formation) or premenstrual symptoms (mood replacement preparations are available in oral, transdermal,
changes or breast tenderness). Some patients will maintain subcutaneous, and vaginal routes of administration. The dose

54
Reproductive Disorders

of estrogen required by young women is titrated to prevent KEY POINTS


vasomotor symptoms and vaginal dryness and may be higher
• After excluding pregnancy, the laboratory evaluation of
than that used in an older age group. Because spontaneous
primary and secondary amenorrhea includes measure­
ovulation may occur (although it is infrequent), counseling on
ments of prolactin, follicle-stimulating hormone, lute­
contraceptive options should also be provided for sexually
inizing hormone, estradiol, and thyroid-stimulating
active women not desiring pregnancy. Cyclic progesterone
hormone.
exposure should be considered in patients with an intact
uterus to prevent excessive unopposed endometrial prolifera­ • If hormonal evaluation for amenorrhea is negative, the
tion. Oocyte donation may be considered for fertility options next step is a progesterone challenge test; if the patient
for this patient population. bleeds within 1 week of completing 7 to 10 days of pro­
gesterone, estrogen deficiency is not the cause and
KEY POINT
PCOS should be considered
• The most common causes of secondary amenorrhea are
pregnancy, structural abnormalities, and polycystic
ovary syndrome. Hyperandrogenism Syndromes
Hirsutism and Polycystic Ovary Syndrome
Evaluation of Amenorrhea When hirsutism is present, the patient should be assessed for
A thorough history and physical examination, including a virilization, or development of male characteristics. Rapid
pelvic examination, are needed to evaluate both primary onset and progression of deepening of the voice, severe acne,
(no history of menstruation) and secondary amenorrhea clitoromegaly, and male pattern balding are signs of viriliza­
(cessation of menstruation after menarche). Urine or serum tion and are concerning for an ovarian or adrenal tumor. Age
human chorionic gonadotropin (HCG) testing should be of onset after 30 years is also a risk factor for an androgen­
done first to exclude pregnancy, as this is the most common secreting tumor.
cause of amenorrhea. In patients with primary amenor­ In patients with hirsutism or virilization, recommended
rhea, a karyotype is recommended if a pregnancy test is initial laboratory tests include measurement of plasma dehy­
negative. Serum levels of prolactin, FSH, LH, estradiol, and droepiandrosterone sulfate (DHEAS) level and serum levels of
thyroid-stimulating hormone (TSH) should then be TSH, prolactin, total testosterone, and follicular-phase
obtained in the evaluation of primary and secondary amen­ 17-hydroxyprogesterone. Normal levels exclude adrenal tumors,
orrhea. Abnormal levels of prolactin and/or TSH support a hypothyroidism, hyperprolactinemia, and ovarian tumor.
nonovarian cause of amenorrhea. Elevations of FSH and LH Common forms of late-onset congenital adrenal hyperplasia,
levels in the presence of a low estradiol level support the often mistaken for PCOS, can be excluded with a normal
diagnosis of POI. 17-hydroxyprogesterone level. Pelvic ultrasound and adrenal CT
If no elevations in these hormones are found, a progester­ should be performed to exclude an ovarian or adrenal neoplasm
one challenge test (oral medroxyprogesterone acetate, 10 mg if the serum total testosterone level is greater than 200 ng/dL
for 7-10 days) may be used to determine if the amenorrhea is (6.9 nmol/L}. Adrenal CT is necessary to exclude an adrenal
due to estrogen deficiency. If the patient has menstrual bleed­ cortisol-secreting and/or androgen-secreting neoplasm if the
ing within 1 week of completing 7 to 10 days of medroxypro­ plasma DHEAS level is greater than 700 µg/mL (18.9 µmol/L).
gesterone, estrogen deficiency is not the cause. In this case Hirsutism is typically a benign condition, most commonly from
PCOS (or a similar diagnosis) should be considered. If no PCOS. A marked elevation of total testosterone or DHEAS is not
menstrual bleeding occurs, the patient has a low-estrogen compatible with a diagnosis of PCOS.
state, and hypogonadotropic hypogonadism is the diagnosis PCOS has a prevalence of 7% to 10% and is one of the most
(see Disorders of the Pituitary Gland). common endocrine disorders in young women. Two sets of
Pelvic ultrasound is helpful to identify structural diagnostic criteria are commonly used. The 2003 American
causes of amenorrhea such as mullerian agenesis and Society for Reproductive Medicine and the European Society of
intrauterine synechiae. Saline-infusion sonohysterogram Human Reproduction criteria for PCOS require two of the fol­
can identify intrauterine synechiae, and transvaginal or lowing three findings in the absence of other endocrine disor­
transabdominal ultrasound can identify absence of a uterus ders: (1) oligo-ovulation or anovulation, (2) clinical or
in patients with mullerian agenesis. Endocrinology consul­ biochemical evidence of hyperandrogenism (such as hirsutism
tation for further evaluation and testing of patients with or acne), or (3) ultrasound findings of polycystic ovarian mor­
findings suspicious for a genetic cause of amenorrhea may phology in at least one ovary. The 1990 criteria from the National
be appropriate. A pituitary MRI may be indicated to exclude Institutes of Health and the National Institute of Child Health
other intracranial causes of hypogonadotropic hypog­ and Human Development require all of the following for diag­
onadism when diagnosing HA or FHA, and consultation nosis of PCOS: oligo-ovulation, signs of androgen excess (clini­
with an endocrinologist is recommended before imaging is cal or biochemical), exclusion of other disorders that can result
pursued. in menstrual irregularity, and hyperandrogenism.

55
Reproductive Disorders

A constant stagnant follicular stage is seen in PCOS, KEY PO I NT S (continued)


resulting in unopposed estradiol secretion from small ovarian
• Polycystic ovary syndrome can be diagnosed if two of
follicles. Owing to disordered secretion of LH by the anterior the following three findings are present: (1) oligo-ovula­
pituitary, intraovarian androgen production is also increased tion or anovulation, (2) clinical or biochemical evidence
in PCOS, resulting in the hyperandrogenism associated with of hyperandrogenism (such as hirsutism or acne), or (3)
the disorder. Women with PCOS typically have elevated rest­
ultrasound findings of polycystic ovarian morphology in
ing LH levels. In patients trying to conceive, this can lead to
at least one ovary.
false-positive indication of the ovulatory LH surge on home
urinary LH kits for ovulation. • The 1990 criteria from the National Institutes of Health
Estradiol secretion results in proliferation of the endome­ and the National Institute of Child Health and Human
trium in the absence of progesterone secretion from a corpus Development require all of the following for diagnosis
luteum. This predisposes patients to endometrial hyperplasia of PCOS: oligo-ovulation, signs of androgen excess
and heavy menstrual bleeding as a result of anovulatory bleed­ (clinical or biochemical), exclusion of other disorders
ing. Oligo-ovulation and anovulation result in infertility but that can result in menstrual irregularity, and hyperan­
are typically correctable with clomiphene citrate or letrozole drogenism.
for ovulation induction if fertility is desired. If fertility is not
desired, oral contraceptives should be considered if not con­ Androgen Abuse in Women
traindicated. Addition of oral contraceptives will increase Anabolic steroids may be abused by some women to enhance
secretion of sex hormone-binding globulin (SHBG) and their athletic performance or physique. Such exogenous
decrease circulating levels of free testosterone. !f a patient has a administration may result in absence of GnRH pulsatility and
contraindication to oral contraceptives, a progestin-secreting resultant hypogonadotropic hypogonadism and amenorrhea.
intrauterine device or cyclic oral or vaginal progesterone should Adverse effects may include hirsutism, acne, deepening of the
be given to prevent prolonged unopposed estrogen exposure. voice, decreased breast size, and clitoromegaly. Withdrawal of
Hyperandrogenism may present as hirsutism, acne, or exogenous androgens does not result in severe hypogonadism
androgenic alopecia. In patients with hirsutism desiring as it does in men, and most women return to regular men­
treatment, existing terminal hairs will need to be removed strual cycles.
with depilatory methods, but the rate of hair growth while on
treatment will decrease. Spironolactone, an aldosterone and
androgen inhibitor, may be added after 6 months if acne and
Female Infertility
hirsutism are still cosmetically bothersome. Before initiation Infertility is defined as the absence of conception after 1 year
of therapy, the patient should be counseled about teratogenic of unprotected intercourse (on average twice weekly) in a
effects on a male fetus, and contraceptive counseling should woman younger than 35 years of age. Investigation should
be provided. begin after 6 months if no conception has occurred in a
Both obese and lean women with PCOS also have insulin woman 35 years of age or older. Infertility evaluation should
resistance, and studies have identified an increased incidence include a careful medical history of both partners with special
of metabolic syndrome, obesity, impaired glucose tolerance, focus on menstrual history, previous exposure to sexually
and type 2 diabetes mellitus in these women. Although insulin transmitted infections, pelvic surgery, and previous obstetric
resistance may improve with weight loss, the use of insulin­ complications such as miscarriage or cesarean delivery. If a
sensitizing agents such as metformin is associated with a report of oligomenorrhea is elicited, measurement of serum
decrease in serum androgens; however, it is not very effective TSH and prolactin levels is appropriate to exclude thyroid dis­
as a single agent for ovulation induction. ease and hyperprolactinemia as causes of oligo-ovulation.
Evaluation of patients with PCOS should include assess­ Further evaluation of infertility causes typically includes
ment for signs of sleep apnea, hypercholesterolemia, and fatty semen analysis of the male partner, confirmation of ovulation
liver. In women with a thickened endometrium or menome­ with measurement of midluteal progesterone level (>3 ng/mL
trorrhagia, endometrial sampling with endometrial biopsy [9.5 nmol!L]), and because Fallopian tubes may be obstructed
should be considered to evaluate for endometrial hyperplasia. due to infection such as pelvic inflammatory disease, tubal
Weight loss and exercise should be emphasized. patency evaluation with hysterosalpingogram. A common
cause of tubal occlusion and resultant infertility is past pelvic
KEY POINTS
inflammatory disease. Laparoscopy for evaluation of pelvic
• Polycystic ovary syndrome has a prevalence of 7% to adhesions or mild endometriosis may be warranted in patients
10% and is one of the most common endocrine disor­ with dysmenorrhea, previous exposure to sexually transmit­
ders in young women; it is often associated with insulin ted infections, or previous pelvic surgery.
resistance, metabolic syndrome, obesity and type 2 dia­ If no abnormalities are found, treatment to enhance
betes mellitus. endogenous gonadotropin release and increase the numbers
(Continued)
of oocytes ovulated monthly may be warranted. Some studies

56
Reproductive Disorders

support moving directly to in vitro fertilization treatment for Leydig cell production of testosterone occurs in a diurnal
women with infertility at age 40 years. In women treated pattern, with the highest concentration observed in the morn­
with ovarian stimulation, oral medications such as clomi­ ing. A large percentage of circulating testosterone is bound
phene citrate or Ietrozole are typically used. This therapy is either to SHBG or albumin. A serum total testosterone level
not appropriate in patients with POI. Patients should be measured in the early morning is generally considered to be an
counseled about the 5% to 8% risk of multiple gestation with accurate measurement of a patient's androgen status, but it
these therapies. Referral to a reproductive endocrinologist is does not account for decreased SHBG as seen in obesity (see
recommended. Evaluation of Male Hypogonadism).
KEY POINT

• Infertility evaluation in women should include a med­


ical history of both partners with special focus on Hypogonadism
menstrual history, previous exposure to sexually Primary Hypogonadism
transmitted infections, pelvic surgery, and previous Primary hypogonadism, or testicular failure, represents a
obstetric complications such as miscarriage or cesar­ decrease in testosterone or sperm production. Primary hypo­
ean delivery. gonadism is uncommon and may have congenital or acquired
causes. Klinefelter syndrome is the most common cause of
congenital primary hypogonadism. Klinefelter syndrome is a
Physiology of Male Reproduction common cause of hypergonadotropic hypogonadism and azo­
Control of spermatogenesis and testosterone production ospermia, resulting in infertility. A 47,XXY karyotype is diag­
depends on the pulsatile secretion of GnRH from the hypo­ nostic of Klinefelter syndrome. Mosaic variants of this condi­
thalamus as well as subsequent downstream stimulation of tion exist but typically present with oligoasthenospermia,
the anterior pituitary and male gonads. In the testicle, FSH testicular failure or hypogonadism. Concomitant symptoms
stimulates Sertoli cell spermatogenesis, and LH stimulates often include sexual dysfunction and generalized fatigue. Tall
Leydig cell testosterone production. Negative feedback from stature is a common finding. Patients with Klinefelter syn­
testosterone production inhibits FSH and LH secretion at the drome may fail to achieve puberty or may present after sexual
level of the anterior pituitary as well as pulsatile hypothalamic maturation with azoospermia. Exposure to chemotherapy or
GnRH secretion. Inhibin B, produced by the Sertoli cells, also radiation may also result in primary testicular failure. Local
inhibits FSH (Figure 12). injury as a result of torsion, orchitis, or trauma may result in
ischemia and necrosis of testicular tissues.

Secondary Hypogonadism
GnRH
Typically secondary hypogonadism is a result of insufficient
GnRH production by the hypothalamus or deficient LH/FSH
secretion by the anterior pituitary. Causes may be congenital

9
8 8 or acquired. Idiopathic hypogonadotropic hypogonadism,

GF
with anosmia (Kallmann syndrome) or without anosmia, is
the most common cause of congenital secondary hypog­
onadism. Acquired secondary hypogonadism is most com­
Leydig cells Sertol i cells lnhibin B monly iatrogenic due to exogenous testosterone administra­
tion. Untreated sleep apnea and obesity are other common
causes. Other acquired causes include hyperprolactinemia,
Testosterone Spermatogenesis I chronic opioid use, glucocorticoid use, or infiltrative disease
(lymphoma or hemochromatosis).

Estradiol Androgen Deficiency in the Aging Male


Di hyd rotestosterone The natural progression of male aging involves testosterone
level decline. Most men will not become hypogonadal, and the
FIGURE 1 2. Male reproductive axis. Pulses of GnRH elicit pulses of LH and decline in testosterone production is highly variable with each
FSH. FSH acts on Sertoli cells, which assist sperm maturation and produce inhibin person. "Low T" has become a part of the popular vernacular,
B, the major negative regulator of basal FSH production. The Leydig cells produce owing to aggressive direct-to-consumer marketing, and
testosterone, which feeds back to inhibit GnRH and LH release. Some testosterone
describes many symptoms that may or may not be associated
is irreversibly converted to dihydrotestosterone or estradiol, which are both more
potent than testosterone in suppressing GnRH and LH. FSH = follicle-stimulating with a low serum testosterone level in men. Both prescription
hormone; GnRH = gonadotropin-releasing hormone; LH = luteinizing hormone; and over-the-counter testosterone and derivative sales have
- (circled)= negative feedback. surged in the United States and many other countries. With

57
Reproductive Disorders

the increased sales of testosterone formulations to treat aging hypogonadotropic state is revealed, transferrin saturation and
men, questions have emerged about the potential adverse ferritin levels should be evaluated to exclude hemochromato­
effects of exogenous testosterone therapy, particularly in men sis. MRI of the pituitary should be performed to evaluate for
who have no biochemical evidence of testosterone deficiency. hypothalamic or pituitary masses as the cause of the hypogon­
Potential adverse effects include increased risk of cardiovascu­ adotropic state if no confounding medications or reversible
lar disease and death, venous thromboembolism, and prostate secondary causes are discovered. Figure 13 shows an algorithm
cancer. for evaluating male hypogonadism.
KEY POINTS

Evaluation of Male Hypogonadism • Measurement of testosterone levels is not recommended HVC


A thorough history and physical examination are essential in if a patient is having regular morning erections, does
the evaluation of hypogonadism. A sleep history is especially not have true gynecomastia on examination, and has a
helpful. A large constellation of nonspecific symptoms is asso­ normal testicular examination, as it is highly unlikely
ciated with male hypogonadism, which makes diagnosis and that he has testosterone deficiency.
treatment based on symptoms alone not advisable. Nonspecific • Testosterone deficiency is diagnosed with two early
symptoms include fatigue, decreased muscle strength, morning total testosterone levels below the reference
decreased libido, amotivational state, or decreased robustness range.
or frequency of erections . Testosterone measurements are not • Once testosterone deficiency is confirmed, the cause
recommended if only nonspecific symptoms are present; should be further investigated prior to initiation of tes­
rather, investigation of other causes of the patient's symptoms tosterone replacement.
is appropriate. More specific symptoms include gynecomastia,
diminished testicular volumes, and absence of morning erec­
tions. Measurement of testosterone levels is not recommended
if a patient is having regular morning erections , does not have
Testosterone Replacement
true gynecomastia on examination, and has a normal testicu­ Therapy
lar examination, as it is highly unlikely that he has testoster­ Testosterone replacement therapy is a widely used treatment
one deficiency. for men with hypogonadism. Possible benefits seen with
Testosterone deficiency is diagnosed with two early testosterone replacement therapy, such as improved libido,
morning serum total testosterone levels below the refer­ energy level, and bone density, have been described but
ence range. Because illness and strenuous activity can remain controversial. Testosterone therapy has been associ­
falsely lower testosterone levels, measurement should ated with increased hemoglobin and hematocrit levels,
occur in healthy men who have avoided strenuous activity worsened obstructive sleep apnea, and a decrease in HDL
for several days. Measurements of the testosterone level cholesterol levels. LDL cholesterol levels do not appear to be
occurring later in the morning or in the afternoon are not affected.
useful for interpretation. Consultation with an endocri­ Although hypogonadism remains an independent risk
nologist should be considered if two early morning total factor for mortality, recent studies have examined the asso­
testosterone levels are low. In certain clinical scenarios, ciation between testosterone therapy and cardiovascular risk.
such as morbid obesity, total testosterone may be low but The association between testosterone therapy and mortality
free testosterone may be normal. Free testosterone assays has remained controversial. Physicians prescribing testoste­
can be unreliable, and routine measurement of free testos­ rone therapy to elderly men with biochemically proven
terone is not recommended. Free testosterone by equilib­ testosterone deficiency and comorbidities should use it pru­
rium dialysis is the gold-standard assay. dently with close follow-up. Cardiovascular disease risk as
Once confirmed, the cause of hypogonadism (primary or well as risk for thrombosis should be discussed with patients
secondary) should be further investigated prior to initiation of before pursuing therapy. Prescribing testosterone therapy in
testosterone replacement. Serum LH, FSH, prolactin, and TSH the absence of biochemically proven testosterone deficiency
levels should be measured. Primary testosterone deficiency puts the patient at risk for iatrogenic hyperandrogenism with
(hypergonadotropic hypogonadism) is diagnosed when FSH subsequent increased risk of myocardial infarction, stroke,
and LH levels are frankly elevated in the presence of a simul­ death, venous thromboembolism, polycythemia, and
taneously low testosterone level. Low or inappropriately nor­ obstructive sleep apnea. Prescribing testosterone therapy in
mal FSH and LH levels in the presence of simultaneous low the absence of a full evaluation may delay treatment for sec­
testosterone levels are diagnostic of secondary hypogonadism ondary causes such as prolactinoma, hemochromatosis, or
(hypogonadotropic hypogonadism). intracranial mass.
A hypergonadotropic state (elevated LH and FSH levels) Although implantable pellets and injectable testosterone
should be further investigated with a karyotype if no history preparations are available, the most popular testosterone prep­
of gonadotoxic therapy or testicular insult is elicited. If a arations currently are topical (most commonly hydroalcoholic

58
Reproductive Disorders

Suspect hypogonadism

Measure: total serum testosterone level (x2)

Low testosterone Normal testosterone

Bioavailable/free testosterone Follow


and/or SHBG in appropriate patients

Confirmed low testosterone:


Check FSH and LH levels

High FSH or LH level Low or normal FSH or LH level

Primary hypogonadism Secondary hypogonadism:


Check PRL level, iron studies

Elevated PRL level, other pituitary deficiencies Elevated transferrin saturation, ferritin level
Signs/symptoms of mass effect, testosterone level <150 ng/dL
(5.2 nmol/L)

Pituitary MRI Hemochromatosis

FIGURE 13. Algorithm for evaluating male hypogonadism. FSH = follicle-stimulating hormone; LH = luteinizing hormone; PRL = prolactin;
SHBG = sex hormone-binding globulin; x2 = two separate measurements.

gels). They require daily use and may incur significant cost to rone level should be in the mid-normal range. Monitoring of
the patient, but the steady level of testosterone achieved the prostate specific antigen and hematocrit level should
within 30 minutes of application is an appealing feature. follow Endocrine Society guidelines (Table 29).
Inadvertent absorption by patient contacts may occur; users
KEY POINTS
should be informed that virilization of contacts is not uncom­
mon and premature puberty can occur in exposed children. • Testosterone deficiency should be diagnosed biochemi­
The patient should also be counseled that decline in endog­ cally, and its cause should be definitively determined
enous testosterone production and spermatogenesis may before initiation of testosterone replacement therapy.
occur. If fertility is desired, testosterone therapy should be • Patients requiring testosterone replacement therapy
avoided, and consultation with a reproductive endocrinolo­ should have testosterone, prostate specific antigen, and
gist is recommended. hematocrit levels monitored.
Patients requiring testosterone replacement therapy • Goal total testosterone level should be in the mid­
should have testosterone levels monitored at 3 and 6 months normal range for patients requiring testosterone therapy.
after initiation and annually thereafter; the goal total testoste-

59
Reproductive Disor d e r s

TABLE 29. Endocrine Society Clinical Guidelines for KEY POINTS


Monitoring Adverse Effects of Testosterone Replacement
Therapy
• Excessive muscle bulk, acne, gynecomastia, and
decreased testicular volume may be seen on physical
Parameter Recommended Alerts
Screening Schedule
examination in male patients using anabolic steroids.
• Exogenous testosterone use may result in irreversible
Hematocrit Value obtained at Value>54%
baseline and then at 3
decline in spermatogenesis and resultant infertility, as
months and 6 months well as permanent inability to produce endogenous tes­
after therapy initiation, tosterone.
followed by yearly
measurements

PSA level For patients>40 years


of age with a baseline
Increase>1.4 ng/ml
( 1.4 µg/L) in 1 year
Male Infertility
value>0.6 ng/ml or >0.4 ng/ml Physical examination should include assessment for the pres­
(0.6 µg/L), DRE and PSA (0.4 µg/L) after
level (determined at 6 months of use;
ence or absence of the vas deferens, evaluation for congenital
3 and 6 months after abnormal results on bilateral absence of the vas deferens (as seen in cystic fibrosis),
therapy initiation DRE; AUA prostate assessment of testicular volume, and evaluation for the pres­
followed by regular symptoms score/
screening) IPSS >19
ence of hernia, varicocele, or tumor. Semen analysis obtained
after 48 to 72 hours of abstinence from sexual activity is the
AUA = American Urological Association; DRE = digital rectal examination;
IPSS = International Prostate Symptom Score; PSA = prostate-specific antigen. best test to assess male fertility. For accurate results, analysis of
Data from Bhasin S, Cunningham GR, Hayes FJ, et al. Testosterone therapy in men
the sample should occur within 1 hour of ejaculation. Extended
with androgen deficiency syndromes: an Endocrine Society Clinical Practice abstinence periods may diminish fructose in the ejaculate and
Guideline. J Clin Endocrinol Metab. 2010;95(6):2550. IPMID: 20525905]
artificially lower sperm motility. If the physical examination is
abnormal, evaluation by a urologist may be appropriate. If
semen analysis results are abnormal, the test should be
Anabolic Steroid Abuse in Men repeated, and referral, if abnormal, to a reproductive endocri­
Testicular testosterone production is suppressed in the pres­ nologist is warranted.
ence of exogenous testosterone administration. Many elite
KEY POINT
athletes abuse androgens in injectable form, and herbal prep­
arations of oral testosterone are readily available. Commonly • Semen analysis obtained after 48 to 72 hours of absti­
used androgens include injectable testosterone esters and nence from sexual activity is the best test to assess male
oral alkylated testosterone preparations. HCG injections fertility; if abnormal, the test should be repeated for
mimic LH stimulation to the Leydig cells and result in ele­ confirmation.
vated testosterone levels. Although this therapy is appropri­
ate in men with hypogonadotropic hypogonadism, it may
also be abused. Aromatase inhibitors are frequently used Gynecomastia
concurrently with exogenous testosterone preparations to Gynecomastia is glandular breast tissue enlargement in men
prevent adipose conversion of estrogens to androgens and due to imbalance in the levels or activity of testosterone and
development of gynecomastia. Androstenedione supple­ estrogen. This imbalance results in an increased estrogen-to­
ments are commonly abused. testosterone ratio, which in turn results in decreased inhibi­
Excessive muscle bulk, acne, gynecomastia, and decreased tory action of testosterone on the breast tissue. The less testos­
testicular volume may be found on physical examination in terone and/or more estrogen the breast tissue is exposed to,
patients using anabolic steroids. Irreversible hypogonadism may the more likely gynecomastia will develop. Although abnor­
result and often presents as male infertility with oligospermia or mal in the postpubertal man, it is usually benign. It is typically
azoospermia on sperm analysis. Permanent inability to produce bilateral but not always symmetric. Unilateral gynecomastia is
endogenous testosterone may occur. Extratesticular effects may uncommon and should be evaluated with mammogram as
also be noted, including low HDL cholesterol level, hepatotoxic­ soon as possible owing to risk of breast cancer.
ity, erythrocytosis, and increased risk of obstructive sleep apnea. There are many causes of gynecomastia, ranging from
Mood disorders are common in anabolic steroid users. drug-induced (marijuana, alcohol, Sa.-reductase inhibitors,
Laboratory studies showing low or normal gonadotropin H2-receptor antagonists, spironolactone, digoxin, ketocona­
levels and a low testosterone level with clinical evidence of hyper­ zole, calcium channel blockers, ACE inhibitors, antiretroviral
androgenism are consistent with use of a non-testosterone­ agents, tricyclic antidepressants, selective serotonin reuptake
containing product, such as one containing androstenedione, inhibitors) and hypogonadism (primary, secondary) to chronic
or cessation of long-standing (typically greater than 1 year) illness (hepatic cirrhosis, chronic kidney disease) and endo­
anabolic steroid use, with failure to recover endogenous tes­ crine disorders (hyperprolactinemia, acromegaly, hyperthy­
tosterone function. roidism, Cushing syndrome). Obesity and aging are associated

60
Calcium and Bone Disorders

with gynecomastia owing to increased aromatase activity in In both cases, ionized calcium should be measured. It will
the periphery. Estrogen-secreting tumors (such as Leydig or usually be normal, indicating normal circulating free levels of
Sertoli cell tumors or adrenal cortical carcinoma) and HCG­ calcium. There are also instances of artificially increased cal­
secreting tumors (such as germ cell tumors and hepatic carci­ cium levels due to high protein states as in multiple myeloma
nomas) are associated with gynecomastia. (elevated monoclonal immunoglobulins), hyperalbumine­
A thorough history should be obtained. The breasts mia, Waldenstrom macroglobulinemia, and thrombocytosis.
should be examined for glandular enlargement, which typi­ In these patients, ionized calcium would be normal with ele­
cally extends concentrically from under the areolae, and is vated total serum calcium.
firm, mobile, and rubbery. The breasts may be tender if the Vitamin D is a fat-soluble vitamin, and body sources
time course is acute. Pseudogynecomastia is subareolar adi­ include de novo production from the skin, through forms
pose tissue, without glandular proliferation, that is associ­ found in food, and through supplementation (Table 30). There
ated with obesity. True gynecomastia typically distorts the are two forms of vitamin D supplementation: vitamin D2
normally flat contour of the male nipple, causing it to pro­ (ergocalciferol) and vitamin D 3 (cholecalciferoi). Although
trude owing to the mass of glandular tissue beneath it. In both forms are useful in raising vitamin D levels, vitamin D 3
pseudogynecomastia, the nipple is typically still flat but soft, may be more beneficial because of tighter bonding to vitamin
and nondescript subcutaneous fat tissue is present in the D receptors, longer shelf life, greater potency than vitamin D 2,
breast area. and being identical to the vitamin D that naturally occurs in
Mild, chronic, asymptomatic gynecomastia does not war­ humans after ultraviolet light exposure.
rant evaluation. Evaluation of gynecomastia that is asymmet­ Regardless of the method of ingestion, vitamin D 3 and D 2
ric or concerning for malignancy (bloody nipple discharge, are both inactive forms that must be hydroxylated twice before
hard and fixed, associated with regional lymphadenopathy), of becoming active. The first occurs in the liver and converts
rapid and recent onset, or larger than 2 cm (>5 cm in obese vitamin D to 25-hydroxyvitamin D [25(0H)D], also known as
men owing to the known increase in aromatase activity in calcidiol. The second occurs primarily in the kidney and forms
obesity), should include measurement of total testosterone,
LH, FSH, and TSH levels, as well as assessment of liver and
TABLE 30. Sources of Vitamin D '
kidney function. If indicated by findings on history and/or
physical examination, measurement of prolactin, estradiol, Sources Type of Amount of
Vitamin D Vitamin D
and HCG may also be indicated. If the biochemical evaluation
demonstrates abnormalities, further evaluation with testicular Food Sources
ultrasound, adrenal CT, or pituitary MRI may be indicated; Cod liver oil Cholecalciferol 400-1000 U/
consultation with an endocrinologist is recommended before teaspoon
imaging is ordered. Salmon, wild caught Cholecalciferol 600-1000 U/4 oz

KEY POINTS Salmon, canned Cholecalciferol 300 U/4 oz

• Unilateral gynecomastia in the male patient is concern­ Mackerel, canned Cholecalciferol 250 U/4 oz

ing for malignancy and warrants immediate evaluation Sundried shitake Ergocalciferol 1600 U/4 oz
with a mammogram. mushrooms

Egg yolk Ergocalciferol 20 U/yolk


HVC • Mild, chronic, asymptomatic gynecomastia in the male
patient does not warrant evaluation. Sunlight(one minimal 20,000 U in
erythermal dose) bathing suit

Fortified Foods

Calcium and Bone


Milk Cholecalciferol

Orange juice Cholecalciferol

Disorders Infant formula Cholecalciferol

Calcium Homeostasis Pharmaceutical Sources

and Bone Physiology Vitamin D2 Ergocalciferol 50,000 U/capsule


Liquid vitamin D2 Ergocalciferol 8000 U/capsule
Serum calcium levels are tightly regulated on a moment-to­
Multivitamin Ergocalciferol 400,500, or1000
moment basis by the actions of vitamin D and parathyroid and U/capsule
hormone (PTH). The amount of calcium that is albumin bound cholecalciferol
can be affected by hydration and nutritional status. If albumin Vitamin D3 Cholecalciferol 400,800, 1000,
levels decrease, total serum calcium levels may appear low 2000, 5000,
(pseudohypocalcernia). Conversely, if albumin levels increase, 10,000, 50,000
U/capsule
total calcium levels will appear elevated (pseudohypercalcernia).

61
Calcium and Bone Disorders

the physiologically active 1,25-dihydroxyvitamin D [l ,25(0H)p],


Hypercalcemia
also known as calcitriol (Figure 14).
Because 25-hydroxyvitamin D has a relatively long half­ Clinical Features of Hypercalcemia
life of several weeks, it is the best indicator of whole body Hypercalcemia is marked by serum calcium levels above the
vitamin D status. Active vitamin D acts on three organ systems normal range, usually greater than 10.5 mg/dL (2.6 mmol/L).
to achieve and maintain normal serum calcium: bone, intes­ Most patients are asymptomatic, and hypercalcemia may be
tine, and kidney. With adequate vitamin D, bone resorption is noted incidentally on laboratory tests obtained for other rea­
increased, intestinal uptake of dietary calcium is increased, sons. Symptoms may occur with any degree of hypercalce­
and excretion of calcium through the kidney is decreased. PTH mia but are more likely when serum calcium levels exceed
is secreted to increase the calcium in the blood in response to 12 mg/dL (3 mmol!L). Classic symptoms of polyuria, polydip­
even the slightest degree of hypocalcemia; it acts on the kidney sia, and nocturia sometimes occur with elevated calcium
to increase production of active vitamin D and promote cal­ levels ofll mg/dL (2.8 mmol!L) or less. Other symptoms such
cium reabsorption in the distal convoluted tubule and loop of as anorexia, nausea, abdominal pain, constipation, increased
Henle, and increased resorption in bones, thereby increasing creatinine levels, and mild mental status changes are more
release of calcium into the blood. likely to occur with levels greater than 11 mg/dL (2.8 mmol/L).
As serum calcium levels continue to increase beyond 12 mg/
dL (3 mmol/L), symptoms become more severe such as pro­
found mental status changes, obtundation, acute kidney
injury due to profound dehydration, and increased creatinine
Sunlight concentration.
(UVB)

l
KEY POINTS
Skin • Classic symptoms of hypercalcernia are polyuria, poly­
dipsia, anorexia, nausea, abdominal pain, constipation,
and mental status changes; as serum calcium levels
7-Dehydrocholesterol
increase and/or the rate of change increases, symptoms
become more severe, with profound mental status
Cholecalciferol
(vitamin D3) changes, obtundation, and acute kidney injury.
• 25-Hydroxyvitamin D has a relatively long half-life of HVC
several weeks, is the best indicator of whole body
vitamin D status, and is the recommended test for
Dietary Sources of Vitamin D
Cholecalciferol D3 (fish, meat) vitamin D deficiency.
Ergocalciferol D 2 (supplements)

Diagnosis and Causes of Hypercalcemia


When serum calcium elevation is incidentally noted, repeat
25-hydroxyvitamin D3 measurement of serum calcium is indicated, and if a second
hypercalcemic level is noted, further evaluation is warranted
to determine the cause (Table 31). The next step is determin­
ing if the hypercalcemia is PTH- or non-PTH-mediated by
simultaneous measurement of serum calcium and intact PTH
levels (Figure 15). Ionized calcium may be used in evaluating
1,25-dihydroxyvitamin D3 hypercalcemia, but it is rarely helpful in diagnosing hypercal­
Maintains calcium balance
in the body
cemia in patients with normal albumin levels or no acid-base
disturbances.

Parathyroid Hormone-Mediated Hypercalcemia


Primary Hyperparathyroidism
Primary hyperparathyroidism is the most common cause of
Calcium
Calcification PTH-mediated hypercalcemia, and is diagnosed with a simul­
resorption
taneously elevated serum calcium level, with an inappropri­
ately normal or elevated intact PTH level. The incidence peaks
Blood calcium and phosphorus in the seventh decade and affects mostly women (75%). Before
the age of 45 years, rates are similar in men and women.
FIGURE 1 4. Production of vitamin D. PTH = parathyroid hormone; Approximately 80% of patients will have elevated PTH levels
UVB = ultraviolet B. with simultaneously elevated calcium levels. Most commonly,

62
Calcium and Bon e Disord ers

TABLE 31. Causes of Hypercalcemia


may facilitate management. Serum phosphorus levels are typi­
cally low or low-normal in these patients. In contrast, phos­
Parathyroid Horm one-Mediated Hypercalcemia
phorus levels will be elevated in patients with vitamin D
Primary hyperparathyroidism (adenoma, hyperplasia) toxicity. Approximately 50% of patients with primary hyper­
Parathyroid cancer parathyroidism will have elevated urine calcium levels, and the
Tertiary hyperparathyroidism other 50% will have normal levels. Occasionally, urine calcium
can be low in those patients with concomitant primary hyper­
Familial hypocalciuric hypercalcemia
parathyroidism and vitamin D deficiency. Additionally, patients
Normocalcemic primary hyperparathyroidism
with vitamin D deficiency convert more 25-hydroxyvitamin D
Non-Parathyroid-Mediated Hypercalcemia to 1,25-dihydroxyvitamin D so they may have elevated levels of
Hypercalcemia of malignancy (humoral and local osteolytic) 1,25-dihydroxyvitamin D.
Parathyroidectomy is the treatment for primary hyper­
Vitamin D toxicity
parathyroidism. Surgical management is curative in roughly
Vitamin A toxicity
90% of patients, but evidence that the benefit outweighs the
Milk alkali syndrome risk of the surgical procedure is present under only certain
Thyrotoxicosis circumstances. There have been several long-term observa­
Prolonged immobilization tional studies that found stability in biochemical markers and
Granulomatous diseases (sarcoidosis, tuberculosis) bone density in patients who do not meet the surgical inter­
vention criteria listed in Table 32. When one or more of these
Lymphomas
criteria are met, surgery is recommended. Surgery can be
Total parenteral nutrition
considered when surgical criteria are not met, but patients
should be cautioned that there are no robust data to support
that intervention.
primary hyperparathyroidism is due to a single parathyroid It is critical that an experienced surgeon perform the sur­
adenoma; however, rarely it may be attributed to multigland gery to avoid increased risk of postoperative hypoparathy­
hyperplasia (typical in patients with end-stage kidney disease roidism and damage to the recurrent laryngeal nerve.
or multiple endocrine neoplasia syndromes) or parathyroid Historically, a bilateral neck dissection was done to identify
gland carcinoma (calcium is typically >14 mg/dL [3.5 mmol/L] parathyroid glands that appeared to have irregular appearance
and intact PTH levels >250 pg/mL [250 ng/L] on presentation; or increased size. With the increased use of sestamibi scans,
diagnosis is made histopathologically given the overlap with high-definition ultrasound, and intraoperative measurement
benign primary hyperparathyroidism). of PTH levels, the minimally invasive technique is now pre­
Once diagnosed, measurement of serum phosphorus, ferred. Minimally invasive surgery allows for a smaller incision
24-hour urine calcium, and serum 25-hydroxyvitamin D levels and a shortened surgical duration.

200

180

160

� 140
Primary hyperparathyroidism
120

100 Vitamin D
deficiency
80

60
Normal
40

20 Hypoparathyroidism

Hypercalcemia of malignancy
0
6 7 8 9 10 11 12 13 14 15

Serum calcium (mg/dl)

FIGURE 1 5. Relationship of calcium, PTH, and vitamin D status in normal conditions and in several diseases. PTH = parathyroid hormone.

63
Calcium and Bone Dis orders

TABLE 32. Indications for Surgical Intervention in Patients benign primary hyperparathyroidism, parathyroid carcinoma
with Primary Hyperparathyroidism is equally prevalent in both sexes, more commonly presents
with kidney and bone involvement and a neck mass, and fre­
Increase in serum calcium level �1 mg/dL( 0.25 mmol/L) above
upper limit ofnormal• quently is associated with a total serum calcium level greater
than 14 mg/dL (3.5 mmol/L) and very high parathyroid hor­
Creatinine clearance must be <60 mUmin (0.06 Umin)•
mone levels, typically greater than four times the upper limit
T-score (on DEXA scan) of-2.5 or worse at the lumbar spine,
total hip, femoral neck, or distal radius• of normal. Most patients present with single gland involve­
ment. Because parathyroid carcinomas may not appear histo­
Age 50 years or younger•
logically different from benign adenomas, local spread from
Surgery also indicated in patients in whom medical surveillance the capsule, distant metastasis, or lymph node involvement
is neither desired nor possible, including those with significant
bone, kidney, gastrointestinal, or neuromuscular symptoms must be present for carcinoma diagnosis. In these patients,
typical of primary hyperparathyroidism surgical resection is the treatment of choice with calcimimet­
DEXA = dual-energy x-ray absorptiometry. ics used for residual disease or in patients who are poor surgi­
.:i1n otherwise asymptomatic patients.
cal candidates. Any patient found to have parathyroid carci­
noma should be screened for the HRPT2 gene, and if positive,
Recommendations from Bilezikian JP, Khan AA, Potts JT Jr. Third International
Workshop on the Management of Asymptomatic Primary Hyperparathyroidism. family members should be screened as well.
Guidelines for the management of asymptomatic primary hyperparathyroidism:
summary statement from the Third International Workshop. J Clin Endocrinol
Metab. 2009 Feb;94(2):335-9 IPMID: 191939081
Tertiary Hyperparathyroidism
Tertiary hyperparathyroidism is the result of the prolonged
PTH stimulation needed to maintain normocalcemia resulting
In patients with osteoporosis who are poor surgical can­ from decreased 1,25-dihydroxyvitamin D levels from kidney
didates or refuse surgery, intravenous bisphosphonate therapy impairment. This prolonged stimulation results in increased
will slow bone resorption and temporarily decrease serum calcium levels and severe hyperparathyroid hyperplasia and
calcium levels. Intravenous bisphosphonate should be redosed elevated PTH levels that do not respond to phosphate binders
when hypercalcemia recurs. and calcitriol therapy. Severe bone loss and other symptoms
In patients who do not meet the criteria for surgery, sur­ make surgical resection the treatment of choice.
veillance is recommended. Patients should have annual meas­
urement of serum calcium and creatinine levels. A three-site Normocalcemic Primary Hyperparathyroidism
dual-energy x-ray absorptiometry (DEXA) scan should be Normocalcernic primary hyperparathyroidism is defined as
performed every 1 to 2 years to evaluate bone mineral density increased PTH levels in the absence of elevated calcium levels.
of the lumbar spine, hip, and distal radius. The frequency of This is a diagnosis of exclusion used in patients being evalu­
DEXA scanning should increase to yearly when any treatment ated for low bone density in which all secondary causes have
has been initiated for bone health or other medications have been ruled out, including vitamin D deficiency. Approximately
been added that may affect bone health (antiandrogen, anties­ 20% of these patients will develop hypercalcemia within
trogen , antiseizure medications, or glucocorticoids). 3 years, so they should be monitored closely.
In patients with asymptomatic primary hyperparathy­
roidism, other precautions can be taken to prevent disease­ Familial Hypocalciuric Hypercalcemia
related complications. These patients should maintain The most common form of familial hypercalcernia is familial
adequate vitamin D (400-600 U daily) intake to prevent fur­ hypocalciuric hypercalcemia (FHH). It is a rare autosomal dom­
ther PTH stimulation. In patients with concomitant vitamin D inant condition with a high penetrance that often occurs in
deficiency, repletion is recommended to replete patients childhood. These patients are frequently asymptomatic, but in
whose levels are below 30 ng/dL (75 nmol/L) with careful rare cases the calcium-sensing receptor (CASR) gene mutation
attention to urine calcium excretion and serum calcium once can increase the risk of pancreatitis. Elevated serum calcium
values are greater than 30 ng/dL (75 nmol!L). A large study did levels are caused by a mutation in the G-coupled protein CASR
not show worsening calcium levels when repleting vitamin D gene. These receptors are in the parathyroid glands and the kid­
in patients with levels less than 30 ng/dL (75 nmol/L)_ neys. The sensor mutation results in a shift upward in the "nor­
Adequate physical activity to prevent bone resorption and mal" range of calcium that the receptor recognizes, resulting in a
adequate hydration to prevent kidney damage are imperative. mildly elevated serum calcium level (usually less than 11.0 mg/
dL [2.8 mmol/L]) and high normal or mildly elevated PTH level.
Parathyroid Carcinoma An elevated PTH level is more commonly seen in patients with
Parathyroid carcinoma is very rare, accounting for less than 1 "lo concomitant vitamin D deficiency. The diagnosis is made by
of all persons with primary hyperparathyroidism. Mutations measuring 24-hour urine calcium excretion levels. Typically,
in the HRPT2 gene are thought to be the major genetic link in patients with FHH will have a 24-hour urine calcium level less
parathyroid carcinoma, and inactivation of this gene leads to than 200 mg/24 h (5.0 mmol/24 h). The preferred standard is the
familial hyperparathyroidism as well. When compared with calcium-creatinine clearance ratio, using the following formula:

64
Calcium and Bone Disorders

Ca/Cr clearance ratio= [24-hour urine Ca x serum Cr] � [serum hypercalcemia through increased bone resorption.
Ca x 24-hour urine Cr]. A ratio less than 0.01 confirms the diag­ Resolution of the thyrotoxicosis should lead to normaliza­
nosis if all other causes of hypocalciuria (thiazides, lithium, tion of calcium levels. Prolonged immobilization and
vitamin D deficiency) have been excluded. FHH is usually a increased vitamin A levels can lead to increased bone
benign condition that requires no intervention but should be resorption. Increased levels of calcium absorption from the
recognized to prevent unnecessary parathyroidectomy. gut can be from markedly high vitamin D levels or increased
intake of calcium carbonate products (milk-alkali syn­
Other Familial Hypercalcemias drome). Granulomatous diseases, such as sarcoidosis and
Familial hyperparathyroidism is another rare cause of hyper­ Wegener granulomatosis, and malignant lymphomas cause
caicemia. The disease presentation is almost identical to spo­ hypercalcemia through increased I-a-hydroxylation activ­
radic primary hyperparathyroidism, and a careful family his­ ity that increases 1,25 -dihydroxyvitamin D levels and cal­
tory will suggest the diagnosis. Once a diagnosis of primary cium reabsorption in the gastrointestinal tract.
hyperparathyroidism is made, screening for familial causes
should be done if the patient (1) is younger than 30 years of age Treatment of Hypercalcemia
at the time of diagnosis, (2) has a family history of hypercalce­ The treatment of hypercalcemia should focus on decreasing the
mia, or (3) has a medical history of other endocrinopathies. serum calcium level by increasing calcium excretion and
These patients should be tested for multiple endocrine neopla­ decreasing bone resorption or intestinal calcium absorption, as
sia syndrome types 1 and 2 (MENl and MEN2). well as volume repletion. Polyuria, due to the decreased concen­
MENl is characterized by functional pituitary adenoma, tration ability of the distal tubule, is the main cause of dehydra­
functional pancreatic tumors, and primary hyperparathyroidism. tion in these patients. Although many patients do not require
MEN2A is characterized by medullary thyroid cancer, pheochro­ hospitalization, those with marked mental status changes, acute
mocytoma, and parathyroid gland hyperplasia with the associ­ kidney injury, or calcium levels greater than 12 mg/dL (3 mmol!L)
ated RET oncogene mutation. Patients with MEN would follow should be hospitalized for treatment. First-line therapy is aggres­
similar guidelines for surgical removal of parathyroid glands. sive intravenous fluid resuscitation. Once the patient is volume
replete, an intravenous loop diuretic should be added if the cal­
Medications Causing Hypercalcemia
cium level has not normalized. Intravenous bisphosphonate
Thiazide diuretics decrease the reabsorption of calcium in the
therapy is usually given for longer-term control of hypercalce­
kidney and result in elevated levels. Primary hyperparathy­
mia. Caution should be exercised with these agents in the setting
roidism, however, should also be considered if the patient
of kidney dysfunction. Zoledronic acid, while more expensive, is
remains hypercalcemic despite the discontinuation of the thi­
a more effective therapy for patients with malignancy-related
azide diuretic. In these patients, the thiazide may have been
hypercalcemia. In patients resistant to or intolerant of bisphos­
masking the PTH-mediated hypercalcemia.
phonate therapy, off-label use of denosumab, which also reduces
Lithium decreases the parathyroid glands' sensitivity to
osteoclast-mediated bone resorption can be used. Attention
calcium and may also reduce urine calcium excretion.
should be turned as quickly as possible to treatment of the
Non-Parathyroid Hormone-Mediated Hypercalcemia underlying cause of the patient's hypercalcemia to ensure long­
In contrast to PTH-mediated hypercalcemia, non-PTH­ term maintenance of normocalcemia. If the underlying cause is
mediated hypercalcemia is associated with very low PTH lev­ increased 1,25-dihydroxyvitamin D hydroxylation, glucocorti­
els, typically less than 10 to 15 pg/dL (10-15 ng/L). coids can be effective therapy but may need to be dosed on a
regular basis. For patients who present with serum calcium
Malignancy-Associated Hypercalcemia levels greater than 18 mg/dL (4.5 mmol/L) with neurologic
There are two mechanisms of hypercalcemia of malignancy: symptoms or compromised kidney function, hemodialysis is an
local osteolytic and humoral. When lytic bone metastases are appropriate choice to quickly reduce calcium levels.
present, hypercalcemia is the result of increased mobilization
KEY POINTS
of calcium from the bone. Humoral hypercalcemia is less com­
mon and occurs when the tumor itself produces parathyroid­ • Primary hyperparathyroidism is the most common
related protein (PTHrP) that binds to and activates the cause of parathyroid hormone-mediated hypercalcemia
parathyroid receptor, raising serum calcium levels. Squamous and is diagnosed with simultaneously elevated serum
cell carcinomas, breast cancers, and renal cell carcinomas are calcium levels, with an inappropriately normal or ele­
the tumors most commonly associated with hypercalcemia of vated intact parathyroid hormone level.
malignancy. In multiple myeloma, the hypercalcemia is caused • Parathyroidectomy is curative in approximately 90% of
by the release of factors that stimulate osteoclast activity. patients with primary hyperparathyroidism, but should
be performed by an experienced surgeon using mini­
Other Causes mally invasive techniques.
Non-PTH-mediated hypercalcemia can be caused by sev­
(Continued)
eral other mechanisms. Thyrotoxicosis can lead to mild

65
Calcium and Bone Disorders

KEY POI NT S (continued) KEY POINT


• In contrast to parathyroid hormone-mediated hypercal­ • The most common cause ofhypocalcernia is hypopar­
cemia, nonparathyroid hormone-mediated hypercalce­ athyroidism, which is most often due to trauma during
mia is associated with very low parathyroid hormone surgery to the neck.
levels, typically less than 10 to 15 pg/dL (10-15 ng/L).
• The acute treatment of hypercalcemia focuses on Treatment of Hypocalcemia
decreasing the serum calcium level by increasing cal­ Mild, asymptomatic hypocalcernia (serum calcium 8.0-8.5 mg/
cium excretion with vigorous volume replacement, dL [2.0-2.1 mmol/L]), is a common finding and does not
decreasing bone resorption with bisphosphonates. require treatment.
Calcium carbonate and calcium citrate are the most com­
mon oral calcium formulations. Calcium carbonate requires
Hypocalcemia an acidic environment to be absorbed; therefore, all patients
Clinical Features of Hypocalcemia who are on proton pump inhibitors should be prescribed cal­
Hypocalcemia, defined by serum calcium levels below the cium citrate. Adequate levels of both 25-hydroxyvitamin D
normal range, may be asymptomatic ifmild. As calcium levels and 1,25-dihydroxyvitamin D are also essential. In the acute
decrease, particularly below 8.0 mg/dL (2.0 mmol!L), symp­ phase of repletion, 1,25-dihydroxyvitamin D is more effective
toms may develop, including paresthesias (numbness/tingling than 25-hydroxyvitamin D. If the patient is symptomatic or if
around the mouth, tingling in fingers and toes), muscle the hypocalcemia is acute, calcium gluconate and calcium
cramping (Trousseau and Chvostek signs), decreased muscle citrate are both available in intravenous form. Goal calcium is
strength, electrocardiogram changes (prolonged Q/T interval), 7.0 to 7.5 mg/dL (1.8-1.9 mmol/L) with intravenous repletion,
tetany, and seizures. and oral forms can be used once that goal is achieved. The
overall goal ofrepletion is the low to low-normal range (serum
Diagnosis and Causes of Hypocalcemia calcium 8.0-8.5 mg/dL [2.0-2.1 mmol/L]).
Asymptomatic hypocalcemia may be noted incidentally on If a patient requires chronic replacement, usually due to
routine laboratory tests. When this occurs, the calcium level hypoparathyroidism, care must be taken to avoid hypercalciu­
should be repeated in conjunction with a serum albumin level. ria as calcium nephrolithiasis and decreased glomerular filtra­
If hypocalcemia is confirmed, simultaneous intact PTH and tion rate can occur. Chronic replacement typically includes
serum calcium must be measured to confirm if PTH is calcitriol, calcium, and occasionally magnesium. Calcitriol is
responding appropriately. The appropriate physiologic the vitamin D source of choice because PTH is needed for
response to lower calcium levels is an elevation in PTH levels. optimal conversion of25-hydroxyvitamin D to 1,25-dihydrox­
yvitamin D. Serum calcium, magnesium, creatinine, and urine

CJI Hypoparathyroidism
I lypopar,1thyroidis111. commonly due lo trauma during neck
calcium levels should be measured at each follow-up visit. The
goal calcium levels should be low-normal without hypercal­
surgcrv (t hyroidectomy or parat hyroidcctomy). is the most ciuria. The magnesium level should ideally be greater than
common cause or hypoc,1lcc111ia. l)uri11g hc,1d ,1rnl neck sur 2 mg/dL (0.83 mmol!L) and creatinine levels should remain in
gcrics. the p,m1thyroid glands can he in,1dvcrtc11tly removed or the normal range. If the urine calcium level is greater than
p,1 r..1 thyroid hormone production rn 11 he tra nsicntly decreased 300 mg/24 h (hypercalciuria), calcium and/or vitamin D replace­
due to disruption orhlood supply. Only serial mcasLm:mcnts or ment needs to be decreased. Calcium is usually decreased first if
calciu111 levels wi 11 clctcrminc whether the damagc is t rn nsient. the vitamin D levels are within the normal sufficiency range.
I lypoparnthyroidism Glll also be caused by damage 1·ro111 racli KEY POINT
ation exposure. p,irathyrnid gland inl�1rctio11. inl'iltrative dis
• Ifhypoparathyroidism is the cause ofhypocalcemia, cor­
c,1scs (hcrnochrornatosis. Wilson disease. grc1nulo111,1s). or
rection ofany coexisting hypomagnesemia to serum
autoimmune hypo1x1rathyroiclis111. Cl
magnesium 2 mg/dL (0.8 mmol!L) or higher is necessary.
Other Causes of Hypocalcemia
Other, less common causes of hypocalcemia include poor
calcium intake, activating mutations in the CASR gene, PTH Metabolic Bone Disease
resistance, increased phosphate binding in vascular space Osteopenia and Osteoporosis
(rhabdomyolysis or tumor lysis syndrome), increased citrate Physiology
chelation with large volume blood transfusions, sepsis, vita­ Bone mineral density begins to increase with puberty, and peak
min D deficiency, and hypomagnesemia. Low levels of mag­ bone mass is achieved in early adulthood. Sex hormones, estro­
nesium (due to alcohol abuse or malnutrition) activate gen and testosterone, are crucial to increasing bone mineral
G-proteins that stimulate calcium-sensing receptors and density in women and men, respectively. Specifically, estrogen
decrease PTH secretion. has impact on osteoclast and osteoblast activity in both men and

66
Calcium and Bone Disorders

women. Towards the end of puberty, estrogen halts bone resorp­ Diagnosis
tion and signals the closure of epiphyseal plates. Bone mass The diagnosis of osteopenia is based on BMD testing.
begins to decline in women after menopause, with decreased Osteoporosis, however, can be diagnosed by BMD testing or
estrogen levels, and in men over age SO years. In men, the loss clinically in the patient with history of fragility fracture, hip
of testosterone typically accelerates bone loss after 70 years of fracture, or vertebral compression fracture.
age. Early cessation of sex hormone production in either sex, for DEXA assesses the density of the vertebral and hip bones
any reason, may accelerate the loss of bone mineral density. compared with healthy young-adult sex-matched reference
Bone loss occurs when the removal of old bone (osteoclastic values. The distal one-third of the radius can be used in
activity) exceeds the replacement with new bone (osteoblastic patients when the hip or vertebral BMD cannot be measured.
activity). Accelerated bone loss can often be attributed to hypo­ The score is based on the number of standard deviations above
gonadism or medications that promote bone loss. or below the mean reference value and is known as theT -score.
Those patients withT -scores at -1.0 and above have normal
Risk Assessment and Screening Guidelines bone density. AT -score between -1.0 and -2.S is defined as
Declining bone mineral density is associated with future frac­ low bone mass (osteopenia). Osteoporosis is defined as a
ture risk and therefore is an important component of fracture T -score below -2.S. Severe osteoporosis is defined as a T-score
risk assessment. Individual peak bone mass is determined by of -2.S or below with one or more fractures. In women and
genetic factors, nutrition, changes in hormone (estrogen, tes­ men younger than SO years, the International Society for
tosterone, thyroxine) levels, concomitant health conditions, Clinical Densitometry recommends that ethnic- or race­
and physical activity level. adjusted Z-scores be used.The Z-score compares a patient's
The National Osteoporosis Foundation recommends that BMD with others of their same age and ethnicity, and osteopo­
all postmenopausal women and men older than SO years of age rosis/osteopenia cannot be diagnosed in these patients. A
be evaluated for osteoporosis.This evaluation includes a thor­ Z-score of -2.0 or lower should be described as "low bone
ough history for potential risk factors and physical examination. mineral density for chronologic age" or "below the expected
This preliminary screening will determine if bone mineral den­ range for age." Patients with Z-scores above -2.0 are "within
sity (BMD) or vertebral imaging is necessary. Table 33 lists sev­ the expected range for age."
eral common risk factors for osteoporosis. If a patient is deemed In 2008, the World Health Organization (WHO) created
high risk, a study of BMD with DEXA may help further assess the Fracture Risk AssessmentTool (FRAX) calculator that fur­
fracture risk.The DEXA is designed to measure BMD and estab­ ther defines the 10-year fracture risk for patients with osteope­
lish risk of fracture in postmenopausal women. In young men nia, defined as aT-score between -1.0 to -2.S on DEXA.The
and premenopausal women, assessment of BMD for fracture FRAX score notes the probability of major osteoporotic fracture
risk is not advised or validated. Table 34 lists the U.S. Preventive and hip fracture in the next 10 years. If the risk of major osteo­
ServicesTask Force recommendations for BMD testing. porotic fracture is greater than or equal to 20% or the risk of hip

TABLE 33. Low Bone Mineral Density Associations


Lifestyle Comorbid Illness Hormonal States Medications Nonmodifiable Risk
Factors

Alcohol use Vitamin D insufficiency Premature menopause Anticonvulsants Race

BMI <17 Hypercalciuria Premature ovarian Glucocorticoids (�5 mg/d Age


insufficiency of prednisone or
Low calcium intake Osteogenesis imperfect Gender
equivalent for �3 months)
Panhypopituitarism
Smoking Homocystinuria First-degree relative
GnRH antagonists and
Hyperprolactinem ia with low bone mineral
Immobilization Hemochromatosis agonists
density
Androgen insufficiency
Weight loss Glycogen storage SSRls
disease Thyrotoxicosis
Malabsorptive bariatric Thiazolidinediones
surgery Cystic fibrosis
Aromatase inibitors
Gastric bypass surgery Celiac disease
Anticoagulants
Recurrent falls Cushing syndrome
Lithium
Inflammatory bowel
disease
Diabetes mellitus
(types 1 and 2)

GnRH = gonadotropin-releasing hormone; SSRI = selective serotonin reuptake inhibitor.

67
Calcium a n d Bo n e Disorders

TABLE 34. U.S. Preventive Services Task Force hypercalciuria) is an appropriate set of laboratory tests to
Recommendations for Measurement of Bone Mineral screen for secondary causes ofBMD loss. These are modifiable
Density and Vertebral Imaging conditions that, if corrected, will result in increased BMD.
Bone Mineral Density Testing• These patients are typically young with either markedly low
BMD for age or a new fracture or patients of any age with mul­
Women age 65 and older and men age 70 and older
tiple fractures.
Postmenopausal women and men age 50 to 69, based on risk With all secondary causes of decreased BMD, reversal of
factor profile
the cause should be the first line of therapy and subsequent
Those who have had a fracture, to determine degree of disease DEXA should be performed. If the BMD has not improved or
severity
the FRAX score suggests increased risk of fracture, treatment
Radiographic findings suggestive of osteoporosis or vertebral
deformity should be started. In the event of a fracture, the underlying
cause should still be addressed in addition to initiating phar­
Glucocorticoid therapy for more than 3 months
macologic therapy for osteoporosis.
Primary hyperparathyroidism
Treatment for osteoporosis (to monitor therapeutic response) Pharmacologic Treatment Options
Vertebral lmagingb
Currently there are six categories of pharmacologic agents that
are FDA approved for the treatment of postmenopausal osteo­
Women �70 and men �80 ifT-score at the spine, total hip, or porosis. These medications are bisphosphonates, calcitonin,
femoral neck is,;; -1.0
estrogens, estrogen agonists, parathyroid hormone, and the
Women aged 65-69 and men aged 75-79 ifT-score at the receptor activator of nuclear factor KB (RANK) ligand inhibitor
spine, total hip, or femoral neck is :51.5
family. Bisphosphonates are usually first-line therapy unless
In postmenopausal women age 50-64 and men aged 50-69 there is a compelling reason why another therapy should be
with the following risk factors:
used. The other modalities are typically used after a bisphos­
Low-trauma fractures
phonate failure or inability to use the medication. Most of these
Historic height loss of 1.5 in or more (4 cm) medications can also be used for prevention of osteoporosis.
Height loss of 0.8 inches or more (2 cm) Prevention therapy may be used in patients with osteopenia
Recent or ongoing long-term glucocorticoid treatment who do not meet FRAX standards for therapy but have multiple
•BMD testing should be performed at DEXA facilities using accepted quality
risk factors such as high-risk medications (glucocorticoids or
assurance measures. antiestrogen, antiandrogen, or antiseizure medications) in
bVertebral imaging should be repeated when a new loss of height is noted or new combination with a strong family history of osteoporosis.
back pain is reported.

Bisphosphonates
fracture is greater than or equal to 3%, the patient's benefit Bisphosphonate medications work by inhibiting osteoclastic
from therapy exceeds the risk, and treatment should be offered. activity.Before starting any bisphosphonate therapy, vitamin D
The FRAX was validated for use in persons 40 to 90 years status and calcium levels should be evaluated, as bisphospho­
of age who are not currently or previously treated with phar­ nates can lead to hypocalcemia. For oral bisphosphonates,
macotherapy for osteoporosis. The WHO has a web site that integrity of the esophageal lining and ability to swallow pills
offers an online FRAX calculator at www.shef.ac. uk/FRAX. are important. Kidney function should be assessed as bispho­
In additional toBMD testing, vertebral imaging using radi­ sphonates are contraindicated in patients with an estimated
ographs is recommended in high-risk groups since many ver­ glomerular filtration rate less than 35 mL/min/1.73 m2 .
tebral fractures are asymptomatic. The presence of a vertebral Although rare, osteonecrosis of the jaw has been reported with
compression fracture establishes the clinical diagnosis of osteo­ bisphosphonate usage, particularly with high-dose intrave­
porosis, regardless of T -score on DEXA, and treatment is rec­ nous administration and increased duration of the bisphos­
ommended. Once a vertebral image is obtained, it only needs to phonate. Additionally, atypical femur fractures have been
be repeated if there is noted height loss or new back pain. reported with long-term usage. Regular questioning about
pain in the thigh or groin area is recommended for patients on
Evaluation of Secondary Causes of bisphosphonates. If patients report discomfort, a radiograph
Bone Mineral Density Loss should be obtained. To reduce the risk of these side effects, a
Most cases of osteoporosis are due to declining levels of sex drug holiday has been suggested in patients with low-risk
hormones which are non-modifiable, such as age, sex, meno­ osteoporosis (T-score greater than -2.S or single fractures) who
pause, height, and build. Some patients, however, have osteo­ have been on therapy for 3 to S years with stableBMD. During
porosis caused by secondary causes. Measurement of complete bisphosphonate therapy for prevention or treatment, men SO
blood count (for malignancy), complete metabolic panel (for to 70 years of age should consume 1000 mg/d of calcium and
calcium levels and kidney function), thyroid-stimulating hor­ women aged 51 years and older and men aged 71 years and
mone, 25-hydroxyvitamin D, and urine calcium (screening for older consume 1200 mg/d of calcium.

68
Calcium and Bone Disorders

Alendronate has been approved by the FDA for preven­ Parathyroid Hormone
tion and treatment of osteoporosis. The prevention dose is a Teriparatide (recombinant human PTH [1-34]) has been FDA
5-mg tablet daily or a 35-mg tablet weekly. The treatment dose approved for the treatment of osteoporosis in postmenopausal
is a 10-mg tablet daily or a 70-mg tablet weekly. Alendronate is women and men who are at high risk for fracture. "High risk"
also approved for treatment of men with osteoporosis as well is defined as patients with a T -score of -3.0 or less or patients
as treatment of both women and men with glucocorticoid­ who have either had a fracture or decreased BMD while on
induced osteoporosis. Alendronate has been shown to reduce bisphosphonate therapy. It is also approved for men and
the incidence of spine and hip fractures by approximately SO% women at high risk of fracture due to long-time glucocorticoid
over 3 years in patients with previous fractures. use. Teriparatide has bone-building properties in addition to
lbandronate is FDA approved for the treatment and pre­ the antiresorptive properties of the other agents. It is the only
vention of postmenopausal osteoporosis. The dosage is a bone-building treatment option for osteoporosis.
150-mg tablet monthly or 3 mg every 3 months by intravenous Teriparatide is an anabolic steroid that is administered by
injection. There is FDA approval for the oral formulation for a 20-µg daily subcutaneous injection; it is approved for up to
osteoporosis prevention only. lbandronate primarily reduces 24 months over a patient's lifetime. After the 24-month dura­
risk of vertebral fractures by 50% in 3 years. tion, an antiresorptive agent (such as bisphosphonates or den­
Risedronate is FDA approved for the prevention and treat­ osumab) can be administered to maintain BMD gains achieved
ment of osteoporosis. The treatment dose is a 5-mg tablet daily, with teriparatide.
a 35-mg tablet weekly, a 75-mg tablet on two consecutive days
every month, or a 150-mg tablet monthly. Risedronate is also Receptor Activator of Nuclear Factor KB (RANK)
approved for treatment of osteoporosis in men and women Ligand Inhibitors
with glucocorticoid-induced osteoporosis. Risedronate Denosumab is a receptor activator of nuclear factor KB (RANK)
reduces the incidence of vertebral fracture by approximately ligand inhibitors that is FDA approved for the treatment of
45% and nonvertebral fractures by one-third over 3 years. osteoporosis in postmenopausal women who are at high risk
Zoledronic acid has been FDA approved for the preven­ of fracture. It is an antiresorptive agent, like the bisphospho­
tion and treatment of postmenopausal osteoporosis in women, nates with much the same effect and outcomes. It is also
for improvement of bone mass in men with osteoporosis, and approved for treatment of osteoporosis in men and those
for the prevention and treatment of glucocorticoid-induced undergoing treatment of certain cancers, such as prostate can­
osteoporosis in men and women. Recently zoledronic acid has cer, who are at high risk for fractures. Denosumab is given by
been approved for secondary prevention of fractures in subcutaneous injection (60 mg every 6 months).
patients who have had recent low-trauma hip fracture. The
treatment dose of zoledronic acid is 5 mg intravenously annu­ Annual Reassessment of Patients with Low Bone Mass
ally or once every 2 years for prevention. Once an initial DEXA scan has been obtained, every effort
should be made to have subsequent scans done on the same
Calcitonin machine. Once a repeat scan has been done, change in BMD,
Calcitonin is FDA approved for the treatment of osteoporosis not T -score, from year to year is the appropriate way to inter­
in women who are Sor more years postmenopausal. Calcitonin pret whether there has been a significant change in BMD. Most
(200 U) is delivered in a single daily intranasal spray. reports will not show a statistically significant change in the
Subcutaneous administration is also available but is used less BMD from the previous test. If not noted on the report, a cal­
f requently. Calcitonin should be used with caution with culated change of about 4% likely represents a statistically
patients who have an allergy to salmon, allergic rhinitis, or significant change. Annually, a complete clinical evaluation of
epistaxis. Very rarely, patients can also have an anaphylactic the patient, determination of risk factors for bone loss, and
response that requires emergency attention and discontinua­ evaluation for development of secondary causes of bone loss
tion of the medication. should be performed, starting with a history and physical
examination.
Estrogen Agonists and Antagonists
The use of estrogen to maintain bone health in postmenopau­ Vitamin D Deficiency
sal women has fallen out of favor because of data indicating In promoting absorption from the gut, vitamin D enables proper
that estrogen increases the risk of cardiovascular disease and bone mineralization by maintenance of calcium and phospho­
breast cancer. Therefore, estrogen use for osteoporosis preven­ rus levels. Vitamin D also modulates the actions of osteoblasts
tion should be limited to younger women with premature and osteoclasts to ensure proper bone growth and remodeling.
ovarian failure and postmenopausal women who also require Chronically low levels of vitamin D can lead to rickets in chil­
its beneficial effects for hot flushes or vaginal dryness. dren and osteomalacia in adults (see MKSAP 17 Nephrology).
Raloxifine has been approved for the treatment of post­ In addition to bone health, vitamin D plays a role in
menopausal osteoporosis. The treatment dose is a 60-mg tab­ inflammation reduction, growth regulation of various cell
let to be taken with or without food. types, immune function, and neuromuscular signaling.

69
Calcium and Bone Disorders

In assessing serum levels of vitamin D, concentrations of Vitamin D toxicity is a very rare entity but one to be aware
25-hydroxyvitamin D are the best indicator of vitamin D sta­ of. The effects of vitamin D levels greater than 90 ng/mL (225
tus. It reflects vitamin D produced cutaneously and that nmol/L) include hypercalcemia. As a fat-soluble vitamin,
obtained from food and supplements and has a circulating decreasing vitamin D levels that are once elevated can be a
half-life of 15 days. slow process requiring continued monitoring.
There are three levels of vitamin D status: sufficient
KEY POINTS
(25-hydroxyvitamin D 2':30 ng/mL [75 nmol/L]), insufficient
(25-hydroxyvitamin D 21-29 ng/mL [52.4-72.4 nmol/L]), and • The U.S. Preventive Services Task Force recommends
deficient (25-hydroxyvitamin D ::::20 ng/mL [50 nmol!L]). screening for osteoporosis in women aged 65 years and
Because vitamin D levels can be affected by sun exposure, fall older and in younger women whose fracture risk is
through winter months are ideal times to measure vitamin D equal to or greater than that of a 65-year old white
levels. It is best to measure vitamin D levels at the same time women who has no additional risk factors.
each year unless treatment is being followed. In general, the • The presence of a vertebral compression fracture makes
optimal levels of vitamin D are those that prevent PTH levels the clinical diagnosis of osteoporosis, regardless of
from increasing to above normal levels. Increased PTH levels T-score on dual-energy x-ray absorptiometry (DEXA)
will lead to increased calcium withdrawal from the bones. In scan, and treatment is recommended.
an attempt to find the optimal vitamin D level, several studies • Fracture Risk Assessment Tool (FRAX) score can help
have looked at vitamin D levels related to cancer incidence, identify which patients are most likely to benefit from
muscle stability and falls, immune status, and mood, in addi­ osteoporosis treatment; bisphosphonate therapy is first­
tion to bone health. Among most experts, a level between 30 line therapy for postmenopausal osteoporosis treatment
and 40 ng/mL (75-100 nmol/L) is deemed sufficient for pre­ and prevention.
ventive health. Based on these levels, about 30% to 60% of
• A 25-hydroxyvitamin D level between 30 and 40 ng/mL HVC
Americans have low vitamin D levels, therefore most expert
(75-100 nmo!/L} is deemed sufficient for bone health;
groups recommend screening all patients at least once. Those most expert groups recommend screening all groups at
with darker skin, decreased sun exposure, or increased
least once for evidence of deficiency since U.S. inci-
demands (pregnancy) often have low levels. dence is 30% to 60% of the population, however, it
Special populations will have lower levels of vitamin D
should not be a serial, recurring screening test.
owing to medical conditions or medication side effects. In addi­
tion, obesity has been correlated with lower vitamin D levels
possibly related to fat sequestration. Certain antiseizure medica­ Paget Disease of Bone
tions (phenobarbital and phenytoin) may increase the metabo­ Paget disease of bone is characterized by rapid and chaotic
lism of vitamin D to inactive forms. Glucocorticoids can decrease bone remodeling leading to disorganized bone microarchitec­
vitamin D metabolism. Agents that decrease absorption such as ture. This disease more commonly affects persons ofEuropean
orlistat and cholesterol lowering agents can decrease vitamin D descent during the sixth decade, with a prevalence of 3% to
absorption. Similarly, patients with malabsorption disorders, 10% in the elderly. Paramyxovirus infection of osteoclastic
including those with celiac disease and those who had bariatric precursors is thought to be one possible cause, although about
surgery, can have decreased levels of vitamin D. In these special 15% of people with Paget disease have a family member with
patient populations, not only does screening for deficiency need the disease. Paget disease appears to be inherited in an autoso­
to be more frequent, but repletion may be more challenging. It is mal dominant manner with incomplete penetrance.
recommended that these populations be given at least two to
three times more vitamin D to maintain adequate levels. Clinical Presentation
The most common clinical manifestation is asymptomatic
Recommendations elevated serum alkaline phosphatase levels; only 30% of
The current National Osteoporosis Foundation andEndocrine patients have symptoms at diagnosis. In some patients, the
Society recommendation for adults 19 to 70 years of age is at dysfunctional bone structure creates expansion in the bone
least 600 U/d of vitamin D to maximize bone health, however, leading to pain, swelling, and warmth. Bones of the axial skel­
to raise blood levels consistently above 30 ng/dL (75 nmol/L} eton are most frequently affected, namely the pelvis (70%),
may require 1500 to 2000 U/d. In adults older than 70 years of femur (55%), lumbar spine (53%), skull (42%), and tibia (30%).
age, 800 U of supplemental vitamin D per day is recom­ As a result, patients tend to have headache, sensorineural
mended to maximize bone health; however, 1500 to 2000 U/d hearing loss, and bowing of the long bones (Figure 16). The
may be required to keep levels consistently above 30 ng/dL abnormal bone growth may also lead to nerve impingement
(75 nmol/L). In treating the deficient patient, 50,000 U of causing pain or neurologic deficits. Rarely, patients can
either ergocalciferol or cholecalciferol is recommended, once develop increased vascular shunting to bones with resultant
weekly for 8 weeks. Once sufficiency is attained, maintenance right-sided heart failure, cellular transformation to osteosar­
therapy of 1500 to 2000 U/d is recommended. coma, and hypercalcemia of immobilization.

70
Bibliography

Bibliography
Disorders of Glucose Metabolism
Bergenstal RM, Klonoff DC, Garg SK, et al; ASPIRE In-HOME Study Group.
Threshold-based insulin-pump interruption for reduction of hypoglyce­
mia. N Engl J Med. 2013 Jul 18;369(3):224-32. [PMID: 23789889]
Bergenstal RM, Tamborlane WV, Ahmann A, et al; STAR 3 STUDY Group.
Effectiveness of sensor-augmented insulin-pump therapy in type l diabe­
tes. N Engl J Med. 2010 Jul 22;363(4):311-20. [PMID: 20587585]
Committee on Practice Bulletins-Obstetrics. Practice Bulletin No. 137:
Gestational diabetes mellitus_ Obstet Gynecol. 2013 Aug;l22(2 Pt 1):406-16.
[PMID: 23969827]
FIGURE 16. Radiograph showing Paget disease of bone in the left lateral Cowie CC, Rust KF. Byrd-Holt DD, et al. Prevalence of diabetes and high risk
tibia of a 71-year-old woman. Note the anterior bone, cortical thickening, and for diabetes using A\c criteria in the U.S. population in 1988-2006. Diabetes
bone enlargement as compared with the normal radiograph on the right. The Care. 2010 Mar;33(3J:562-8. [PMID: 20067953]
long-standing Paget disease resulted in a left leg that was 2.5 cm (1 in) shorter Cryer PE, Axelrod L, Grossman AB, et al. Evaluation and management of adult
than the right. hypoglycemic disorders: an Endocrine Society Clinical Practice Guideline.
J Endocrinol Metab. 2009 Mar;94(3):709-28. [PMID: 19088155]
Gregg EW. Chen H, Wagenknecht LE. et al. Association of an intensive lifestyle
intervention with remission of type 2 diabetes. JAMA. 2012 Dec
19;308(23):2489-96. [PMID: 23288372]
Diagnosis Malanda UL, Welschen LM, Riphagen II, et al. Self-monitoring of blood glu­
The diagnosis of Paget disease should be suspected in asymp­ cose in patients with type 2 diabetes mellitus who are not using insulin.
Cochrane Database Syst Rev. 2012 Jan 18;1:CD005060. [PMID: 22258959]
tomatic patients with an isolated elevation of alkaline phos­
Nathan DM; DCCT/EDIC Research Group. The Diabetes Control and
phatase without evidence of liver disease. In these patients Complications Trial/Epidemiology of Diabetes Interventions and
the most sensitive test is a nuclear bone scan which will Complications Study at 30 years: Overview. Diabetes Care. 2014Jan;37(1):9-
16. [PMID: 24356592]
detect areas of increased metabolic activity. Plain films of Qaseem A. Humphrey LL. Sweet DE, et al. Oral pharmacologic treatment of type
these areas should be obtained to identify pathognomonic 2 diabetes mellitus: a clinical practice guideline from the American College
of Physicians. Ann Intern Med. 2012 Feb 7;156(3):218-31. [PMID: 22312141]
pagetic lesions such as focal osteolysis with coarsening of the
Yeh HC, Brown TT, Maruthur N, et al. Comparative effectiveness and safety of
trabecular pattern and cortical thickening. In symptomatic methods on insulin delivery and glucose monitoring for diabetes mellitus.
patients with bone pain, plain films of painful areas may be Ann Intern Med. 2012 Sep 4;157(5):336-47. [PMID: 22777524]
the initial imaging test, although many experts recommend a Disorders of the Pituitary Gland
baseline bone scan once the diagnosis is confirmed prior to Colao A, Bronstein MD, Freda P, et al. Pasireotide versus octreotide in acro­
megaly: A head-to-head superiority study. J Clin Endocrinol Metab. 2014
initiating treatment. Mar:99(3):791-9. [PMID: 24423324]
Colao A, Petersenn S. Newell-Price J, et al. A 12-month phase 3 study of
Treatment pasireotide in Cushing's disease. N Engl J Med. 2012 Mar 8;366(10):914-24.
[PMID: 22397653]
The main therapies for Paget disease of bone are the nitrogen­
Freda PU, Beckers AM, Katznelson L. et al. Pituitary incidentaloma: an endo­
containing bisphosphonate medications (alendronate, crine society clinical practice guideline. J Clin Endocrinol Metab. 2011
pamidronate, risedronate, and zoledronic acid). The main Apr:96(4):894-904. [PMID: 21474686]
indications for antiresorptive therapy are (1) pain caused by Katznelson L, Laws ER Jr, Melmed S, et al. Acromegaly: an endocrine society
clinical practice guideline. J Clin Endocrinol Metab. 2014 Nov;99(11):3933-
the increased metabolic activity; (2) planned surgery at site of 51. [PMID: 25356808]
pagetic bone disease, and (3) hypercalcemia due to multiple Klibanski A. Clinical Practice. Prolactinomas. N Engl J Med. 2010 Apr
l:362(13):1219-26. [PMID: 20357284]
affected sites. There is no evidence that antiresorptive therapy
Korbonits M, Storr H, Kumar! AV. Familial pituitary adenomas-who should be
is beneficial in asymptomatic patients. These medications are tested for AIP mutations? Clin Endocrinol (OxO. 2012 Sep;77(3):351-6.
ideal because they suppress the rapid bone turnover that is [PMID: 22612670]
characteristic of Paget disease. Decreases in alkaline phos­ Malchiodi E. Profka E, Ferrante E. et al. Thyrotropin-secreting pituitary adeno­
mas, Outcome of pituitary surgery and irradiation. J Clin Endocrinol Metab.
phatase can be noted within 10 to 14 days after the initiation of 2014 Jun:99(6):2069-76. [PMID: 24552222]
therapy with a nadir reached in 3 to 6 months. NSA!Ds and Melmed S, Casanueva FF, Hoffman AR. et al. Diagnosis and treatment of
antineuropathic medications can also be used for pain control hyperprolactinemia: An Endocrine Society clinical practice guideline.
J Clin Endocrinol Metab. 2011 Feb;96(2):273-88. [PMID: 21296991]
in these patients. For patients with pseudofractures, orthope­ Nieman LK, Biller BMK, Findling JW, et al. The diagnosis of Cushing's syn­
dic stabilization may be required. drome: an Endocrine Society clinical practice guideline. J Clin Endocrinol
Metab. 2008 May;93(5):1526-40. [PMID: 18334580]
KEY POINTS Rajasekaran S. Vanderpump M, Baldeweq S, et al. UK guidelines for the man­
agement of pituitary apoplexy. Clin Endocrinol (OxO. 2011 Jan;74(1):9-20.
• The most common clinical manifestation of Paget dis­ [PMID: 21044119]
ease of bone is asymptomatic elevated alkaline phos­ Vasilev V, Daily AF, Petrossians P, et al. Familial pituitary tumor syndromes_
phate levels. Endocrine Practice. 2011 Jul-Aug:17 Suppl 3:41-6. [PMID: 21613050]

• The main therapy for Paget disease of bone is the Disorders of the Adrenal Glands
Cordera F. Grant C. van Heerden J, et al. Androgen-secreting adrenal tumors.
nitrogen-containing bisphosphonate medications Surgery. 2003 Dec;l34(6):874-80. [PMID: 14668717]
(alendronate, pamidronate, risedronate, and zole­ Eisenhofer G, Goldstein D, Walther M, et al_ Biochemical diagnosis of pheo­
dronic acid). chromocytoma: How to distinguish true- from false-positive test results. J
Clin Endocrinol Metab. 2003 Jun;88(6):2656-66. [PMID: 12788870]

71
Bibliography

Fishbein L, Orlowski R, Cohen D. Pheochromocytoma/Paraganglioma: Review Barbieri RL, Makris A. Randall RW, Daniels G. Kistner RW, Ryan KJ. Insulin
of perioperative management of blood pressure and update on genetic stimulates androgen accumulation in incubations of ovarian stroma
mutations associated with pheochromocytoma. J Clin Hypertens obtained from women with hyperandrogenism. J Clin Endocrinol Metab.
(Greenwich). 2013 Jun;l5(6):428-34. [PMID: 23730992] 1986 May:62(5):904-10. [PMIO: 3514651]
Funder JW, Carey RM, Fardella C. et al. Case detection, diagnosis, and treat­ Belvisi L, Bornbelli F, Sironi L. Doldi N. Organ-specific autoimmunity in
ment of patients with primary aldosteronism: an endocrine society clinical patients with premature ovarian failure. J Endocrinol Invest. 1993
practice guideline. J Clin Endocrinol Metab. 2008 Sep;93(9):3266-81. Dec;l6(11):889-92. [PMID: 8144865]
[PMID: 18552288] de Moraes Ruehsen M, Blizzard RM. Garcia-Bunuel R, Jones GS. Autoimmunity
Lenders J W, Duh QY, Eisenhofer G. et al. Pheochromocytoma and paragan­ and ovarian failure. Am J Obstet Gynecol. 1972 Mar:112(5):693-703. [PMID:
glioma: An endocrine society clinical practice guideline. J Clin Endocrinol 4551032]
Metab. 2014 Jun:99:1915-1942. [PMID: 24893135] Ferrnindez-Balsells MM, Murad MH, Lane M, et al. Clinical review 1: Adverse
Morelli V, Reimondo G. Giordano R. et al. Long-term follow-up in adrenal effects of testosterone therapy in adult men: a systematic review and rneta­
incidentalomas: an Italian multicenter study. J Clin Endocrinol Metab. 2014 analysis. J Clio Endocrinol Metab. 2010 Jun:95(6):2560-75. [PMID: 20525906]
Mar:99(3):827-34. [PMID: 24423350] Hoek A, Schoemaker J. Drexhage HA. Premature ovarian failure and ovarian
Neary N. Nieman L. Adrenal insufficiency: etiology, diagnosis and treatment. Curr autoimmunity. Endocr Rev. 1997 Feb;l8(1):107-34. [PMID: 9034788]
Opin Endocrinol Diabetes Obesity. 2010 Jun:17(3):217-23. [PMID: 20375886] Kauffman RP, CastracaneVD. Premature ovarian failure associated with auto­
Nieman LK, Biller BM, Findling JW, et al. The diagnosis of Cushing's syndrome: immune polyglandular syndrome: pathophysiological mechanisms and
an Endocrine Society Clinical Practice Guideline. J Clin Endocrinol Metab. future fertility. J Womens Health (Larchrnt). 2003 Jun:12(5):513-20. [PMID:
2008 May;93(5):1526-40. [PMID: 18334580] 12869299]
Sprung CL, Annane D, Keh D, et al. Hydrocortisone therapy for patients with Kong MF, Jeffcoate W. Eighty-six cases of Addison's disease. Clio Endocrinol
septic shock. N Engl J Med. 2008 Jan 10;358(2):lll-24. [PMID: 18184957] (OxO. 1994 Dec;41(6):757-61. [PMID: 7889611]
Young WF, Jr. Clinical practice. The incidentally discovered adrenal mass. N Legro RS, Barnhart HX. Schlaff WO, et al; Cooperative Multicenter
Engl J Med. 2007 Feb 8;356(6):601-10. [PMID: 17287480] Reproductive Medicine Network. Clomiphene, metforrnin, or both for
Zeiger MA, Thompson GB, Duh QY, et al. The American Association of Clinical infertility in the polycystic ovary syndrome. N Engl J Med. 2007 Feb
Endocrinologists and American Association of Endocrine Surgeons medi­ 8;356(6):551-66. [PMID: 17287476]
cal guidelines for the management of adrenal incidentalomas. Endocr Mignot MH, Schoemaker J. Kleingeld M, Rao BR, Drexhage HA. Premature
Pract. 2009 Jul-Aug;l5 Suppl 1:1-20. [PMID: 19632967] ovarian failure. I: The association with autoimmunity. Eur J Obstet Gynecol
Reprod Biol. 1989 Jan;30(1) :59-66. [PMID: 2647538]
Disorders of the Thyroid Gland
Moncayo-Naveda H, Moncayo R. Benz R, Wolf A, Lauritzen C. Organ-specific
American Thyroid Association (ATA) Guidelines Taskforce on Thyroid Nodules and antibodies against ovary in patients with systemic lupus erythematosus.
Differentiated Thyroid Cancer. Cooper OS, Doherty GM, Haugen BR, et al. Am J Obstet Gynecol. 1989 May;l60(5 Pt 1):1227-9. [PMID: 2729399]
Revised American Thyroid Association management guidelines for patients
with thyroid nodules and differentiated thyroid cancer. Thyroid. 2009 Plymale SR. Matej LA, Jones RE, Friedl KE. Inhibition of sex hom1one-binding
Nov;l9(ll):ll67-214. Erratum in: Thyroid. 2010Aug;20(8):942. [PMID: 19860577] globulin production in the human hepatoma (Hep G2) cell line by insulin and
prolactin. J Clin Endocrinol Metab. 1988 Sep;67(3):460-4. [PMID: 2842359]
Bartalena L. Diagnosis and management of Graves disease: a global overview.
Nat Rev Endocrinol. 2013 Dec;9(12):724-34. [PMID: 24126481] Ryan MM, Jones HR Jr. Myasthenia gravis and premature ovarian failure.
Muscle Nerve. 2004 Aug;30(2):231-3. [PMID: 15266640]
Biondi B. Natural history, diagnosis and management of subclinical thyroid
dysfunction. Best Pract Res Clio Endocrinol Metab. 2012 Aug;26(4):431-46. Vigen R, O'Donnell CI, Baron AE. et al. Association of testosterone therapy
[PMID: 22863386] with mortality, myocardial infarction, and stroke in men with low testos­
terone levels. JAMA. 2013 Nov 6;310(17):1829-36. Erratum in: JAMA. 2014
Bogazzi F, Bartalena L, Martino E. Approach to the patient with amiodarone­ Mar 5;311(9):967. [PMID: 24193080]
induced thyrotoxicosis. J Clin Endocrinol Metab. 2010 Jun;95(6):2529-35.
[PMID: 20525904] Calcium and Bone Disorders
Cooper OS, Biondi B. Subclinical thyroid disease. Lancet. 2012 Mar Al-Azem H, Khan AA. Hypoparathyroidism. Best Pract Res Clin Endocrinol
24;379(9821):1142-54. [PMID: 22273398] Metab 2012 Aug:26(4):517-22. [PMID: 22863393]
Demers LM, Spencer CA. Laboratory medicine practice guidelines: laboratory Bischoff-Ferrari H A. Willert WC. Orav E J, et al. A pooled analysis of vitamin
support for the diagnosis and monitoring of thyroid disease. Clin D dose requirements for fracture prevention. N Engl J Med. 2012 Jul
Endocrinol (OxO. 2003 Feb:58(2):138-40. [PMID: 12580927] 5;367(1):40-9. [PMID: 22762317]
Devdhar M, Ousman YH, Burman KO. Hypothyroidism. Endocrinol Metab Halick MF. Vitamin D deficiency. N Engl J Med. 2007 Jul 19;357(3):266-81.
Clin North Am. 2007 Sep:36(3):595-615, v. [PMID: 17673121] [PMID: 17634462]
Farwell AP. Nonthyroidal illness syndrome. Curr Opin Endocrinol Diabetes Halick MF, Binkley N, Bischoff-Ferrari HA, et al. Evaluation. treatment and
Obes. 2013 Oct:20(5):478-84. [PMID: 23974778] prevention of vitamin D deficiency: an Endocrine Society clinical practice
Klubo-Gwiezdzinska J, Wartofsky L. Thyroid emergencies. Med Clin North guideline. J Clio Endocrinol Metab. 2011 Jul:96(7):1911-30. [PMID: 21646368]
Am. 2012 Mar:96(2):385-403. [PMID: 22443982] Marcocci C, Cetani F. Primary hyperparathyroidism. N Engl J Med. 2011 Dec
Pearce EN, Farwell AP, Braverman LE. Thyroiditis. N Engl J Med. 2003 Jun 22;365(25):2389-97. [PMID: 22187986]
26;348(26):2646-55. Review. Erratum in: N Engl J Med. 2003 Aug Institute of Medicine. Dietary Reference Intakes for Calcium and Vitamin D.
7:349(6):620. [PMID: 12826640] Washington, DC: National Academy Press, 2010.
Siegel RD, Lee SL Toxic nodular goiter. Toxic adenoma and toxic multinodular National Osteoporosis Foundation. Clinician's Guide to Prevention and
goiter. Endocrinol Metab Clin North Arn.1998 Mar:27(1):151-68. [PMID: 9534034] Treatment of Osteoporosis. Washington, DC: National Osteoporosis
Stagnaro-Green A, Abalovich M, Alexander E. et al; American Thyroid Foundation, 2014.
Association Taskforce on Thyroid Disease During Pregnancy and Ralston SH. Pager's disease of the bone. N Engl J Med. 2013 Feb 14:368(7):644-
Postpartum. Guidelines of the American Thyroid Association for the diag­ 50. [PMID: 23406029]
nosis and management of thyroid disease during pregnancy and postpar­ Sharma OP. Hypercalcemia in granulomatous disorders: a clinical review. Curr
tum. Thyroid. 2011 Oct:21(10):1081-125. [PMID: 21787128] Opin Pulm Med. 2000 Sep:6(5):442-7. [PMID: 10958237]
Reproductive Disorders Shoback D. Hypoparathyroidism. N Engl J Med. 2008 Jul 24:359(4):391-403.
Bakalov VK,VanderhoofVH, Bondy CA. Nelson LM. Adrenal antibodies detect [PMID: 18650515]
asymptomatic auto-immune adrenal insufficiency in young women with Wolpowitz D, Gilchrest BA. The vitamin D questions: how much do you need
spontaneous premature ovarian failure. Hum Reprod. 2002 Aug;l7(8):2096- and how should you get it? J Arn Acad Dermatol. 2006 Feb:54(2):301-17.
100. [PMID: 12151443] [PMID: 16443061]

72
Endocrinology and Metabolism
Self-Assessment Test
This self-assessment test contains one-best-answer multiple-choice questions. Please read these directions carefully
before answering the questions. Answers, critiques, and bibliographies immediately follow these multiple-choice
questions. The American College of Physicians is accredited by the Accreditation Council for Continuing Medical
Education (ACCME) to provide continuing medical education for physicians.

The American College of Physicians designates MKSAP 17 Endocrinology and Metabolism for a maximum of14 AMA
PRA Category 1 CreditsTM _ Physicians should claim only the credit commensurate with the extent of their partici­
pation in the activity.

Earn "Instantaneous" CME Credits Online


Print subscribers can enter their answers online to earn CME credits instantaneously. You can submit your answers
using online answer sheets that are provided at mksap.acponline.org, where a record of your MKSAP 17 credits will
be available. To earn CME credits, you need to answer all of the questions in a test and earn a score of at least 50%
correct (number of correct answers divided by the total number of questions). Take any of the following approaches:
� Use the printed answer sheet at the back of this book to record your answers. Go to mksap.acponline.org,
access the appropriate online answer sheet, transcribe your answers, and submit your test for instantaneous
CME credits. There is no additional fee for this service.
� Go to mksap.acponline.org, access the appropriate online answer sheet, directly enter your answers, and
submit your test for instantaneous CME credits. There is no additional fee for this service.
� Pay a $15 processing fee per answer sheet and submit the printed answer sheet at the back of this book by
mail or fax, as instructed on the answer sheet. Make sure you calculate your score and fax the answer sheet
to 215-351-2799 or mail the answer sheet to Member and Customer Service, American College of Physicians,
190 N. Independence Mall West, Philadelphia, PA 19106-1572, using the courtesy envelope provided in your
MKSAP 17 slipcase. You will need your 10-digit order number and 8-digit ACP ID number, which are printed
on your packing slip. Please allow 4 to 6 weeks for your score report to be emailed back to you. Be sure to
include your email address for a response.

If you do not have a IO-digit order number and 8-digit ACP ID number or if you need help creating a username and
password to access the MKSAP 17 online answer sheets, go to mksap.acponline.org or email custserv@acponline.org.

CME credit is available from the publication date of December 31, 2015, until December 31, 2018. You may submit
your answer sheets at any time during this period.

73
Directions
Each of the numbered items is followed by lettered answers. Select the ONE lettered answer that is BEST in each case.

Item 1 Item 3
A 23-year-old woman is evaluated because of a 1-week his­ A 31-year-old woman is evaluated following her recent
tory of palpitations. She also reports some heat intolerance discovery that she is pregnant at approximately 10 weeks'
and mild anxiety during the last several weeks, but she oth­ gestation. Medical history is significant for a prolactinoma
erwise feels well. She is in the first trimester of an otherwise diagnosed 2 years ago during an evaluation for amenor­
uncomplicated first pregnancy. Her only medication is a rhea. At the time of diagnosis, her serum prolactin level
prenatal vitamin. was 184 ng/mL (184 µg/L), and a 1.4-cm pituitary adenoma
On physical examination, she is afebrile, blood pres­ extending above the sella was detected on MRI without
sure is 110/72 mm Hg, pulse rate is 105/min, and respi­ evidence of mass effect. She was treated with bromocrip­
ration rate is 13/min. BM! is 20. The skin is warm and tine with return of regular menses. She discontinued the
moist. There is no proptosis or lid lag. Examination of the bromocriptine when she found that she was pregnant. She
neck shows a diffusely enlarged thyroid with an audible is currently without symptoms. She does not have new
bruit over both lobes. Cardiopulmonary and abdominal or severe headache. Medical history is otherwise unre­
examinations are unremarkable. Neurologic examina­ markable, and her only current medication is a prenatal
tion reveals a fine resting tremor of the hands and brisk multivitamin.
reflexes. On physical examination, vital signs are normal. Visual
Laboratory studies: fields are full to confrontation, and the remainder of her
Thyroid-stimulating hormone <0.008 µU/mL examination is normal.
(0.008 mU/L) Which of the following is the most appropriate next step in
Free thyroxine (T4 ) 5.5 ng/dL (70.9 pmol/L) management?
Total triiodothyronine (T} 400 ng/dL (6.2 nmol/L)
Thyroid-stimulating 4.5 (normal <1.3) (A) Check serum prolactin level
immunoglobulin index (B) Formal visual field testing
(C) Repeat pituitary MRI
Which of the following is the most appropriate treatment? (D) Restart bromocriptine
(A) Methimazole
(B) Propylthiouracil Item 4
(C) Radioactive iodine A 48-year-old woman returns for a follow-up visit for man­
(D) Thyroidectomy agement of type 1 diabetes mellitus. She reports doing well
since the last visit. Overall, she believes that most of her
blood glucose levels are at goal, but is concerned about occa­
Item 2 sional episodes of hyperglycemia occurring in the morning
A 34-year-old man is evaluated for episodic palpitations of before breakfast. She eats a bedtime snack every night that
8 months' duration. The palpitations last 5 to 10 minutes is not covered with mealtime insulin. Review of her blood
and then resolve spontaneously. They are usually associated glucose log demonstrates morning fasting blood glucose
with sweating and anxiety. Medical history is significant for values from 80 to 190 mg/dL (4.4-10.5 mmol/L). Her other
thyroidectomy for medullary thyroid carcinoma diagnosed premeal and bedtime values range from 100 to 120 mg/dL
at 12 years of age. His father has also undergone thyroidec­ (5.5-6.7 mmol/L). She exercises two to three times per week
tomy for medullary thyroid cancer. His only medication is in the evening. Medical history is significant for hyperten­
levothyroxine. sion and hyperlipidemia.
On physical examination, blood pressure is 164/ Medications are insulin glargine, insulin lispro, rami­
92 mm Hg, pulse rate is 106/min, and respiration rate is 12/ pril, simvastatin, and aspirin.
min. Auscultation of the heart reveals a regular tachycardia On physical examination, blood pressure is 130/72
without murmurs. The remainder of his examination is mm Hg and pulse rate is 67/min. BM! is 24. The remainder
unremarkable. of the examination is unremarkable.
Laboratory studies show a 24-hour urine excretion of Results of laboratory studies show a hemoglobin
catecholamines of 310 µg/m2/24 h (1832.1 nmol/m2/24 h) A 1 c level of 6.9% and serum creatinine level of 1.0 mg/dL
and metanephrines of 3400 µg/24 h (17,238 nmol/24 h). (88.4 µmol/L). Serum electrolytes are normal.
In addition to the presenting diag nosis, which of Which of the following is the most appropriate manage­
the follow ing disorders is this patient most likely to ment of this patient's occasional fasting hyperglycemia?
develop? (A) Add insulin lispro at bedtime
(A) lnsulinoma (B) Add metformin
(B) Neurofibroma (C) Increase insulin glargine dose
(C) Primary hyperparathyroidism (D) Measure 3 AM blood glucose level
(D) Prolactinoma (E) Continue current regimen

75
Self-Assessment Test

Item 5 (C) Repeat measurement of antibodies to glutamic acid


A 70-year-old woman is seen for follow-up evaluation for decarboxylase 65 and islet antigen 2
possible Cushing syndrome. She presented with new-onset (D) Repeat measurement of fasting C-peptide and glucose
diabetes mellitus and a 9.1-kg (20-lb) weight gain over the levels
last 6 months. Medical history is otherwise unremarkable,
and she is currently taking no medications and has had no
exposure to exogenous glucocorticoids in the past year.
On physical examination, blood pressure is 160/90
Item 7
A ,13-year-old man is evaluated in the emergency depart­ Cl
mm Hg, pulse rate is 80/min, and respiration rate is 12/ ment for the ·'worst headache of my life.·· It occurred sud­
min. BM! is 30. Facial plethora, central obesity, and bilat­ denly without warning. He has had mild headaches that
eral supraclavicular fat pads are noted. There are viola­ come and go over the past 3 years. but nothing this severe.
ceous abdominal striae measuring 1 cm wide and multiple Soon after the headache began. he lost vision in his left eye.
ecchymoses on the extremities. and the vision in his right eye became blurry. He vomited
Initial laboratory studies show a serum cortisol level of twice in the emergency department. His medical history is
9 µg/dL (248.4 nmol/L} following a 1-mg dose of dexameth­ significant for progressive erectile dysfunction and loss of
asone the night before, and a 24-hour urine free cortisol libido over the past 3 years.
level that is greater than 3 times the upper limit of normal, On physical examination. temperature is 37.4 °C (99.3 °F).
which is confirmed on a second measurement. A plasma blood pressure is 156/92 111111 Hg. pulse rate is 104/min. and
adrenocorticotropic hormone (ACTH) level is undetectable. respiration rate is 16/min. BMI is 28. He has loss of vision
in his left eye and in the upper quadrants of his right eye.
Which of the following is the most appropriate diagnostic
He also has left eye ptosis. Other cranial nerves are intact.
test to perform next?
Strength and sensation in all extremities are normal as are
(A) CT scan of the adrenal glands his speech and gait.
(B) Inferior petrosal sinus sampling CT of the head shows acute pituitary hemorrhage. Pitu­
itary MRI shows a 3.1 x 2.5 x 2.2-cm pituitary mass with
(C) Late night salivary cortisol measurement
central hemorrhage. The mass compresses the optic chiasm
(D) MRI of the pituitary gland and the left cavernous sinus.
After administering high-dose glucocorticoids, which
Item 6 of the following is the most appropriate immediate
A 44-year-old man is evaluated for management of type 2 management?
diabetes mellitus. He was diagnosed with diabetes 6 months (A) Assess pituitary function
ago after being admitted to the hospital with diabetic keto­
(B) Repeat imaging in 2 weeks
acidosis. He was discharged from the hospital on a basal and
preprandial insulin regimen. Medications are regular insulin (C) Urgent transsphenoidal pituitary decompression
before meals and neutral protamine Hagedorn (NPH) insulin (D) Whole brain external beam radiation
at bedtime. He completed diabetes education and nutrition
classes and has been adherent with lifestyle modifications.
His insulin doses have been decreased gradually over the last Item 8
4 to 5 months. His most recent hemoglobin A,c level is 6.7%. A 42-year-old woman is evaluated during an annual
Blood glucose values from his log book average 130 mg/dL physical examination. She feels well. She has no perti­
(7.2 mmol!L). nent personal or fa mily medical history, and she takes no
On physical examination, temperature is 37.2 °c (99.0 °F), medications.
blood pressure is 128/68 mm Hg, and pulse rate is 72/min. On physical examination, vital signs are normal. Pal­
BM! is 30. His physical examination is unremarkable. pation of the thyroid reveals a possible nodule in the right
Laboratory studies at the time of hospital admission: lobe that is not mobile with swallowing. The remainder of
Glucose, fasting 825 mg/dL (45.8 mmol/L} the gland is unremarkable, and there is no palpable cervical
Antibody to glutamic Negative lymphadenopathy. Other physical examination findings
acid decarboxylase 65 are normal.
(GAD-65) Laboratory studies reveal a serum thyroid-stimulating
Antibody to islet antigen Negative hormone level ofl.7 µU/mL (1.7 mU/L).
2 (IA-2) Ultrasound of the neck shows a right 1.5-cm hypoechoic
C-peptide, fasting 0.5 ng/mL (0.16 nmol!L) nodule with internal microcalcifications.
Normal range: 0.8-3.1 ng/mL
(0.26-1.03 nmol!L) Which of the following is the most appropriate next step in
management?
Which of the following is the most appropriate next step in (A) CT with contrast of the neck
his management? (B) Fine-needle aspiration of the nodule
(A) Discontinue current insulin regimen, initiate sliding­ (C) Levothyroxine therapy
scale insulin (0) Measurement of serum thyroglobulin level
(B) Discontinue insulin, initiate metformin (E) Thyroid scan with technetium

76
Self-Assessment Test

Item 9 twice. Her last menstrual period was 4 months ago. She
An 18-year-old woman is evaluated for primary amenor­ also notes low libido and dyspareunia. She has not had
rhea. Her cognitive function is normal, and she is not sex­ weight changes, constipation, hair loss or hirsutism, or
ually active. Her personal and family medical history is skin changes.
unremarkable. She takes no medications. Her medical history is significant for primary hypothy­
On physical examination, temperature is 36.1 °C (97.0 °F), roidism and bipolar disorder. Medications are levothyrox­
blood pressure is 110/70 mm Hg, pulse rate is 72/min, and ine, lithium, and risperidone. She reports that she has been
respiration rate is 16/min; BM! is 20. Her height is 147 cm stable on these medications for a few years and feels well.
(58 in). Physical examination and secondary sex character­ She plans to discuss her medications with her psychiatrist
istics are normal, with Tanner stage IV breast and pubic hair prior to pregnancy.
development. On physical examination, blood pressure is 118/72 mm
Pregnancy testing is negative. On subsequent lab­ Hg and pulse rate is 82/min. BM! is 24. The thyroid is normal.
oratory studies estradiol level was undetectable, serum Visual fields are intact.
follicle-stimulating hormone level is 72 mU/mL (72 Laboratory studies:
U/L}, and serum luteinizing hormone level is 46 mU/mL Follicle-stimulating hormone 1.3 mU/mL (1.3 U/L}
(46 U/L}. Luteinizing hormone 2.0 mU/mL (2.0 U!L)
Prolactin 102 ng/mL (102 µg/L}
Which of the following is the most appropriate manage­ Thyroid-stimulating hormone 1.1 µU/mL (1.1 mU/L}
ment?
(A) Initiate estrogen and progestin therapy Which of the following is the most likely cause of her
(B) Measure serum prolactin hyperprolactinemia?
(C) Measure thyroid-stimulating hormone (A) Hypothyroidism
(D) Perform pituitary MRI (B) Lithium
(C) Pituitary adenoma
Item 10 (D) Risperidone
A 32-year-old man is evaluated for a 1-week history of
severe neck pain. He also has heat intolerance, palpitations, Item 12
and insomnia. Medical history is significant only for a viral A 55-year-old woman is evaluated for a new-patient
upper respiratory tract infection 3 weeks ago. He takes no visit. Medical history is significant for an eating disorder.
medications. Although she has maintained a normal weight for the past
On physical examination, he appears anxious and is 20 years, she notes that prior to that time her weight would
sweating. There is no proptosis or lid lag. Examination of fluctuate in a range correlating with BM!s of 17 to 19. She
the thyroid reveals a normal-sized gland that is very tender has otherwise been healthy and currently feels well. She
to palpation. There are no thyroid nodules. The heart rate is is postmenopausal and a never-smoker. Family history is
regular but tachycardic. The lungs are clear. significant for postmenopausal osteoporosis in her mother.
Laboratory studies: Her medications are over-the-counter calcium and vitamin
Thyroid-stimulating <0.008 µU/mL (0.008 mU/L} D supplements.
hormone On physical examination, temperature is 36.3 °c (97.3 °F},
Free thyroxine (T.) 3.2 ng/dL (41.3 pmol/L blood pressure is 137/81 mm Hg, pulse rate is 76/min, and
Total triiodothyronine (T) 310 ng/dL (4.8 nmol/L} respiration rate is 11/min. BM! is 21. She has mild thoracic
Thyroid-stimulating <1.3 (normal, <1.3) kyphosis but no skeletal tenderness. The remainder of the
immunoglobulin index examination is unremarkable.
24-Hour radioactive iodine 5% (low) Results of laboratory studies are significant for a serum
uptake calcium level of 9.1 mg/dL (2.3 mmol/L} and 25-hydroxyvi­
tamin D level of 40 ng/mL (99.8 nmol/L}; thyroid function
Which of the following is the most appropriate treatment? studies are normal.
Dual-energy x-ray absorptiometry (DEXA) scan shows
(A) Methimazole T-scores of -1.8 in the femoral neck and -1.9 in the lumbar
(B) Metoprolol spine. Ten-year fracture risk using the Fracture Risk Assess­
(C) Propylthiouracil ment Tool (FRAX) is 6.9% for major osteoporotic fracture
(D} Radioactive iodine and 0.7% for hip fracture. Plain radiographs of the spine
show no evidence of compression fracture.
Which of the following is the most appropriate manage­
Item 11 ment of this patient?
A 30-year-old woman is evaluated for amenorrhea. She (A) Begin raloxifene
and her husband are interested in pregnancy in the
next year, and they are concerned that they will not be (B) Repeat DEXA scan in 2 years
able to conceive. Her menses became irregular about (C) Replace calcium with cholecalciferol
2 years ago. In the past 12 months, she has had menses (D) Start bisphosphonate therapy

77
Self-Assessment Test

Item 13 weekly, 6 weeks ago. Medical history is significant for vit­


A 64-year-old man in the ICU is evaluated because of abnor­ iligo and chronic fatigue. Medications are vitamin 02 and
mal thyroid runction tests. He was admitted 3 days ago calcium carbonate.
f'or community-acquired pneumonia requiring intubation. On physical examination, temperature is 36.l °C (96.9 ° F),
mechanical ventilalion. intravenous fluids. and dopamine blood pressure is 132/71 mm Hg, pulse rate is 83/min, and
support for his blood pressure. respiration rate is 12/min. BM! is 19. The remainder of her
On physical examination. temperature is 38.8 °C examination is unremarkable.
(101.8 °F), blood pressure is 95/60 mm Hg. and pulse rate Laboratory studies:
is 130/min. ·1 he skin is warm and clry. There is no proptosis. Calcium 9.1 mg/dL (2.3 mmol/L)
l::xamination or the neck shows a normal-sized thyroid Creatinine 0.9 mg/dL (79.6 µmol/L)
without nodules. Cardiovascular examination reveals reg­ Parathyroid hormone 101 pg/mL (101 ng/L)
ular tachycardia. On neurologic examination. renexes are 25-Hydroxyvitamin 0, 7 ng/mL (17.5 nmol/L)
slightly clclayecl. after 6 weeks
The serum thyroid-stimulating hormone (TS!-1) level is 24-Hour urine calcium 150 mg/24 h (3.7 mmol/24 h)
0.1 µU ml (0.1 mU U. the serum free thyroxine level (T)
is 0.9 ng dL (11.6 pmol U. and the serum tot,11 triiodothy­ Which of the following is the most appropriate next step in
ronine (T,) level is SO ng di. (0.8 nmol U. management?
Which of the following is the most likely cause of this (A) Parathyroid sestamibi scan
patient's abnormal thyroid function? (B) Refer for parathyroidectomy
(A) Euthyroicl sick syndrome (C) Switch to vitamin 0 3 (cholecalciferol)
(B) Gra,·es disease (D) Tissue transglutaminase antibody testing
(C) Hashimoto thyroiclitis
(D) Subacule thyroiclitis
Item 16
A 54-year-old woman is evaluated because of fatigue.
Item 14 Although she follows a daily 1400-kcal diet and exercises
A 43-year-old man is evaluated during a follow-up visit for 3 to 4 nights per week for 30 minutes, she has gained
management of type 1 diabetes mellitus. He was diagnosed at 2.3 kg (5.0 lb) in the last month. She has hypercholesterol­
18 years of age and has multiple chronic complications from emia requiring statin therapy. Her mother was diagnosed
his diabetes, including end-stage kidney disease requiring with hypothyroidism shortly after the birth of her last child.
hemodialysis, gastroparesis, frequent hypoglycemia with On physical examination, blood pressure is 145/90 mm Hg,
hypoglycemic unawareness, painful peripheral neuropathy, pulse rate is 80/min, and BM! is 25. The skin is dry. The thyroid
and proliferative retinopathy. The patient uses an insulin is mildly enlarged with a diffusely nodular texture. No discrete
pump and a continuous glucose monitoring system to man­ thyroid nodules are palpated. Reflexes are normal.
age his diabetes. He is adherent with his regimen and per­ Laboratory studies:
forms multiple fingerstick blood glucose measurements with Thyroid-stimulating hormone 6.5 µU/mL (6.5 mU/L)
values ranging from 65 to 250 mg/dL (3.6-13.9 mmol/L). His (TSH)
most recent hemoglobin A1 c level is 7.5%. Free thyroxine (T) 0.9 ng/dL (11.6 pmol/L)
Which of the following is the most appropriate next step in Thyroid peroxidase antibody Positive
the management of this patient?
Similar results for TSH and T4 were obtained 4 months
(A) Alter insulin pump settings to attain a hemoglobin A1c ago.
goal of less than 7.0% Which of the following is the most appropriate next step in
(B) Alter insulin pump settings to decrease the insulin management?
doses
(C) Discontinue the insulin pump, start subcutaneous (A) Initiate levothyroxine therapy
insulin injections (B) Measure thyroid-stimulating immunoglobulins
(D) Start gabapentin for treatment of painful peripheral (C) Repeat serum TSH measurement in 12 months
neuropathy (D) Schedule thyroid radioactive iodine uptake and scan

Item 15 Item 17
A 55-year-old woman is seen in follow-up for low bone A 74-year-old woman is evaluated for a diagnosis of primary
mass and vitamin D deficiency. Cortical bone thinning was hyperparathyroidism made after an elevated serum calcium
noted on radiographs of her right ankle following a fall 3 level was incidentally discovered on laboratory studies. She
months ago. Subsequent evaluation included a dual-energy has no symptoms associated with hypercalcemia. Medical
x-ray absorptiometry (DEXA) scan showing osteopenia. Her history is significant for hypertension and chronic kidney
serum 25-hydroxyvitamin D level is 4 ng/mL (10 nmol/L). disease. Her only medication is amlodipine. She has never
She was started on 50,000 U of vitamin D2 (ergocalciferol) smoked.

78
Self-Assessment Test

On physical examination, temperature is 36.8 °c (98.3 °F), hot flushes and fatigue. She has noted galactorrhea. She
blood pressure is 134/87 mm Hg, pulse rate is 92/min, and began having headaches 2 years ago. In addition, she notes
respiration rate is 14/min. BM! is 27. The remainder of her blurry peripheral vision.
examination is unremarkable. The rest of her medical history is unremarkable. She
takes no medications.
Laboratory studies:
On physical examination, blood pressure is 112/72
Calcium 11.3 mg/dL (2.8 mmol/L)
1.3 mg/dL (114.9 µmol!L) mm Hg and pulse rate is 68/min. BMI is 21. White milky
Creatinine
76 pg/mL (76 ng/L) substance is expressed from her breasts bilaterally. Ocular
Parathyroid hormone
40 mL/min/1.73 m2 movements and cranial nerves are intact. There are no stig­
Estimated glomerular
mata of Cushing disease or acromegaly.
filtration rate
Laboratory studies:
Dual-energy x-ray absorptiometry (DEXA) scan shows Cortisol 8 AM after 1 mg of 16 µg/dL (441.6 nmol/L)
a T-score of - 1.3 in the right femoral neck. Her Fracture dexamethasone the night
Risk Assessment Tool (FRAX) score indicates a 2.1% 10-year before
probability of hip fracture and 17% 10-year probability of Estradiol <32 pg/mL (117.4 pmol/L)
any fracture. Follicle-stimulating 1.1 mU/mL (1.1 U/L)
Which of the following is the most appropriate therapy to hormone
Luteinizing hormone 0.8 mU/mL (0.8 U/L)
recommend to this patient?
Prolactin 472 ng/mL (472 µg/L)
(A) Alendronate Thyroid-stimulating 1.1 µU/mL (1.1 mU/L)
(B) Cinacalcet hormone
(C) Parathyroidectomy MR! shows a 2.4-cm pituitary tumor that elevates the
(D) Clinical observation optic chiasm and surrounds the left carotid artery.
Which of the following is the most appropriate treatment?
Item 18 (A) Cabergoline
A 74-year-old woman is evaluated because of new-onset (B) Octreotide
anxiety and insomnia. For the last 6 weeks, she has been (C) Radiation
waking up multiple times each night. She does not have
(D) Surgery
heat intolerance, change in bowel habits, palpitations, or
dyspnea on exertion. She takes no medications.

Cl
On physical examination, blood pressure is 125/68 mm Item 20
Hg and pulse rate is 89/min. BM! is 18. There is no proptosis
A 74-year-old woman is evaluated in the emergency depart­
or lid lag. Examination of the thyroid reveals a 1.5-cm firm
ment for several hours of' altered mental status. She is from
nodule in the left lobe that moves upward with swallowing.
out-of-state and is visiting with relatives. One of her young
A fine resting hand tremor is present bilaterally.
relatives was recently ill with gastrointestinal symptoms.
Laboratory studies reveal a serum thyroid-stimulat­
The patient developed anorexia 3 days ago and vomiting
ing hormone level of 0.05 µU/mL (0.05 mU/L), a serum
2 days ago. She has been unable to tolerate any liquid or solid
free thyroxine (T,) level of 2.9 ng/dL (37.4 pmol/L), and
foods for the last 24 hours. Medical history is significant for
a serum total triiodothyronine (T) level of 250 ng/dL
Lype 2 diabetes rnellitus. hypertension. hyperlipidemia, and
(3.8 nmol/L).
hypothyroidism. Medications are aspirin. lisinopril. glime­
Ultrasound of the neck shows two thyroid nodules, a
piride, levothyroxine, and atorvastatin. Her last dose of med­
1.5-cm nodule in the right lobe and a 2.0-cm nodule in the
ications was 48 hours ago.
left lobe.
On physical examination, her temperature is 37.5 °C
Which of the following is the most appropriate next step in (99.5 °F). blood pressure is 115/65 mm Hg, and pulse
management? rate is 95/min. She is arousable but confused. Mucous
membranes are dry. Her neck is supple. Cardiac exam­
(A) Fine-needle aspiration of both thyroid nodules ination reveals no murmurs. Her chest is clear Lo auscul­
(B) Initiation of methimazole tation. Bowel sounds are present. and mild tenderness
(C) Radioactive iodine (123 1) uptake and scan of the thy­ Lo palpation is noted throughout the abdomen. ·n1ere is
roid no rebound or guarding. There are no focal neurologic
(D) Total thyroidectomy deficits.
Laboratory studies are pending.
Which of the following is the most likely cause of this
Item 19 patient's altered mental status?
A 34-year-old woman is evaluated for amenorrhea, head­
(A) Cerebrovascular accident
ache, and fatigue. She reports that from the time of menarche
until 2.5 years ago, her menses were regular and predictable. (B) Hypoglycemia
Two and a half years ago, her menses became irregular and (CJ Hypothyroidism
then stopped completely 6 months ago. She has had a few (D) Slatin toxicity

79
Self-Assessment Test

Item 21 with a round face. She has terminal hairs on her chin, upper
A 38-year-old woman is evaluated because of a 3-week lip, chest, and back. Mild facial acne, central obesity, and a
history of palpitations. She notes that her heart "races" few wide purple striae on the back of her arms are also noted.
at night and after minimal exertion. She also report: l·�?t She has supraclavicular fat. Her skin has psoriatic plaques.
intolerance but has no change in bowel habits or menses. Muscle strength in the upper and lower extremities is 4/5.
On physical examination, the patient is restless and Which of the following is the most likely diagnosis?
has pressured speech. Temperature is 36.8 °c (98.2 °F),
blood pressure is 130/60 mm Hg, pulse rate in llO/min, and (A) Adrenocortical carcinoma
respiration rate is 12/min. Her skin is warm and moist, and (B) Cushing disease
a bilateral hand tremor is present. There is no proptosis or (C) Ectopic adrenocorticotropic hormone production
lid lag. The thyroid is enlarged without nodules or bruits. (D) Iatrogenic Cushing syndrome
Serum thyroid-stimulating hormone level is 0.08 µU/rnl
(0.08 mU/L), and the serum free thyroxine (T4) level is 1.7
ng/dL (21.9 pmol/L).
Which of the following is the most appropriate next step in
Item 24
A 20-year-old woman is evaluated in the emergency depart­ Cl
management? ment for polyuria. polydipsia, polyphagia. and an uninten­
(A) Measure serum triiodothyronine (T) level tional 5.Ll-kg (ll.9-lb) weight loss over the past month.
She has had increasing lethargy over the last 24 hours. Her
(B) Measure serum thyroid peroxidase antibody titer
medical history and family history are unremarkable. She
(C) Repeat thyroid function tests in 6 weeks takes no medications.
(D) Schedule ultrasound of the neck On physical examination. temperature is 37.5 °C
(99.5 °F). blood pressure is 98/52111111 Hg, pulse rate is 120/
min. and respiration rate is 30/min. BMI is 17. She is lethar­
Item 22 gic with dry mucous membrane . tachypnea, and tachy­
A 34-year-old woman is evaluated for a diagnosis of hyper­ cardia. Chest auscultation is clear. Abdominal examination
calcemia after presenting to the emergency department 3 shows difl'use mild tenderness and normal bowel sounds.
days ago for treatment of a kidney stone. She presented with There is no rebound tenderness or guarding with palpation.
severe right flank pain with nausea and vomiting. A 2-mm Laboratory studies:
kidney stone was identified in the right ureter by ultraso­ Hemoglobin 17 g/dL (170 g/L)
nography that passed spontaneously; multiple additional Leukocyte count 14.200/�lL (14.2 x 10"/L)
intrarenal calcifications were noted to be present. Labora­ Blood gases. arterial
tory studies at that time showed normal kidney function pH 7.25
and a serum calcium level ofll .5 mg/dL (2.9 mmol/L). Med­ PCO., 21111111 Hg (2.8 kPa)
ical history is otherwise significant for hypertension and Creatinine 1.3 mgldL (114.9 µmollL)
sarcoidosis. Her only medication is hydrochlorothiazide. Electrolytes
On physical examination, temperature is 36.8 °C (98.2 °F), Sodiu111 130 mEq/L (13011111101/L)
blood pressure is 138/87 mm Hg, pulse rate is 89/min, and Potassium 3.0 mEqlL (3.011111101/L)
respiration rate is 12. BM! is 32. The remainder of the exam­ Chloride 99 mEq/L (99 mmol/L)
ination is unremarkable. Bicarbonate 9 mEq/L (9 11111101/L)
Laboratory studies are significant for a parathyroid Glucose 620 mg/dL (34.4 mmol/L)
hormone level of 4 pg/mL (4 ng/L). Lactic acid 8 mg/dL (0.89 mmol/L)
Which of the following is most likely responsible for caus­ Intravenous 0.9'Y., saline is initiated through a central
ing this patient's hypercalcemia? venous catheter.
(A) Calcium-sensing receptor mutation An electrocardiogram shows sinus tachycardia 120/min.
Chest radiograph is normal.
(B) Elevated 1,25-dihydroxyvitamin D levels
(C) 25-Hydroxyvitamin D level Which of the following is the most appropriate next step
(D) Thiazide-induced renal calcium reabsorption in the management?
(A) Administer intravenous ceftriaxone
(Fl) Administer intravenous potassium chloride
Item 23
(C) Administer intravenous sodium bicarbonate
A 40-year-old woman is evaluated for amenorrhea of 4 (D) Initiate intravenous insulin therapy
months' duration. She has had weight gain, facial hair,
alopecia, and debilitating fatigue. Her medical history is
significant for psoriasis. She seems to be gaining weight in
her face, abdomen, and neck. She also bruises easily. Her Item 25
only medication is clobetasol for psoriasis. A 65-year-old woman is evaluated following a recent diag­
On physical examination, temperature is 37.6 °C nosis of osteoporosis discovered on a screening dual-energy
(99.7 °F), blood pressure is 148/90 mm Hg, pulse rate is 88/ x-ray absorptiometry (DEXA) scan that showed T-scores
min, and respiration rate is 12/min. BM! is 38. She is obese of -2.5 at the femoral neck and -2.7 at the hip. Overall she

80
Self-Assessment Test

feels well, although she notes a 5-cm (2-in) loss of height striae measuring 8 to 12 mm wide are noted on his upper
over the past 15 years and a 2.3-kg (5-lb) weight loss over arms and abdomen. There isl+ bilateral lower extremity
the last year. She is postmenopausal. Medical history is edema. Multiple ecchymoses and acanthosis nigricans
unremarkable, and she is a never-smoker. Family history are present.
is negative for osteoporosis or nontraumatic fractures. She Laboratory studies:
takes no medications. Adrenocorticotropic <5 pg/mL (1.1 pmol/L)
On physical examination, temperature is 36.8 °c hormone
(98.2 °F), blood pressure is 144/68 mm Hg, pulse rate is 92/ 24-Hour urine cortisol
min, and respiration rate is 14/min. BM! is 22. The remainder excretion
of the examination is unremarkable. Initial measurement 280 µg/24 h (771.6 nmol/24 h)
Laboratory studies are significant for a normal basic Repeat measurement 300 µg/24 h (826.7 nmol/24 h)
metabolic profile and complete blood count. Serum calcium Cortisol, serum 46 µg/dL (1269.6 nmol/L)
is 9.5 mg/dL (2.4 mmol/L) and serum phosphorus is 3.8 mg/ Urine
dL (1.2 mmol/L). Serum 25-hydroxyvitamin D level is 32 ng/ Catecholamines 40 µg/m2/24 h (236.4 nmol/m2/
dL (79.9 nmol/L). 24 h)
Which ofthe following tests is indicated prior to initiation Metanephrines 1000 µg/24 h (5070 nmol/24 h)
ofpharmacologic therapy?
CT scan of the abdomen with and without contrast
(A) Serum and urine markers of bone turnover reveals a 5.6-cm heterogeneous right adrenal mass with
(B) Serum estradiol level focal areas of calcifications and hemorrhage. The density of
(C) Serum parathyroid hormone level the mass is SO Hounsfield units, and the contrast washout
at 10 minutes is 20%.
(D) Serum thyroid-stimulating hormone level
Which ofthe following is the most appropriate next step in
the management ofthis patient's adrenal mass?
Item 26
(A) Chemotherapy
A 24-year-old woman is evaluated for excessive men­
strual bleeding. She was recently diagnosed with polycys­ (B) Fine-needle biopsy
tic ovary syndrome during an evaluation for hirsutism. (C) Radiation therapy
Menarche occurred at age 11 years, and she has always (D) Surgical excision
had irregular menses occurring approximately every 60
days. However, her periods over the past year have been
associated with heavy bleeding that is increasingly both­
ersome. Medical history is otherwise unremarkable, and
Item 28
/1. 47-year old man presents to the emergency department CJ
she takes no medications. She currently does not desire with weakness and shakiness. I-le has a long history ofalco
fertility. hol abuse but significantly decreased his usual daily alcohol
On physical examination, she is afebrile. Blood pres­ int<1ke over the past week because of gastrointestim1I upset.
sure is ll0/70 mm Hg, pulse rate is 78/min, and respiration nausea. and diarrhea. Medical history is ol hcrwise unre
rate is 14/min. BM! is 32. Excess terminal hair growth is markable. and he takes no medications.
present on the upper lip, chin, and chest. The physical On physical examination. the patient is awake and
examination is otherwise normal. oriented but tremulous. Mucous membranes are dry.
A urine pregnancy test is negative. Temperature is 37.:3 °C (99.1 °F). blood pressure is 139 76
111111 I lg, pulse rate is IOI/min. and respiration rate is IS
Which ofthe following is the most appropriate therapy for min. f3MI is 19. Cardiopulmonary examination is unre
this patient? mark<1ble. The abdomen is diffusely tender lo palpation.
(A) Combined oral contraceptive pills Periodic spontaneous twitching is noted in the major
(B) Levonorgestrel intrauterine system muscle groups.
Laboratory studies arc signifkanl for a serum cal­
(C) Metformin
cium level or 6.5 mg di. (1.6 mmol/1.). serum albumin
(D) Periodic progestin withdrawal level of 2.6 g/dL (26 g/L). serum potc1ssium or 3.4 mEq/L
(:l.4 mmol/L). and norm,11 kidney runction studies. Serum
Item 27 parathyroid hormone and 25 hydroxyvitamin D levels
are pending.
A 55-year-old man is evaluated for abdominal fullness and An electrocardiogram shows prolongation or the QT
nausea of 2 weeks' duration. He has no vomiting or fever. interval.
One month ago, he was diagnosed with type 2 diabetes
mellitus. He reports an unintentional weight loss ors kg (11 Which of"the following is the most appropriate next diag­
lb) over the past month, generalized weakness, and poor nostic test to evaluate this patient's hypocalcemia?
appetite. Metformin is his only medication. (A) l.25-Dihydroxyvitamin D level
On physical examination, blood pressure is 158/90
mm Hg and pulse rate is 90/min. BM! is 29. His face (B) 24-l-lour urine calcium cxcretion
is round and red. A dorsocervical fat pad is present. (C) Ionized calcium level
His abdomen is distended, but nontender. Violaceous (D) Magnesium level

81
Self-Assessment Test

Item 29 (C) Serum ferritin level and transferrin saturation


A 52-year-old woman presents for follow-up evaluation (D) Testicular ultrasound
after being diagnosed with type 2 diabetes mellitus 6 weeks
ago. Her initial hemoglobin A1c level was 8.0%. Management
at this time is with lifestyle modifications. She has worked Item 31
closely with a diabetes educator and a nutritionist since her A 59-year-old man is evaluated for hypercalcemia. He was
diagnosis. She has lost 3.2 kg (7 lb) by making changes to her recently diagnosed with multiple myeloma. He does not
diet and activity level. Review of her blood glucose log for have anorexia, nausea, constipation, polydipsia, polyuria,
the past 2 weeks shows preprandial blood glucose values or confusion. Medical history is otherwise unremarkable,
in the 150 to 160 mg/dL (8.3-8.9 mmol/L) range and several and he takes no medications.
2-hour postprandial blood glucose values ofl 90 to 200 mg/dL On physical examination, temperature is 36.4 °C
(10.5-11.l rnmol!L). Her only other medical problem is ( 97.5 °F), blood pressure is 134/80 mm Hg, pulse rate is 80/
hypertension for which she takes lisinopril. min, and respiration rate is 12/min. BM! is 30. The remainder
On physical examination, blood pressure is 125/70 mm of his physical examination is normal, and no weakness is
Hg and pulse rate is 74/min. BMI is 28. There is no evidence noted on neurologic examination.
of diabetic retinopathy. She has normal rnonofilarnent and Serum calcium level is 10.8 mg/dL (2.7 rnmol/L).
vibratory sensation in her extremities.
Except for her blood glucose parameters, basic labo­ Which of the following is the most appropriate next labora­
ratory studies obtained at the time of her initial diagnosis tory test for evaluating this patient's hypercalcemia?
were normal. (A) 1,25-Dihydroxyvitamin D level
In addition to continuing lifestyle modifications, which of (B) Ionized calcium level
the following is the most appropriate management for this (C) Parathyroid hormone level
patient's diabetes? (D) Parathyroid hormone-related protein level
(A) Initiate dapagliflozin
(B) Initiate glipizide
Item 32
(C) Initiate rnetformin
A 37-year-old man is evaluated for a 2-year history of low
(D) Initiate sitagliptin
libido, loss of morning erections, fatigue, and decreasing
muscle mass. His medical history is otherwise unremark­
Item 30 able. He takes no medications.
On physical examination, vital signs are normal. BM!
A 28-year-old man is evaluated for fatigue and erectile is 35. The remainder of the examination, including genital
dysfunction. His symptoms have been progressive over
examination, is normal.
the past year. He notes decreased libido and reports loss
of morning erections. He also feels tired, has difficulty Laboratory studies:
concentrating, and notes diffuse joint aches. He believes Follicle-stimulating hormone 12 mU/rnL (12 U/L)
he has less strength and has had to decrease his level of Luteinizing hormone 10 mU/rnL (10 U/L)
exercise. Prolactin Normal
Medical history is unremarkable. He had normal Morning testosterone (total) 148 ng/dL (5.1 nrnol/L)
puberty and normal growth. He takes no medications. Confirmatory morning 137 ng/dL (4.7 nmol/L))
On physical examination, temperature is 37.4 °c (99.3 °F), testosterone (total)
blood pressure is 108/72 mm Hg, pulse rate is 68/rnin, and Thyroid-stimulating hormone Normal
respiration rate is 14/rnin. BM! is 23. The liver edge is pal­
A pituitary MRI is normal.
pable 4 cm below the costar margin. The penis is normal,
and the testes are normal volume but soft and freely mobile Before initiating therapy for this patient, which of the fol­
without masses. Visual fields are intact. lowing should be determined?
Laboratory studies: (A) Bone mineral density
Follicle-stimulating hormone 3.0 rnU/rnL (3.0 U/L)
(B) Desire for fertility
Luteinizing hormone 2.2 rnU/rnL (2.2 U/L)
(C) Fasting plasma glucose level
Prolactin 12 ng/rnL (12 µg/L)
Testosterone. total (8 AM) 178 ng/dL (6.2 nrnol/L) (D ) Scrotal ultrasound
Testosterone, total (8 AM), 162 ng/dL (5.6 nrnol/L)
repeated
TI1yroid-stirnulating hormone
Pituitary MRI is normal.
2.3 µU/rnL (2.3 rnU/L) Item 33
A 55 year-old woman is evaluated in the emergency C]
department because of' a 1-week history of' palpitations,
Which of the following is the most appropriate next step in chest pain. shortness of' breath. diarrhea. ancl weight loss.
management? Medical history is significant f'or an episode of' chest pain
--1 weeks ago: the chest pain was evaluated with an cxcr
(A) Begin testosterone replacement therapy cise stress test. vv hich was positive. i\ subsequent c<irck1c
(B) Karyotyping cathcterization was negative f'or epica rdial coronary artery

82
Self-Assessment Test

Cl disease. She also h,is a history or Grnves disease treated


with daily methimazole: she slopped this medication on
(C) Continue current treatment regimen
CONT.
(D) Decrease calcium supplementation
the day of her cathelerization and has not restarted it since
that time.
On physical examination, she is restless and conf"used. Item 35
Temperature is 38.9 °C (102.0 °F). blood pressure is 175/
A 40-year-old man with type 1 diabetes mellitus presents to
94 111111 Hg. pulse rate is 135/rnin and regular, and respirn­
the office. He seeks advice on his diabetes management as
tion rnte is 20/min. The skin is warm and moist. "!here is
he intensifies his exercise routine in preparation for partic­
mild proptosis with sclcral injection. [xamination or the
ipation in a 10-K race. He reports prolonged hypoglycemia
thyroid reveals a diffusely enlarged gland lhal is nonlender
during intense exercise, despite eating a meal prior to the
lo palpation. A thyroid bruit is heard. i\bclominal examina­
activity. His insulin regimen is insulin glargine and insulin
tion is unremarkable. 1l1ere is edema or lhe lower extremi­
glulisine. His most recent hemoglobin A,c level was 7.0%.
ties to lhe mid-lower leg. On neurologic examination, she is
oriented lo place but not lo time, as she does nol know the Which of the following is the most appropriate manage­
correct year. ment of this patient's hypoglycemia on the days that he
Laboratory studies: exercises?
Leukocyte count 10,.500/µl. ( 10 . .5 X 10''/L) (A) Decrease meal-time insulin glulisine dose prior to
Crealinine l.3 mg/dL (1 ts µ1110! L) exercise, continue insulin glargine dose
1l1yroid-slimulal ing <0.008 µU/ml. (0.008 rnU I.)
(B) Discontinue insulin glargine, continue insulin glulis­
hormone
ine dose
Free thyroxine (T,) 7.5 ng/dl_ (96.7 prnol/L)
(C) Increase meal-time protein prior to exercise, continue
current insulin doses
Which of the following is the most likely diagnosis?
(D) Switch insulin glulisine to a sliding-scale regimen,
(A) Euthyroid sick syndrome continue insulin glargine dose
(B) Pheochromocytoma
(C) Subacule thyroiditis
Item 36
(D) ·niyroid storm
A 54-year-old man is evaluated because of fatigue. He also notes
constipation and cold intolerance. Medical history is significant
Item 34 for tonsillar squamous cell carcinoma treated with radiation
3 years ago. There is no family history of thyroid disorders.
A 39-year-old man is seen for a follow-up examination. On physical examination, the skin is dry. Mild peri­
Three months ago, he underwent an organ-sparing pro­ orbital edema is present. The thyroid is of normal size and
cedure for squamous cell carcinoma of the throat. As part without nodules. Reflexes are delayed.
of his surgery, his thyroid and all four parathyroid glands Laboratory studies show a hemoglobin level ofll g/dL
were removed. He was started on 1,25-dihydroxyvitamin D (110 g/L), a serum sodium level of 129 mEq/L (129 mmol/L),
(calcitriol) and supplemental calcium carbonate following and a serum thyroid-stimulating hormone (TSH) level of
surgery. He subsequently completed a course of concur­ 1.4 µU/mL (1.4 mU/L).
rent cisplatin-based chemotherapy and radiation therapy
without significant complications and currently feels well Which of the following is the most appropriate next step in
with no new signs or symptoms. Medical history is oth­ management?
erwise unremarkable except for a 25-pack-year smoking
(A) Free thyroxine (T) measurement
history; he discontinued tobacco use at the time of diag­
nosis. Medications are calcitriol, calcium carbonate, and (B) Repeat TSH measurement in 4 weeks
levothyroxine. (C) Thyroid scintigraphy
On physical examination, temperature is 37.0 °C (98.6 °F), (D) Thyroid ultrasound
blood pressure is 125/84 mm Hg, pulse rate is 85/min, and
respiration rate is 12/min. BM! is 24. Well-healed surgical
incisions are noted on the anterior neck. The remainder of Item 37
the examination is unremarkable. A 24-year-old woman is evaluated for hypercalcemia inci­
Laboratory studies: dentally discovered on laboratory studies performed for
Calcium 8.5 mg/dL (2.1 mmol!L) another indication. She reports no hypercalcemia-related
Creatinine 0.8 mg/dL (70.7 µmol/L) symptoms. Medical history is significant for gastroesopha­
1,25-Dihydroxyvitamin D Within therapeutic range geal reflux disease and menstrual migraine. Family history
24-Hour urine calcium 375 mg/24 h (9.4 mmol/24 h) is notable for a brother who has a "calcium" problem. Medi­
cations are omeprazole and sumatriptan as needed.
Which of the following is the most appropriate manage­ On physical exaniination, temperature is 36.2 °C (97.2 °F),
ment of this patient's postsurgical calcium therapy? blood pressure is 127/68 mm Hg, pulse rate is 73/min, and
respiration rate is 12/min. BM! is 25. Chest, heart, and
(A) Check 25-hydroxyvitamin D level abdominal examinations are normal, as is the remainder of
(B) Check parathyroid hormone level her examination.

83
Self-Assessment Test

Laboratory studies are significant for a serum calcium serum cortisol following a low-dose overnight dexameth­
level ofll .2 mg/dL (2.8 mmol!L), parathyroid hormone level asone suppression test. Subsequent testing revealed a sup
of 55 pg/mL (55 ng/L), and 25-hydroxyvitamin D level of 35 pressed adrenocorticotropic hormone level.
ng/mL (87.4 nmol!L). Kidney and thyroid function studies A contrast-enhanced adrenal CT scan revealed a
are normal. well-circumscribed 3.7-cm right adrenal mass with a con­
trast washout of greater than 50% at 10 minutes.
Which of the following is the most appropriate next step in
management? Which of the following is the most appropriate perioper­
ative management?
(A) Bone densitometry
(B) Measurement of urine calcium and creatinine levels (A) Postoperative hydrocortisone
(C) Parathyroid sestamibi scan (B) Postoperative mitotane
(D) Referral for parathyroidectomy (C) Postoperative norepinephrine
(D) Preoperative phenoxybenzamine

Item 38
A 25-year-old woman with type 1 diabetes mellitus is eval­ Item 40
uated for recent-onset glycemic fluctuations without symp­ A 62-year-old man is evaluated for right leg pain. The pain
tomatic hypoglycemia. She was diagnosed with diabetes 7 has developed progressively over the past 6 months and
years ago. Her hemoglobin A1c levels since diagnosis have worsens with prolonged activity, such as playing golf. Med­
ranged from 6.4% to 7.3%, with the most recent value at ical history is unremarkable, and he takes no medications.
7.3%. She reports eating a carbohydrate-consistent diet at On physical examination, temperature is 36.3 °C (97.3 °F),
each meal, with little variation in her selection of meals blood pressure is 128/67 mm Hg, pulse rate is 73/min, and
or snacks. She started a new job several months ago but respiration rate is 10/min. BMl is 29. There is no pain to
continues her daily exercise routine and sleep schedule. She palpation over the femoral region, and he has normal range
has no other medical problems or symptoms. Her diabetes of motion.
treatment regimen is insulin glargine once daily and insulin Laboratory studies are significant for a serum alkaline
lispro three times daily. phosphatase level of 200 U/L and serum calcium level of 9.0
Physical examination findings and vital signs are nor­ mg/dL (2.3 mmol!L).
mal. Plain radiograph of the right femur is shown.
Estimate glomerular filtration rate, serum creatinine
level, and urine albumin-creatinine ratio are normal. Her
blood glucose values from the previous week are shown
below.
Blood glucose values:
Breakfast Lunch Dinner Bedtime
(mg/dL (mg/dL (mg/dL (mg/dL
[mmol!L]) [mmol/L]) [mmol!L]) [mmol!L])
124 (6.9) 190 (10.5) 109 (6.1) 210 (11.6)
110 (6.1) 92 (5.1) 112 (6.2) 126 (7.0)
115 (6.4) 118 (6.5) 112 (6.2) 126 (7.0)
117 (6.5) 127 (7.0) 204 (11.3) 110 (6.1)
108 (6.0) 101 (5.6) 122 (6.8) 114 (6.3)
101 (5.6) 111 (6.2) 106 (5.9) 72 (4.0)
126 (7.0) 187 (10.4) 102 (5.7) 196 (10.9)

Which of the following is the most likely cause of the fluc­


tuating glycemic control?
(A) Antibodies to exogenous insulin
(B) Gastroparesis
(C) Inadequate insulin doses
(D) Inappropriate insulin timing

Which of the following is the most appropriate next step in


Cl A 20-year-old woman is admitted to the hospital for an
Item 39 management?
(A) Antiresorptive therapy
elective adrenalectomy. Her history is significant for recent
onset of hypertension. weight gain. generalized weakness. (B) Bone biopsy
and easy bruising. Laboratory evaluation demonstrated (C) Evaluation for multiple myeloma
an elevated urine rree cortisol and lack of suppression of (D) Clinical observation

84
Self-Assessment Test

Item 41 Item 43
A 29-year-old woman is evaluated during her first prenatal A 44-year-old woman is evaluated for weight gain, muscle
visit. She feels well. Medical history is significant only for weakness, and metabolic syndrome. She has hirsutism but
hypothyroidism, for which she has taken levothyroxine, also notes hair loss on her head. She has been amenorrheic
100 µg/d, for the last 3 years. Her only other medication is a for 2 years.
prenatal vitamin. Medical history is significant for hyperlipidemia, type
On physical examination, she is afebrile. Blood pres­ 2 diabetes mellitus, hypertension, and obesity. Medications
sure is 98/72 mm Hg, pulse rate is 88/min, and respiration are atorvastatin, metformin, and lisinopril.
rate is 12/min. The thyroid is of normal size and without On physical examination, blood pressure is 156/92 mm
nodules. There is no cervical lymphadenopathy. Cardio­ Hg and pulse rate is 78/min. BM! is 42. She has a cushingoid
vascular and pulmonary examinations are unremarkable. appearance, acne, and moderate hirsutism affecting the chin,
Abdominal examination reveals normal bowel sounds. The upper lip, breasts, back, and chest. There are several wide vio­
uterus is not palpable. laceous striae across the abdomen and the back of her arms.
Laboratory studies show a serum thyroid-stimulating Laboratory studies:
hormone level of 3.6 µU/mL (3.6 mU/L) and a serum total Adrenocorticotropic 52 pg/mL (11.4 pmol/L)
thyroxine (T4) level of 4.5 µg/dL (58 nmol/L). hormone (ACTH)
Which of the following is the most appropriate treatment of Cortisol 8 AM after 1 mg of 5.2 µg/dL (143.5 nmol/L)
this patient's hypothyroidism? dexamethasone the night
before
(A) Continue levothyroxine dose 24- Hour urine cortisol
(B) Decrease levothyroxine dose excretion
(C) Discontinue levothyroxine Initial measurement 124 µg/24 h (341.7 nmol/24 h)
Repeat measurement 98 µg/24 h (270.0 nmol/24 h)
(D) Increase levothyroxine dose
lntrapetrosal sinus sampling identifies a pituitary
microadenoma as the source of the high ACTH level.
Item 42 Which of the following is the most appropriate test to per­
A 62-year-old man is evaluated in the hospital for several form next?
hours of nausea. lightheadedness. and back and abdominal
pain. He underwent uncomplicated mechanical aortic valve (A) 8-mg dexamethasone suppression test
replacement 3 days ago. He had been doing well postop­ (B) 24-Hour urine free catecholamine and metanephrine
eratively until the development of his current symptoms. measurement
Medical history is significant only for hypothyroidism. Med­ (C) Dual-energy x-ray absorptiometry scan
ications are therapeutic unfractionated heparin, levothy­ (D) PETscan
roxine. and as-needed oxycodone.
On physical examination. temperature is 37.2 °C
(99.0 °F), blood pressure is 80/50 111111 Hg. pulse rate
Item 44
is 110/rnin, and respiration rate is 1 8/min. BMI is 26.
Examination of the chest shows a clean and dry sternal A 28-year-old woman is being discharged from the hospital
wound. Cardiac examination reveals regular tachycar­ with a diagnosis of autoimmune adrenalitis. Medical history
dia and mechanical S.,. There is no pain with palpation is otherwise unremarkable. and she was on no medications
of the abdomen or lov\rer back. His skin pigmentation is prior to admission.
normal. On physical examination. temperature is 37.0 °C (98.6 °F).
blood pressure is 120/80 mm Hg. pulse rate is 80/min. and
Laboratory studies: respiration rate is 12/min. BMI is 22. Increased skin pigmen­
Hemoglobin 7.3 g/dL (73 g/1.) tation is noted over the extensor surfaces of the extremities
9.0 g/dL (90 g/1.) bilaterally. Tl1e remainder of" her examination is normal.
postoperatively
Leukocyte count 11.000/µL (II x 10''/i.) Which of the following is the most appropriate long-term
Activated partial 70 s medication regimen for this patient?
thromboplastin time
1.0 mg/dL (88.4 µmol/L) (A) Dexamethasone. I mg once daily
Creatinine
Sodium 130 mEq/L (130 mrnol/L) (B) Fludrocortisone. 0.05 mg once daily
Potassium 6.0 mEq/L (6.0 mmol/U (C) Hydrocortisone.10 mg three times daily
Cortisol, random <2 µg/dL (55.2 nmol/L) (D) Prednisone, 5 mg once daily. and fludrocortisone.
0.05 mg once daily
Which of the following is the most likely diagnosis?
(A) Autoimmune adrenalitis Item 45
(BJ Bilateral adrenal hemorrhage A 78-year-old woman is evaluated for a 2-week history of
(C) Opiate-induced adrenal insun'iciency unintentional weight loss, night sweats, and neck swelling.
(D) Pituitary apoplexy She has more recently also had difficulty swallowi11g solid

85
363

Self-Assessment Test

foods and positional shortness of breath. She does not have Which of the following is the most appropriate manage­
hoarseness, palpitations, or change in bowel habits. Prior ment of this patient?
to the development of these symptoms, she had been in her
(A) Refer for parathyroidectomy
usual state of health with no illnesses. She has hypothyroid­
ism and has been taking levothyroxine for 40 years, with a (B) Start alendronate
stable dose for the past 10 years. (C) Start calcitonin
On physical examination, temperature is 36.9 °c (98.4 °F), (D) Start cinacalcet
blood pressure is 140/88 mm Hg, pulse rate is 75/min, and (E) Start vitamin D3 (cholecalciferol)
respiration rate is 12/min. Tile thyroid is symmetrically
enlarged, firm, and fixed. 1here is no thyroid bruit.
CT scan of the neck is shown. Item 47
A 57-year-old man is ad111itted to the hospital for evaluation Cl
of' substernal chest pain. His medical history is significant
f'or type 2 diabetes 111ellitus. coronary artery disease, hyper­
tension. and hyperlipide111ia. He manages his diabetes as
an outpatient with diet. exercise, and 111etfor111in. His other
111edications are aspirin. metoprolol. atorvastatin. and sub­
lingual nitroglycerin as needed. His inpatient plas111a glu­
cose values are 170 to 210 111g/dL (9.4-11.6 rnmol!L). Results
of'all other laboratory studies are normal.
Which of the following is the most appropriate treatment
for this patient's diabetes while hospitalized?
(A) Basal and prandial insulin
(13) Glipizicle
(C) Metf'or111in
(D) Slicling-scale insulin

Item 48
Which of the following is the most likely diagnosis? A 43-year-old woman is evaluated for progressive weight
(A) Graves disease gain over the past 2 years. Her previous weight was 72.6 kg
(160 lb) but has steadily risen to her current weight of 106.6
(B) Papillary thyroid cancer kg (235 lb). She notes a slight increase in her appetite but
(C) Primary thyroid lymphoma minimal change in her lifestyle or activity level. She has
(D) Subacute thyroiditis tried to lose weight with increased exercise and nutritional
counseling but without significant results. More recently
she reports having trouble sleeping and decreased exercise
Item 46 tolerance with activites such as walking up steps. Medical
A 67-year-old man is evaluated for a recent diagnosis of history is significant for impaired fasting glucose, hyper­
primary hyperparathyroidism after an elevated serum cal­ tension, and hyperlipidemia. Medications are hydrochlo­
cium level was incidentally detected on laboratory testing. rothiazide and atorvastatin. She has not been prescribed
Medical history is significant only for hypertension, and his glucocorticoids or had glucocorticoid joint injections.
only medication is ramipril. On physical examination, temperature is 37.2 °C (99.0 °F),
On physical examination, temperature is 35.8 °C (96.4 °F), blood pressure is 136/86 mm Hg, pulse rate is 88/min, and
blood pressure is 120/68 mm Hg, pulse rate is 62/rnin, and respiration rate is 12/min. BM! is 38. She has rounded facies,
respiration rate is 14/rnin. BM! is 32. 1he remainder of his thin hair, mild hirsutism, and prominent fat deposition in
examination is unremarkable. the dorsocervical and supraclavicular areas. Her skin is thin,
Laboratory studies: and she bruises easily, although striae are not present. Her
Calcium 10.9 mg/dL (2.7 mmol/L) examination is otherwise unremarkable.
Creatinine 0. 9 mg/dL (79.6 µmol/L) Laboratory studies are significant for a fasting plasma
Parathyroid hormone 98 pg/mL (98 ng/L) glucose level of 120 mg/dL (6.7 mmol/L) and normal thy­
25-Hydroxyvitamin D 19 ng/mL (47.4 nmol/L) roid-stimulating hormone level.
Estimated glomerular >60 mL/min/1.73 m2 Which of the following is the most appropriate next step in
filtration rate
evaluation?
A dual-energy x-ray absorptiometry (DEXA) scan (A) Adrenocorticotropic hormone measurement
shows T-scores of -1.3 in the right femoral neck, -1.0 in the (13) 1-mg dexamethasone suppression test
lumbar spine, and -1.4 in the non-dominant forearm. Frac­
ture Risk Assessment Tool (FRAX) score indicates a 13% risk (C) 8-mg dexarnethasone suppression test
of major osteoporotic fracture and a 1. 9% risk of hip fracture (D) Pituitary MRI
over the next 10 years. (E) Serum cortisol measurement

86
Self-Assessment Test

Item 49 Item 51
A SO-year-old man undergoes follow-up evalua­ A 64-year-old man with type 2 diabetes mellitus and stage 4
tion for type 2 diabetes mellitus. His daily log demon­ chronic kidney disease is evaluated for continued glycemic
strates average blood glucose levels of 120 to 150 mg/dL management. He is followed closely by the nephrology ser­
(6.7-8.3 mmol!L), with hypoglycemia in the 50 mg/ vice in preparation for impending hemodialysis, with initi­
dL (2.8 mmol/L) range noted once or twice per week ation of erythropoietin therapy within the last 3 months. His
without a discernible pattern. He is unable to detect the average fasting and preprandial blood glucose values are in
hypoglycemia. the 145 to 190 mg/dL (8.0-10.5 mmol/L) range. He does not
The patient has a medical history of diabetic ret­ have hypoglycemia. His insulin regimen consists of insulin
inopathy, chronic kidney disease, peripheral neurop­ detemir at bedtime and insulin glulisine before meals. His
athy, hypertension, hyperlipidemia, obstructive sleep most recent hemoglobin A1e value has decreased from 7.5%
apnea (on bilevel positive airway pressure), gastro­ to 6.2%.
esophageal reflux disease, and osteoarthritis in both
Which of the following is the most appropriate manage­
knees. He reports intolerance to strenuous exercise due
ment for this patient's diabetes?
to knee pain. He is able to walk 15 minutes daily. He has
worked closely with a nutritionist, resulting in a 5.0- (A) Continue current therapy
kg (11-lb) weight loss over 1 year, which has plateaued (B) Decrease insulin detemir dose
recently. (C) Discontinue preprandial insulin glulisine
Medications are insulin glargine, insulin aspart, lis­
inopril, carvedilol, pantoprazole, aspirin, and atorvas­ (D) Measure postprandial glucose level
tatin.
On physical examination, blood pressure is 140/
90 mm Hg and pulse rate is 65/min. BM! is 37. Bilateral Item 52
proliferative retinopathy is present. There are no carotid A 34-year-old woman is evaluated for hirsutism and acne
bruits or cardiac murmurs. Bilateral loss of monofilament that began 4 months ago. She also reports that her voice
and vibratory sensation on the feet and decreased ankle has become deeper. Her menstrual periods have remained
reflexes are noted. The remainder of the examination is regular, occurring every 28 days.
normal. She has one son 5 years of age. She reports that her
Results of laboratory studies show hemoglobin pregnancy was uncomplicated and she had no difficulty
A1e level of 8.2% and serum creatinine level of 1.7 mg/dL conceiving. She had a tubal ligation following the birth of
(150.3 µmol/L). her child. She has a sister with polycystic ovary syndrome.
She takes no medications.
Which of the following is the most appropriate treatment
On physical examination, blood pressure is 140/84 mm
of this patient?
Hg, and pulse rate is 62/min. BM! is 21. She has hyperpig­
(A) Bariatric surgery mented terminal hairs on the chin, neck, abdomen, and
(B) Increase insulin lower back and comedones and pustules on the face and
(C) upper back. There is frontotemporal hair loss. The remain­
Initiate metformin
der of the examination is unremarkable.
(D) Initiate pramlintide
Laboratory studies:
Cortisol, free (urine) 26 µg/24 h (71.6 nmol/24 h)
Dehydroepiandrosterone 8.2 µg/mL (22.1 µmol/L)
Item 50 sulfate
A 47-year-old woman is evaluated for an incidentally dis­ Prolactin 8 ng/mL (8 µg/L)
covered right adrenal mass. Testosterone 96 ng/dL (3.3 nmol/L)
On physical examination, blood pressure is 120/80 mm
Hg in both arms and pulse rate is 84/min. The abdomen is
Which of the following is the most appropriate diagnostic
nontender, and there are no palpable masses. The remain­ test to perform next?
der of the examination is unremarkable.
Noncontrast CT of the abdomen demonstrates a 3.2-cm (A) Abdominal CT scan
well-circumscribed, partially cystic right adrenal lesion (B) Low-dose dexamethasone suppression test
with a density of 30 Hounsfield units. A low-dose dexa­ (C) Pelvic ultrasound
methasone suppression test is negative for evidence of cor­
(D ) Pituitary MRI
tisol hypersecretion.
Which of the following is the most appropriate next step in
management? Item 53
(A) Adrenalectomy A 27-year-old woman is evaluated for management of her
(B) type 1 diabetes mellitus. She was diagnosed 10 years ago. She
CT-guided transcutaneous biopsy
has no known complications from her diabetes. She eats a
(C) Plasma aldosterone to plasma renin ratio healthy diet and exercises an average of 60 minutes per day in
(D) Plasma free metanephrines the evening. She takes insulin glargine and insulin aspart. She
(E) No additional testing is indicated is adherent with her insulin regimen and checks her blood

87
Self-Assessment Test

glucose level three to five times per day. Her average blood erections but reports low libido and occasional erectile dys­
glucose value is 125 mg/dL (6.9 mmol/L), with fasting glucose function during intercourse. He wakes once during the night
values ranging from 80 to 150 mg/dL (4.4-8.3 mmol/L). She to urinate and drink water. He estimates that he urinates 5 to
routinely measures her 2-hour postprandial glucose values, 8 times during the day, which is unchanged.
and they are consistently less than 150 mg/dL (8.3 mmol/L). Medical history is significant for transsphenoidal
She has several overnight blood glucose values ranging from resection of a craniopharyngioma at age 18 years. He has
90 to 140 mg/dL (5.0-7.8 mrnol/L). Her hemoglobin A 1c values required anterior pituitary hormone replacement and des­
over the last 6 months have been 7.3% to 7.5%. She is discour­ mopressin since that time. Medications are desmopressin,
aged that her hemoglobin A 1c values remain above 7.0%. hydrocortisone, levothyroxine, somatropin, and testoster­
Laboratory studies, including creatinine and complete one enanthate.
blood count, are normal. On physical examination, temperature is 37.0 °C (98.6 °F),
blood pressure is 118/64 mm Hg, pulse rate is 74/min, and
Which of the following is the most appropriate manage­
respiration rate is 14/min. No orthostatic changes are noted.
ment of her elevated hemoglobin A1 c level?
BM! is 24. There are no facial changes suggestive of acromeg­
(A) Begin continuous glucose monitoring aly or Cushing syndrome. The thyroid is normal without
(B) Increase exercise goiter or nodules. Hair distribution and skin turgor are
normal. There is no gynecornastia or striae. Normal penis
(C) Increase insulin aspart
and testicular volume are noted. Visual fields are intact on
(D) Increase insulin glargine neurologic examination.
Laboratory studies:
Item 54 Sodium 138 mEq/L (138 mmol!L)
Insulin-like growth factor 1 Normal
A 25-year-old woman is evaluated after a recent diagnosis of Prolactin 18 ng/mL (18 µg/L)
polycystic ovary syndrome. She is concerned about hirsut­ 1hyroid-stimulating hormone 0.8 µU/mL (0.8 rnU/L)
ism and irregular menses. She has no desire to be pregnant Thyroxine (T), free 0.7 ng/dL (9.0 pmol!L)
at this time. She takes no medications. Testosterone (11 days after 482 ng/dL (16.7 nmol!L)
On physical examination, she is afebrile. Blood pres­ injection)
sure is 110/60 mm Hg, pulse rate is 68/min, and respiration
rate is 16/min; BM! is 37. Coarse hair is noted on the chin, Follow-up MR!s show no residual tumor.
jawline, and periumbilical area. The remainder of her phys­
ical examination, including pelvic examination, is normal. Which of the following is the most appropriate manage­
ment?
Which of the following is the most appropriate treatment?
(A) Increase desmopressin
(A) Combined oral contraceptive pills (B) Increase hydrocortisone
(B) Intermittent progestin withdrawal (C) Increase levothyroxine
(C) Levonorgestrel intrauterine system (D) Increase testosterone enanthate
(D) Spironolactone therapy (E) Stop somatropin

Item 55 Item 57
A 46-year-old man is evaluated following a diagnosis of pheo­ A 55-year-old man is evaluated following a screening for
chromocytoma. He has no signs or symptoms at this time. type 2 diabetes mellitus. He is asymptomatic. He has a his­
Except for hypertension, his medical history is unremark­ tory of hypertension and hyperlipidemia. 1here is no history
able. Medications are lisinopril and hydrochlorothiazide. of anemia, liver disease, or kidney disease. Medications are
On physical examination, blood pressure is 170/90 mm lisinopril and rosuvastatin.
Hg and pulse rate is 90/min. The remainder of the physical On physical examination, blood pressure is 123/
examination is unrevealing. 76 mm Hg and pulse rate is 72/min. BM! is 28. The remainder
of the examination is unremarkable.
Which of the following is the most appropriate next step in
management? Laboratory studies:
Hematocrit 45.6%
(A) Increase lisinopril dosage Creatinine 1.0 mg/dL (88.4 µmol/L)
(B) Perform contrast-enhanced adrenal CT scan Glucose, fasting 128 mg/dL (7.1 mmol/L)
(C) Start phenoxybenzamine Hemoglobin A,c 5.6%
(D) Start propranolol
Which of the following is the most appropriate diagnostic
test to perform next?
Item 56
(A) Fasting plasma glucose
A 38-year-old man with panhypopituitarism is evaluated for
worsening fatigue and weight gain over the past 3 months. He (B) Hemoglobin A,c
is sleeping 9 hours each night, but he feels tired during the day (C) Oral glucose tolerance test
and has decreased h.is usual exercise level. He has morning (D) Random blood glucose

88
Self-Assessment Test

Item 58 Her medical history is significant for diet-controlled


A 34-year-old man is evaluated for a 2-year history of type 2 diabetes mellitus diagnosed I year ago and oste­
fatigue, low libido, and infertility. His family history is nota­ oporosis diagnosed 4 years ago. Her only medication is
ble for a brother with infertility. His medical history is unre­ alendronate.
markable, and he takes no medications. On physical examination, temperatme is 37.0 °C (98.6 °F},
On physical examination, vital signs are normal. His blood pressure is 120/80 mm Hg, and pulse rate is 70/min.
height is 195.6 cm (77 in), and his weight is 86.2 kg (190 lb). BM! is 26. The remainder of the physical examination is
Genital examination reveals small, firm testes bilaterally. normal.
Laboratory studies reveal a morning serum total tes­ Laboratory evaluation reveals a serum sodium level
tosterone level of 140 ng/dL (4.9 nmol/L} (which was con­ of 139 mEq/L (139 mmol/L} and serum potassium level of
firmed on repeat testing), a serum follicle-stimulating hor­ 4.1 mEq/L (4.1 mmol/L}. The previously obtained CT scan
mone level of 24 mU/mL (24 U/L}, and a serum luteinizing shows a 2.0-cm well-circumscribed, left adrenal lesion with
hormone level of 18 mU/mL (18 U/L}. Repeated semen anal­ a density of 5 Hounsfield units.
yses reveal azoospermia. In addition to screening tests for pheochromocytoma,
Which of the following is the most appropriate diagnostic which of the following is the most appropriate diagnostic
test to perform next? test to perform next?

(A) Karyotype (A) Adrenal vein sampling


(B) MRI of the pituitary (B) Low-dose dexarnethasone suppression test
(C) Scrotal ultrasound (C) Plasma renin activity and aldosterone concentration
measurernent
(D) Serum prolactin measurement
(D) No further testing

Item 59
Item 61
An 84-year-old man is evaluated for moderate fatigue. He
otherwise feels well and does not have constipation, cold A 68-year-old man is seen in follow-up for a recent diagno­
intolerance, weight gain or loss, anxiety, tremor, palpita­ sis of acromegaly. He presented with chronic fatigue, joint
tions, or dyspnea. Medical history is significant for hyper­ and back pain, and an increase in his shoe size over the past
tension, and his only medication is felodipine. 2 years. Medical history is significant for hypertension and
On physical examination, he is alert and oriented. Blood type 2 diabetes mellitus. Current medications are lisinopril,
pressure is 144/83 mm Hg; other vital signs are normal. The rnetformin, and as-needed acetaminophen.
thyroid is not palpable. Cardiopulmonary examination is On physical examination, blood pressure is 146/88 mm
normal. TI1ere is no peripheral edema. Neurologic examina­ Hg and pulse rate is 90/min. BM! is 29. He has a prominent
tion is nonfocal, and deep tendon reflexes are normal. brow. Macroglossia is present. Lung and heart examinations
are unremarkable. Musculoskeletal examination reveals
Laboratory studies:
large hands and knees with bone swelling and crepitus.
Complete blood count Normal Skin is thickened, and there is excessive perspiration. On
Comprehensive metabolic Normal neurologic examination, bitemporal hemianopsia is noted.
profile Laboratory studies are significant for an elevated serum
TI1yroid-stimulating hormone 6.4 µU/mL (6.4 mU/L} insulin-like growth factor 1 level of 996 ng/mL (996 µg/L}
(TSH)
and serum prolactin level of 42 ng/rnL (42 µg/L}.
Free thyroxine (T,) 1.3 ng/dL (16.8 pmol/L} MRI shows a 2.5 x 1.8-cm pituitary macroadenoma that
elevates the optic chiasm and appears to envelop the left
Which of the following is the most appropriate manage­ carotid artery and invade the left cavernous sinus. TI1e optic
ment? chiasm is mildly atrophied.
(A) Levothyroxine therapy Which of the following is the most appropriate next step in
(B) Measurement of serum total triiodothyronine (T3) therapy for this patient?
level
(A) Dopamine agonist
(C) Measurement of serum total T,1 level
(B) Growth hormone receptor blockade
(D) Repeat TSH and free T4 measurement in 6 to 12 weeks
(C) Somatostatin analogue
(D) Stereotactic radiation therapy
Item 60 (E) Transsphenoidal pituitary surgery
A 62-year-old woman is evaluated for an incidentally dis­
covered left adrenal mass. Two weeks ago, the patient was
evaluated in the emergency department for diffuse abdom­ Item 62
inal pain and vomiting. A CT scan was obtained that was A 47-year-old man is evaluated postoperatively following
normal except for the adrenal mass. TI1ree hours after pre­ thyroidectomy for papillary thyroid cancer. Preoperative
sentation to the emergency department, the pain resolved evaluation showed no evidence of distant metastatic dis­
spontaneously. ease, and he underwent total thyroidectomy with central

89
Self-Assessment Test

and left lateral neck dissections. The patient's medical his­ Which of the following is the most appropriate treatment?
tory is otherwise unremarkable.
(A) Clomiphene citrate
On physical examination, vital signs are normal. The
neck surgical sites are clean and dry. The remainder of the (B) In vitro fertilization
examination is unremarkable. (C) Injectable gonadotropin
The surgical pathology report reveals a 3.5-cm papillary (D) Metformin
thyroid cancer in the left lobe of the thyroid and six malig­
nant lymph nodes out of 35 dissected. There is evidence of
minor extrathyroidal extension and vascular invasion by Item 65
the primary tumor. A 64-year-old woman is seen for follow-up evaluation. Two
Which of the following is the most appropriate postopera­ weeks ago, she was in a car accident, and an incidental pitu­
tive treatment? itary adenoma was found on a cervical spine CT scan. She
has no residual injuries from the car accident.
(A) Doxorubicin chemotherapy She is otherwise healthy and takes no medications. She
(B) External-beam radiotherapy went through menopause at age 51. She has night sweats
(C) Radioactive iodine therapy two to three times per month and occasional hot flushes.
(D) No additional therapy These have improved over the past decade and are not
bothersome. She is not sexually active. She has never taken
hormone replacement therapy. She has had no change in
vision, headaches, or galactorrhea.
Item 63
On physical examination, temperature is 37.5 °C
A 66-year-old man is evaluated in the office after being (99.5 °F), blood pressure is 110/63 mm Hg, pulse rate is
treated in the emergency department for an exacerbation of 82/min, and respiration rate is 14/min. BM! is 26. There
chronic obstructive pulmonary disease. While in the emer­ is axillary and pubic hair loss. Visual fields are intact.
gency department, he was noted to have a random blood There are no findings suggestive of Cushing syndrome or
glucose level of 211 mg/dL (11.7 mmol/L). His hemoglobin acromegaly.
A,c was 7.8% at the time. A repeat random fingerstick blood
Laboratory studies:
glucose level in office is 204 mg/dL (11.3 mmol/L).
Estradiol <20 pg/mL (73.4 pmol!L)
The patient reports recent polyuria and polydipsia. He
Follicle-stimulating hormone 6.4 mU/mL (6.4 U/L)
has lost 6 kg (13.2 lb) over the last 3 months. He has chronic
Luteinizing hormone 3.2 mU/mL (3.2 U/L)
epigastric pain associated with loose, oily stools due to
Prolactin 53 ng/mL (53 µg/L)
chronic pancreatitis.
Thyroid-stimulating hormone 3.2 µU/mL (3.2 mU/L)
He has a 20-pack-year history of tobacco use and
l11yroxine (T), free 1.1 ng/dL (14.2 pmol!L)
prior alcohol use, however, he does not currently use alco­
hol. Current medications are enteric-coated pancreatic Pituitary MRI shows a 7-mm adenoma in the anterior
enzymes, vitamins, tiotropium inhaler, and an albuterol sella. The tumor is not invasive. It does not approximate the
inhaler as needed. optic chiasm. The pituitary stalk is mid-line.
On physical examination, temperature is 37.1 °C (98.8 °F),
blood pressure is 130/75 mm Hg, and pulse rate is 90/min. Which of the following is the most appropriate manage­
BM! is 22. He has mild epigastric pain on palpation without ment?
rebound tenderness or guarding. The rest of his examination (A) Begin dopamine agonist
is unremarkable.
(B) Gamma knife stereotactic radiosurgery
Which of the following is the most appropriate treatment (C) Repeat testing in 12 months
for his diabetes? (D) Transphenoidal resection
(A) Exenatide
(B) Glipizide
Item 66
(C) Insulin
(D) Metformin A 57-year-old man with a 15-year history of type 2 diabe­
tes mellitus is evaluated for bilateral burning sensation in
his feet for the last 6 to 12 months. The sensation worsens
at night. His hemoglobin A 1 c levels have remained less
Item 64 than 7.0% for the last 2 years but were between 8.0% and
A 34-year-old woman is evaluated because she and her male 9.0% before implementing significant lifestyle changes
partner have been trying to conceive for 13 months without and transitioning to insulin therapy from metformin ther­
success. Her medical history is notable for a 6-year history apy 2 years ago.
of irregular menses and a recent diagnosis of polycystic His medical history includes coronary artery disease,
ovary syndrome. Her only medication is a prenatal vitamin. first-degree atrioventricular block, nonproliferative dia­
On physical examination, vital signs are normal. BM! is betic retinopathy, hypertension, and hyperlipidemia.
36. Terminal hair growth on the chin, upper lip, and sides Medications are regular insulin, neutral protamine
of the face is noted. No evidence of abdominal or pelvic Hagedorn (NPH) insulin, aspirin, metoprolol, atorvastatin,
masses, clitoromegaly, or galactorrhea is detected. and lisinopril.

90
Self-Assessment Test

On physical examination, findings are compatible with rate is 16/min. BM! is 24. Pelvic examination reveals a non­
distal polyneuropathy. tender, mobile uterus.
Which of the following is the most appropriate manage­ Laboratory studies:
ment of this patient's neuropathy? Estradiol <80 pg/mL (293.6 pmol/L)
Follicle-stimulating hormone 6.2 mU/mL (6.2 U/L)
(A) Amitriptyline Luteinizing hormone 4.1 mU/mL (4.1 U/L)
(B) Duloxetine Prolactin Normal
(C) Nerve conduction study Thyroid-stimulating hormone Normal
(D) Vitamin B 12 measurement Urine human chorionic Negative
gonadotropin

CJ Item 67
A 72-year-olcl man presents to the emergency department
Which of the following is the most appropriate diagnostic
test to perform next?
overnight. His wife noted that he was cliaphoretic and
(A) MRI of the pituitary
restless during the night, and his initial blood glucose level
measured by emergency medical services was 41 mg/ell (B) Peripheral karyotype
(2.3 mmol/L). He was given a single dose of' intravenous (C) Progestin withdrawal test
glucose, which increased his blood glucose level to 85 mg/ell (D) Transvaginal ultrasound
(4.7 mmol/L). His symptoms recurred en route to the hos­
pital, and he responded to a second dose of' intravenous
glucose. While in the emergency department, he was Item 69
provided food and his glucose level has remained above A 29-year-old woman comes to the office for a follow-up
80 mg/dL (4.4 mmol/L) without further treatment. The evaluation. She was diagnosed with type 2 diabetes mellitus
patient ·s wife reports several recent episodes or overnight 2 years ago. Her fasting and premeal blood glucose values
diaphoresis over the last several weeks. His last meal range from 120 to 150 mgidL (6.7-8.3 mmol/L). She has had
prior to this episode was approximately 7 hours before no hypoglycemic events. She has been adherent with her
symptom onset. insulin regimen along with diet and exercise modifications.
His medical history is significant for hypertension and Her hemoglobin A,c level has decreased from 9.0% to 7.5%
osteoarthritis. Medications are lisinopril and as-neeclecl ibu­ over the last 12 months. She expresses frustration over the
profen. need for multiple medications to treat her diabetes. Medical
On physical examination. the patient is alert and ori­ history is otherwise unremarkable.
ented. Blood pressure is 135/84 mm Hg, and pulse rate is 82/ Medications are insulin glargine, insulin aspart, and
min. BMI is 23. ll1e general physical and neurologic exam metformin.
inations are normal. On physical examination, blood pressure is 120/72 mm
Laboratory studies are significant for a normal com Hg and pulse rate 80/min. BM! is 27. The remainder of the
plete metabolic profile and blood count. An insulin secre­ examination is normal.
tagogue screen is obtained. and hypoglycemic studies con­ Hemoglobin A,c level is currently 7.4%. Results of all
sisting of measurement of insulin, C-pepticle. proinsulin. other laboratory studies are normal.
and �-hydroxybutyrate levels are planned.
Which of the following is the most appropriate manage­
Which of the following is the most appropriate diagnostic ment of this patient's diabetes?
strategy for this patient?
(A) Increase insulin glargine dose
(A) Hypoglycemic studies now (B) Measure 3 AM blood glucose level
(B) Mixed-meal testing with hypoglycemic studies at the (C) Measure postprandial blood glucose levels
time of symptomatic hypoglycemia (D) Switch metformin to sitagliptin
(C) Oral glucose tolerance testing with hypoglycemic
studies at the time of symptomatic hypoglycemia
(D) 72-Hour fast with hypoglycemic studies at the time Item 70
or symptomatic hypoglycemia A 32-year-old woman is evaluated for a 2-week history of
foreign body sensation in her eyes. Despite flushing the eyes
numerous times with saline, the sensation remains. She also
Item 68 feels pressure behind her eyes. She is not pregnant. Medical
A 24-year-old woman is evaluated for a 3-month history of history is unremarkable.
amenorrhea accompanied by cyclic pelvic pain. Preceding On physical examination, scleral injection and peri­
the onset of amenorrhea, she had a recent dilatation and orbital edema are present bilaterally, right worse than left.
curettage to remove retained products of conception after Diplopia is noted on lateral gaze. There is no lid lag or
a first-trimester spontaneous abortion. Her personal and proptosis. ll1e thyroid is diffusely enlarged, and a bruit is
family medical history is unremarkable. She takes no med­ noted. Neurologic examination discloses brisk reflexes and
ications. a bilateral hand tremor.
On physical examination, she is afebrile. Blood pres­ Laboratory studies reveal a serum thyroid-stimulating
sure is 110/60 mm Hg, pulse rate is 68/min, and respiration hormone level of less than 0.008 µU/mL (0.008 mU/L), a

91
Self-Assessment Test

serum free thyroxine (T) level of 4.5 ng/dL (58.0 pmol/L),


and a serum total triiodothyronine (T) level of 365 ng/dL
(5.6 nmol/L).
Which of the following is the most appropriate next step in
treatment?
(A) External-beam radiotherapy to the orbits
(B) Methimazole
(C) Radioactive iodine
(D) Total thyroidectomy

Item 71
A 42-year-old man is evaluated for resistant hypertension.
He was diagnosed with hypertension at age 35 years and
reports that his blood pressure has never been well con­
trolled. He is taking his medications as prescribed. He does
not have headaches, chest pain, palpitations, shortness of
breath, or symptoms of panic attack. He has no history of TI1ere is diffuse enlargement of the thyroid, and the
cardiovascular disease, does not smoke, and does not drink inferior edge of the gland cannot be visualized. CT scan
alcohol. Medications are lisinopril, amlodipine, hydrochlo­ shows substernal extension of the goiter in the left lobe
rothiazide, metoprolol, and potassium chloride supplemen­ with mild tracheal narrowing and tracheal deviation to
tation. He is not taking any over-the-counter medications. the right.
On physical examination, blood pressure is 150/86 mm Which of the following is the most appropriate treatment?
Hg and pulse rate is 65/min. BM! is 24. Examination of the
heart is significant for an S,1 but no murmurs. TI1e remainder (A) External-beam radiotherapy
of his examination is unremarkable. (B) Levothyroxine
Laboratory studies are significant for a serum creati­ (C) Radioactive iodine
nine level of 1.0 mg/dL (88.4 µmol/L), fasting plasma glu­
(D) TI1yroidectomy
cose level of 82 mg/dL (4.5 mmol/L), and serum potassium
level of3.2 mEq/L (3.2 mmol/L).
Which of the following is the most appropriate next d.iag­ Item 73
nostic step? A 26-year-old woman is evaluated because she and her
(A) Dexamethasone suppression test husband have been trying to conceive for 14 months with­
out success. Her husband fathered a child in a previous
(B) Plasma aldosterone-plasma renin activity ratio marriage. Her medical history is notable for pelvic inflam­
(C) Plasma metanephrines and catecholamines matory disease, which was diagnosed and successfully
(D) Renal artery Doppler flow study treated at age 18 years. Her only medication is a prenatal
vitamin.
Physical examination findings are unremarkable.
Item 72
Which of the following studies is most likely to be diag­
A 65-year-old man is evaluated because of painless neck
nostic?
swelling and difficulty swallowing that has progressively
worsened over the last year. He does not have hoarseness, (A) Diagnostic laparoscopy
but he feels as though his voice is not as strong as it was in (B) Hysterosalpingogram
the past. Medical history is significant for multiple thyroid (C) Peripheral karyotype of the patient
nodules. Fine-needle aspiration of three of the largest nod­
ules 5 years ago showed that all nodules were benign on (D) Semen analysis of the patient's husband
cytologic examination. Medical history is otherwise unre­ (E) Transvaginal ultrasound assessment of follicle count
markable, and he takes no medications.
On physical examination, vital signs are normal. ·n1e
thyroid is diffusely enlarged and mobile with swallowing. Item 74
He has facial flushing when raising his hands above his head A 46-year-old man is evaluated for a right lateral neck mass.
as shown (see top of next column). He first noted the mass while shaving 2 weeks ago. He does
No cervical lymphadenopathy is noted. Cardiovascular not have hoarseness or difficulty swallowing. Medical his­
and pulmonary examinations are normal. Reflexes are nor­ tory is significant only for hypertension. His only medica­
mal, and there is no visible tremor. tion is hydrochlorothiazide.
Laboratory studies show a serum thyroid-stimulating On physical examination, blood pressure is 165/95 mm
hormone (TSH) level of 4.0 µU/mL (4.0 mU/L). Hg, pulse rate is 88/min, and respiration rate is 12/min.
Thyroid ultrasound shows innumerable coales­ TI1ere is a firm 2-cm mass in the right lateral neck and a
cent nodules with no suspicious sonographic features. mobile 1-cm right thyroid nodule.

92
Self-Assessment Test

Laboratory studies: ranging from 50 to 190 mg/dL (2.8-10.5 mmol/L). His over­
Calcium 10.5 mg/dL (2.6 mmol/L) night blood glucose values range from 120 to 140 mg/dL
Creatinine 1.8 mg/dL (159.1 µmol/L) (6.7-7.8 mmol/L). He is unable to detect hypoglycemia.
Thyroid-stimulating 4.8 µU/mL (4.8 mU/L) The patient is concerned about hyperglycemia, and he
hormone desires to reach a hemoglobin A,c level of less than 7%.
Medical history is significant for diabetic retinopathy,
Ultrasound of the neck reveals a hypoechoic thyroid peripheral neuropathy, hypertension, and hyperlipidemia.
nodule with microcalcifications measuring 1.6 cm. There is Medications are neutral protamine Hagedorn (NPH) insulin,
a 2.2-cm mass with internal calcifications in the area of the regular insulin, losartan, chlorthalidone, metformin, rosu­
palpable abnormality. vastatin, and aspirin.
Fine-needle aspiration of the thyroid nodule and the On physical examination, blood pressure is 125/82
lateral neck mass reveals medullary thyroid cancer. RET mm Hg and pulse rate is 80/min. BM! is 24. Retinal
testing is positive for a mutation in codon 634. examination demonstrates nonproliferative retinopa­
Which of the following is the most appropriate next step in thy. His lower extremities have diminished sensation
management? to a 10-g monofilament and vibration with a 128-Hz
tuning fork.
(A) 18-Fluoro-deoxyglucose positron emission tomogra- Hemoglobin A,c level is 7.2%, and the results of all other
phy scan laboratory studies are normal.
(B) Plasma fractionated metanephrine levels
Which of the following is the most appropriate treatment of
(C) Serum parathyroid hormone level this patient's diabetes?
(D) Total thyroidectomy and lateral neck dissection
(A) Continue current insulin and metformin doses
(B) Continue current insulin, increase metformin dose
CJ Item 75 (C)
(D)
Decrease meal-time insulin, add pramlintide
Decrease meal-time insulin, continue metformin
A 55-year-old woman is evaluated in the emergency
department for altered mental status that has developed
over the past week. Her family notes that she has become Item 77
increasingly confused, and her oral intake has decreased
signiflcantly. Her medical history includes breast cancer A 28-year-old woman is evaluated for a 6-month history of
that was diagnosed s years ago. Additional medical his­ infertility. In order to predict her fertile period, the patient
tory is significant for hypertension. Her only medication is has been using a home urinary luteinizing hormone (LH)
chlorthalidone. kit. The kit can identify an LH surge and therefore predict
On physical examination, the patient is minimally ovulation and the optimal time for intercourse. Her uri­
responsive. Mucous membranes are dry. Temperature is nary LH kit result has been consistently positive but her
36.2 °C (97.2 °F). blood pressure is 100/59 111111 Hg. pulse pregnancy test result consistently negative. She has been
rate is l lO/min, and respiration rate is 12/min. BM! is 22. exercising recently in an attempt to lose weight. Her med­
There is mild, difluse abdominal tenderness to palpation. ical history is otherwise unremarkable, and she takes no
The remainder of the examination is unremarkable. medications.
Laboratoty studies are significant for a serum sodium On physical examination, vital signs are normal. BMI is
level or 148 mEq/L (l48 mmol/L). creatinine level or 38. Coarse terminal hairs are noted on the chin, neck, and
4.0 mg/dL (353.6 µmol/L). baseline creatinine level is anterior chest. Pelvic examination is normal.
1.6 mg/dL (141.4 µmol/L), and calcium level of 18.2 mg/dL Pelvic ultrasound reveals a thickened endometrium
(4.6 mmol/L). and polycystic-appearing ovaries bilaterally.
Chlorthalidone is discontinued. and intravenous nor­ Which of the following is the most likely diagnosis?
mal saline is started.
(A) Hypothyroidism
Which of the following is the most appropriate immediate
(B) Late-onset congenital adrenal hyperplasia
next step in management?
(C) Polycystic ovary syndrome
(A) Calcitonin (D) Prolactinoma
(B) Cinacalcet
(C) Hemodialysis
Item 78
(D) Zoledronic acid
A 72-year-old woman is evaluated in follow-up for oste­
oporosis. Medical history is significant for a hip fracture
5 years ago sustained after a mild fall. Evaluation at that
Item 76 time included a dual-energy x-ray absorptiometry (DEXA)
A 60-year-old man is evaluated during a routine follow-up scan showing a left hip T-score of -2.8 and vertebral
examination. He has type 2 diabetes mellitus. Review of T-score of -2.7. She has been maintained on alendronate
his blood glucose log demonstrates fasting blood glucose therapy since that time. Medical history is also significant
values ranging from 120 to 160 mg/dL (6.7-8.9 mmol/L) for hypertension. Medications are alendronate, hydro­
and variable 2-hour postprandial blood glucose values chlorothiazide, calcium, and vitamin D. Family history

93
Self-Assessment Test

is significant for osteoporosis in her mother, sister, and On physical examination, her temperature is 37.2 °C
maternal aunt. She has a 35-pack-year tobacco use history (99.0 °F), blood pressure is 118/68 mm Hg, pulse rate is
and continues to smoke. 72/min, and respiration rate is 10/min. BM! is 28. Other­
On physical examination, temperature is 36.9 °c wise, her physical examination is unremarkable.
(98.4 °F), blood pressure is 138/87 mm Hg, pulse rate is Laboratory studies:
89/min, and respiration rate is 12/min. BM! is 28. She Human chorionic gonadotropin Negative
has marked thoracic kyphosis and increased central Prolactin 68 ng/mL (68 µg/L)
adiposity. TI1yroid-stimulating hormone 15 µU/mL (15 mU/L)
Laboratory studies show a serum calcium level of TI1yroxine (T4), free 0.82 ng/dL (10.6 pmol/L)
8.6 mg/dL (2.2 mmol/L) and 25-hydroxyvitamin D level
of 44 ng/mL (109.8 nmol/L); kidney function studies are
normal. Which of the following is the most appropriate manage­
Repeat DEXA shows a stable bone mineral density. ment?

Which of the following is the most appropriate treatment of (A) Begin cabergoline
this patient's osteoporosis? (B) Begin levothyroxine
(C) Hold sertraline and retest
(A) Change to denosumab
(D) Order pituitary MRI
(B) Change to teriparatide
(C) Continue alendronate therapy
(D) Initiate a drug holiday
Item 81
A 33-year-old man is evaluated for infertility. As part of
Item 79 this evaluation, azoospermia is identified on two sequential
A 55-year-old man presents for follow-up of a recent diag­ semen analyses. His medical history is otherwise notable for
nosis of hyperaldosteronism. Medical and family history chronic sinusitis. He takes no medications.
are unremarkable. Medications are losartan, metoprolol, On physical examination, vital signs are normal. BM!
amlodipine, and chlorthalidone. is 24. There is no evidence of inguinal hernia. A small left­
On physical examination, blood pressure is 150/90 mm sided varicocele is present that empties when the patient is
Hg and pulse rate is 70/min. TI1e rest of the examination is recumbent. Testicles are of normal size. Absence of the vas
unremarkable. deferens is noted bilaterally.
Additional studies include a noncontrast CT scan of the
Which of the following is most likely responsible for this
abdomen that shows a 1.2-cm homogeneous left adrenal
patient's infertility?
lesion with a density of -5 Hounsfield units. Adrenal vein
sampling indicates similar ratios of aldosterone to cortisol (A) Anabolic steroid abuse
bilaterally. (B) Cystic fibrosis
Which of the following is the most appropriate therapy for (C) Klinefelter syndrome
this patient? (D) Varicocele
(A) Bilateral adrenalectomy (E) Y chromosome microdeletions
(B) Dexamethasone
(C) Left adrenalectomy
(D) Spironolactone Item 82
A 22-year-old woman is evaluated for fatigue and weight
loss over the last 2 months. She describes a lack of energy,
Item 80 decreased appetite, and an unintentional 3.6-kg (8-lb)
A 24-year-old woman is evaluated for 3 months of galac­ weight loss. Over the last week she has experienced nau­
torrhea, worsening fatigue, and malaise. She was well sea and occasional vomiting. She notes no other focal
6 months ago and without explanation began feeling symptoms. She has no significant medical history. Family
tired and lethargic. She has noted milk leaking sponta­ history is remarkable for hypothyroidism in her mother
neously from her breasts. Her menses have been unpre­ and type 1 diabetes mellitus in her brother. She takes no
dictable. She had a normal cycle 3 months ago and medications.
4 months ago, but then her next cycle was delayed. Her On physical examination, temperature is normal,
last menstrual period was 6 weeks ago, and she had blood pressure is 100/68 mm Hg, pulse rate is 90/min, and
excessive bleeding. She has stopped exercising and has respiration rate is 12/min. BM! is 19. Hyperpigmentation
gained 3.6 kg (8 lb). She also reports headaches that is pronounced over the knuckles of the hands and elbows
occur about 2 to 3 times per week. She has noted consti­ bilaterally. The remainder of the physical examination is
pation and breast tenderness. She has no libido, which unremarkable.
is a significant change. She took a home pregnancy test, Laboratory evaluation demonstrates an 8 AM serum
which was negative. She has a history of depression. cortisol level of 2 µg/dL (55.2 nmol/L), a serum sodium
There is no family history of pituitary disease. Her only level ofl33 mEq/L (133 mmol/L), and a serum potassium
medication is sertraline for depression. level of 5.8 mEq/L (5.8 mmol/L).

94
Self-Assessment Test

Which of the following is the most appropriate next step in (C) Discontinue insulin glargine, change insulin aspart to
management? a sliding-scale regimen
(A) Cosyntropin stimulation test (D) Discontinue insulin glargine, discontinue insulin
aspart
(B) Hydrocortisone and fludrocortisone
(C) Plasma adrenocorticotropic hormone (ACTH) mea­
surement
Item 84
(D) Prednisone
A 34-year-old man is evaluated for a 1-year history
of decreased libido and severe erectile dysfunction,
Item 83 including absence of morning erections. He and his
A 21-year-old woman is evaluated for management of type female partner would like to conceive. His medical
1 diabetes. She was diagnosed 3 months ago after presenting history is otherwise unremarkable, and he takes no
to the emergency department with diabetic ketoacidosis. medications.
Her hemoglobin A1e level at the time of diagnosis was 15.2%. On physical examination, vital signs are normal. BM! is
She was discharged from the hospital on a basal and pran­ 27. Atrophic testes are noted bilaterally.
dial insulin regimen with minor adjustments required as Laboratory studies:
an outpatient. Her blood glucose log demonstrates a recent Follicle-stimulating hormone 1.2 mU/mL (1.2 U/L}
change manifesting as frequent symptomatic episodes of Luteinizing hormone 0.8 mU/mL (0.8 U/L}
both fasting and postprandial hypoglycemia with blood glu­ Prolactin llO ng/mL (llO µg/L}
cose levels ranging from SO to 65 mg/dL (2.8-3.6 mmol/L}. Morning testosterone (total) 120 ng/dL (4.2 nmol/L}
Medications are insulin glargine and insulin aspart. Thyroid-stimulating hormone Normal
On physical examination, blood pressure is ll0/70 mm
Hg and pulse rate is 70/min. BM! is 19. The remainder of the MRI of the pituitary reveals a 0.8-cm anterior pituitary
examination is unremarkable. mass consistent with an adenoma.
Hemoglobin A 1e level is 6.2%. Which of the following is the most appropriate
Which of the following is the most appropriate manage­ treatment?
ment for this patient's hypoglycemia? (A) Cabergoline
(A) Decrease insulin glargine, decrease insulin aspart (B) Clomiphene citrate
(B) Decrease insulin glargine, decrease insulin aspart, add (C) Sildenafil
pramlintide (D) Testosterone replacement therapy

95
Answers and Critiques

Item 1 Answer: B roidism due to multiple gland hyperplasia frequently occurs


Educational Objective: Treat hyperthyroidism during in patients with MEN2A. Patients with primary hyperparathy­
pregnancy. roidism may present with symptoms related to hypercalcemia
(polydipsia. polyuria. and constipation), or the hyperparathy­
This patient has hyperthyroidism occurring during pregnancy roidism may be found during an evaluation for osteoporosis
and should be treated with propylthiouracil (PTU) . Based on or nephrnlithiasis. Pheochromocytomas in MEN2A are usu­
this patient's age and sex, the most likely cause of her hyper­ ally benign and intra-adrenal in location, but can be multiple
thyroidism is Graves disease. Clinical findings supporting or bilateral.
this djagnosis are the diffuse thyromegaly, thyroid brnit, and lnsulinoma and prolactinoma occur in multiple endo­
elevated thyroid-stimulating immunoglobulin index. Treat­ crine neoplasia type 1 (MENl) . [n MENI. one mutated allele
ment of hyperthyroidism during pregnancy is typically with of the MENl gene is usually inherited. and a somatic muta­
medical management, and PTU, rather than methimazole, tion in the other allele is later acquired and results in the for­
is the drng of choice during the first trimester. Although mation of neoplasia. TI1e most common endocrine disorder
rare, methimazole has been associated with development of in MENl is primary hyperparathyroidism resulting from one
aplasia cutis (absence of a portion of skin on the scalp in a or more parathyroid adenomas.
localized or widespread area) and choanal atresia (blockage Neurofibroma is not a clinical feature of MEN2A. Neu­
of the posterior nasal passage due to failed recanalization of rofibromas. cafe-au-lait spots, and pheochromocytoma are
the nasal fossae during fetal development) when used in the among the clinical features of the autosomal dominant dis­
first trimester. Once organogenesis is complete, methimazole order neurofibromatosis type 1.
can be substituted for PTU. Methimazole is typically the pre­
KEY POINT
ferred agent in patients with hyperthyroidism because it has
a longer intrathyroidal half-life than PTU and usually can be • Medullary thyroid cancer, pheochromocytoma, and
administered once daily. primary hyperparathyroidism occur in patients with
Radioactive iodine is contraindicated during pregnancy multiple endocrine neoplasia type 2A (MEN2A).
because it can cause destruction of the fetal thyroid.
TI1yroidectomy is generally avoided during pregnancy Bibliography
unless the hyperthyroidism cannot be controlled or the Krampitz GW. Norton JA. RET gene mutations (genotype and phenotype) of
patient cannot tolerate either PTU or methimazole. When multiple endocrine neoplasia type 2 and familial medullary thyroid
carcinoma. Cancer. 2014 Jul l:120(13):1920-3 1. [PMlD: 24699901]
needed, surgery is typically performed during the second
trimester, if possible.
KEY POINT Item 3 Answer: B
• In pregnant patients with hyperthyroidism, propylthi­ Educational Objective: Treat prolactinoma during
ouracil, rather than methimazole, should be used in pregnancy.
the first trimester because of the teratogenic effects of The patient should undergo formal visual field testing. During
methimazole during thjs time. pregnancy, there is concern that prolactinomas can grow
due to estrogenic stimulation. TI1e risk of significant growth
Bibliography depends on the size of the prolactinoma prior to pregnancy.
Lazams JH. Management of hyperthyroidism in pregnancy. Endocrine. 2014
Mar;45(2):190-4. [PMID: 24174179]
With microadenomas (<10 mm). the risk is considered to
be low, whereas the risk is higher with macroprolactinomas
(�10 mm). Significant expansion may cause vision loss by
Item 2 Answer: C compressing the optic chiasm. It is therefore appropriate to
obtain visual field testing during pregnancy in women with
Educational Objective: Diagnose the multiple endocrine
macroadenomas even without symptoms to diagnose vision
neoplasia type 2 (MEN2) syndrome.
field loss (such as bitemporal hemianopsia) that may be due
This patient is most likely to develop primary hyperparathy­ to an enlarging prolactinoma. Testing in these women is rec­
roidism. He has symptoms related to catecholamine excess ommended every trimester of pregnancy. Because this patient
from a pheochromocytoma and has a personal and family is in her first trimester and has a history of a macroadenoma,
history of medullary thyroid cancer. This is typical of multi­ visual field testing now is indicated.
ple endocrine neoplasia type 2A (MEN2A) resulting from a It is not helpful to check the serum prolactin level. Pro­
mutation in the RETproto-oncogene. Primary hyperparathy- lactin is normally elevated in pregnancy and can be greater

97
Answers and Critiques
_ _

than 200 ng/mL (200 µg/L). Additionally, this patient has a measuring the glucose level at 3 AM. Medications that affect
known prolactinoma. so her prolactin will likely be elevated the overnight glucose level need to be decreased if the 3 AM
for both reasons. Elevated prolactin itself is not necessarily glucose level is low. Medications that affect the overnight
harmful. Prolactinomas cause concern when they cause glucose should be increased or added if the 3 AM glucose level
hypogonadism or mass effect. ln this patient, an elevated is elevated.
prolactin level will not change the treatment plan because it Fast-acting insulin such as insulin lispro at bedtime
is expected to be elevated and does not help clarify whether increases the risk of hypoglycemia.
the prolactinoma is causing harm. Metformin will decrease gluconeogenesis from the liver
Routine monitoring of women with prolactinomas with and improve fasting hyperglycemia. However, for similar
MRI during pregnancy is not indicated because the absolute reasons, overnight hypoglycemia must be excluded before
risk of significant enlargement of pituitary adenomas is this treatment could be safely initiated.
low. However, MRI is indicated in women with a pituitary Increasing the insulin glargine dose could also worsen
macroadenoma and abnormalities on visual field testing or overnight hypoglycemia if that is the cause of the fasting
change in headache possibly attributable to expansion of the hyperglycemia.
adenoma. Despite the hemoglobin A 10 level of less than 7%, the
Bromocriptine is avoided in pregnancy when possible etiology of the patient's fasting hyperglycemia should be
because its safety during gestation has not been established, investigated. Detection of overnight hypoglycemia would
although when a patient has symptoms of mass effect during necessitate immediate changes in her insulin regimen or
pregnancy, bromocriptine may need to be restarted. How­ food intake regardless of her hemoglobin A 10 value.
ever, it would not be appropriate to restart dopamine agonist
KEY POINT
therapy in this patient without a clear indication for treat­
ment such as an enlarging pituitary adenoma. • Overnight blood glucose monitoring can help detect
hypoglycemia or dawn phenomenon.
KEY POINT

• Because prolactinomas may increase in size in preg­ Bibliography


nant women and lead to loss of vision, close clinical Cryer PE. Axelrod L. Grossman AB, et al. Evaluation and management of
monitoring and formal visual field testing should be adult hypoglycemic disorders: an Endocrine Society Clinical Practice
Guideline. J Clin Endocrinol Metab. 2009 Mar:94(3):709-28. [PMlD:
performed during each trimester. 19088155]

Bibliography
Melmed S. Casanueva FF, Hoffman AR. et al. Diagnosis and treatment of
hyperprolactinemia: an Endocrine Society clinical practice guideline. J Item 5 Answer: A
Clin Enclocrinol Metab. 2011 Feb:96(2):273-88. [PMID: 21296991) Educational Objective: Evaluate the cause of Cushing
syndrome.

Item 4 Answer: D The most appropriate diagnostic test to perform next is a CT


scan of the adrenal glands. This patient was suspected to have
Educational Objective: Manage overnight
Cushing syndrome (CS) based on pathognomonic clinical
hypoglycemia.
findings and is not taking exogenous glucocorticoids. Two
This patient should measure her blood glucose level at 3 AM. screening tests for CS are abnormal, which is adequate to
The etiology of fluctuating fasting glucose values in diabe­ establish the diagnosis. The next step in evaluation is mea­
tes can be multifactorial, including overnight hypoglycemia, surement of the plasma adrenocorticotropic hormone (ACTH)
dawn phenomenon. or inadequate insulin doses. To maintain level to differentiate bet\,veen ACTH-dependent and ACTH-in­
normal blood glucose levels upon rising, an early morning dependent CS. With ACTH-dependent causes, the plasma
physiologic release of catecholamines, cortisol, and growth ACTH is usually greater than 20 pg/mL (4.4 pmol!L). How­
hormone occurs to stimulate endogenous glucose produc­ ever, the low plasma ACTH reported in this patient is consis­
tion from the liver. Overnight hypoglycemia caused by over­ tent with ACTH-independent CS. A CT scan of the abdomen to
treatment of diabetes or prolonged effects of recent physical evaluate the adrenal glands is therefore indicated because the
exertion can lead to low-normal fasting glucose values and most common causes of ACTH-independent CS are adenomas
amplify the release of catecholamines, cortisol, grovvth hor­ and carcinomas of the adrenal cortex.
mone, and glucagon to increase endogenous glucose produc­ Inferior petrosal sinus sampling and MRI of the pituitary
tion, which can lead to hyperglycemia (Somogyi effect). With gland are used to confirm the presence of a corticotroph ade­
the dawn phenomenon, fasting hyperglycemia occurs in the noma of the pituitary gland in ACTH-dependent CS, which
setting of inadequate basal insulin coverage to maintain the is the most common cause of CS overall. However, neither is
endogenous glucose value within a normal range. Food intake indicated in this patient because biochemical testing has not
in the evening can also contribute to fasting hyperglycemia if revealed ACTH-dependent CS.
it is inadequately covered with insulin. Overnight hypogly­ The diagnosis of CS requires that at least two first­
cemia and the dawn phenomenon can be distinguished by line screening tests be abnormal, including the low-dose

98
Answers and Critiques

dexamethasone suppression test (LOST) (both standard and development of ketoacidosis, which would require restarting
overnight), 24-hour urine free cortisol (UFC), or late-night insulin therapy.
salivary cortisol. This patient failed to suppress cortisol levels Determining autoimmunity, in conjunction with beta
following an overnight LOST and had an elevated 24-hour cell function, is helpful in assessing whether a patient has
UFC level confirmed by a repeat collection. Therefore, the potential to become insulin-independent in the future.
measurement of late night salivary cortisol is unnecessary His autoantibodies were negative at the time of his presen­
because the diagnosis of CS has already been established. tation, and it is unlikely that these would now be positive. It
KEY POINT
is not necessary to retest antibodies in this setting.

• CT of the adrenal glands should be performed in KEY POINT


patients with ACTH-independent Cushing syndrome • Prior to switching from insulin to oral therapy in
because adenomas and carcinomas of the adrenal cor­ patients with ketosis-prone diabetes, fasting
tex are common causes. C-peptide and glucose levels should be checked.

Bibliography Bibliography
Nieman LK. Biller BM. Findling JW. et al. The diagnosis of Cushing·s Maldonado MR. Otiniano ME, Cheema F. et al. Factors associated with insu­
syndrome: an Endocrine Society Clinical Practice Guideline. J Clin lin discontinuation in subjects with ketosis-prone diabetes but preserved
Endocrinol Metab. 2008 May:93(5):1526-40. [PMID: 18334580] �-cell function. Diabet Med. 2005 Dec:22(12):1744-50. [PMID: 16401322]

ltem6 Answer: D
Educational Objective: Manage ketosis-prone type 2
Item 7 Answer: C
Educational Objective: Treat pituitary apoplexy.
Cl
diabetes mellitus. The patient has acute apoplexy caused by pituitary hemor­
This patient should have a repeat measurement of fast­ rhage and requires urgent transsphenoidal decompression
ing C-peptide and glucose levels. He has ketosis-prone of the hemorrhage Lo preserve vision. Given his history of
type 2 diabetes mellitus. Patients with ketosis-prone type previous headache, loss of I ibiclo. and erectile dysfunction.
2 diabetes do not fulfill the classic phenotype associated he likely had a preexisting prolactinoma that acutely bled.
with autoimmune type 1 diabetes. These patients are often Prolactinomas are almost always treated wilh dopamine ago­
older, overweight or obese, and of black or Latino ethnic­ nisls. but this patient requires urgent surgery to decrease
ity. Patients with new-onset ketosis-prone diabetes require pressure on Lhe optic chiasm to save his vision. Transsphe­
insulin therapy initially but might be able to be managed noidal resection is the preferred method of pituitary surgery.
with oral agents in the future. Prior to switching from He also requires urgent stress dose glucocorticoids because
insulin to oral therapy, his pancreatic beta-cell function of risk of' secondary cortisol deficiency, which would be
should be assessed with fasting C-peptide and glucose mea­ lifc-Lhreatcning.
surements. Ketosis-prone type 2 diabetes is heterogeneous ·1 he patient has appropriately received glucoconicoids
condition in that the presence of autoantibodies is variable fcir possible adrcnoconicot ropic hormone deficiency. -r here
across the population, as is the degree of pancreatic beta cell is no indication to assess other pituitary function. In the
function. His initial C-peptide level in the setting of hyper­ next 2 Lo 4 weeks. thyroid f'unction will need to be assessed.
glycemia and diabetic ketoacidosis is not an accurate indi­ but thyroxine Cl) has a long half' life and does not need to
cation of his pancreatic function. Due to the toxic effects be emcrgently measured or replaced. Tr·eatment or hypogo
of prolonged hyperglycemia on the pancreatic beta cells, naclism or growl h hormone deficiency is not urgent: surgical
the fasting C-peptide and glucose or a glucagon-stimulated decompression is.
C-peptide should be measured 7 to 14 days after the correc­ Waiting 2 weeks Lo rcpcat imaging could lead to per
tion of the acidosis in order to better assess function. If his mancnt vision loss. I le needs immediate intervention Lo
repeat fasting C-peptide value is greater than or equal to 1.0 preserve his vision. Similarly. whole brain external beam
ng/mL (0.33 nmol/L) or his glucagon-stimulated C-peptide raclialion is not appropriate fi>r this patient who needs rapid
value is greater than or equal to 1.5 ng/mL (0.5 nmol!L), his surgic.1 I decompression.
beta cell function is preserved. KEY POINT
A sliding-scale insulin regimen that does not include
• Patients with pituitary apoplexy and vision loss
basal insulin and does not begin insulin administration
should receive immediate stress-dose glucocorticoids
unless the blood glucose level is at or above a threshold level
in addition to undergoing urgent transsphenoidal
will cause wide swings from hyperglycemia to hypoglyce­
pituitary decompression.
mia, and this is inappropriate treatment.
Discontinuation of his insulin and switching to an oral
agent such as metformin could be attempted with evidence Bibliography
Rajasekaran S. Vanderpump M. Baldeweg S. et al. UK guidelines for the
of beta cell function preservation. Close follow-up would management of pituitary apoplexy. Clin Endocrinol. 2011 Jan;74(1):9-20.
be necessary to monitor for worsening hyperglycemia or [PMID: 21044119]

99
Answers and Critiques

Item 8 Answer: B Item 9 Answer: A


Educational Objective: Manage a solitary thyroid Educational Objective: Manage primary amenorrhea.
nodule.
The most appropriate management is to initiate estro­
Fine-needle aspiration (FNA) is the most appropriate next step gen and progestin therapy in this patient with primary
in the evaluation to determine whether this patient's thyroid amenorrhea. Primary amenorrhea is defined as the lack
nodule is malignant or benign. She has already had an ultra­ of menses by age 16 years accompanied by a normal body
sound examination. Ultrasonography is a sensitive means of hair pattern and normal breast development. Pregnancy
identifying nodules and providing further characterization of must be ruled out in all patients with primary amenor­
the nodules, which is more preclictive of malignancy than size rhea. Approximately SO% of patients with primary amen­
alone. The ultrasonographic features that are considered more orrhea have a chromosomal abnormality. Primary ovarian
suspicious for malignancy include hypoechogenicity, micro­ insufficiency due to Turner syndrome, a syndrome char­
calcifications, irregular margins, and increased intranodular acterized by short stature and the loss of a portion or all
Doppler flow. According to American Thyroid Association of one X chromosome, is one of the most common causes
guidelines, this patient with a hypoechoic nodule that is of primary amenorrhea. TI1e diagnosis of Turner syn­
1.5 cm should have an FNA to rule out malignancy. drome can be made on the basis of a karyotype, and this
Nodular features are not readily identified on CT, and should be the next diagnostic test for this patient. Such a
this study has a lower sensitivity than ultrasonography for patient may also have fragile X premutation; however, no
identifying the presence of nodules. CT of the neck would cognitive impairment is typically seen in this patient pop­
therefore not provide any additional information about this ulation. Diagnosing Turner syndrome is critical because
patient, who has already had an ultrasound examination. In affected patients have a higher incidence of cardiovascular
addition, CT would expose this patient to unnecessary radi­ disease, metabolic syndrome, and thyroid dysfunction.
ation. If a patient has a substernal goiter, CT is beneficial for Patients with Turner syndrome may have either primary
determining the extent of the goiter. However, this patient or secondary amenorrhea and commonly have normal
has no extension of the thyroid into the mediastinum. secondary sexual characteristics. The mechanism involved
Levothyroxine therapy to suppress growth of benign nod­ appears to be early follicular depletion, such that ovaries
ules is no longer reconm1ended. Randomized clinical trials have are devoid of follicles and oocytes. Serum evaluations in
failed to show a significant effect on nodule volume. Adclition­ these patients will reveal low estradiol levels (typically
ally, the required dose can induce thyrotoxicosis, which is asso­ below the detectable level in the assay) and markedly
ciated with significant risk for carcliovascular complications. elevated gonadotropin levels. This constellation of find­
Measurement of the serum thyroglobulin level is ings is consistent with hypergonadotropic hypogonadism.
reserved for patients who have had a total thyroidectomy Such patients should receive hormone replacement ther­
and is useful as a tumor marker for detection of residual or apy with estrogen and cyclic progestin to prevent endo­
recurrent thyroid cancer. In a patient with an intact thyroid, metrial hyperplasia, osteoporosis, and other sequelae of
serum thyroglobulin measurement is an insensitive and hypoestrogenism.
nonspecific means of testing for malignancy. A pituitary prolactinoma causes secondary amenor­
TI1yroid scanning with technetium is unlikely to be rhea through direct inhibition of gonadotropin-releasing
helpful in this patient. Thyroid scanning is used to deter­ hormone secretion by prolactin. Because this patient has
mine the functional status of the nodule. Isotope scanning is elevated levels of gonadotropins, neither a pituitary MRI nor
most useful in the setting of a nodule accompanied by a low a prolactin measurement is necessary.
serum thyroid-stimulating hormone level because toxic (or Both hypothyroidism and hyperthyroidism also cause
hyperfunctioning) nodules typically do not require FNA as secondary amenorrhea. Hypothyroidism results in increased
the vast majority are benign. This patient has a normal sernm levels of thyrotropin-releasing hormone through negative
thyroid-stimulating hormone level and is likely to have an feedback, and this hormone, in turn, stimulates prolactin
indeterminate result ("cold" or "warm") on thyroid scanning. secretion and suppresses gonadotropin secretion. Hyper­
thyroidism can cause rapid weight loss, which is known
KEY POINT
to cause functional hypothalamic amenorrhea. Since this
• Fine-needle aspiration is the most appropriate next patient has elevated gonadotropin levels and no signs of
step in the management of a patient with a solid thy­ hyperthyroidism, thyroid-stimulating hormone measure­
roid nodule measuring greater than 1 cm on ultra­ ment is not needed.
sonography.
KEY POINT

Bibliography • Patients with primary amenorrhea associated with


American Thyroid Association (ATA) Guidelines Taskforce on Thyroid hypergonadotropic hypogonadism should receive
Nodules and Differentiated Thyroid Cancer, Cooper DS. Doherty GM,
Haugen BR, et al. Revised American Thyroid Association management hormone therapy with estrogen to prevent endome­
guidelines for patients with thyroid nodules and differentiated thyroid trial hyperplasia and osteoporosis and cyclic progestin
cancer. Thyroid. 2009 Nov:19(11):1167-214. Erratum in: Thyroid. 2010
Aug:20(8):942. [PMID: 198605771 to prevent endometrial hyperplasia.

100
Answers and Critiques

Bibliography hyperprolactinemia but should only be done under the super­


Cordts EB, Christofolini OM, Dos Santos AA, Bianco B. Barbosa CP. Genetic vision of a psychiatrist. 1he patient's risperidone should not
aspects of premature ovarian failure: a literature review. Arch Gynecol
Obstet. 2011 Mar:283(3):635-43. [PMID: 21188402] be discontinued unless her psychiatrist is consulted.
Hypothyroidism can cause hyperprolactinemia when
uncontrolled, but her hypothyroidism is well treated. She
Item 10 Answer: B has no symptoms of hypothyroidism, and her thyroid func­
Educational Objective: Treat subacute thyroiditis. tion tests are normal, so uncontrolled hypothyroidism is not
the cause of her hyperprolactinemia.
AP-blocker, such as metoprolol, is the most appropriate treat­ Antipsychotic agents cause hyperprolactinemia by
ment for this patient with thyrotoxicosis who has subacute blocking dopamine, but lithium does not cause hyperpro­
granulomatous (de Quervain) thyroiditis based on the low lactinemia.
radioactive iodine uptake (RAJU) and the painful thyroid on Although the patient has a likely explanation for her
examination. An antecedent event, in this case a viral ill­ hyperprolactinemia, a prolactinoma is still possible. Risperi­
ness, destroys the thyroid follicles and triggers the release of done can cause prolactin levels above 200 ng/mL (200 µg/L),
preformed thyroid hormones into the bloodstream, creating so her level of 102 ng/mL (102 µg/L) is not unreasonable.
the thyrotoxic phase. During this phase, further release of Remeasuring the prolactin level following discontinuation
thyroid hormones ceases, resulting in a low RAJU as seen in of the drug is recommended before further evaluation for
this patient. P-Blockers are beneficial in the thyrotoxic phase a pituitary adenoma; the patient's psychiatrist should be
to block the adrenergic effects of the high circulating thyroid consulted before withholding the risperidone for testing. A
hormone levels. Treatment of subacute thyroiditis is other­ patient with hyperprolactinemia without a clear secondary
wise typically supportive, with NSAIDs for pain as needed. In or drug-induced cause should be assessed by an imaging
patients with severe pain, glucocorticoids may occasionally study (preferably, MRI of the pituitary gland) to a exclude
be used. pituitary lesion.
Blocking further release of thyroid hormones with a
KEY POINT
thionamide (either methimazole or propylthiouracil) is inef­
fective because the thyroid has already released preformed • Antipsychotic agents block dopamine and decrease
thyroid hormone into the bloodstream and is currently not inhibition of prolactin release at the pituitary, causing
producing or secreting additional thyroxine. hyperprolactinemia.
Administration of radioactive iodine for treatment of
the hyperthyroidism is also ineffective because this patient's Bibliography
thyroid is not currently taking in iodine, as evidenced by the Melmed S, Casanueva FF. Hoffman AR. et al. Diagnosis and treatment of
low uptake on his thyroid scan. More importantly, radioac­ hyperprolactinemia: an Endocrine Society clinical practice guideline.
J Clin Endocrinol Metab. 2011 Feb:96(2):273-88. [PMID: 2129699!]
tive iodine will not treat this patient's underlying problem of
damaged thyroid follicles and release of preformed thyroid
hormone into the bloodstream.
Item 12 Answer: B
KEY POINT Educational Objective: Manage low bone mass.
• Treatment of subacute granulomatous (de Quervain) A repeat dual-energy x-ray absorptiometry (DEXA) scan
thyroiditis typically is supportive, with NSAIDs and should be repeated in 2 years in this patient with low bone
occasionally glucocorticoids for severe pain; mass and relatively low 10-year fracture risk. The Fracture
P-blockers are beneficial in the thyrotoxic phase to Risk Assessment Tool (FRAX) calculator defines the 10-year
block the adrenergic effects of the high circulating fracture risk for patients with T -scores in the -1.0 to - 2.S
thyroid hormone levels. ranges. The FRAX calculator (www.shef.ac.uk/FRAX) incor­
porates multiple risk factors including sex, fracture history,
Bibliography femoral neck bone mineral density, glucocorticoid use, smok­
Sweeney LB. Stewart C, Gaitonde DY. Thyroiditis: an integrated approach. ing, BM!, age, and alcohol intake to determine projected frac­
Am Fam Physician. 2014 Sep 15:90(6):389-96. [PMID: 25251231]
ture risk. If the risk of major osteoporotic fracture is greater
than or equal to 20% or the risk of hip fracture is greater than
Item 11 Answer: D or equal to 3%, then the patient's benefit from therapy exceeds
the risk, and she should be offered treatment. Because of her
Educational Objective: Diagnose hyperprolactinemia
history of low body weight and limited nutritional intake
caused by an antipsychotic agent.
during the time of development of peak bone mass, she is at
Hyperprolactinemia is a known side effect of antipsychotic increased risk for low bone mass or osteoporosis and is there­
agents, and this patient's hyperprolactinemia is likely due fore an appropriate candidate for early screening. Her DEXA
to risperidone. Antipsychotics block dopamine and decrease scan shows low bone mass. Spine film shows no evidence of
inhibition of prolactin release at the pituitary, causing hyper­ fracture. Additionally, her calcium and vitamin D levels are
prolactinemia. Stopping the medication can reverse the normal. Continuing lifestyle activities (such as maximizing

101
Answers and Critiques

weight-bearing exercise and avoidance of tobacco or excessive occur during an acute illness is lowering or the serum total
alcohol) in addition to calcium and vitamin D supplementa­ triiodolhyronine (T.) level. 1l1e serum thyroid-stimulating
tion is appropriate management of this patient. hormone (TSH) and serum free thyroxine (T) levels may also
Raloxifene is a selective estrogen receptor modulator decline as the illness increases in severity. '!he pattern of these
(SERM) that is a treatment option for women with osteo­ test results is often indistinguishable from that seen with
porosis because it has been shown to increase bone mineral ccntra I hypothyroidism. In fact. some con Lroversy exists about
density and reduce the risk of vertebral (but not nonverte­ whether the clinical picture of' a low sernrn TSH level and low
bral) fractures. However, raloxifene is also associated with serum T 1 and T� levels is an adaptive response to the critical
an increased risk of thromboembolic events and vasomotor illness in order to alter the body's metabolism and thereby aid
symptoms. There is limited data supporting use of raloxifene in recovery from the acute illness.
or other SERMs for treating patients with low bone mass, ·111is patient does not have any definite findings of
although some guidelines recommend considering treat­ underlying Graves disease. 1l1e tachycardia and fever are
ment in patients with low bone mass and 10-year fracture likely the result of his severe infection. If' he was also
risk determined by the FRAX calculator of greater than or experiencing thyroid storm, high serum T I and T3 levels
equal to 20% for a major osteoporotic fracture or greater than would most likely be associated with the low serum TSH
or equal to 3% for hip fracture. Raloxifene would therefore concentration.
not be appropriate therapy for this patient. ·n,e most common cause of hypothyroidism is
Cholecalciferol (D), a metabolite of vitamin D, is com­ Hashimoto thyroiditis. Physical examination findings can
monly used to supplement low serum vitamin D levels in include a reduced basal temperature. diastolic hypertension.
patients with vitamin D deficiency. This patient has normal an enlarged thyroid gland. bradycardia. p,1llor. dry and cold
serum vitamin D levels; therefore, there is no indication for skin, brittle hair. hoarseness. and a delayed recovery phase
treatment with vitamin D metabolites. of deep tendon renexes. Patients with Hashimoto thyroiditis
Bisphosphonates are considered first-line therapy for have low T.1 and T.1 levels and elevated TSH. This patient's
osteoporosis, although they are not used routinely in women low TSH level is not compatible with Hashimoto thyroiditis.
with low bone mass. Similar to the use of SERM therapy, TI1is patient is unlikely to have subacute thyroiditis
guidelines recommend consideration of treatment with a based on the timing of the illness. Although the infection
bisphosphonate for low bone mass only if there is 10-year could have triggered destruction of the thyroid. the subse­
fracture risk determined by the FRAX calculator of greater quent release of preformed thyroid hormones into the serum
than or equal to 20% for a major osteoporotic fracture or should result in an elevation of serum T4 and T1 levels in the
greater than or equal to 3% for hip fracture. early phase of the disease. which is not consistent with the
findings in this case.
KEY POINT

• Treatment for low bone mass in postmenopausal KEY POINT

women involves lifestyle modification (maximizing • Unless there is a strong suspicion of an underlying
weight-bearing exercise and avoidance of tobacco or thyroid disorder that may be contributing to a
excessive alcohol) and vitamin D and calcium supple­ patient's clinical findings, thyroid function tests
mentation; the need for pharmacologic therapy is should not be performed during critical illness
based on the 10-year estimated fracture risk (:2'.20% for because test results are highly likely to be abnormal.
a major osteoporotic fracture or :2'.3% for hip fracture).
Bibliography
Bibliography Farwell AP. Nonthyroidal illness syndrome. Curr Opin Endocrinol Diabetes
Obes. 2013 Oct:20(5):478-84. [PMID: 23974778]
Kling JM. Clarke BL. Sandhu NP. Osteoporosis prevention, screening. and
treatment: a review. J Womens Health. 2014 Jul:23(7):563-72. [PMID:
24766381]
Item 14 Answer: B
Educational Objective: Liberalize glycemic targets in

Cl Item 13 Answer: A
Educational Objective: Diagnose euthyroid sick
a patient with multiple diabetic complications and
advanced microvascular disease.
syndrome.
1l1e most appropriate next step in management of this
-111is paliem·s clinical picture is consistent with euthyroid patient is to decrease the risk of hypoglycemia by decreas­
sick syndrome (also called nonthyroidal illness syndrome). ing the insulin doses delivered by the pump. This patient
Unless there is a strong suspicion of an underlying thyroid has had type 1 diabetes for 25 years with subsequent devel­
disorder that may be contributing to a patient's clinical find­ opment of advanced microvascular disease. His frequent
ings. lhyroid function tests should not be performed during hypoglycemic events and hypoglycemic unawareness
critical illness. In hospitalized patients. especially ones as ill increase the risk of morbidity and mortality from recur­
as the patient described here. thyroid function test results rent hypoglycemia that may occur with stringent glycemic
are highly likely to be abnormal. The first deviation that may goals. Glycernic goals should be individualized to account

102
Answers and Critiques

for patient-specific factors, such as age and comorbidities. Parathyroid imaging with a sestamibi scan is not indi­
The American Diabetes Association suggests a hemoglo­ cated because the patient does not have primary hyper­
bin A 1 e goal of less than 8.0% in patients with a decreased parathyroidism. Her parathyroid hormone level is elevated
life expectancy, history of severe hypoglycemia, multiple as an appropriate physiologic response to her markedly
comorbidities, or advanced microvascular or macrovascular low vitamin D levels. Once her vitamin D levels are suffi­
disease. lhe less stringent hemoglobin A1 e goal should be cient (>30 ng/dL [75 nmol!L]), her parathyroid hormone
implemented to avoid recurrent hypoglycemia; however, level should be remeasured to ensure that it has returned to
the goal may need to be increased above 8.0% if it cannot the normal range. lhe parathyroid hormone level should be
be achieved safely. remeasured in approximately 4 weeks.
Altering the pump settings to deliver more insulin to Referral for parathyroidectomy would also not be indi­
attain a hemoglobin A1e goal of less than 7.0% will increase cated in this patient without an established diagnosis of
his risk of hypoglycemia. His hemoglobin A1e goal should be primary hyperparathyroidism.
liberalized to avoid hypoglycemia. A person her age with normal diet and minimal sun
The risk of hypoglycemia can be reduced with lower exposure should require about 1000 U daily of vitamin D to
insulin doses delivered by either an insulin pump or sub­ maintain adequate vitamin D stores. The choice to use chole­
cutaneous injections. Since the patient is already using an calciferol versus ergocalciferol is often based on the level of
insulin pump, alteration of his insulin pump settings to vitamin D deficit. Since the ergocalciferol is more readily
deliver less insulin should occur next. available in the 50,000 U form and has a shorter half-life, it
Gabapentin for treatment of his painful peripheral neu­ is recommended when a patient's vitamin D level is less than
ropathy is appropriate, but avoidance of recurrent hypogly­ 10 ng/mL (25 nmol/L). Cholecalciferol is often used when
cemia is the most serious issue that needs to be addressed the level is between 20 and 30 ng/mL (50-75 nmol/L) or for
next due to the associated increased risk of morbidity and maintenance and therefore would not be ideal to replete this
mortality. patient. Clinical discretion can be used for levels between 10
and 20 ng/mL (25-50 nmol!L).
KEV POINT
Although vitamin 03 (cholecalciferoi) is a reasonable
• A less stringent hemoglobin A1 e goal is appropriate for option for treatment of vitamin D deficiency, as mentioned
persons with diabetes mellitus with a decreased life above, cholecalciferol is best used for maintenance of vita­
expectancy, history of severe hypoglycemia, multiple min D levels or repletion when the 25-hydroxyvitamin
comorbidities, or advanced rnicrovascular or macro­ D level is between 20 ng/mL (50 nmol/L) and 30 ng/mL
vascular disease. (75 nmol/L). Neither form of repletion, however, will be
effective in the presence of significant malabsorption.
Bibliography
KEV POINT
American Diabetes Association. (6) Glycemic targets. In: Standards of
Medical Care in Diabetes-2015. Diabetes Care. 2015 Jan:38 Suppl l:S33- • When vitamin D repletion efforts fail, secondary
40. [PMID: 255377051
causes, such as malabsorption, should be considered.

Bibliography
Item 15 Answer: D
Tavakkol A. DiGiacorno D. Green PH, Lebwohl B. Vitamin D status and con­
Educational Objective: Determine causes of vitamin comitant autoimmunity in celiac disease. J Clin Gastroenterol. 2013
Jul:47(6):515-9. [PMID: 23328299]
D deficiency.

This patient with refractory vitamin D deficiency, despite


aggressive attempts at repletion, should be screened for celiac Item 16 Answer: A
disease. The fact that supplementation with a therapeutic
Educational Objective: Treat subclinical
dose of vitamin D has failed to replete her body stores should
hypothyroidism.
raise concern for a malabsorption disorder. Based on her his­
tory of another autoimmune disorder (vitiligo), lower range The most appropriate next step in management is to initiate
BM!, and nonspecific signs and symptoms such as fatigue, levothyroxine therapy. This patient has subclinical hypothy­
subclinical celiac disease would be a reasonable cause of roidism with a mild elevation of the serum thyroid-stimulat­
her malabsorption. Celiac disease may present with classic ing hormone (TSH) level and normal serum free thyroxine (T,)
symptoms of diarrhea, overt malabsorption, and weight loss, level. Her family history of hypothyroidism and her thyroid
but may also exist in a very mild form and may go largely peroxidase antibody positivity increase the likelihood of pro­
undetected since patients have only nonspecific symptoms gression to overt thyroid failure. ln patients with a mild serum
and subclinical rnalabsorption. If this patient tests positive for TSH elevation (between the upper limit of normal and 10 µU/
celiac disease, removing gluten from her diet will improve her mL [10 mU/L]), beginning levothyroxine therapy is reasonable
intestinal lining and improve absorption of vitamin D. Even if symptoms suggestive of hypothyroidism are present.
with therapy, patients with malabsorption will likely require Thyroid-stimulating immunoglobulins (TSls) are highly
increased doses of vitamin D supplementation. associated with Graves disease. When the diagnosis of

103
Answers and Critiques

Graves disease cannot be made clinically in a patient with A bisphosphonate, such as alendronate, would not be
hyperthyroidism, measurement of the serum level of these the treatment of choice for this patient as she does not
antibodies is recommended, especially if radioactive iodine have osteoporosis nor does she meet Fracture Risk Assess­
uptake studies are not available or if radioactive iodine expo­ ment Tool (FRAX) criteria for therapy. 1he FRAX calculator
sure is contraindicated, as in pregnancy and breastfeeding. defines the 10-year fracture risk for patients with T -scores
This patient does not have hyperthyroidism, and these tests in the -1.0 to -2.5 range. The FRAX calculator (www.shef.
are therefore not indicated. ac.uk/FRAX) incorporates multiple risk factors including
Repeating a serum TSH measurement in this patient gender, fracture history, femoral neck bone mineral den­
was reasonable, since up to 30% of patients with an initially sity, glucocorticoid usage, smoking, BM!, age, and alcohol
abnormal serum TSH level will have a normalization of intake to determine projected fracture risk. If the risk of
this value upon retesting. Since this patient has persistent major osteoporotic fracture is greater than or equal to 20%
elevation of TSH and symptoms that may be attributable to or the risk of hip fracture is greater than or equal to 3%,
hypothyroidism, waiting 12 months before initiating therapy the patient's benefit from therapy exceeds the risk, and she
is not appropriate. should be offered treatment. Additionally, bisphosphonates
A radioactive iodine uptake (RAJU) scan is reserved for should be used with caution in patients with compromised
patients with hyperthyroidism. Patients with Graves disease kidney function.
typically have an elevated RAJU. Conversely, in patients with If the patient declines surgical intervention, a calci­
thyroiditis or exposure to exogenous thyroid hormone, the mimetic agent such as cinacalcet would be an appropriate
RAJU will be low (<5%) despite biochemical hyperthyroid­ therapy. Cinacalcet has recently been FDA approved as an
ism. Obtaining a thyroid RAJU in this patient is not indicated. alternative for patients unable or unwilling to undergo para­
thyroidectomy. Cinacalcet lowers calcium levels by stimulat­
KEY POINT
ing the calcium-sensing receptors of the parathyroid glands
• Levothyroxine therapy is reasonable in a patient with and inhibiting parathyroid hormone secretion. However, it is
subclinical hypothyroidism and symptoms suggestive expensive and has multiple drug interactions, which make it
of hypothyroidism. less desirable in this patient.
Because this patient is considered to be at increased risk
Bibliography of complications due to untreated hypercalcemia, clinical
Garber JR. Cabin RH. Gharib H. et al; American Association of Clinical observation alone would not be appropriate.
Endocrinologists and American Thyroid Association Taskforce on
Hypothyroidism in Adults. Clinical practice guidelines for hypothyroid­ KEY POINT
ism in adults: cosponsored by the American Association of Clinical
Endocrinologists and the American Thyroid Association. Endocr Pract. • D ecreased estimated glomerular filtration rate (<60
2012 Nov-Oec:18(6):988-1028. Erratum in: Endocr Pract. 2013 Jan­
Feb;l9(l):175. [PMID: 23246686] mL/min/1.73 m2) is an indication for surgical treat­
ment of primary hyperparathyroidism in otherwise
asymptomatic persons.
Item 17 Answer: C
Bibliography
Educational Objective: Define surgical indications for Bilezikian JP, Brandi ML. Eastell R, Silverberg SJ. Udelsman R. Marcocci C,
primary hyperparathyroidism. et al. Guidelines for the Management of Asymptomatic Primary
Hyperparathyroidism: Summary Statement from the Fourth
The most appropriate treatment recommendation for this International Workshop. J Clin Endocrinol Metab. 2014;99(10):3561-9.
[PMIO: 251626651
patient is parathyroidectomy. She has primary hyperpara­
thyroidism as shown by her elevated serum calcium and
parathyroid hormone levels. She also has evidence of kidney Item 18 Answer: C
compromise with an elevated creatinine level and decreased
Educational Objective: Evaluate a toxic thyroid nodule.
estimated glomerular filtration rate (eGFR). Impaired kid­
ney function (defined as eGFR <60 mL/min/1.73 m2 , 24-h Performing a radioactive iodine (123 1) uptake and scan is the
urine calcium >400 mg/24 h [10 mmol/24 h], or the presence most appropriate management for this patient, who most
of nephrolithiasis or nephrocalcinosis by radiograph, ultra­ likely has a toxic nodule or nodules as the cause for her mild
sound, or CT) is an indication for surgical treatment ofhyper­ hyperthyroidism. Since multiple nodules were identified on
parathyroidism in otherwise asymptomatic patients. Other ultrasound, it is important to ascertain which of the nodules
indications for surgery in asymptomatic patients include age are autonomous, as the likelihood of malignancy in such
younger than 50 years; a serum calcium level greater than lesions is very low. Performing a thyroid scan will help iden­
or equal to 1 mg/dL (0.25 mmol/L) above upper limit of nor­ tify whether one or more of the nodules is responsible for her
mal; a T-score of -2.5 or worse at the lumbar spine, total hip, thyroid function abnormalities.
femoral neck, or distal radius: or in those in whom medical A thyroid scan will also determine if fine-needle aspi­
surveillance is neither desired nor possible. Patients with ration is indicated elsewhere in the gland. Nodules identi­
these indications are considered to have the highest potential fied as autonomously functioning (or "hot") do not require
benefit from surgery. fine-needle aspiration, whereas nodules that are either "cold"

104
Answers and Critiques

or "warm" (of similar uptake to the surrounding non-nodular specific kind of radiation therapy) would be considered in
thyroid tissue) may require cytologic evaluation. a patient with a tumor that is refractory to medical therapy
This patient may ultimately benefit from methimazole and incompletely resectable.
therapy, but since her hyperthyroidism is mild, it is safe to Because this patient has a prolactinorna, the treatment
wait to initiate medical therapy until after the thyroid scan of choice is a dopamine agonist, not surgery. There is an
is performed . In patients with a greater degree of thyrotoxi­ excellent chance that the prolactinoma will respond to cab­
cosis, beginning methimazole to lower the thyroid hormone ergoline, and the patient can avoid surgery.
levels prior to performing additional testing is reasonable.
KEY POINT
Once the hormone levels are nearing the normal range,
the thionamide should be withheld for s days prior to the • Dopamine agonists, such as cabergoline, are first-line
thyroid scan. therapy for symptomatic patients with prolactinomas.
If the patient is found to have a toxic nodule or multiple
toxic nodules, surgical removal is a reasonable treatment Bibliography
approach, particularly if there are compressive symptoms Melmed S. Casanueva FF, Hoffman AR. et al. Diagnosis and treatment of
hyperprolactinemia: An Endocrine Society clinical practice guideline. J
from the goiter. Since this is considered an elective proce­ Clin Enclocrinol Metab. 2011 Feb:96(2):273-88. [PMID: 21296991]
dure, administering low-dose methimazole to normalize
of her thyroid hormone levels would be advisable prior to

Cl
surgery.
Item 20 Answer: B
KEY POINT Educational Objective: Diagnose hypoglycemia in a
• Performing a radioactive iodine (123 1) thyroid uptake patient with diabetes mellitus taking a sulfonylurea.
and scan is the most appropriate initial management Hypoglycemia is the most likely cause of this patient's
for a patient who has mild hyperthyroidism most altered mental status. In patients taking a sulfonylurea for
likely caused by a toxic nodule or nodules. diabetes who develop dehydration, such as this patient with
decreased oral intake in conjunction with nausea and vom­
Bibliography iting. impaired kidney perfusion may lead to altered phar­
Jameson JL. Minimizing unnecessary surgery for thyroid nodules. N Engl J macokinetics and an increased risk of hypoglycemia related
Med. 2012 Aug 23:367(8):765-7. [PMID: 227316711
to ongoing effects of the medication and minimal carbo­
hydrate intake. Although sulfonylureas are very effective
Item 19 Answer: A antihyperglycemic medications. most agents have relatively
long half-lives. allowing convenient daily dosing. However,
Educational Objective: Treat a macroprolactinoma.
this slower clearance predisposes to hypoglycemia com­
The patient has a macroprolactinoma, which is best treated pared with other antiglycemic medications, particularly
with a dopamine agonist such as cabergoline. The most com­ when kidney function is impaired. Glyburide, in particular.
mon cause of hyperprolactinemia is a prolactinoma, which is has a longer half-life than olher sulfonylureas and its use is
a benign adenoma. Microprolactinomas are less than 10 mm recommended against in older patients by the Beers Crite­
in diameter, and macroprolactinomas are 10 mm or greater ria. a list of medications that should be avoided or used with
in diameter. Prolactinomas are the most common type of caution in older pal ients. Although she last took glimepiride
secretory pituitary adenoma. The patient's tumor is causing more Lhan 24 hours before her presentation with altered
significant mass effect, including compression of the optic mental status. she was dehydrated. thus prolonging the
chiasm, invasion into the cavernous sinus, secondary hypo­ glucose-lowering effects or this insulin secretagogue for
thyroidism, and likely growth hormone deficiency; however, several clays.
even with these complications, the macroprolactinoma is Although patients with diabetes are at increased risk
best treated with medication instead of surgery. A dopamine for atherosclerolic cardiovascular disease. this patient does
agonist can cause a rapid decrease in the serum prolactin not have local abnormalities suggesting a cerebrovascular
level and shrinkage of the prolactinoma. More specifically, in accident. and stroke itself' is nol a common cause of isolated
as many as 90% of patients, it can normalize prolactin levels, altered mental status.
reverse hypogonadism, and shrink tumors by at least 50%. Levothyroxine also has a relatively long half-life allow­
Because of these rapid decreases in tumor size, dopamine ing once daily dosing in most patients. Missing one or sev­
agonists can be used as first-line therapy, even in patients eral closes of levothyroxine would therefore not likely lead
with mild visual field defects, as long as visual acuity is not to a degree of hypothyroidism causing acute mental status
threatened by rapid progression of the h1mor or recent tumor changes.
hemorrhage. Statin toxicity is unusual. and the most common toxic­
Octreotide is used to treat acromegaly but has no role in ity associated with statin use is musculoskeletal symptoms.
the first-line treatment of prolactinoma. Increased stat in levels are not typically associated with men­
Medical therapy is preferred over surgery and radiation tal status changes and would likely not be the cause of this
for the treatment of prolactinomas. Stereotactic surgery (a patient's menLal status changes.

105
Answers and Critiques

KEY POINT Item 22 Answer: B


• Sulfonylureas with long half-lives, such as glimepir­ Educational Objective: Diagnose hypercalcemia in a
ide, may lead to acute kidney injury and hypoglyce­ patient with sarcoidosis.
mia in older persons with diabetes mellitus. This patient most likely has hypercalcemia due to increased
1,25-dihydroxyvitamin D levels. She has an elevated cal­
Bibliography cium level with a low parathyroid hormone level, indicating
American Diabetes Association. (7) Approaches lo glycemic treatment. In: non-parathyroid hormone (PTH)-mediated hypercalcemia.
Standards of Medical Care in Diabetes-2015. Diabetes Care. 2015 Jan:38
Suppl l:S41-8. [PMID: 25537707] The differential diagnosis of non-PTH-mediated hypercalce­
mia includes cancer-related hypercalcemia caused by osteo­
lytic lesions of bone, humorally mediated by tumor-secreted
Item 21 Answer: A parathyroid hormone-related protein (PTHrP), or granuloma­
Educational Objective: Evaluate triiodothyronine (T3) tous diseases, such as sarcoidosis. In granulomatous diseases,
hyperthyroidism. the hydroxylase in disease-associated macrophages actively
converts 25-hydroxyvitamin D to the highly active 1,25-
The serum triiodothyronine (T) level should be measured dihydroxyvitamin D metabolite. The increased levels of active
next. This patient exhibits signs and symptoms of hyperthy­ vitamin D lead to increased absorption of calcium in the
roidism. Even though the available laboratory data are consis­ gut, promotion of increased bone resorption of calcium, and
tent with subclinical hyperthyroidism, the diagnostic evalu­ decreased calcium and phosphate excretion by the kidney.
ation is not complete until the T3 level is checked. Although Elevated serum calcium can be due to a mutation in the
rare, an elevated T 3 level and a normal serum free thyroxine G-coupled protein calcium-sensing receptor (CASR) gene.
(T) level may be present in patients with hyperthyroidism. These receptors are in the parathyroid glands and the kid­
Measurement of the T3 concentration should therefore be neys. The sensor mutation results in a shift upward in the
obtained in all patients suspected of having thyrotoxicosis. "normal" range of calcium that the receptor recognizes,
Measurement of the T3 level is not indicated in patients with resulting in a mildly elevated serum calcium level (usually
hypothyroidism because the T3 concentration is conserved <11.0 mg/dL [2.7 mmol!L]) and high normal or mildly ele­
and may remain within the normal range, even in patients vated PTH level, unlike this patient whose PTH level was low.
with significant hypothyroidism. 25-Hydroxyvitamin D is the major circulating form of
Measuring the thyroid peroxidase (TPO) antibody titer vitamin D and is minimally metabolically active relative to
will not provide additional information about this patient. 1,25-dihydroxyvitamin D. Levels in this patient are likely to be
Determining TPO antibody status is helpful in patients with low due to excessive conversion to the 1,25-dihydroxyvitamin
a mildly elevated senun thyroid-stimulating hormone (TSH) D form, and would not identify the likely mechanism of
level and is associated with Hashimoto thyroiditis and future hypercalcemia in this patient.
risk of developing permanent hypothyroidism. However, Hydrochlorothiazide, a thiazide diuretic, reduces blood
this patient demonstrates signs and symptoms consistent volume by acting on the kidneys to reduce sodium (Na+) reab­
with hyperthyroidism. sorption in the distal convoluted tubule. Thiazides increase
Repeating the thyroid function tests (TFTs) in 6 the reabsorption of calcium in the distal convoluted tubule
weeks may be appropriate if the total or free T 3 level is by their action on a Na+ -c1- calcium co-transporter. However,
found to be normal. If the T 3 is normal, the patient has any increase in calcium caused by thiazides is mild and rarely
subclinical hyperthyroidism, and repeating the TFTs may reduces the PTH level below the lower range of normal.
be indicated to determine if the abnormality is transient
or permanent. KEY POINT
Ultrasound of the neck is appropriate for the eval­ • Granulomatous diseases, such as sarcoidosis, cause
uation of this patient if a nodule is suspected. However, hypercalcemia through increased !-a-hydroxylation
this patient's diffusely enlarged thyroid gland is not activity that increases 1,25-dihydroxyvitamin D levels
suggestive of nodular disease. In addition, even if the and calcium reabsorption.
physical examination were suggestive of nodular dis­
ease, the first step would be evaluation of the functional Bibliography
thyroid status. Sharma OP. Hypercalcemia in granulomatous disorders: a clinical review.
Curr Opin Pulm Med. 2000 Sep:6(5):442-7. [PMID: 10958237]
KEY POINT

• Most patients with thyrotoxicosis will have elevations


of both free thyroxine (T 4) and triiodothyronine (T), Item 23 Answer: D
but isolated T3 elevation is rare. Educational Objective: Diagnose Cushing syndrome
from exogenous glucocorticoids.
Bibliography
Vaidya B. Pearce SH. Diagnosis and management of thyrotoxicosis. BMJ. The patient has iatrogenic Cushing syndrome caused
2014 Aug 21:349:g5128. [PMID: 25146390] by use of topical glucocorticoids in the treatment of

106
Answers and Critiques

psoriasis. Cushing syndrome presents similarly whether it KEY POINT


is due to a pituitary adenoma (Cushing disease), adrenal
• During the initial management of diabetic ketoacido­
tumor cortisol production, ectopic adrenocorticotropic
sis, hypokalemia should be corrected before initiation
hormone production, or excessive use of glucocorticoids.
Her presentation would be consistent with any of these of intravenous insulin therapy to avoid significant
diagnoses; however, she is on high-potency topical glu­ worsening of the serum potassium levels that could
cocorticoids, so this alone explains her symptoms and cause cardiac arrhythmias.
presentation. Exogenous glucocorticoid use as a cause of
Cushing syndrome is common, whereas the other causes Bibliography
are rare. Wilson JF. In the clinic. Diabetic ketoacidosis. Ann Intern Med. 2010 Jan
5;152(1):ITC!-IS. [PMID: 20048266]
KEY POINT

• The most common cause ofCushing syndrome is an


elevated level of cortisol resulting from both endoge­
Item 25 Answer: D
nous and exogenous exposure to glucocorticoids. Educational Objective: Identify secondary causes of
bone loss prior to initiating bisphosphonate therapy.
Bibliography Checking this patient's serum thyroid-stimulating hormone
Nieman LK, Biller BMK, Findling JW, et al. The diagnosis of Cushing's syn­ (TSH) level would be the most appropriate test to obtain
drome: an Endocrine Society clinical practice guideline. J Clin Endocrinol
Metab. 2008 May;93(5):1526-40. [PMID: 183345801 prior to initiation ofpharmacologic therapy. Although osteo­
porosis in postmenopausal women is most commonly asso­

CJ Item 24
ciated with nonmodifiable risk factors such as age, sex,
Answer: B menopausal status, height, and build, it is always important
to assess for possible secondary causes of bone loss that
Educational Objective: Treat a patient with diabetic
might be amenable to treatment, particularly if there is clin­
ketoacidosis.
ical suspicion in a specific patient. Appropriate laboratory
This patient has diabetic ketoacidosis (OKA) and a low serum testing for most patients with newly diagnosed osteoporosis
potassium level, so potassium chloride should be adminis­ includes complete blood count (for malignancy), complete
tered. The patient should be making adequate urine before metabolic profile (for calcium levels and kidney function),
adminjstration of potassium replacement. Low potassium TSH, 25-hydroxyvitamin D, and urine calcium (screening for
stores in the body will need correction prior to initiation hypercalciuria), most of which were normal in this patient.
of insulin therapy to avoid cardiac arrhythmias. Potassium However, this patient's history of unintentional weight loss
chloride should be added to each liter of intravenous fluids to over the past year may be her only symptom of hyperthy­
maintain the serum potassium level in the 4.0 to 5.0 mEq/L roidism, which could be contributing to her osteoporosis
(4.0-5.0 mmol/L) range with the continued use of intrave­ and would be important to treat in addition to therapy
nous insulin tl1erapy. directed toward her osteoporosis. Therefore, measuring this
Antibiotic therapy is not warranted at this time. The patient's TSH level would be appropriate prior to starting
mild leukocytosis upon presentation is most likely related to therapy for osteoporosis.
stress from OKA rather than an infectious process. ln addi­ Both serum and urine markers of bone turnover mea­
tion. the chest radiograph is normal. sure collagen breakdown products and other chemicals
Sodium bicarbonate provides no added benefit when released from osteoclasts and osteoblasts as part of bone
the arterial blood pH is greater than 6. 9 and may be asso­ metabolism. However, they are not commonly used in most
ciated with harm. A 2011 systematic review found that patients with osteoporosis primarily because there is signif­
bicarbonate administration worsened ketonemia. Several icant variability in the different measures in an individual
studies found higher potassium requirements in patients patient or between different patients, making standard­
receiving bicarbonate. Studies in children found a possi­ ization of results difficult. Therefore, their use is typically
ble association between bicarbonate therapy and cerebral limited to research settings or in management of specific
edema. patients who have failed to respond to usual therapy for
During the treatment of OKA, initiation of insulin osteoporosis.
therapy and correction of acidosis will shift the extracel­ There is evidence that estrogen is effective for preven­
lular potassium back into the intracellular space. Signifi­ tion and possibly treatment for osteoporosis, although the
cant worsening of the hypokalemia noted at presentation significant nonskeletal risks associated with this therapy
could occur if this is not corrected to a serum potassium have led to its not being used in favor of bisphosphonates.
level higher than 3.3 rnEq/L (3.3 mrnol/L) prior to initi­ As this patient is postmenopausal and has no clinical sug­
ation of insulin therapy, which could lead to the devel­ gestion of excess estradiol secretion, and bisphosphonate
opment of arrhythmias. Insulin therapy can be safely therapy would be considered preferable to estrogen despite
initiated once the serum potassium level is greater than serum levels, testing for estradiol would not be indicated in
3.3 mEq/L (3.3 mmol/L). this patient.

107
Answers and Critiques

Hyperparathyroidism should always be considered as KEY POINT


a possible secondary cause of bone loss. However, since
• In women with polycystic ovary syndrome, heavy
this patient had normal calcium and 25-hydroxyvitamin
menstrual bleeding, and hirsutism who do not desire
D levels, hyperparathyroidism would be highly unlikely,
fertility, estrogen-progestin oral contraceptive pills are
and checking a parathyroid hormone level would not be
first-line therapy to provide endometrial protection
indicated.
and suppress androgen production.
KEY POINT

• Testing for secondary causes of bone loss is appropri­ Bibliography


ate before beginning pharmacologic therapy for newly Legro RS, Arslanian SA, Ehrmann DA, et al; Endocrine Society. Diagnosis
and treatment of polycystic ovary syndrome: an Endocrine Society clini­
diagnosed osteoporosis. cal practice guideline. J Clin Endocrinol Metab. 2013 Dec;98(12):4565-92.
[PMID: 24151290]
Bibliography
Hudec SM. Camacho PM. Secondary causes of osteoporosis. Endocr Pract.
2013;19:120-8. [PMID: 23186949] Item 27 Answer: D
Educational Objective: Treat adrenocortical
carcinoma.
Item 26 Answer: A
Adrenocortical carcinoma (ACC) is the most likely cause of
Educational Objective: Manage heavy menstrual
this patient's Cushing syndrome, and surgical excision is the
bleeding caused by polycystic ovary syndrome.
most appropriate management. The patient has classic clinical
This patient with polycystic ovary syndrome (PCOS) manifestations of Cushing syndrome (CS), and the 24-hour
with heavy menstrual bleeding and hirsutism should be urine cortisol is markedly elevated on repeated measure­
treated with combined estrogen-progestin oral contra­ ments. Plasma adrenocorticotropic hormone (ACTH) is sup­
ceptive pills. Patients with PCOS remain in a stagnant pressed consistent with an ACTH-independent cause. The
follicular stage resulting in unopposed estradiol secretion imaging characteristics of ACC include a large mass with
from small ovarian follicles, causing proliferation of the irregular borders or shape, calcification, high attenuation
endometrium in the absence of progesterone secretion (high Hounsfield units) on CT, and delay in contrast medium
from a corpus luteum. This predisposes patients to endo­ washout (less than SO% at 10 minutes), findings all present in
metrial hyperplasia and heavy menstrual bleeding as a this patient.
result of anovulatory bleeding. lntraovarian androgen The treatment of ACC depends on the extent of dis­
production is also increased in PCOS, resulting in the ease at presentation. Surgical removal after appropriate bio­
hyperandrogenism and hirsutism associated with the chemical assessment remains the best option, especially in
disorder. Estrogen-progestin oral contraceptive pills are patients with early disease. Even after apparent complete
first-line therapy for the menstrual irregularities and resection, adjuvant therapy with mitotane, a known adrenal
hirsutism associated with PCOS. This therapy prevents cytotoxic drug, may be beneficial. Treatment with mito­
unopposed estrogen-induced proliferation of the endo­ tane is recommended for patients with persistent disease
metrium and suppresses the excess androgen production and others with known metastases and is associated with
associated with PCOS. This would be appropriate therapy objective remissions in approximately 25% of patients. The
to treat both issues in this patient, who does not currently main factors limiting the use of mltotane include nausea,
desire fertility. vomiting, lethargy, and neurologic side effects. Experience
Progestin therapy alone, either through periodic pro­ with other cytotoxic chemotherapy is limited, but has usu­
gestin withdrawal or use of a progesterone-eluting intrauter­ ally been ineffective. A poorer prognosis is associated with
ine device, will provide endometrial protection and treat this advanced stages of the disease, the presence of metastasis at
patient's menstrual irregularity. However, progestin therapy diagnosis, an older age, and cortisol hypersecretion by the
alone does not suppress androgen production and would not tumor. In patients without clinically evident disease after
treat this patient's hirsutism. initial surgery, the median survival rate is 60% at S years.
Metformin has several favorable metabolic effects Fine-needle biopsy cannot distinguish a benign ade­
in patients with PCOS, including increased insulin sen­ noma from a carcinoma and is not used in the evaluation
sitivity and reduced serum free testosterone. However, it of ACC. It is sometimes used to distinguish ACC from meta­
has been shown to be less effective than oral contracep­ static disease. Its use in this patient is both unnecessary and
tives for improving the menstrual pattern and reducing inappropriate.
serum androgens. It also does not provide endometrial Radiation therapy is not used as the principal initial
protection and is considered a second-line therapy for treatment in ACC; however, it may be used as adjuvant ther­
patients with PCOS with significant menstrual irreg­ apy after surgery to prevent tumor recurrence. Radiation
ularities and hirsutism who are unable to tolerate oral therapy can also be used to treat areas of metastasis, such as
contraceptive pills. to the bones or brain.

108
Answers and Critiques

KEY POINT Bibliography


Shoback, D. Hypoparathyroidism. N Engl J Med. 2008 Jul 24;359(4):391-403.
• Surgical removal after appropriate biochemical [PMID: 18650515]
assessment is the most appropriate treatment for
adrenocortical carcinoma, especially in patients with
early disease.
Item 29 Answer: C

Bibliography Educational Objective: Manage early type 2 diabetes


Berruti A, Baudin E, Gelderblom H, et al. Adrenal cancer: ESMO Clinical
mellitus.
Practice Guidelines for diagnosis, treatment and follow-up. Ann Oncol.
2012;23 Suppl 7:viil31-138. [PMID: 22997446] The most appropriate management for this patient is to
initiate metformin. The patient is early in her diabetes

c::J
disease course without evidence of microvascular dis­
Item 28 Answer: D ease. For other wise healthy adults meeting these criteria,
Educational Objective: Diagnose hypomagnesemia the American Diabetes Association recommends a hemo­
as a cause of hypocalcemia. globin A 1c level of less than 7.0%, preprandial glucose
values of 70 to 130 mg/dL (3.9-7.2 mmol/L), and 1- to
This patient's magnesium level should be checked. He has a 2-hour postprandial glucose values of less than 180 mg/dL
very low calcium level that is likely contributing to his clinical (10 mmol/L). Because the patient has not met these goals,
findings of tremulousness, muscle irritability, and electro­ a pharmacologic agent should be added at this time. Life­
cardiogram changes. 1-le struggles with alcohol abuse, and style recommendations consisting of increased physical
his low albumin level suggests malnutrition. likely due to his activity, dietary modifications, and weight loss (if BMI is
chronic alcohol intake as a primary source of calories. Mag­ elevated) are the initial first step in treating diabetes. When
nesium deficiency is common in persons who abuse alcohol. lifestyle modifications fail to meet glycemic goals within
Furthermore, he has had diarrhea for several days, which will 6 weeks, metformin is the recommended first-line ther­
also deplete his magnesium stores. Th.is patient's low calcium apy to be started in conjunction with continued lifestyle
level should promote parathyroid hormone (PIH) secretion modifications. If glycemic goals are not met after 3 months
to help correct the hypocalcemia. However, decreased levels of lifestyle modifications and metformin use, additional
of magnesium impair release of PIH; levels in patients with agents should be added to the regimen every 3 months
significant magnesium deficiency are either low or inappro­ until glucose goals are met.
priately in the normal range. Low magnesium levels are also Dapagliflozin, a sodium-glucose transporter-2 (SGLI-
associated with resistance to PIH activity at the level of bone. 2) inhibitor, increases excretion of glucose through the
further contributing to hypocalcemia. ·n1erefore, in patients kidney. It is a second-line agent that should be used after
with hypocalcemia and hypomagnesemia. it is crucial to cor­ lifestyle modifications and metformin fail to reach glyce­
rect the magnesium level (to at least 2 mg/c!L [0.83 mmol/L]), mic goals.
as it is clifficult to increase calcium levels until this is done. The sulfonylurea glipizide stimulates insulin secretion
1,25-Dihydroxyvitamin D has a short half-life, and from the pancreatic beta cells. This agent could improve
its measurement only reflects the active levels of vitamin the patient's postprandial hyperglycemia, but it may also
D. Since active levels tend to vacillate frequently based on induce weight gain in a patient actively working on weight
immediate need, this level would not be diagnostically help­ loss. Glipizide is a second-line agent that should be used
ful. 25-Hydroxyvitamin D levels have a longer half-life and. after lifestyle modifications and metformin fail to reach
therefore. more accurately reflect the total body stores of glycemic goals.
vitamin D. Sitagliptin, a dipeptidyl peptidase-4 (DPP-4) inhib­
Measuring 24-hour urine calcium excretion in this itor, improves glycemic control by slowing gastric emp­
patient would not be diagnostically helpful as significant tying and suppressing glucagon secretion. It is also con­
urinary loss of calcium is uncommon, and the urine calcium sidered a second-line agent that might be considered if
level in this patient would be expected to be low due to com­ lifestyle modifications and metformin fail to reach glyce­
pensatory kidney retention of calcium. mic goals.
Checking the ionized calcium level is inclicated in settings
KEY POINT
where abnormal serum protein binding of calcium is possible.
TI1is test is not needed in this symptomatic patient who has no • For most patients with type 2 diabetes mellitus, life­
suggestion of excessive protein binding of calcium. style modifications and metformin therapy are the
most appropriate initial treatments.
KEY POINT

• Low serum magnesium levels impair parathyroid hor­ Bibliography


mone secretion and require repletion before serum American Diabetes Association. (7) Approaches to glycemic treatment. In:
calcium levels may be corrected. Standards of Medical Care in Diabetes-2015. Diabetes Care. 2015 Jan;38
Suppl l:S41-8. [PMID: 25537707]

109
Answers and Critiques

Item 30 Answer: C symptoms associated with hypercalcemia. Approximately


40% to 45% of the calcium in serum is bound to protein,
Educational Objective: Identify hemochromatosis as
a cause of hypogonadotropic hypogonadism.
principally albumin, although the physiologically active
form of calcium is in an ionized (or free) state. In most
The patient has a clinical history suspicious for hemochro­ patients with relatively normal serum albumin levels, the
matosis and should be further evaluated by measuring serum total calcium usually accurately reflects the ionized calcium
transferrin saturation and ferritin levels. The patient has fraction. However, in clinical settings where increased pro­
hypogonadism based on his clinical symptoms of decreased tein binding of calcium may occur, the serum total calcium
libido and erectile dysfunction, associated with a low morn­ level may be elevated without a rise in the actual serum
ing serum testosterone level. A hypogonadotropic etiology ionized calcium concentration. This may occur in patients
is indicated by his low luteinizing and follicle-stimulating with hyperalbuminuria (as may occur in those who are
hormone levels. Causes of hypogonadotropic hypogonadism severely dehydrated), and in patients with a paraprotein
include infiltrative diseases such as hemochromatosis, sar­ capable of binding calcium (such as occasionally occurs in
coidosis, cancer metastatic to the pituitary, and lymphoma. some patients with multiple myeloma). This phenomenon
Pituitary tumors that impair gonadotropin function may is sometimes termed pseudohypercalcemia (or factitious
also be a cause. This patient has several clinical findings hypercalcemia). If present, a normal ionized calcium level
suggestive of possible hemochromatosis, including a report may indicate that the elevated total calcium levels are due
of arthralgia and hepatomegaly on physical examination. to excessive protein binding and potentially eliminate the
Therefore, the next step in evaluation of this patient's hypo­ need for further evaluation for hypercalcemia.
gonadotropic hypogonadism is measurement of serum fer­ Measuring the 1,25-dihydroxyvitamin D level is useful
ritin level and transferrin saturation to evaluate for possible in further assessing patients with non-parathyroid hormone
hemochromatosis. (PTH)-mediated hypercalcemia to assess for excess vita­
The cause of hypogonadism must be evaluated prior min D production. PIH testing is indicated in patients with
to the initiation of testosterone replacement. If testosterone hypercalcemia to differentiate between PIH-mediated and
therapy is started without testing for hemochromatosis, the non-PIH-mediated hypercalcemia.
diagnosis may be missed. The hypercalcemia associated with multiple myeloma
Although genetic disorders such as Klinefelter syn­ is caused primarily by tumor-induced, osteoclast-mediated
drome (47,)CXY) may cause hypogonadism, patients with bone resorption due to cytokines released by myeloma cells
this syndrome have hypergonadotropic hypogonadism with and is not PIH-mediated or due to excessive vitamin D lev­
elevated luteinizing and follicle-stimulating hormone values, els. Measurement of PIH level and 1,25-dihydroxyvitamin
unlike this patient. Therefore, karyotyping is not indicated. levels may be indicated as part of this patient's evaluation
A testicular ultrasound is used to evaluate the cause of only after true hypercalcemia has been established.
primary testicular failure and is not indicated in the eval­ Parathyroid hormone-related protein (PTHrP) level
uation of hypogonadotropic hypogonadism. This patient's measurement is useful in evaluating patients with non­
low gonadotropin levels indicate either a hypothalamic or PTH-mediated hypercalcemia but would not be indicated as
pituitary disorder, instead of testicular disease. Although a next study in this patient in whom pseudohypercalcemia
hemochromatosis may also directly affect testicular func­ has not been excluded.
tion in addition to its central hypogonadal effect, testicular
KEY POINT
ultrasound is not helpful in establishing the diagnosis of
hemochromatosis as a cause of hypogonadism. • In patients with conditions in which increased pro­
tein binding of calcium may occur, such as hyperal­
KEY POINT
buminemia or paraproteinemia, an artificially ele­
• Patients with symptoms of hypogonadotropic hypo­ vated total serum calcium level must be excluded.
gonadism should have serum transferrin saturation
and ferritin concentration levels measured to identify Bibliography
hemochromatosis prior to initiating any therapy. Clines GA. Mechanisms and treatment of hy percalcemia of malignancy.
Curr Opin Endocrinol Diabetes Obes. 2011 Dec;l8(6):339-46. [PMID:
21897221]
Bibliography
Bacon BR, Adam PC, Kowdley KV, et al. Diagnosis and management of
hemochromatosis: 2011 Practice guidelines by the American Association
for the Study of Liver Diseases. Hepatology. 2011:54(1):328-43. [PMID: Item 32 Answer: B
21452290]
Educational Objective: Manage androgen therapy in
the setting of hypogonadism.
Item 31 Answer: B
Before initiating therapy for this patient with hypogonad­
Educational Objective: Diagnose pseudohypercalcemia.
ism, his desire for fertility should be explored. Testoster­
This patient's ionized calcium level should be checked. one replacement therapy can be associated with decreased
His total calcium level is elevated, but he is without clear spermatogenesis and infertility. Exogenous testosterone

110
Answers and Critiques

suppresses both hypothalamic gonadotropin-releasing hor­


mone and pituitary follicle-stimulating hormone and lutein­
Item 33 Answer: D
Educational Objective: Diagnose thyroid storm.
Cl
izing hormone (LH) production, resulting in depletion of
intratesticular testosterone. The effect is suppression of sper­ '!his 1x11ient h,1s thyroid storm. most likely precipitated by the
matogenesis so pronounced that testosterone replacement iodine contained in the dye load from the CcltTliac catheteriza­
therapy has been studied as a male hormonal contraceptive. tion in ihe context or underireated Graves disease following
Based on Endocrine Society guidelines, men with hypogo­ her discontinuation or methimazolc. Graves disease is the
nadism should be treated with exogenous testosterone when most common underlying condition ,1ssociated with thyroid
they have consistent signs and symptoms of hypogonadism storm. and onset may be triggered by a number or factors in
and low serum testosterone levels. Symptomatic men may addition to iodine. including inlection. surgery. myocardial
report reduced libido, erectile dysfunction, mood changes, infarction. trauma. or parrurition. ·1his patient has typical
irritability, fatigue, or memory loss. Although this patient's leatures or thyroid storm: lever. tachycardia. heart failure, gas­
symptoms would likely be improved with exogenous admin­ trointestinal dysrunction. and mental st,1t1.1s changes. Scoring
istration of androgen, replacement therapy may also result in systems exist that can be used to provide a more objective
infertility due to oligospermia. Patients with hypogonadism measurement or the severity or the thyrotoxicosis. Treatment
who desire fertility may require treatment with human cho­ or thyroid storm is directed toward supportive care. reduction
rionic gonadotropin (HCG). HCG has LH-like activity and or thyroid hormone production. decreasing peripheral con­
stimulates the production of intratesticular testosterone, \·ersion or thyroxine (T 1 ) to triiodothyronine (T). addressing
resulting in the high concentrations required for induction adrenergic symptoms and thermoregulatory changes, and
and maintenance of spermatogenesis. treating any identified precipitating !actors. Thionamides and
Asymptomatic men with low serum testosterone levels �-blockers arc the mainstay or treatment to reduce thyroid
may experience decreased bone mineral density and oste­ hormone production and control adrenergic symptoms.
oporosis. Hormone replacement therapy will decrease the ·111is paticnt·s clinical presentation is not consistent with
risk of osteoporosis. A bone mineral density measurement cuthyroid sick syndrome. Although this patient·s low level
prior to the initiation of hormone replacement therapy is or serum thyroid stimulating hormone (TSH) is consistent
not needed. with euthyroid sick syndrome. her elevated serum thyroid
Male hypogonadism is associated with increased vis­ hormone level is not. In euthyroid sick syndrome, serum
ceral fat and insulin resistance, and hormone replacement thyroid hormone levels are typically low, creating a clinical
therapy improves these metabolic parameters. There is no picture similar to that seen in central hypothyroidism.
recommendation that hypogonadal patients initiating hor­ Pheochromocytoma typically presents with hyperten­
mone replacement therapy be screened for diabetes mel­ sion and tachycardia but thyromegaly is not a clinical feature
litus with fasting plasma glucose measurement or other or this disorder. r\dditionally. conrusion and altered mental
testing. status arc not presenting signs or the disease.
A 2010 systematic review of hypogonadal men receiv­ Subacute thyroiditis is typically precipitated by a recent
ing testosterone therapy found no evidence of increased \·iral illness. An iodine load. such as this patient had. is an
risk of prostate cancer when compared with the placebo/ unlikely cause. Although patients with subacute thyroiditis
nonintervention group. The Endocrine Society guideline on may present with significant thyrotoxicosis. the thyroid is
testosterone replacement therapy recommends a digital rec­ typically painful to palpation. a clinical finding that was not
tal examination and prostate-specific antigen (PSA) level present in this patient.
determination at 3 and 6 months following the initiation of KEY POINT
replacement therapy. Continued regular screening is recom­
mended for men older than 40 years of age with a baseline • In patients with underlying Graves disease, thyroid
PSA level greater than 6 ng/mL (6 µg/L). Scrotal ultrasound storm may be precipitated by the iodine content
is unnecessary prior to initiation of testosterone therapy; a found in contrast media.
clinical testicular examination to rule out abnormalities or a
testicular mass is sufficient. Bibliography
Klubo-Gwiezdzinska JI. Wartofsky L. Thyroid emergencies. Med Clin North
KEY POINT Am. 2012 Mar:96(2):385-403. [PMID: 224439821

• Prior to initiation of testosterone therapy for hypo­


gonadism, the desire for fertility should be ascer­
Item 34 Answer: D
tained because exogenous testosterone replacement
Educational Objective: Manage postsurgical
therapy may result in oligospermia and infertility.
hypoparathyroidism.

Bibliography This patient without parathyroid function should be advised


Samplaski MK. Loai Y. Wong K, Lo KC, Grober ED. Jarvi KA. Testosterone use to decrease his calcium supplementation intake. In patients
in the male infertility population: prescribing patterns and effects on without parathyroid function, stimulation of the kidney to
semen and hormonal parameters. Fertil Steril. 2014 Jan:101(1):64-9.
[PMID: 24094422] convert 25-hydroxyvitamin D (from the liver) into the active

111
Answers and Critiques

form, 1.25- dihydroxyvitamin D (calcitriol), is lost, as is the rapid-acting insulin, before intensive exercise. Since the dura­
signal to increase reabsorption of calcium in the kidney in the tion of action of insulin glulisine can extend up to 4 hours, cov­
distal convoluted tubule and loop of Henle. As a result, both ering the meal consumption prior to exercise with a smaller
calcitriol and calcium supplementation must be provicl�d. dose of insulin glulisine can reduce the risk of hypoglycemia
as was done in this patient. However, without reabsorption in the setting of intense or prolonged exercise.
of calcium by the kidney, oral calcium will be absorbed and Discontinuation of insulin glargine in a patient with
passed through the kidney, resulting in higher levels of urine type 1 diabetes will lead to hyperglycemia if the rapid-acting
calcium than in patients with normal parathyroid hormone insulin isn't adjusted to provide basal insulin coverage. The
levels. In monitoring calcium status in hypoparathyroid hyperglycemia and insulin deficiency that develop in the
patients, the recommended goal should be a 24-hour urine absence of basal insulin coupled with the stress associated
calcium level of less than 300 mg/24 hours (7.5 mmol/24 h) with exercise will lead to an increase in the release of coun­
with a concomitant serum calcium level in the low normal terregulatory hormones. In this scenario, there may not
range (8.0-8.5 mg/dL [2.0-2.l mmol!L]). lf the urine calcium be sufficient insulin to decrease lipolysis and subsequent
levels are surpassed, it is appropriate to decrease the calcium oxidation of free fatty acids. This could lead to diabetic
intake first; in patients with serum calcium levels greater ketoacidosis.
than 8.5 mg/dL (2.1 mmol!L) and urine calcium greater than The meal-time insulin prior to exercise should be
300 mg/24 hours (7.5 mmol/24 h), concurrent decreases in decreased; however, modification of the diet with increase
both calcium supplementation and calcitriol are indicated. in carbohydrates, rather than protein, can also help avoid
Although 25-hydroxyvitamin D levels are the best indi­ exercise-induced hypoglycemia. Consumption of 15 to 30
cator of total body vitamin D stores in patients with normal grams of carbohydrates prior to exercise and/or a snack with
parathyroid function, it would not be helpful for therapeutic complex carbohydrates after prolonged exercise can help
decision-making in this patient as the lack of PTH requires mitigate the risk of hypoglycemia. Carbohydrates, especially
treatment with activated (1,25-dihydroxy) vitamin D. In this simple ones, can rapidly provide glucose to the bloodstream
patient, 1,25-dihydroxyvitarnin D levels measured concur­ and maximize glycogen stores in the liver that can be utilized
rently with the serum and urine calcium levels are the most for fuel during exercise. The digestion time for protein is
appropriate indicators of therapeutic effect. prolonged compared with carbohydrates, thus providing a
Removal of the parathyroid glands does not allow the slower source of energy during exercise
PTH level to be used to monitor appropriateness of therapy. A sliding-scale regimen of insulin glulisine is a reactive
It would be expected to be low or undetectable in this patient management plan for glucose control. In this scenario, it
who has had resection of his parathyroid glands and chemo­ is possible that the patient could have an increased risk of
therapy and radiation. hyperglycemia or hypoglycemia prior to, during, or after
It would be inappropriate to continue the current exercise secondary to insufficient or excessive doses of insu­
regimen given the elevated 24-hour urine calcium levels. lin from the sliding-scale regimen.
Persistently elevated levels could lead to nephrolithiasis or
KEY POINT
nephrocalcinosis.
• Because exercise can increase glucose utilization by
KEY POINT
the muscles, reducing the doses of mealtime insulin in
• The urine and serum calcium goals are different in a patient with diabetes mellitus will decrease the risk
patients with hypoparathyroidism; the urine calcium of hypoglycemia with intense or prolonged exercise.
goal is less than 300 mg/24 hours (7.5 mmol/24 h)
and serum calcium goal is between 8.0 and 8.5 rng/dL Bibliography
(2-2.1 mmol!L) in these patients. American Diabetes Association. (4) Foundations of care: education. nutri­
tion. physical activity, smoking cessation. psychosocial care. and immu­
nization. In: Standards of Medical Care in Diabetes-2015. Diabetes Care.
Bibliography 2015 Jan:38 Suppl l:S20-30. [PMID: 25537702]
Khan MS. Waguespack SG. Hu M. Medical management of postsurgical
hypoparathyroidism. Endocr Pract. 2011 Mar-Apr:17 Suppl 1:18-25.
[PMID: 21134871]
Item 36 Answer: A
Educational Objective: Diagnose central
Item 35 Answer: A hypothyroidism.
Educational Objective: Manage exercise-induced
hypoglycemia.
Measurement of the serum free thyroxine (T) level is the
most appropriate next step in management for this patient
This patient should decrease his meal-time insulin glulisine who has clinical evidence of hypothyroidism (fatigue, con­
dose prior to exercise and continue his insulin glargine. Exer­ stipation, cold intolerance, dry skin, delayed reflexes, ane­
cise can increase glucose utilization by the muscles, which can mia, and mild hyponatrernia) but has had radiation to the
induce hypoglycemia in the setting of exogenous insulin. This base of the skull, including the pituitary gland. Although
patient consumes a meal and administers insulin glulisine, a measurement of thyroid-stimulating hormone (TSH) is the

112
Answers and Critiques

most accurate reflection of thyroid status in patients with an rarely associated with hypercalcemia and therefore does not
intact hypothalamic-pituitary-thyroid axis, it is not a reliable require therapy to lower serum calcium levels. Screening
measure of thyroid function in patients in whom there is other family members for the disorder is indicated.
loss of hypothalamic-pituitary function, such as seen in this Bone densitometry is not indicated in this age group in
patient. His low-normal TSH in the context of clinical hypo­ the absence of fragility fractures or other risk factors such as
thyroidism suggests possible central hypothyroidism, and long-term high-dose glucocorticoid use or primary hyper­
measurement of the circulating level of thyroid hormone, the parathyroidism.
free serum T 4, would therefore be a more accurate indication A parathyroid sestamibi scan is a very useful nuclear
of his thyroid status. imaging study for localization of adenomas in patients with
Repeating the TSH measurement would not be appro­ primary hyperparathyroidism or parathyroid cancer. How­
priate in this patient with signs and symptoms of hypothy­ ever, primary hyperparathyroidism has not been confirmed
roidism, as untreated hypothyroidism leads to increased in this patient with suspicion for FHH, making this study
cardiovascular morbidity and mortality. In addition, because premature.
of likely central hypothyroidism, the TSH level would remain The need for surgical treatment in this patient has also
an inaccurate indicator of thyroid function. not been established. Therefore, surgical referral for parathy­
Thyroid scintigraphy is unlikely to distinguish the roidectomy would not be appropriate.
source of the hypothyroidism, as patients with primary
KEY POINT
or secondary hypothyroidism have decreased radioactive
uptake. Thyroid scanning is most helpful in elucidating the • Measurement of 24-hour urine calcium and
cause of hyperthyroidism. creatinine levels will distinguish between primary
Ultrasound of the neck is normal in patients with cen­ hyperparathyroidism and familial hypocalciuric
tral hyperthyroidism and would be unlikely to provide any hypercalcemia.
additional information about this patient's thyroid status.
Bibliography
KEY POINT
Shinall MC Jr, Dahir KM, Broome JT. Differentiating familial hypocalciuric
• When central hypothyroidism is suspected, measure­ hypercalcemia from primary hyperparathyroidism. Endocr Pract. 2013
Jul-Aug;l9(4):697-702. [PMID: 234256441
ment of the serum free thyroxine (T 4) level is essential.

Bibliography
Item 38 Answer: D
Persani L. Clinical review: Central hypothyroidism: pathogenic, diagnostic,
and therapeutic challenges. J Clin Endocrinol Metab. 2012Sep;97(9):3068- Educational Objective: Evaluate timing of prandial
78. [PMID: 22851492]
insulin in a patient with diabetes mellitus.

The mismatch of timing of insulin administration to food


Item 37 Answer: B intake with meals, possibly related to the time demands of
her new job, is the most likely explanation for the erratic
Educational Objective: Diagnose familial hypocalciuric
glycemic fluctuations noted on this patient's blood glucose
hypercalcemia.
log. The adequacy of her nocturnal long-acting insulin is
This patient's urine calcium and creatinine levels should be reflected in her near-goal pre-breakfast blood glucose levels.
measured. Her laboratory values are consistent with hyper­ However, the major fluctuations occurring around mealtimes
calcemia, and her parathyroid hormone level is toward the are best explained by inconsistent use of her immediate-act­
upper end of the normal range. In her age group and with ing insulin relative to food intake. Meal coverage with insulin
a family member with a suspicious history, it is important should mimic the physiologic pattern seen with endogenous
to distinguish between primary hyperparathyroidism and insulin secreted from pancreatic beta cells. Administration of
familial hypocalciuric hypercalcemia (FHH). The distinc­ immediate-acting insulins should therefore ideally occur just
tion bet,veen primary hyperparathyroidism and FHH can prior to or at the time of the meal consumption. Because of
be made by a 24-hour urine collection for calcium and cre­ the rapid onset of action with these agents, shifting the tim­
atinine, which will establish the amount of kidney calcium ing of administration away from this physiologic pattern may
excretion and will allow evaluation of the calcium-creati­ result in the blood glucose fluctuations seen in this patient.
nine ratio. Total urine calcium of less than 200 mg/24 h (5 An important aspect of diabetes education is helping patients
mmol/24 h) and a calcium-creatinine ratio less than 0.01 are understand the actions of their prescribed insulin regimen
highly suggestive of familial hypocalciuric hypercalcemia. and the importance of timing issues when using them.
FHH results from a mutation in a specific calcium-sens­ Antibodies can develop in response to exposure to
ing receptor in the parathyroids and kidneys, and results exogenous insulin; however, these antibodies are rarely
in an upward shift in the range of calcium and PTH leading clinically significant and would not adequately explain the
to these clinical findings. Although a rare entity, making blood glucose pattern seen in this patient.
this diagnosis is crucial because it may prevent unnecessary Gastroparesis can cause erratic blood glucose values due
parathyroidectomy for the patient. The course of FHH is to either rapid transit or delayed emptying of food within the

113
Answers and Critiques

digestive system. However, the lack of gastrointestinal symp­ Postoperative norepinephrine is not inclicatecl. If the
toms, absence of clinical evidence of other diabetes-related patient were to experience hypotension or shock postop­
complications, and her hemoglobin A 1c history suggesting eratively. treatment with vasopressors in addition to gluco­
good diabetic control make gastroparesis a less likely cause corticoid replacement would be considered: however. this
of her erratic blood glucose readings. would not replace the administration of' glucocorticoids in
Inadequate insulin dosing can cause fluctuations in gly­ this population at risk for acute hypocortisolism.
cemic control. However, this patient reports little variability Preoperative phenoxybenzamine is indicated in the
in her daily diet. She also has several days with evidence of management of patients with pheochromocytoma. not CS.
adequate glycemic control on her current insulin regimen The purpose of preoperative ex-blockade is to provide blood
doses. Therefore, inadequate dosing is less likely to be the pressure control and decrease the risk of cardiovascular
cause of her glycemic variability. complications related to excessive catecholamine release
during intraoperative manipulation or the tumor.
KEY POINT
KEY POINT
• Because meal coverage with insulin should mimic the
physiologic pattern seen with endogenous insulin • Following adrenalectomy, patients with adrenal
secreted from the pancreatic beta cells as closely as Cushing syndrome should be treated with stress-dose
possible, insulin administration in patients with dia­ glucocorticoids during the postoperative period to
betes mellitus should ideally occur prior to or at the avoid the risk of acute hypocortisolism.
time of the meal consumption.
Bibliography
Bibliography Di Dalmazi G, Berr CM, Fassnacht M. el al. Adrenal function after adrenal­
ectomy for subclinical hypercortisolism and Cushing's syndrome: A
DeWitt DE, Hirsch 18. Outpatient insulin therapy in type 1 and type 2 diabe­ systematic review of the literature. JCEM. 2014;99:2637-2645. [PMID:
tes mellitus. JAMA. 2003 May 7:289(17):2254-64. [PMID: 12734137] 24878052]

CJ Item 39 Answer: A
Educational Objective: Manage a patient with
Item 40 Answer: A
Educational Objective: Manage Paget disease of bone.
Cushing syndrome following adrenalectomy.
Initiating antiresorptive therapy in this patient with symp­
The most appropriate management of this patient under­ tomatic Paget disease of bone is the most appropriate next step
going right adrenalectomy for Cushing syndrome (CS) in management. Paget disease of bone is characterized by focal
is postoperative hydrocortisone. The patient has adre­ areas of accelerated bone remodeling that ultimately causes
nocorticotropic hormone ( ACTH)-independent CS. and overgrowth of bone at one or more sites that may impair the
a contrnst-enhanced adrenal CT scan demonstrated a integrity of affected bone. Areas commonly affected include
right adrenal mass with imaging characteristics consis­ the skull, spine, pelvis, and long bones of the lower extrem­
tent with a benign adenoma. Adrenocortical adenomas ities, such as in this patient who has thickened cortical bone
typically have low attenuation on unenhanced CT scan and coarsening of the trabecular bone of the femur. The main
(density Jess than 10 Hounsfield units) and exhibit rapid indications for antiresorptive therapy in most patients include
washout of intravenous iodine contrast media (>50% pain caused by increased bone metabolic activity (as in this
at LO minutes). Following adrenalectomy. patients with patient) and hypercalcemia due to multiple affected sites. The
adrenal CS may develop acute adrenal railure because 01· most commonly used treatment agents are nitrogen-contain­
hypothalamic-pituitary-adrenal (I-IPA) axis suppression ing bisphosphonates (alendronate, pamidronate, risedronate,
and contralateral adrenal atrophy. All patients should be and zoledronic acid); these are the newer bisphosphonates
treated with stress-dose glucocorticoids and tapered to and have been the most extensively studied for treatment of
physiologic replacement until HPA axis recovery is con­ Paget disease of bone. Bisphosphonates stabilize bone turn­
l'irmed. Most patients have adrenal insufficiency lasting over by suppressing bone resorption and new bone forn1ation
as long as l2 months. with a resulting reduction in serum alkaline phosphatase
Postoperative administration or mitotane. an adreno­ levels. There is no evidence that antiresorptive therapy is ben­
lytic drug. is recommended as acljuvant therapy for patients eficial in asymptomatic patients.
with locally persistent or metastatic adrenocortical carci­ Bone biopsy is rarely needed to establish the diagnosis
noma (ACC). In these patients. mitotane is associated with of Paget disease of bone when there are characteristic radio­
objective remissions in approximately 25% of patients. ll1is graphic findings of bone turnover (concurrent osteolytic and
patient's CT scan findings are not consistent with ACC. Adre­ osteoblastic changes) and consistent laboratory studies (such
nal cancers are typically large (>4-6 cm) with irregular mar­ as elevated serum alkaline phosphatase levels). Bone biopsy
gins and areas of necrosis or calcification. Unenhanced CT may be useful in certain situations in which bone lesions are
will demonstrate high attenuation (density >LO Hounsfleld primarily osteoblastic (suggesting possible metastatic dis­
units). and washout of intravenous iodine contrast media is ease) or osteolytic (possibly indicating multiple myeloma),
less than 50% at 10 minutes. neither of which are present in this patient.

114
Answers and Critiques

The bone lesions of multiple myeloma are primarily proteins. The increased thyroid hormone requirements of the
osteolytic, in which case further evaluation for that diagnosis fetus also contribute to this change in serum T4 levels.
with a serum and urine protein electrophoresis would be In this patient, with a serum TSH value above 2.5 µU/mL
appropriate. However, this patient's radiographic findings (2.5 mU/L) during the first trimester, the dose of levothyrox­
are not consistent with a diagnosis of multiple myeloma, ine needs to be increased rather than decreased or discon­
and further evaluation for this disorder would therefore not tinued. The serum TSH level should be rechecked in 4 weeks
be indicated. to ensure that the dose adjustment continues to be adequate.
Most patients with Paget disease of bone are asymp­ Likewise, thyroid function tests should be repeated at least
tomatic and are identified only by elevated serum alkaline once during each trimester to ensure that additional adjust­
phosphatase levels detected on laboratory studies obtained ments in the levothyroxine dose are not needed.
for other reasons. In many patients with mild disease, clini­
KEY POINT
cal observation without initiation of therapy is appropriate.
However, in this patient with symptomatic disease in a • On confirmation of pregnancy in a patient taking lev­
critical weight-bearing skeletal area, clinical observation othyroxine, serum thyroid-stimulating hormone
without treatment would not be appropriate. should be checked to determine the need for levothy­
roxine dose adjustment.
KEY POINT

• Treatment of Paget disease of bone with antiresorptive Bibliography


agents is indicated in symptomatic patients, those Stagnaro-Green A. Abalovich \II. Alexander E. et al: American Thyroid
with elevated calcium levels, or patients with involve­ Association Taskforce on Thyroid Disease During Pregnancy and
Postpartum. Guidelines of the American Thyroid Association for the
ment of skeletal areas at high risk of complications, diagnosis and management of thyroid disease during pregnancy and
including fractures. postpartum. Thyroid. 2011 Oct:21(10):1081-125. [PMID: 21787128]

Bibliography Item 42 Answer: B


Ralston SH. Pager's disease ofthe bone. N Engl J Med. 2013 Feb 14:368(7) :644-
50. [PMID: 23406029] Educational Objective: Diagnose the cause of adrenal
failure.

Item 41 Answer: D ll1is patient most likely has primary adrenal failure due to
bilateral adrenal hemorrhage. I-le has acute onset nausea.
Educational Objective: Treat hypothyroidism in
lightheadedness. back and abdominal pain. and hypoten­
pregnancy.
sion. While nonspecific. these findings are consistent with
This pregnant patient's levothyroxine dose should be adrenal failure. Additionally. laboratory studies show hypo­
increased to lower the serum thyroid-stimulating hormone. natremia, hyperkalemia. and hypocortisolemia. which are
Maternal thyroid hormone production typically increases by also consistent with the diagnosis. A sudden drop in the
30% to 50% during pregnancy; therefore, in pregnant patients hemoglobin c1nci hematocrit. as seen in this patient, may be
requiring levothyroxine supplementation, the replacement present without evidence of' bleeding elsewhere. Risk fac­
dose usually needs to be increased to provide adequate thy­ tors for adrenal hemorrhage include anticoagulant therapy
roxine (T4) for the neurologic development of the fetus. The (and may occur with treat mcnt levels within the therapeutic
combination of this patient's elevated thyroid-stimulating range), the postoperative slate. abnormalities of hemostasis
hormone (TSH) level and low total T,1 level suggest that her (such as heparin induced thrombocytopenia or antiphos­
levothyroxine dose should be increased. During pregnancy, pholipid antibody syndrome). and sepsis. Because adrenal
the physiologic changes in thyroid hormone levels include a hemorrhage is uncommon and the associated f·indings may
reduction in the serum TSH level and an increase in the serum be relatively nonspecific. an increased level of' suspicion is
total T,1 level. This change in the serum TSH level is partly required for the diagnosis in at-risk patients: failure to iden­
due to the rise in the serum human chorionic gonadotropin tify acute adrenal failure in a timely manner may lead to car­
(HCG) level; both hormones share sequence homology in diovascular collapse. Adrenal hemorrhage can often be visu­
their a subunit. As the serum HCG level rises with progression alized on abdominal er imaging. Treatment of'acute adrenal
of pregnancy, the hormone can bind to the TSH receptors, failure is with stress-close glucocorticoicis (hydrocortisone.
resulting in a reduction in serum TSH levels. Consequently, 50-100 mg intravenously every 6-8 hours) and supportive
the normal reference range for serum TSH during pregnancy care with intravenous fluids and vasopressors as needed for
shifts to a lower value. from 0.5 to 5.0 µU/mL (0.5-5.0 mU/L} hcmoc\ynD 111 ic compromise.
pre-pregnancy to 0.03 to 2.5 µU/mL (0.03-2.S mU/L) during Although autoimmune adrenalitis may cause primary
the first trimester. Additionally, the serum total T4 level rises ,idrenal failure and is associated with the presence of' other
LS-fold above the normal nonpregnant reference range. Part autoi mmune diseases (such as hypothyroiclism in this
of this increase is due to the higher levels of estrogen associ­ patient). the onset of' sy mptoms related to hypocortisolism
ated with pregnancy, which cause an increase in serum total are usually more gradual. and skin hyperpigmentalion is
protein levels, including serum thyroid hormone-binding often seen on examination.

115
Answers and Critiques

Cl
CONT.
Chronic ad 111 in isl rat ion of long-acting opiate medica
tions is a known cause or hypogonadotropic hypogonad
Measuring 24-hour urine free catecholamine and meta­
nephrine levels would be used to screen for pheochromo­
ism and a potential etiology or secondary adrenal insuffi­ cytoma. If this patient had an adrenal adenoma, she would
ciency. ·111e aclrninistrat ion or relatively short-acting opiates certainly need to be screened for pheochromocytoma prior
(oxycoclone) on an as-neeclcd basis for postoperative pain to surgery, but she does not have a known adrenal lesion. Her
is unlikely to cause clinically significant hypothalamic­ symptoms and comorbidities are explained by her diagnosis
pituitary adrenal axis disturbance. Hypogonaclotropic of Cushing disease, and she has no specific signs or symp­
hypogonaclisrn would also not present with hyperkalemia toms that require an evaluation for pheochromocytoma.
because rnineralocorticoid secretion is preserved. A PET scan is not necessary or indicated in the evalu­
Pituitary apoplexy results from acute hemorrhage ation of Cushing syndrome. Cushing disease is caused by a
into the pituitary and may result in adrenal insufficiency. pituitary adenoma and is not malignant. Cushing syndrome
Although this patient might be at increased risk clue to due to ectopic ACTH production is often caused by a can­
anticoagulation. he does not have headache or visual distur­ cer. PET scan may be indicated to try to localize the source
bances, which arc common in pituitary apoplexy. Because of ectopic ACTH, but the source of the excessive ACTH is
pituitary apoplexy causes secondary adrenal insufficiency. it already known to be from a pituitary adenoma.
would not result in hyperkalemia.
KEY POINT
KEY POINT • Patients with hypercortisolism should have a screen­
• Patients with bilateral adrenal hemorrhage typically ing dual-energy x-ray absorptiometry scan because
present with clinical features of acute cortisol and they are at a high risk of osteoporosis and fracture.
aldosterone deficiency, including gastrointestinal dis­
turbance, lethargy, weakness, hypotension, shock, Bibliography
hypoglycemia, and electrolyte imbalances, such as Nieman LK. Biller BMK. Findling JW, et al. The diagnosis of Cushing's syn­
drome: an Endocrine Society clinical practice guideline. J Clin Endocrinol
hyponatremia and hyperkalemia. Metab. 2008 May;93(5):l526-40. [PMID: 18334580]

Bibliography

Cl
Rosenberger LH. Smith PW, Sawyer RG, et al. Bilateral adrenal hemorrhage:
the unrecognized cause ofhemodynamic collapse associated with hepa­
Item 44 Answer: D
rin-induced thrombocytopenia. Crit Care Med. 2011 Apr;39(4):833-8 Educational Objective: Manage primary adrenal failure.
[PMID: 21242799]
1 he most appropriate regimen for the long-term treatment of
this patient's primary adrenal failure would be preclnisone,
Item 43 Answer: C 5 mg once daily, and fluclrocortisone, 0.05 mg once daily. In
prima1y adrenal failure, there is a failure in the production
Educational Objective: Evaluate a patient with a new
or all the hormones or the adrenal cortex. Patients therefore
diagnosis of hypercortisolism for osteoporosis.
require both glucocorticoicl and mineralocorticoicl replace­
1l1is patient with newly diagnosed hypercortisolism is at ment. Because she is no longer ill, physiologic replacement
high risk for osteoporosis and fracture, and a screening dual­ doses are appropriate. Prcdnisone primarily has glucocor­
energy x-ray absorptiometry (DEXA) scan is indicated. 1l1e ticoid activity. with 5 mg being consiclerecl a physiologic
mechanism of osteoporosis in patients with Cushing syn­ replacement dose. It is also long-acting so may be adminis­
drome is related to decreased intestinal calcium absorp­ tered once daily in combination with fludrocortisone. which
tion, decreased bone formation, increased bone resorption, has almost pure mineralocorticoicl properties. Physiologic
and decreased renal calcium reabsorption. Patients with low replacement closes or fludrocortisone are 0.05 to 2 mg per clay.
bone density should be considered for bisphosphonate treat­ Dexamethasone is primarily a glucocorticoid and
ment to reduce the risk of fracture. could be used as the glucocorticoicl portion or combination
An 8-mg dexamethasone suppression test can be used replacement therapy with a mineralocorticoid such as flucl­
to help localize the source of adrenocorticotropic hormone rocortisone. However, dexamethasone alone would not be
(ACTH) in a patient with Cushing syndrome. Patients with appropriate therapy for primary adrenal failure due to its
pituitary Cushing disease respond to 8 mg of dexamethasone intrinsic lack ofmineralocorticoicl activity.
with a suppressed cortisol level. In comparison, patients Fluclrocortisone alone is also inappropriate because it
with ectopic ACTH production do not respond to 8 mg of would not provide glucocorticoid replacement.
dexamethasone, and their cortisol levels remain elevated. Hydrocortisone has both glucocorticoid and mineralo­
However, this test is associated with a number of false­ corticoicl properties. with primarily glucocorticoid activity
positive results. The 8-mg dexamethasone suppression test is at physiologic replacement doses of 12.5 to 25 mg in two to
not indicated for this patient because she has already had a three divided doses daily. At total daily doses above 50 mg.
more definitive test. Intrapetrosal sinus sampling has better hyclrocortisone has adequate 111 ineralocort icoid activity to
sensitivity and specificity when completed by a skilled inter­ allow for its use as monotherapy. However, treatment with
ventional radiologist. hydrocortisone, 10 mg three times daily. does not provide

116
Answers and Critiques

Cl
CONT
adequate mineralocorlicoid replacement. while il supplies
a supraphysiologic amount ofglucocorticoid lh,1l could lead
CT are typically a patchy infiltrate with minimal lymphade­
nopathy. This patient's image reveals marked diffuse enlarge­
to iatrogenic Cushing syndrome if administered on a long­ ment of the thyroid, and she did not have a history of prior
term basis. illness.
Patients with primary adrenal failure require adclilional
KEY POINT
glucocorticoicl at times or physiologic stress. Treatment for
minor stress (upper respiratory infection, lever. minor sur­ • Primary thyroid lymphoma most often occurs in
gery under local anesthesia) is typically two to three limes elderly women with underlying hypothyroidism; the
the basal close of hyclrocortisone (or equivalent). for moder­ typical presentation includes rapidly enlarging goiter,
ate stress (minor or moderate surgery with general anesthe­ weight loss, and night sweats, and imaging reveals a
sia) usually 45 to 75 mg/clay, and major stress (major smgery, diffusely enlarged thyroid.
trauma , critical illness. or childbirth) up to 150 lo 200 mg/
day with a gradual taper following resolution or the stress. Bibliography
Kim HC. Han MH. Kim KH, et al. Primary thyroid lymphoma: CT findings.
KEY POINT Eur J Radial. 2003 Jun;46(3):233-9. [PMID: 127581171
• Patients with primary adrenal failure require both
glucocorticoid and mineralocorticoid replacement in
Item 46 Answer: E
physiologic doses.
Educational Objective: Treat primary hyperparathy­
Bibliography roidism and concomitant vitamin D deficiency.
Neary N, Nieman L. Adrenal insufficiency: etiology, diagnosis and treatment. The most appropriate treatment of this patient is to replete
Curr Opin Endocrinol Diabetes Obes. 2010 Jun:17(3):217-23.(PMID:
20375886] his vitamin D deficiency with a supplement such as vitamin
D 3 (cholecalciferol). He has primary hyperparathyroidism as
shown by his elevated serum calcium and parathyroid hor­
Item 45 Answer: C mone levels. However, there is a high prevalence of concurrent
vitamin D deficiency and insufficiency in patients with pri­
Educational Objective: Diagnose primary thyroid
mary hyperparathyroidism, and low levels of 25-hydroxyvi­
lymphoma.
tamin D can stimulate parathyroid hormone secretion in
The patient has primary thyroid lymphoma, which most often non-adenomatous glands. Because of this, measurement of
occurs in elderly women with a long-standing history of vitamin D levels should be ordered as part of the evaluation of
Hashimoto thyroiditis. The clinical presentation is typically primary hyperparathyroidism, and repletion should be pro­
one of rapid onset (weeks) of an enlarging goiter with weight vided if identified. In these patients, it is in1portant to replace
loss and night sweats. The diagnosis is made by biopsy of the their vitamin D to a level of at least 30 ng/dL (75 nrnol/L).
thyroid with flow cytometry. Treatment typically involves After this level is reached, the patient should be placed on a
chemotherapy and/or radiation therapy. Thyroidectomy is vitamin D dosage to maintain that value. The choice to use
usually not needed. cholecalciferol versus ergocalciferol is often based on the level
CT scan of the neck, rather than ultrasound, was of vitamin D deficiency. Since ergocalciferol is more readily
ordered in this patient because of the compressive symp­ available in the 50,000 U form and has a shorter half-life, it
toms and positional breathing issues. CT scan allows visual­ is recommended when a patient's vitamin D level is less than
ization of the enlarged thyroid gland and assessment of the 10 ng/mL (25 nmol/L). Cholecalciferol is often used when
patency of the trachea. In this image, the "doughnut" sign the level is between 20 and 30 ng/mL (50-75 nmol/L) or for
can be seen, whereby the enlarged thyroid extends behind maintenance. Since this patient already has hypercalcemia
and completely encircles the trachea. and low-dose repletion is desired, the lower doses (400 U
New-onset Graves disease is unlikely to occur in a daily) of vitamin D3 (over-the-counter cholecalciferol) should
patient of this age, particularly with her long-standing his­ be used. This patient's serum calcium should be monitored at
tory of hypothyroidism. Furthermore, there is no bruit or least monthly.
other clinical sign of Graves disease, and the thyromegaly This patient does not meet the threshold for surgery. His
associated with Graves disease is not acute in onset. serum total calcium level is less than 1 standard deviation
This patient is unlikely to have papillary thyroid cancer, from upper limit of normal, his bone density score is not in
as these tumors typically grow very slowly, in contrast to the osteoporotic or treatment range, he is older than SO years
the acute onset of her findings. Additionally, the thy roid of age, and his glomerular filtration rate is preserved.
is not typically diffusely enlarged, as seen on this CT scan. Alendronate would be an excellent option for calcium
Rather, a distinct nodule and potentially concomitant cervi­ reduction and simultaneous treatment of osteoporosis if
cal lymphadenopathy would be expected. his T -scores were lower or Fracture Risk Assessment Tool
Subacute (de Quervain) thyroiditis is associated with (FRAX) scores were higher. The FRAX calculator defines the
acute onset of anterior neck pain. It is typically seen follow­ 10-year fracture risk for patients with T -scores in the - LO
ing a viral illness in the preceding months. The changes on to -2.S range. The FRAX calculator (www.shef.ac.uk/FRAX)

117
Answers and Critiques

incorporates multiple risk factors including sex, fracture organ function is stable and glucose levels have returned
history, femoral neck bone mineral density, steroid usage, to baseline values.
smoking, BM!, age, and alcohol intake to determine pro­ Sliding-scale insulin is nonphysiologic and can result
jected fracture risk. If the risk of major osteoporotic fracture in large fluctuations in blood glucose levels. Sliding-scale
is greater than or equal to 20% or the risk of hip fracture is insulin is not recommendecl as the sole insulin therapy in
greater than or equal to 3%, the patient's benefit from ther­ the hospital setting.
apy exceeds the risk and treatment should be offered.
KEY POINT
Calcitonin is an option for reducing his calcium levels,
but he is currently asymptomatic and does not warrant cal­ • For non-critically ill hospitalized patients with diabe­
cium lowering. tes mellitus and hyperglycemia, a weight-based treat­
Cinacalcet, a calcimimetic agent, is another option for ment plan that includes basal and prandial insulin is
lowering calcium for symptomatic patients with kidney recommended.
involvement. This patient has preserved kidney function
and no symptoms. Due to the cost and potential side effects Bibliography
of cinacalcet, it would not be indicated at this time. American Diabetes Association. (13) Diabetes care in the hospital, nursing
home, and skilled nursing facility. In: Standards of Medical Care in
KEY POINT Diabetes-2015. Diabetes Care. 2015 Jan:38 Suppl:S80-5. [PMID:
25537715]
• In patients with primary hyperparathyroidism and con­
comitant vitamin D deficiency, 25-hydroxyvitamin D
levels should be repleted to at least 30 ng/dL (75 nmol/L)
Item 48 Answer: B
to prevent further parathyroid hormone stimulation.
Educational Objective: Diagnose Cushing syndrome.

Bibliography The most appropriate next diagnostic test for this patient is
Holick MF, Binkley N. Bischoff-Ferrari HA, Gordon C, Hanley D. Heaney R. the 1-mg dexamethasone suppression test. She has the typical
Weaver C. Evaluation, treatment and prevention of vitamin D deficiency :
a n Endocrine Society clinical practice guideline. J Clin Endocrinol Metab. clinical features and findings of cortisol excess, or Cushing
2011 Jul;96(7):19ll-30. [PMlD: 21646368] syndrome. The most common cause of Cushing syndrome is
exogeneous glucocorticoid use; however, she has not received
glucocorticoids. To evaluate for Cushing syndrome, biochem­
Item 47 Answer: A ical evidence of hypercortisolism must be confirmed by use
of several screening tests. Three screening tests are used for
Educational Objective: Manage diabetes mellitus in a
Cushing syndrome: the 1-mg dexamethasone suppression
hospitalized patient.
test (given late at night with assessment of cortisol suppres­
The most appropriate treatment for this patient's diabetes sion the next morning), 24-hour urine free cortisol excretion
mellitus while hospitalized is a weight-based treatment (to quantify total daily cortisol secretion), and measurement
plan that includes basal and prandial insulin. Hypergly­ of evening salivary cortisol (which normally reaches a nadir
cemia in the hospital is associated with poor outcomes. at that time but remains elevated in patients with Cushing
According to the American Diabetes Association and syndrome). At least two abnormal first-line screening tests
American Association of Clinical Endocrinologists. glu­ are required for diagnosis. Only after establishing biochemical
cose goals in hospitalized patients in a non-ICU setting are hypercortisolism should the source of excess cortisol produc­
premeal values less than 140 mg/ell (7.8 mmol/l) and ran­ tion be sought.
dom values less than 180 mg/ell (10 mmol/l). The Amer­ Measurement of adrenocorticotropic hormone (ACTH)
ican College or Physicians recommends avoiding values is not a screening test for Cushing syndrome. After docu­
less than 140 mg/ell (7.8 mmol/L) to decrease the risk of mentation of excess cortisol production, ACTH levels may
hypoglycemic complications. The patient's plasma glucose be useful in determining if hypercortisolism is ACTH-de­
values exceed the recommenclecl guidelines and require pendent or -independent; however, it is not an appropriate
treatment. initial screening test.
Oral agents clo not have safety or efficacy data for use An 8-mg dexamethasone suppression test is helpful in
in the J1ospital. Glipizide is an insulin secretagogue that can differentiating between Cushing disease (pituitary tumor­
potentially induce hypoglycemia in the hospital setting. par­ secreting ACTH) and ectopic ACTH production. However,
ticularly with unpredictable changes in oral intake. it is not a screening test for Cushing syndrome and would
With rnetformin use, hospitalized patients can be appropriate only in specific situations after Cushing syn­
develop poor organ perfusion. which can increase the drome is diagnosed.
risk of lactic acidosis. Intravenous contrast dye can also A pituitary MRI should be ordered only after hypercor­
impair kidney function in the setting of metformin use in tisolism and Cushing syndrome are diagnosed and a pitu­
the hospital. Reinitiation of the patient's home regimen itary adenoma is suspected as a cause.
or metformin at or near the time of discharge is most Measurement of serum cortisol levels lacks sensitivity
appropriate arter all procedures have been completed and and specificity for diagnosing Cushing syndrome, primarily

118
Answers and Critiques

due to the pulsatile nature of cortisol secretion, and is not Item 50 Answer: D
used as a screening test. Educational Objective: Diagnose an incidentally
KEY POINT noted adrenal mass.

• To screen for Cushing syndrome, biochemical evi­ 1l1e most appropriate next step in management is to evalu­
dence of hypercortisolism must be confirmed by a ate for pheochromocytoma, preferably by measurement of
1-mg dexamethasone suppression test, 24-hour urine plasma free metanephrines in this patient. Approximately
free cortisol testing, and/or measurement of evening 10% to 15% of incidentally discovered adrenal masses are
salivary cortisol levels. functional, although most have no overt clinical manifesta­
tions. Therefore, all patients with an incidentally noted adre­
Bibliography nal mass should be evaluated for the autonomous secretion
Nieman LK, Biller BMK, Findling JW, el al. The diagnosis or· Cushing·s syn­ of cortisol and catecholamines, and those with hypertension
drome: An Endocrine Society clinical practice guideline. J Clin should also undergo testing for primary hyperaldosteronism.
Endocrinol Metab. 2008 May;93(5):l526-40. [PMID: 18334580]
The low-dose dexamethasone suppression test should be
performed to evaluate for subclinical Cushing syndrome
Item 49 Answer: A given its superior sensitivity compared with other screen­
Educational Objective: Treat an obese patient with ing tests (24-hour urine free cortisol and late-night salivary
type 2 diabetes mellitus with bariatric surgery.
co11isol); this test was negative in this patient. Measurement
of 24-hour urine metanephrines and catecholamines is the
Patients with a BM! between 35 and 40 with one or more usual first test in most asymptomatic patients to evaluate
complications associated with obesity should be con­ for catecholamine hypersecretion, although in those with
sidered for bariatric surgery, with the goal of significant imaging suggestive of pheochromocytoma, measurement of
weight loss and improvement in metabolic abnormali­ plasma free metanephrines is the preferred study because of
ties. This obese patient has type 2 diabetes mellitus with its very high sensitivity and high negative predictive value for
advanced microvascular disease along with other compli­ a normal study. Imaging in this patient reveals a well-circum­
cations associated with obesity including hypertension, scribed partially cystic lesion with high attenuation on non­
hyperlipidemia, obstructive sleep apnea, gastroesophageal contrast CT scan, which is in keeping with a pheochromocy­
acid reflux disease, and osteoarthritis. His attempts at life­ toma and is not typical of an adrenocortical adenoma, which
style management with diet and exercise did not result characteristically has low attenuation on CT scan (density <10
in weight loss that substantially improved his metabolic Hounsfield units) due to relatively high lipid content.
abnormalities or obesity. Management of an adrenal incidentaloma depends
Increasing the insulin doses could potentially exac­ on its size, imaging characteristics (phenotype), and hor­
erbate the weekly hypoglycemic events occurring in a monal functioning. Almost all adrenal tumors that are
patient with hypoglycemic unawareness. 1l1is could also overtly functional are larger than 6 cm in size or have unfa­
lead to more weight gain due to frequent treatments of vorable imaging characteristics should be considered for
hypoglycemia. surgical removal. However, biopsy or surgical resection of
Metformin is contraindicated in men with a serum any adrenal mass prior to ruling out a pheochromocytoma
creatinine level above 1.5 mg/dL (132.6 µmol/L) due to the is not recommended, as any manipulation of a catechol­
possibility of lactic acidosis and will not address the under­ amine-secreting tumor without appropriate preoperative
lying problem of obesity. management can precipitate a hypertensive crisis.
Pramlintide slows gastric emptying, which can decrease Measurement of the plasma aldosterone to plasma renin
appetite. The weight loss associated with the use of pramlin­ ratio is indicated as part of the evaluation of an incidentally
tide is modest, and it may not be sufficient to improve the discovered adrenal mass in a patient with hypertension, but
metabolic abnormalities and obesity-related complications not in this individual who has normal blood pressure.
in this patient. In addition, the patient's hypoglycemia may Not performing additional testing may miss a subclin­
be exacerbated by pramlintide. ical pheochromocytoma and would therefore not be an
KEY POINT
appropriate next step in management.

• Obese persons (BM! between 35 and 40) with type 2 KEY POINT
diabetes mellitus and associated complications should • A patient with an incidentally noted adrenal mass
be considered for bariatric surgery. should undergo biochemical testing for subclinical
Cushing syndrome and pheochromocytoma, and
Bibliography those with hypertension should also be evaluated for
Mechanick JI, Youdim A, Jones DB. el al. Clinical practice guidelines for the primary hyperaldosteronism.
perioperative nutritional, metabolic, and nonsurgical support of the
bariatric surgery patient-20l3 update: cosponsored by American
Association of Clinical Endocrinologisls. The Obesity Society. and Bibliography
American Society for Metabolic & Bariatric Surgery. Endocr Pract. 2013 Arnaldi G, Boscaro M. Adrenal incidentaloma. Best Pract Res Clin Endocrinol
Mar-Apr:19(2):337-72. [PMID: 23529351] Metab. 20l2: 26:405-419. [PMID: 22863384]

119
Answers and Critiques

Item 51 Answer: D androgen-producing adrenal tumor. Approximately 50% of


Educational Objective: Manage the limitations of
androgen-producing adrenal tumors are benign adenomas,
hemoglobin A1 c measurements in a patient with
while the other half are malignant. Symptoms are usually
diabetes mellitus and chronic kidney disease. minimal or absent in adult men. Women typically present
with rapidly progressive signs and symptoms of androgen
This patient should measure his postprandial glucose level. excess, including acne, hirsutism, and virilization (deepening
The hemoglobin A1e measurement is not always reliable in the of the voice, clitoromegaly, and male-pattern hair Joss), and
setting of chronic kidney disease; thus fingerstick blood glu­ may also have irregular menses. In patients with clinical evi­
cose measurements should be closely evaluated to help guide dence of hyperandrogenism, biochemical testing is performed
therapy. This patient's slightly elevated fasting and premeal prior to imaging and should include measurement of serum
blood glucose values may be indicative of postprandial hyper­ testosterone and dehydroepiandrosterone sulfate (DHEAS),
glycemia that could be detected with postprandial glucose an adrenal androgen. 1l1is patient's biochemical evaluation
measurements and used to guide therapy. For patients trying has revealed marked elevation of DHEAS and mild eleva­
to achieve hemoglobin A1e levels less than 7.0%, fasting and tion of testosterone, making an androgen-producing adrenal
premeal glucose targets usually are set at approximately 80 to tumor the most likely diagnosis. DHEAS levels above 8 µg/mL
130 mg/dL (4.4-7.2 mmol/L). This patient's slightly out of range (21.6 µmol/L) are diagnostic of an androgen-producing adre­
fasting and premeal glucose measurements seem at odds with nal tumor. 1l1e elevated serum testosterone level seen in this
the recent drop in his hemoglobin A1e level to 6.2%. Specific patient is likely a consequence of the peripheral metabolism
scenarios unique to end-stage kidney disease can affect the of adrenal androgens to testosterone. The serum testosterone
accuracy of the hemoglobin A1e measurement. Hemoglobin A1e level would be more than 150 to 200 ng/dL (5.2-6.9 nmol/L)
can be falsely elevated in the setting of chronic kidney disease in the setting of an androgen-producing ovarian tumor.
due to carbamylated hemoglobin secondary to uremia inter­ Performing a low-dose dexamethasone suppression test
fering with some of the assays. Hemoglobin A 1e can be falsely or pituitary MRI is not indicated because Cushing syndrome
decreased in the setting of a reduced erythrocyte lifespan, iron is unlikely with a normal 24-hour urine free cortisol level
deficiency, blood transfusions, and increased erythropoiesis and in the absence of specific features of hypercortisolism
,vith erythropoietin use. In this patient, the fingerstick blood (facial plethora, violaceous striae, and supraclavicular or
glucose measurements do not correlate with the most recent dorsocervical fat pads).
hemoglobin A1e after initiation of erythropoietin. The hemo­ Pelvic ultrasound is not an appropriate initial imag­
globin A1e value is falsely decreased after erythropoietin ther­ ing test because the marked elevation of DHEAS makes an
apy as a result of a change in the proportion of young and old androgen-producing adrenal tumor much more likely than
erythrocytes and a change in the rate of glycation. an ovarian neoplasm. Although the patient has a family
The patient's fingerstick blood glucose values are ele­ history of polycystic ovary syndrome (PCOS), the tempo and
vated, which further increases risk for microvascular and severity of her clinical presentation are not in keeping with
macrovascular damage. The current regimen should be this disorder. Although adrenal androgen excess occurs in
adjusted to decrease hyperglycemia. 30% to 40% of women with PCOS, only a mild elevation of
Decreasing the insulin detemir dose would increase DHEAS (3 µg/mL [8.1 µmol/L]) is expected.
hyperglycemia based on daily blood glucose data provided
KEY POINT
by the patient.
Discontinuation of the preprandial insulin glulisine based • In female patients, signs of androgen excess such as
on the falsely decreased hemoglobin A1e level would increase progressive hirsutism and virilization over a short
the hyperglycemia noted in the fingerstick blood glucose values. period of time suggest the diagnosis of an androgen­
KEY POINT
producing adrenal or ovarian tumor.

• End-stage kidney disease in patients with diabetes mel­ Bibliography


litus can affect the accuracy of the hemoglobin A1e meas­ Cavlan D, Bharwani N. Grossman A. Androgen- and estrogen-secreting
urement, which should not be used to guide therapy. adrenal cancers. Semin Oncol. 2010 Dec:37(6):638-48. [PMID: 2ll67382]

Bibliography
Ng JM. Cooke M. Bhandari S. e1 al. The effec1 of iron and erythropoietin Item 53 Answer: A
treatment on the Ale of pa1ients with diabetes and chronic kidney Educational Objective: Manage diabetes mellitus
disease. Diabetes Care. 2010 Nov:33(11):2310-3. [PMID: 207983371
with continuous glucose monitoring.

Item 52 Answer: A 1l1ere is a discrepancy between this patient's fingerstick blood


Educational Objective: Diagnose an androgen-pro­
glucose values and her hemoglobin A1e values that can be
quickly reconciled with a 72-hour continuous blood glucose
ducing adrenal tumor.
monitoring system. Continuous blood glucose monitoring
The most appropriate diagnostic test to perform next is systems use an electrochemical enzymatic sensor to mea­
an abdominal CT scan to confirm the diagnosis of an sure the glucose content of interstitial fluid via insertion of

120
Answers and Critiques

a subcutaneous needle. In some systems data recording can terminal hair growth will be slowed with combined oral con­
be made available in real time to the patient, whereas other traceptive use. Oral contraceptives that contain 30 to 35 µg
models store the data for later access and analysis. Since she of ethinyl estradiol appear to be more effective in managing
does not have kidney disease or anemia that could affect hirsutism than formulations containing less ethinyl estradiol.
the accuracy of hemoglobin A,c measurements, she likely has Six months of treatment is considered the minimal interval in
episodes of hyperglycemia not detected by her current moni­ which to determine the level of response. Adherence to an oral
toring efforts. Fingerstick blood glucose values only provide a contraceptive regimen will provide this patient with predict­
small snapshot of the glucose variability that occurs through­ able menses as well as contraceptive benefit. In addition, the
out the day. Undetected hyperglycemia or hypoglycemia can risk of endometrial hyperplasia is diminished.
lead to significant differences between the fingerstick blood Intermittent progesterone withdrawal, although effec­
glucose values and the expected hemoglobin A,c level. Her tive for decreasing the risk of endometrial hyperplasia,
wide range of fasting blood glucose values could be indicative would have no effect on this patient's concern regarding
ofw1detected overnight hypoglycemia. Intermittent continu­ hirsutism.
ous glucose monitoring is recommended when postprandial The levonorgestrel intrauterine system is a long-acting,
hyperglycemia, dawn phenomenon, or overnight hypoglyce­ reversible contraceptive device that diminishes long-term
mia is suspected. risk of endometrial hyperplasia in patients with PCOS; how­
Lifestyle modifications are recommended for glycemic ever, it provides no benefit for hirsutism and has no effect on
management; however, because this patient exercises in the androgen production.
evening, overnight hypoglycemia should be considered and Spironolactone is a potent antiandrogen and is very
evaluated with continuous glucose monitoring. Additional effective against male-pattern hirsutism in patients with
exercise may exacerbate the hypoglycemia. PCOS. However, it offers no benefit for control of the men­
This patient does not have evidence of postprandial strual cycle. When spironolactone is prescribed, patients
hyperglycemia on her fingerstick blood glucose measure­ should be counseled regarding the potential teratogenicity
ments, although it could be missed since she only measures in male fetuses, and a concurrent reliable contraceptive
after her meals periodically. Given the discrepancy in her method should be established to prevent fetal exposure.
blood glucose values and A,c level, hypoglycemia should be Pregnancy can still occur in patients with oligo-ovulatory
ruled out first before increasing her insulin doses that may PCOS, and reliance on menstrual irregularity is not a substi­
increase the risk of hypoglycemia. tute for a more proven contraceptive plan.
KEY POINT KEY POINT

• Continuous glucose monitoring may be useful in per­ • The combined oral contraceptive pill is the optimal
sons with postprandial hyperglycemia, dawn phe­ treatment to address both irregular menses and hir­
nomenon, or overnight hypoglycemia. sutism in patients with polycystic ovary syndrome.

Bibliography Bibliography
Klonof DC, Buckingham B, Christiansen JS. et al. Continuous glucose Amsterdam ESHRE/ASRM-Sponsored 3rd PCOS Consensus Workshop
f

monitoring: An Endocrine Society Clinical Practice Guideline. J Clin Group. Consensus on women·s health aspects of polycystic ovary syn­
Endocrinol Metab. 2011; Oct:96(10):2968-2979. [PMID: 21976745] drome (PCOS). Hum Reprod. 2012 Jan;27(l):14-24. [PMID: 221479201

Item 54 Answer: A Item 55 Answer: C


Educational Objective: Treat irregular menses Educational Objective: Manage a patient with
and hirsutism in a patient with polycystic ovary pheochromocytoma.
syndrome.
The most appropriate next step in management is to begin
The most appropriate treatment is combined oral contracep­ treatment with an a-adrenoceptor antagonist, such as phe­
tive pills. Hirsutism is present in approximately 70% of women noxybenzamine. The purpose of preoperative a-blockade is
with polycystic ovary syndrome (PCOS). The combined oral to provide blood pressure control and decrease the risk of car­
contraceptive pill is the optimal treatment to address both this diovascular complications related to excessive catecholamine
patient's concerns of menstrual irregularity and hirsutism. release during intraoperative manipulation of the tumor. Most
The estrogen component increases hepatic production of sex patients are treated for 1 to 2 weeks before surgery with phe­
hormone-binding globulin, decreasing the patient's circulat­ noxybenzamine with upward titration based on blood pres­
ing free testosterone level. For women in whom hirsutism is sure. The target blood pressure is below 130/80 mm Hg seated
a major concern, treatment is focused on reducing androgen and greater than 90 mm Hg (systolic) standing. Because of
production, decreasing the fraction of circulating free tes­ phenoxybenzarnine's side effects including orthostasis, nasal
tosterone, and limiting androgen bioactivity to hair follicles. stuffiness, fatigue, and retrograde ejaculation, some clinicians
Coarse, thick hairs that are already noted on examination use short-acting specific a-antagonists, such as prazosin, dox­
will need to be removed with a depilatory method; however, azosin, or terazosin. In patients with tachycardia, �-blockers

121
Answers and Critiques

can be added after a-blockade is achieved. Labetalol, a com­ hydrocortisone is adjusted based on symptoms, such as
bined a- and �-blocking agent, also can be used, especially orthostasis, weight loss, nausea, vomiting, and lighthead­
in patients with tachyarrhythmias. A heart rate of 60 to 70/ edness. He does not have these symptoms, so his cortisol
min seated and 70 to 80/min standing can be targeted in most deficiency is adequately treated. Increasing his hydrocor­
patients. Patients with pheochromocytoma who are normo­ tisone to higher than necessary doses increases the risk of
tensive also should be treated with a-blockers because they iatrogenic Cushing syndrome and glucocorticoid-induced
often become hypertensive during surgical resection. osteoporosis.
Increasing the dosage of lisinopril does not address the His testosterone value is normal, and he has normal
need for preoperative pharmacologic management of the morning erections. These are two signs that his hypogonad­
patient's pheochromocytoma with a-blockade. ism is adequately treated. Possibly, his erectile dysfunction
Although indicated for tumor localization following is a result of fatigue from hypothyroidism or is functional
the biochemical diagnosis of pheochromocytoma, a con­ instead of physiologic.
trast-enhanced adrenal CT scan should not be performed There is no reason to stop somatropin. He has no evi­
until after an a-adrenoceptor antagonist has been initiated. dence of residual tumor. Growth hormone (GH) replacement
Administering iodine contrast media to a patient who has can improve lean mass distribution and quality of life in a
not received a-blockade could incite a hypertensive crisis. patient with true GH deficiency, so it is reasonable to con­
Similarly, the �-adrenoceptor antagonist propranolol tinue. Discontinuing GH will likely worsen his fatigue.
should not be given prior to a-blockade because unopposed
KEY POINT
a-adrenoceptor stimulation could also precipitate a hyper­
tensive crisis. • Patients with secondary hypothyroidism from pitui­
tary dysfunction have low or low-normal thyroid­
KEY POINT
stimulating hormone values, so levothyroxine dose
• In patients with a confirmed diagnosis of pheochro­ should be adjusted based on free thyroxine (T4) level.
mocytoma, a-blockade should be instituted before
surgery to reduce the risk of cardiovascular complica­ Bibliography
tions and to control blood pressure. Schneider HJ, Aimaretti G, Kreitschmann-Andermahr, et al. Hypopituitarism.
Lancet. 2007 Apr 28;369(957l):1461-70. [PMID: l74675!7]

Bibliography
Lenders JW, Duh QY, Eisenhofer G, et al. Pheochromocytoma and paragan­
glioma: an endocrine society clinical practice guideline. J Clin Endocrinol Item 57 Answer: A
Metab. 2014 Jun;99(6):l9l5-42. [PMID: 24893135]
Educational Objective: Diagnose type 2 diabetes
mellitus.

Item 56 Answer: C A fasting plasma glucose measurement is the most appro­


Educational Objective: Manage hormone replace­
priate diagnostic test for this patient. Diabetes mellitus
ment therapy in a patient with panhypopituitarism.
can be diagnosed with an abnormal result of one screen­
ing test performed on two separate occasions. Although
The patient's hypothyroidism is inadequately treated, causing the hemoglobin A 1 c is normal in this patient, the fasting
symptoms of fatigue and weight gain and a low free thy­ plasma glucose is abnormally elevated within the diag­
roxine (T) level. Therefore, his levothyroxine dose should nostic range for diabetes mellitus. When discrepant results
be increased. Patients with secondary hypothyroidism from occur among different screening tests for diabetes, the
pituitary dysfunction have low or low-normal thyroid-stimu­ American Diabetes Association recommends repeating the
lating hormone (TSH) values which cannot be used to assess abnormal screening test. If the repeat fasting plasma glu­
the adequacy of thyroid hormone replacement. Because of cose measurement is abnormal, the diagnosis of diabetes
this, the levothyroxine dose is adjusted based on free T4 levels is confirmed. Screening for type 2 diabetes should begin in
instead ofTSH values. His free T4 is low, suggesting inadequate all asymptomatic patients at age 45 years. In adult patients
treatment as the likely cause of his symptoms. with a BM! greater than or equal to 25, screening should
The patient's desmopressin dosing is adequate to treat occur at any age if one or more additional risk factors for
his diabetes insipidus. His symptoms are well controlled diabetes is present.
without evidence of excessive urination, and his serum Use of the hemoglobin A 1c as an initial screening test in
sodium level is normal. In addition, increasing the dose this patient is appropriate as there is no evidence for anemia
would risk potentially causing water retention and hypo­ or kidney or liver disease that could decrease the reliability
natremia. of the test. The value was normal and does not warrant a
He is on a physiologic dose of hydrocortisone. Hydro­ repeat measurement as the next diagnostic test to perform
cortisone dose is not adjusted based on laboratory test results in this scenario.
because his endogenous adrenocorticotropic hormone A 2-hour 75-g oral glucose tolerance test can be used
(ACTH) and cortisol levels will remain low on adequate as a screening tool for diagnosing diabetes. Since this test
therapy and are therefore not used to alter therapy. Instead, was not initially used for screening in this patient, it is most

122
Answers and Critiques

appropriate to repeat the abnormal screening test (fasting nadism with normal prolactin levels. Therefore, measurement
plasma glucose) that was already used for comparison. of serum prolactin is not the most useful test for this patient.
A random blood glucose measurement would be useful
KEY POINT
in this patient if he presented with classic hyperglycemic
symptoms in the setting of a blood glucose level of 200 mg/ • Klinefelter syndrome is a common cause of hypergo­
dL (11.1 mmol/L) or above, as that would be diagnostic of nadotropic hypogonadism and azoospermia.
diabetes. This patient is not symptomatic.
Bibliography
KEY POINT
Krausz C, Chianese C. Genetic testing and counselling for male infertility.
• When discrepant results occur among different Curr Opin Endocrinol Diabetes Obes. 2014 Jun;21{3):244-50. [PMID:
24739313]
screening tests for diabetes mellitus, the American
Diabetes Association recommends repeating the
abnormal screening test. Item 59 Answer: D
Educational Objective: Interpret thyroid function test
Bibliography results in an elderly patient.
American Diabetes Association. (2) Classification and diagnosis of diabetes.
In: Standards of Medical Care in Diabetes-2015. Diabetes Care. 2015;38 Clinical follow-up with repeat measurement of thyroid-stim­
Suppl l:SS-16. [PMID: 25537714]
ulating hormone (TSH) and free thyroxine (T 4) is the most
appropriate management for this patient. In persons over
80 years of age, the serum TSH level may be mildly elevated
Item 58 Answer: A above the typical reference range for younger adults. The
Educational Objective: Diagnose Klinefelter upper limit of the normal range in this elderly population
syndrome. without thyroid dysfunction may be as high as 8.0 µU/mL
(8.0 mUIL). Since this patient's TSH level is 6.4 µU/mL (6.4
The most appropriate diagnostic test to perform next is a
mU/L), his clinical symptoms are nonspecific, and his physi­
karyotype. This patient has evidence of hypergonadotropic
cal examination is normal, additional evaluation or treatment
hypogonadism based on elevated gonadotropin levels and
is not indicated at this time. However, his clinical symptoms
low testosterone level. Klinefelter syndrome is a common
should be monitored, and his serum TSH and free T 4 levels
cause of hypergonadotropic hypogonadism and azoosper­
should be measured repeatedly several times over a period of
mia, resulting in infertility. A 47,XXY karyotype is diagnostic
months to ensure that the TSH value is not part of a trend to
of Klinefelter syndrome. Mosaic variants of this condition
the development of overt thyroid dysfunction.
exist but typically present with oligoasthenospermia, testic­
Because this patient's serum free T 4 level is normal and
ular failure, or hypogonadism. Concomitant symptoms often
a diagnosis of hypothyroidism has not been established, he
include sexual dysfunction and generalized fatigue. Tall stat­
does not require levothyroxine therapy.
ure is a common finding. Patients with Klinefelter syndrome
Measurement of the serum total triiodothyronine (T)
may fail to achieve puberty or may present after sexual
level is not typically helpful in diagnosing hypothyroidism
maturation with azoospermia. Fertility may be achieved
because total T3 levels may remain within the normal range
from ejaculated sperm, if present, or extracted testicular
well into the evolution of hypothyroidism. However, total T 3
sperm; however, advanced reproductive techniques such as
levels are useful in evaluating patients with possible hyperthy­
in vitro fertilization and intracytoplasmic sperm injection
roidism because the value may be elevated out of proportion to
are necessary to achieve pregnancy. Some couples may opt to
the T 4 level, and failure to recognize an elevated T 3 value may
include genetic testing by preimplantation genetic diagnosis
underestimate the degree of hyperthyroidism present.
and embryo biopsy to avoid transmission of the disorder to
Total T4 measures both the bound and unbound thyroid
subsequent generations. Typically, gonadotropin levels are
hormone fractions, whereas the free T4 reflects the unbound
high in patients with Klinefelter syndrome, representing
portion of hormone, and may more accurately reflect avail­
testicular hypofunction. After plans for conception are com­
able hormone levels in patients who may have an abnormal­
pleted, supplementation with exogenous androgens may be
ity in protein metabolism (such as liver or kidney disease).
considered to prevent osteoporosis. Conception with donor
However, measurement of total T 4 in addition to free T 4 would
sperm is an alternative fertility treatment option.
not provide additional diagnostic information in this patient.
MRI of the pituitary would be needed only to rule out
a pituitary mass in the setting of hypogonadotropic hypogo­ KEY POINT
nadism. Because this patient's gonadotropin levels are high, • In patients over 80 years of age, the serum thyroid­
a pituitary mass is unlikely. stimulating hormone level may be mildly elevated
Scrotal ultrasound would identify small testicles in a above the typical reference range.
patient with suspected Klinefelter syndrome, but it would not
identify the cause of this patient's elevated gonadotropin levels. Bibliography
Serum prolactin level would likely be normal, as Tabatabaie V, Surks Ml. The aging thyroid. Curr Opin Endocrinol Diabetes
Klinefelter syndrome is characterized by primary hypogo- Obes. 2013 Oct;20(5):455-9. [PMID: 23974775]

123
Answers and Critiques

Item 60 Answer: B this patient's tumor is invading the left cavernous sinus and
Educational Objective: Evaluate an incidentally
compressing the optic chiasm, complete resection will likely
noted adrenal mass.
not be possible; however, surgery can effectively debulk
the tumor and preserve vision in addition to significantly
The most appropriate diagnostic test to perform next is the decreasing growth hormone (CH) secretion as measured by
low-dose dexamethasone suppression test to screen for the insulin-like growth factor 1 (ICF-1) levels. In patients in whom
autonomous secretion of cortisol. The increasing use of imag­ complete resection is not possible, such as this patient, addi­
ing studies for various medical indications has revealed oth­ tional therapy may be required such as stereotactic radiation
erwise unrecognized adrenal masses in less than 1% of the therapy or medical therapy to inhibit CH secretion or block its
population younger than 30 years and up to 7% of those older effect on the tissues. However, surgical resection remains an
than 70 years. Ten to 15% of adrenal incidentalomas are func­ essential first step in the treatment of acromegaly.
tional, although most have no overt clinical manifestations. A small number of CH-secreting pituitary adenomas
Therefore, testing is usually necessary to identify functional co-secrete prolactin. Although dopamine agonist therapy
tumors secreting catecholamines, cortisol, or aldosterone. Of with agents such as bromocriptine would treat the associ­
the functional adenomas, most secrete excessive amounts of ated prolactin elevation, it is minimally effective in acromeg­
cortisol. In subclinical Cushing syndrome (CS), classic signs aly and would not adequately treat CH secretion or address
or symptoms of cortisol excess are not observed; however, the mass effect of a CH-secreting adenoma.
complications of long-standing hypercortisolism may result. A CH receptor blocker, pegvisomant, is available. Peg­
The patient has two disorders that can be seen in association visomant works in the peripheral tissues as an antagonist to
with subclinical CS: type 2 diabetes mellitus and osteoporosis. CH but does not decrease its production by the tumor. This
Obesity and hypertension are also common. The low-dose patient needs intervention to treat mass effect at this time
overnight dexamethasone suppression test is recommended because the tumor is damaging the optic chiasm and the
as the initial screening test for this condition due to its high patient's vision, and this treatment would not be expected
sensitivity. Screening for pheochromocytoma, such as by to decrease the tumor size.
measuring 24-hour urine fractionated metanephrines and Somatostatin analogues, such as octreotide and lan­
catecholamines, is also indicated in all patients with an inci­ reotide, inhibit CH secretion and are helpful in treating
dentally noted adrenal mass. some patients with acromegaly. They are used primarily in
Adrenal vein sampling (AVS) is not needed. AVS is most patients with unresectable tumors without significant mass
often performed to evaluate for a bilateral versus unilateral effect or those with a contraindication to surgery. They may
adrenal cause of primary hyperaldosteronism. also be used in patients with continued CH secretion fol­
Measurement of plasma renin activity and aldosterone lowing incomplete transsphenoidal resection. However, they
concentration is not indicated in patients without would not be an appropriate treatment in this patient with a
hypertension. large, invasive, vision-threatening pituitary tumor.
No further testing is also inappropriate. Although the Radiation therapy may be added to surgical or medical
imaging characteristics of the mass are in keeping with a therapy to help increase the chance for remission or cure.
benign adrenal adenoma, further diagnostic evaluation is Radiation to the pituitary carries a high risk of causing
needed. This includes testing for autonomous hormonal pituitary insufficiency and damage to surrounding tissues
secretion and subsequent radiographic surveillance (first at (particularly the optic nerves); therefore, it is not usually an
3-6 months and then annually for 1-2 years). initial treatment for acromegaly in most patients. In those in
KEY POINT
whom it is used, stereotactic surgery (gamma knife) is the
preferred approach to minimize potential complications.
• Ten to 15% of adrenal incidentalomas are functional; bio­
chemical testing is needed to identify functional tumors KEY POINT
secreting catecholamines, cortisol, or aldosterone. • The primary therapy for acromegaly is transsphenoi­
dal surgery to remove the causative growth hormone­
Bibliography secreting pituitary adenoma.
Zeiger MA. Thompson GB. Duh QY, et al. The American Association of
Clinical Endocrinologists and American Association of Endocrine
Surgeons medical guidelines for the management ofadrenal incidentalo­ Bibliography
mas. Endocr Pract. 2009 Jul-Aug;lS Suppl 1:1-20. [PMID: 19632967} Melmed, S. Acromegaly pathogenesis and treatment. J Clin Invest. 2009
Nov;ll9:3189-202. [PMID: 19884662]

Item 61 Answer: E
Educational Objective: Treat acromegaly with Item 62 Answer: C
transsphenoidal pituitary surgery. Educational Objective: Treat high-risk thyroid cancer
postoperatively.
Transsphenoidal resection of the pituitary adenoma is the
initial treatment of choice in patients with acromegaly. It is Radioactive iodine (RAI) therapy is the most appropriate post­
also the only treatment that is potentially curative. Because operative treatment in this patient who is at high risk of

124
Answers and Critiques

cancer recurrence based on his age, the size of the primary insulin is the recommended treatment. Unlike autoimmune
tumor, the presence of vascular invasion and extrathyroidal type 1 diabetes, chronic pancreatitis also destroys the pancre­
extension, and the number of involved lymph nodes. He atic alpha cells causing a glucagon deficiency that increases
therefore may benefit from adjuvant RAJ therapy, which may the risk of spontaneous hypoglycemia. Glucagon acts on the
decrease the likelihood of recurrent disease in patients with liver to increase glucose production through glycogenolysis
nodal metastases. This is given in conjunction with levothy­ and gluconeogenesis. The recovery from hypoglycemia is also
roxine suppression therapy, which is indicated in all patients impaired with alpha cell destruction. Early recognition of
who have had a total thyroidectomy. Because of his high-risk hypoglycemic symptoms and strategic hypoglycemic treat­
disease, it would be appropriate to lower the thyroid-stimu­ ment plans should be emphasized with patients with pancre­
lating hormone level to less than 0.1 µU/mL (0.1 mU/L) in the oprivic diabetes.
absence of contraindications such as pre-existing cardiovas­ Exenatide, a glucagon-like protein-1 (GLP-1) mimetic,
cular disease or age greater than 65 years. suppresses glucagon and promotes insulin secretion. The
Traditional chemotherapeutic agents, such as doxoru­ pancreatic beta cell and alpha cell destruction associated
bicin, are generally ineffective in the management of differ­ with chronic pancreatitis precludes this treatment option.
entiated thyroid cancer and would not be indicated for this Postmarketing reports of pancreatitis are also cause for con­
patient. In patients with anaplastic carcinoma of the thyroid, cern for the use of this class of medication in patients with a
however, some studies have demonstrated a possible benefit history of pancreatitis.
with concomitant use of paclitaxel-based chemotherapy and The sulfonylurea glipizide increases insulin secretion.
external-beam radiotherapy. This patient has classic pap­ The effect would likely be minimal to nonexistent in this
illary thyroid cancer histology and would not benefit from patient with hyperglycemia resulting from substantial beta
such treatment. cell destruction from chronic pancreatitis.
External-beam radiotherapy is rarely used in patients Metformin decreases hepatic glucose output by inhibit­
with differentiated thyroid cancer. An exception would be ing gluconeogenesis and increases insulin-mediated glucose
the management of patients with inoperable disease that utilization in peripheral tissues. Metformin is a first-line
threatens to cause local extension into vital structures in the agent for initial treatment of type 2 diabetes; however, this
neck such as the trachea, esophagus, or major blood vessels. patient has an insulin deficiency from pancreatic beta cell
Because of the extent of disease found at surgery and destruction and should be treated as a patient with type 1
this patient's high risk of recurrence, providing no addi­ diabetes.
tional therapy would not be an appropriate next step in
KEY POINT
management.
• Hyperglycemia caused by chronic pancreatitis is an
KEY POINT acquired form of type 1 diabetes mellitus and should
• A patient who has undergone total thyroidectomy be treated with insulin.
for thyroid cancer and is at high risk for disease
recurrence should receive adjuvant radioactive Bibliography
iodine therapy. Mergener K, Baillie J. Chronic pancreatitis. Lancet. 1997 Nov 8;350(9088):
1379-85. [PMID: 9365465]

Bibliography
Jonklaas J, Sarlis NJ, Litofsky D, et al. Outcomes of patients with differenti­ A
ated thyroid carcinoma following initial therapy. Thyroid. 2006 Dec;l6(12):
Item 64 Answer:
1229-42. [PMID: 17199433] Educational Objective: Treat infertility related to
polycystic ovary syndrome.

Item 63 Answer: C The most appropriate treatment is a selective estrogen recep­


tor modulator (SERM) such as clomiphene citrate. SERMs are
Educational Objective: Manage acquired type 1
the established first-line treatment for ovulation induction
diabetes mellitus.
in anovulatory patients with infertility from polycystic ovary
This patient has an acquired form of type 1 diabetes melli­ syndrome (PCOS). Typically, therapy is started after menses
tus caused by chronic pancreatitis (pancreoprivic diabetes), and is given orally for S days. Common side effects include
which necessitates the use of insulin for treatment of the vasomotor symptoms and mood changes. Escalating doses
hyperglycemia. Chronic pancreatitis results in permanent of clomiphene are typically prescribed if a patient does not
destruction of the pancreas and may impair both the endo­ ovulate on lower doses. More recently, evidence suggests the
crine and exocrine functions of the pancreas. The pancre­ effectiveness and possible superiority of aromatase inhibitor
atic exocrine abnormalities arise from loss of the pancreatic therapy (such as with letrozole) in women with PCOS for
enzymes required for digestion and absorption of food. The ovulation induction. However, this therapy is not currently
pancreatic endocrine abnormalities can present in a similar FDA approved for this indication.
manner as type 1 diabetes with hyperglycemia from insulin A small subset of patients with PCOS may require in
deficiency secondary to destruction of beta cells. Therefore vitro fertilization, but this therapy is typically explored only

125
Answers and Critiques

after several failed cycles of ovulation induction with clomi­ KEY POINT
phene citrate.
• Microprolactinomas in asymptomatic patients do not
In patients with clomiphene resistance, gonadotropin
require treatment; however, surveillance is recom­
therapy would be an appropriate next step; however, caution
mended.
is warranted because higher-order multiple gestation may
result.
A 2012 Cochrane review of the effect of insulin-sensi­ Bibliography
Melmed S, Casanueva FF, Hoffman AR, et al. Diagnosis and treatment of
tizing drugs in women with infertility and PCOS included hyperprolactinemia: an Endocrine Society clinical practice guideline. J
44 trials, the majority of which involved metformin. Rates Clin Endocrinol Metab. 2011 Feb:96(2):273-88. [PMID: 21296991]
of pregnancy were improved with metformin compared
with placebo and metformin plus clomiphene compared
with clomiphene alone, but metformin did not change rates Item 66 Answer: B
of live births compared with placebo or with clomiphene Educational Objective: Manage diabetic neuropathy.
compared to clomiphene alone.
Duloxetine is a reasonable initial option for this patient's pain­
KEY POINT ful peripheral neuropathy. The typical presentation for distal
• Selective estrogen receptor modulators such as clomi­ symmetric polyneuropathy is a bilateral "stocking-glove" dis­
phene citrate are the established first-line treatment tribution. Damage to the small nerve fibers can result in pain,
for ovulation induction in anovulator y patients with numbness, burning, and tingling. It can also impair light touch
infertility from polycystic ovary syndrome. and temperature sensation. Damage to the large nerve fibers
leads to abnormal vibration sensation and proprioception.
Diminished or loss of ankle reflexes is commonly seen early
Bibliography
Legro RS, Brzyski RG, Diamond MP, et al: NlCHD Reproductive Medicine
with diabetic polyneuropathy. Motor weakness can occur as
Network. Letrozole versus clomiphene for infertility in the polycystic the polyneuropathy progresses. Several classes of drugs are
ovary syndrome. N Engl J Med. 2014 Jul 10;371(2):119-29. Erratum in: frequently used for symptomatic pain relief, including the tri­
N Engl J Med. 2014 Oct 9;317(15):1465. [PMID: 25006718]
cyclic antidepressants (amitriptyline), other classes of antide­
pressants (duloxetine, venlafaxine), anticonvulsants (pregaba­
lin, gabapentin, valproate), and capsaicin cream. There are few
Item 65 Answer: C head-to-head comparison trials for these classes of drugs for
Educational Objective: Treat a microprolactinoma in distal symmetric polyneuropathy, thus selection must take into
a postmenopausal woman. consideration the potential risks and benefits associated with
each drug for an individual patient. Duloxetine has fewer risks
No therapy is necessary at this time, and the patient should
than amit1iptyline for this patient given his cardiac history.
be retested in 12 months. The patient has a microprolac­
Tricyclic antidepressants, such as amitriptyline, should
tinoma, but she is postmenopausal. Luteinizing hormone
be used cautiously in patients with known cardiac dis­
and follicle-stimulating hormone levels are normally high
ease due to an association between this class of drugs and
in postmenopausal women because of ovarian failure; how­
arrhythmias, heart block, and sudden death. The patient's
ever, her levels are lower than expected, likely because the
history of cardiac disease and a first-degree atrioventricular
elevated prolactin is providing negative feedback. This causes
block may increase his risk of side effects from amitriptyline.
hypogonadism but is not clinically relevant because she is
A nerve conduction study is not routinely required for
already hypogonadal from normal menopause. She has mini­
diagnosis or management in patients with diabetes with a
mal symptoms from menopause and is tolerating it well.
typical presentation of symmetric distal polyneuropathy.
The prolactinoma was found incidentally. On MRI, it
Atypical clinical features should prompt additional work-up,
has no concerning features, and her other pituitary hormone
including electrophysiologic testing.
levels are normal. Although no treatment is necessary for
Vitamin B 12 deficiency has been associated with long-term
this asymptomatic patient, it is advisable to retest in 6 to
use of metformin and can present with peripheral neuropathy.
12 months to make sure that the tumor does not grow.
It is also commonly seen in the setting of megaloblastic anemia.
Dopamine agonists, such as cabergoline, are used to
treat symptomatic prolactinomas, but it is not necessary in It is unlikely that vitamin B 12 deficiency is the cause of this
patient's peripheral neuropathy as he has a classic presentation
this asymptomatic patient.
for symmetric distal polyneuropathy, discontinued metformin
Radiosurgery is not necessary. It is an option to treat
2 years ago, and has a normal complete blood count.
pituitary tumors that are not amenable to standard surgery
or cannot be fully resected, but it is not indicated for this KEY POINT
asymptomatic patient. • Treatment options for diabetic polyneuropathy include
Transsphenoidal resection of the pituitary tumor is the tricyclic antidepressants, other classes of antidepres­
overly invasive and unnecessary because she is doing well.
sants (duloxetine, venlafaxine), anticonvulsants (pregab­
In addition, first-line therapy for symptomatic prolactino­
alin, gabapentin, valproate), and capsaicin cream.
mas are dopamine agonists, not surgery.

126
Answers and Critiques

Bibliography Item 68 Answer: D


American Diabetes Association. (9) Microvascular complications and foot
care. In: Standards of Medical Care in Diabeles-2015. Diabetes Care. 2015 Educational Objective: Diagnose Asherman
Jan:38 Suppl l:S58-66. [PMID: 25537710] syndrome with transvaginal ultrasound.

The most appropriate diagnostic test to perform next is a trans­


Item 67 Answer: D vaginal ultrasound. This patient has likely developed Asher­
man syndrome (AS). Because the hormonal evaluation in this
Educational Objective: Evaluate hypoglycemia in a
patient supports an intact hypothalamic-pituitary-ovarian axis,
patient without diabetes mellitus.
a structural abnonnality such as AS should be suspected. AS
TI1is patient should undergo a 72-hour last \Nilh hypogly­ is an uncommon complication of dilatation and curettage,
cemic testing at the time of symptomatic hypoglycemia. intrauterine device placement, or surgical procedures such as
He had symptomatic hypoglycemia with prolonged lasting hysteroscopic myomectomy; it is caused by lack of basal endo­
(>5 hours) overnight that resolved with glucose adminis­ metrium proliferation and formation of adhesions (synechiae).
tration. Since he is not currently hypoglycemic or symp­ Diagnosis should be considered in any woman with amenor­
tomatic, fasting is necessary in an attempt to recreate the rhea and previous exposure to uterine instrumentation. The
metabolic scenario that induced the hypoglycemia to al!ov,· typical presentation is with secondary amenorrhea, at times
definitive diagnosis. Glucose and hypoglycemic stud­ associated with cyclic pelvic pain, created by distention of
ies. including measurement of insulin, (-peptide. pro the uterine cavity where pockets of functional endometrium
insulin, and �-hydroxybutyrate levels. should be obtained persist but eillux of menstrual flow is blocked or slowed by
at the beginning of the fast and then repeated every 6 adhesion formation. Although some patients with AS may be
hours until the blood glucose level Jails below 60 mg dL completely amenorrheic, others may demonstrate hypomen­
(3.3 mmol/L). al which time they should be repeated every orrhea and report scant menses compared with the volume of
l ro 2 hours until (1) symptomatic hypoglycemia ('.,..JS mg their menstrual flow before the procedure. AS most commonly
dL l2.5 mmol/L]) occurs. (2) asymptomatic hypoglycemia occurs in an inflammatory setting such as endometritis or sep­
('.,55 mg1dL [3.0 mmol/L]) occurs with previously docu­ tic abortion. AS may also occur as a result of an overly aggres­
mented Whipple triad. or (3) the 72-hour fost concludes. sive curettage. In a patient with AS, transvaginal ultrasound will
Whipple triad consists of three components: neuroglycope­ show a thin endometrial stripe and may reveal small pockets of
nia, concurrent hypoglycemia. and resolution of symptoms fluid where menstrual flow has been trapped by neighboring
with correction or hypoglycemia. adhesions. A functional uterine examination, such as hystero­
Hypoglycemic studies are not adequately informa­ salpingogram or saline sonohysterogram, confinns the diag­
tive when the patient is not experiencing symptom­ nosis. Treatment consists of hysteroscopic resection of lesions.
atic hypoglycemia; therefore, testing now would not be Given this patient's normal gonadotropin levels, a pitu­
appropriate. itary cause for her secondary amenorrhea is unlikely; there­
Mixed-meal testing involves ingestion or a meal fore, imaging of the pituitary is not warranted at this time.
containing a mixture of protein. fat, and carbohydrates Premature ovarian insufficiency is not the most likely
intended to raise the plasma blood glucose to assess the diagnosis given this patient's normal estradiol and gonad­
metabolic response to increased glucose. ·1 his test is appro­ otropin levels; therefore, a peripheral karyotype would be
priate when symplomalic hypoglycemia occurs within .S expected to be normal and should not be performed.
hours after meal consumption (postprandial hypoglyce­ Results of a progestin withdrawal test are used to delin­
mia). However. this patient's symptomatic hypoglycemia eate between an estrogen-deficient state (no bleeding) and an
occurred 7 hours af'ter his last meal: therefore, ev::ilu,1- estrogen-sufficient state (withdrawal bleeding). If the patient is
tion for postprandial hypoglycemi::i with 8 mixed-meal test producing estrogen, she will have withdrawal bleeding within
would not be appropriate. 1 week of completing a course of progesterone. If no with­
Oral glucose tolerance Lesli ng was previously used drawal bleeding occurs after the progesterone challenge, then
to evc1luate postprandial hypoglycemia. However, il has the patient has either a low-estrogen state and hypothalamic
not demonstrated effectiveness frJr this purpose and also amenorrhea is the diagnosis, or there is uterine outflow block­
would not be an appropriate study fcir evaluating fasting age. This patient's history of a previous uterine procedure prior
hypoglycemia. to the onset of amenorrhea and the possibility of denuded
endometrium where synechiae are present will make a nega­
KEY POINT
tive test (no withdrawal bleeding) uninterpretable.
• Symptomatic fasting hypoglycemia is best evaluated
KEY POINT
with glucose and hypoglycemic studies preceding and
during a 72-hour fast. • The diagnosis of Asherman syndrome should be con­
sidered in any woman with amenorrhea and previous
Bibliography exposure to uterine instrumentation; the classic pres­
Cryer PE. Axelrod L, Grossman AB, et al. Evaluation and management of entation is with secondary amenorrhea and some­
adult hypoglycemic disorders: an Endocrine Society Clinical Practice times cyclic pelvic pain.
Guideline. J Endocrinol Metab. 2009 Mar:94(3):709-28. [PMID: t9088155l

127
Answers and Critiques

Bibliography Item 70 Answer: B


Conforti A. Alviggi C. Mollo A. De Placido G, Magos A. The management of Educational Objective: Treat Graves ophthalmopathy.
Asherman syndrome: a review of'literatme. Reprod 13iol Endocrinol. 2013
Dec 27:11:118. [PMID: 24373209]
Methimazole is most appropriate for this patient with Graves
ophthalmopathy (GO). GO may be manifested by lateral gaze
Item 69 Answer: C palsy, scleral injection, periorbital edema, and pressure sen­
sation behind the eyes. Additional manifestations include
Educational Objective: Identify postprandial hyper­
proptosis. lid lag, and, if severe, decreased visual acuity.
glycemia as a cause of elevated hemoglobin A1c
Excess deposition of glycosaminoglycans in the retro-orbital
levels.
space results in increased pressure; the ensuing compression
This patient should measure her postprandial blood glucose of the muscles and cranial nerves causes the ocular palsy.
level. Given the patient's young age and lack of other major Most patients with GO have mild, nonprogressive symptoms
comorbidities, her hemoglobin A 1e goal is less than 6.5% to that do not require specific treatment. The decision to treat
7.0%. Postprandial hyperglycemia often remains undetected depends upon the severity and activity of the disease. For
but still contributes to elevated hemoglobin A 1e values. The patients who need treatment, initial therapy should target
ef1ect is more profound when the hemoglobin A 1e is close controlling the hyperthyroidism. Methimazole will rapidly
to 7.0'Yu. Adequate meal-time coverage with insulin can be lower circulating thyroid hormone levels, may reduce serum
determined by measuring postprandial blood glucose levels. thyroid autoantibody titers, and may assist in controlling GO
lf her 1- to 2-hour postprandial blood glucose values are ele­ symptoms. Additional therapy to control symptoms includes
vated above 180 mg/dL (10.0 mmol/L), her meal-time insulin use of ocular lubricants and taping of the eyelids at night
should be increased or the composition of her meals should (if the lids are unable to completely cover the eye). Because
be altered to decrease her blood glucose. Changing her diet if cigarette smoking increases the activity of GO and in1pairs
it is causing postprandial hyperglycemia could eventually lead the response to therapy, smoking cessation is of paramount
to lower insulin requirements. imp011ance.
Checking her postprandial blood glucose values first External-beam radiotherapy is reserved for treatment of
will help identify where the hyperglycemic issue arises that severe GO but typically is employed if symptoms persist or
keeps her hemoglobin A 1e above goal. worsen in spite of return to the euthyroid state.
Increasing her glargine dose will not adequately af1ect Because radioactive iodine (RA[) treatment has been
postprandial hyperglycemia and may lead to overnight or assoch1ted with (at least transient) worsening of GO due to
fasting hypoglycemia. an initial increase in circulating antibody levels, its use is
Overnight glucose abnormalities can be identified not recommended in patients with moderate to severe GO.
with the measurement of a 3 AM blood glucose level. Large When RAJ is used, pretreatment with a glucocorticoid to
fluctuations in fasting blood glucose values or consistent mitigate the rise in antibody levels is recommended prior to
fasting hyperglycemia can be clues to overnight hypogly­ RA! therapy.
cemia with subsequent rebound hyperglycemia (Somogyi Total thyroidectomy is typically recommended for long­
ef1ect) or hyperglycemia as a result of rising catecholamines term control of Graves disease in patients with active GO
(dawn phenomenon). The patient reports stable fasting and when medical therapy fails to induce a remission. However,
preprandial glucose values throughout the day. ll1e most returning the patient to a state of euthyroidism is advisable
likely timing for glucose abnormalities that would af1ect her before surgery. Surgical decompression is also an option to
hemoglobin A 1e value is in the postprandial state. control active GO, particularly if there is compression of the
Sitagliptin is a dipeptidyl peptidase-4 (DPP-4) inhib­ optic nerve.
itor that slows gastric emptying and suppresses glucagon
KEY POINT
secretion. Although it has a modest effect on hemoglobin
A 1e lowering, metformin remains first-line therapy for • Initial treatment of Graves ophthalmopathy is nor­
type 2 diabetes and should be continued as part of her malization of thyroid function.
regimen.
Bibliography
KEY POINT
Phelps PO. Williams K. Thyroid eye disease for the primary care physician.
• Monitoring postprandial blood glucose levels can be Dis Mon. 2014 Jun:60(6):292-8. [PMlD: 24906675]
useful to assess prandial insulin coverage in patients
who have type 2 diabetes mellitus with at-goal pre­
prandial readings but with hemoglobin A,c values Item 71 Answer: B
not at goal. Educational Objective: Diagnose primary hyperaldo­
steronism as a cause of secondary hypertension.
Bibliography The most appropriate diagnostic test to perform next is to mea­
American Diabetes Association. (7) Approaches to glycemic treatment. In: sure the plasma aldosterone-plasma renin activity ratio. This
Standards of Medical Care in Diabetes-2015. Diabetes Care. 2015 Jan;38
Suppl 1:S4 l-8. [PMID: 25537707[ patient has resistant hypertension, defined as blood pressure

128
Answers and Critiques

that remains above goal despite concurrent use of three antihy­ adenomatous hyperplasia, and the sonographic features of
pertensive agents of different classes, one of which is a diuretic. the gland and slow progression of the compressive symptoms
Resistant hype11ension may occur in as many as 10% of patients support a diagnosis of a benign lesion. He has evidence of
with hypertension. Although this patient is being treated with compressive symptoms on physical examination, with facial
a diuretic, he has significant hypokalemia in the presence of flushing when raising his arms (Pemberton sign). The flush­
treatment with an ACE inhibitor and potassium supplementa­ ing indicates venous outflow obstrnction by the goiter as
tion. This raises the possibility of primary hyperaldosteronism the vessels course through the thoracic inlet. Additionally,
as a cause or contributing factor of his resistant hypertension. his symptoms of dysphagia and change in voice quality may
Appropriate initial evaluation for this diagnosis is measure­ suggest a compressive effect on the esophagus and recurrent
ment of the plasma aldosterone-plasma rerlin activity ratio. la1yngeal nerves, respectively. Surgical removal of the thyroid
If positive, confirmatory testing is usually accomplished with offers the best option for immediate relief of symptoms and
intravenous salt loading, fludrocortisone suppression testing, identification of a possible underlying malignancy.
or captopril testing. If confirmed, adrenal imaging is indicated External-beam radiotherapy can result in eventual
to determine if hyperaldosteronism is due to a bilateral or uni­ shrinkage of the thyroid but will not provide immediate
lateral cause. Mineralocotticoid receptor antagonists (such as relief of his obstructive symptoms, is unlikely to be curative,
spironolactone) are indicated for patients with a bilateral cause potentially increases the risk of secondary malignancy, and
of p1irnary hyperaldosteronisrn and those with a wulateral is associated with significant morbidity.
cause who refuse or are not candidates for surgery. Levothyroxine therapy has been shown to reduce thy­
Dexamethasone suppression testing is used to evaluate roid volume by up to 20%, but the clinical significance of this
for glucocorticoid excess. However, this patient is not taking small change in volume with such a large goiter is unlikely to
exogenous glucocorticoids and has no physical examina­ improve symptoms. Additionally, levothyroxine therapy may
tion findings (fat redistribution, striae) or laboratory stud­ take many months to years to reduce the thyroid volume,
ies (glucose metabolism abnormalities) suggesting Cushing and this patient needs immediate relief of his obstructive
syndrome. Therefore, testing for this possibility is not an symptoms.
appropriate next step in diagnosis. Radioactive iodine has been used to shrink multinod­
Measurement of plasma metanephrines and catechol­ ular goiters; the volume change is also gradual and optimal
amines is used to evaluate for the possibility of pheochrorno­ reduction averages 40%. However, radioactive iodine ther­
cytoma, which classically presents with the triad of diapho­ apy may be risky in this patient because acute swelling of
resis, headache, and tachycardia, none of which are present the thyroid after iodine uptake can lead to compromise of
in this patient. In addition, pheochromocytoma would not the vascular structures at the level of the thoracic inlet and
explain the patient's hypokalemia. further compression of the trachea.
Renal artery Doppler flow studies may be helpful in
KEY POINT
evaluating for renovascular hypertension. However, most
cases of renovascular hypertension occur in patients over • In patients with multinodular goiter with compressive
50 years of age and are associated with atherosclerotic car­ features, thyroidectomy is most likely to provide
diovascular disease or evidence of functional impairment immediate relief of symptoms, decrease the risk
of the kidney, neither of which are apparent in this patient. of fatal airway compromise, and possibly identify
underlying malignancy.
KEY POINT

• Patients with suspected primary hyperaldosteronism Bibliography


as a cause of resistant hypertension should be Aslam RI. Steward D. Surgical management of thyroid disease. Otolaryngol
screened with a plasma aldosterone-plasma renin Clin North Am. 2010 Apr:43(2):273-83. [PMID: 20510714]
activity ratio.
Item 73 Answer: B
Bibliography
Educational Objective: Evaluate primary infertility
Funder JW. Carey RM, Fardella C. et al. Case detection. diagnosis. and treat­
ment of patients with primary aldosteronism: an endocrine society with a hysterosalpingogram.
clinical practice guideline. J Clin Endocrinol Metab. 2008:93(9):3266-8!.
[PM ID: 18552288] The test most likely to be diagnostic is a hysterosalpingogram
(HSG) to evaluate tubal patency. Primary infertility due to
Item 72 Answer: D a tubal abnormality is common and is best evaluated with
dynamic testing of the female reproductive tract under flu­
Educational Objective: Treat substernal goiter with
oroscopy with an HSG. TI1is patient, owing to her history of
compressive symptoms.
pelvic inflammatory disease, is likely to have a distal tubal
This patient should undergo thyroidectomy to treat his occlusion and resultant hydrosalpinx. Many women with
obstructive substernal goiter and prevent further airvvay com­ hydrosalpinx have no symptoms; however, symptoms may
promise, wllich may be rapid and fatal. TI1e thyroid nodule include pelvic pain (both unilaterally and bilaterally) and
biopsy findings obtained 5 years ago most likely represent chronic vaginal discharge. Confinnatory evaluation should

129
Answers and Critiques

include diagnostic laparoscopy, but this is not typically per­ and pheochromocytoma. Failure to identify and treat a pheo­
formed as first-line evaluation given the need for general chromocytoma prior to surgery can result in an intraoperative
anesthesia, intubation, and recovery. Repair of the fallopian hypertensive c1isis and, potentially, death.
tubes may be possible with microsurgical techniques; how­ While it is important to assess the risk of metastatic
ever, reocclusion is possible and the risk of subsequent ectopic disease in this high-risk patient, 18-fluoro-deoxyglucose
pregnancy is high. Many women elect to proceed with in vitro positron emission tomography scanning is not the ideal
fertilization in lieu of tubal surgery. imaging modality in patients with MTC, as there is a high
Ovarian reserve assessment and semen analysis are false-negative rate. CT of the lungs and liver is a more effec­
essential when evaluating a couple with infertility. Ovarian tive means of identification of distant metastases .
reserve assessment can be accomplished with early follicu­ While evaluation for hyperparathyroidism by measur­
lar phase testing of follicle-stimulating hormone (FSH) or ing the serum parathyroid hormone level is indicated in this
anti-miillerian hormone (AMH). Transvaginal ultrasound patient with MEN2A, it is not the most appropriate next step
in the early follicular phase allows for counting of antral in management. Testing for hyperparathyroidism is recom­
follicles in each ovary, which if present at greater than eight mended prior to surgery because the parathyroid disease
bilaterally support normal ovarian reserve. However, given can be managed simultaneously with the thyroid cancer, but
this patient's history of pelvic inflammatory disease and her would not be an appropriate step before evaluating a patient
husband's history of fathering a child in a previous marriage. with MEN2A for the presence of pheochromocytoma.
HSG is the diagnostic test that is most likely to reveal an Total thyroidectomy and lateral neck dissection should
abnormality. not be performed until it is confirmed that this patient does
Semen analysis can be performed after a short window not have a coexisting endocrine neoplasm owing to the high
of abstinence on an ejaculated sperm specimen. However, it intraoperative risk associated with untreated pheochromo­
is unlikely to be helpful in this patient's husband. cytoma. If identified, pheochromocytomas should be surgi­
Karyotyping is usually not indicated as part of the initial cally removed prior to thyroidectomy.
evaluation of unexplained female infertility because of the Genetic counseling is a very important component of
low incidence of discovered abnormalities. the treatment plan for all patients with newly diagnosed
Specialized laboratories with andrology services eval­ MEN2A as first-degree relatives are at high risk for also hav­
uate sperm concentration, sperm motility, and sperm mor­ ing the RET mutation. With a nearly 100% penetrance for
phology. Ovarian reserve may be evaluated by serum testing development of MTC in carriers of RET mutations, referral to
(day 3 follicle-stimulating hormone level, AMH level) or by a team with experience in the management of these disor­
transvaginal ultrasound assessment of antral follicle count. ders is critical for timely diagnosis and treatment.
KEY POINT KEY POINT

• Primary infertility due to a tubal abnormality is com­ • All patients with multiple endocrine neoplasia type
mon, particularly in women with a history of pelvic 2A should undergo testing to exclude pheochromocy­
inflammatory disease, and is best evaluated with a toma prior to thyroidectomy; an elevated level of
hysterosalpingogram. plasma fractionated metanephrines should prompt
treatment for pheochromocytoma before addressing
Bibliography the thyroid malignancy.
Maheux-Lacroix S. Boutin A. Moore L. et al. Hysterosalpingosonogrnphy for
diagnosing tubal occlusion in subfertile women: a systematic review
with meta-analysis. Hum Reprod. 2014 May:29(5):953-63. [PMID: Bibliography
24578476] American Thyroid Association Guidelines Task Force,2015 Thyroid. [PMID:
25810047]

Item 74 Answer: B
Educational Objective: Evaluate newly diagnosed
Item 75 Answer: C
Educational Objective: Treat severe hypercalcemia.
Cl
medullary thyroid cancer.
This patient has severe. malignancy-relaied (non-parathy­
The next most appropriate step in the management of this roid hormone [PTI i J-mcdiatcd) hypercalccrnia with associ­
patient with medullary thyroid cancer (MTC) and multiple ated mental status changes and acute kidney injury. Hemo­
endocrine neoplasia type 2A (MEN2A) is to evaluate for the dialysis is generally reserved for patienls wilh extremely
presence of a pheochromocytoma by measurement of fraction­ high scrum calcium levels (>18 mg dL [4.5 mmol L]) asso­
ated plasma metanephrines. All patients with a cytologic diag­ ciated wi!h c1cure kidney injury !hat precludes o!her acute
nosis of MTC should undergo testing of the tumor for genetic treatments becc1use it is able to rnpidly lower serum cal­
abnormalities as the initial step in their evaluation as 25% of cium and provide cc1reful mc1nagement of volume status
patients with MTC will have the inherited form. Identification ,rnd electrolytes. Both hemodialysis and peritoneal dialysis
of a RETmutation in this patient means that the he has MEN2A, 8re options. although hemodialysis more rapidly lowers
an inherited syndrome associated with hyperparathyroidism cc1lcium levels.

130
Answers and Critiques

Cl osteoclast
CONT.
Calcitonin is a rapidly acting agent Lhal inlerleres wiLh
function and promotes kidney excretion of cal­
Continuing the current regimen places the patient at
increased risk for continued hypoglycemia in the setting of
cium. However. its calcium-lowering effecl is relatively hypoglycemic unawareness.
weak. and it is only able to lower serum calcium levels by I to Increasing the metformin dose may improve insulin
2 mg/dL (0.25-0.50 mmol/L) over 4 Lo 6 hours. which would sensitivity and decrease hepatic gluconeogenesis, which
not be optimal lowering of this palienrs very high calcium could improve the hyperglycemia and/or worsen the hypo­
level. Calcilonin will bring her calcium levels down. but nol glycemia. An increased metformin dose does not address the
enough to alleviate her symptoms. serious complication of hypoglycemic unawareness, which
Cinacalcel. a calcimimetic agent. is an option for lower­ must be corrected first.
ing calcium for symptomatic patienls. but it is nol indicated Pramlintide could decrease the hemoglobin A 1e to
to acutely reduce life-Lhreatening hypercalcemi,1. the patient's goal; however, hypoglycemia is a side effect
Intravenous bisphosphonales are used fcir Lrealmenl or of pramlintide in conjunction with insulin use. This may
malignancy-induced hypercalcemia. with zoledronic acid further increase the risk of hypoglycemia in this scenario.
or pamidronate the most common agenls. However. onset or Pramlintide is an amylin mimetic that decreases glucagon
action is nol rapid. and bisphosphonales are conlraindicalecl secretion and increases satiety by decreasing gastric emp­
in severe kidney injury. Therefore, zoleclronic acid would not tying. Pramlintide should be considered when the intended
be an appropriate next step in lreatmenl. reduction in hemoglobin A 1e is modest and the desired
Once the serum calcium has been lowered by dialysis. effects are a reduction in both weight and postprandial
calcitonin or an intravenous bisphosphonale may be used hyperglycemia.
to keep the calcium levels al a manageable level. The use or
KEY POINT
dialysis in the patient is strictly for resolution of Lhe acute
mental status changes from Lhe elevaled calcium level, and • Treatment for hypoglycemic unawareness is to reduce
longer-term management of hypercalcemia is requi reel. the insulin dose and allow permissive hyperglycemia
at all times for several weeks to provide the body an
KEY POINT
opportunity to reset the counterregulatory response
• For patients with serum calcium levels greater than to hypoglycemia to a more appropriate blood glucose
18 mg/dL (4.5 mmol/L) with neurologic symptoms or range.
acute kidney injury, hemodialysis is an appropriate
choice to quickly reduce calcium levels. Bibliography
Cryer PE, Axelrod L. Grossman AB. et al. Evaluation and management of
Bibliography adult hypoglycemic disorders: an Endocrine Society Clinical Practice
Guideline. J Endocrinol Metab. 2009 Mar;94(3):709-28. [PMID: 19088155]
Rosner MH, Daikin AC. Onco-nephrotogy: the pathophysiology and treat­
ment of malignancy-associated hypercalcemia. Clin J Am Soc Nephrol.
2012:7:1722-9. [PMID: 22879438]
Item 77 Answer: C
Educational Objective: Diagnose elevation of lutein­
Item 76 Answer: D izing hormone related to polycystic ovary syndrome.
Educational Objective: Treat hypoglycemic
The most likely cause of this patient's false-positive results
unawareness.
using the urinary luteinizing hormone (LH) kit is polycystic
This patient should decrease his meal-time insuHn and con­ ovary syndrome (PCOS). Women with PCOS typically have
tinue metformin. He has hypoglycemic unawareness sec­ elevated resting LH levels, which may be mistaken on home
ondary to a diminished counterregulatory response that has urinary LH kits for ovulation. The primary clinical manifes­
developed in the setting of repeated episodes of hypoglyce­ tations of PCOS are menstrual irregularity (oligomenorrhea
mia. His hypoglycemia occurs in the postprandial state, thus or amenorrhea), ovulatory dysfunction with resultant infer­
the meal-time insuHn should be decreased to allow permis­ tility, insulin resistance, and hyperandrogenism. Oligo-ovu­
sive hyperglycemia for 2 to 3 weeks. Permissive hyperglycemia lation or anovulation can result in endometrial hyperplasia
is defined as allowing an increase in blood glucose values to and/or infertility. Hyperandrogenism presents as hirsutism,
the level at which no further episodes of hypoglycemia occur. acne, or androgenic alopecia. Most patients with PCOS also
Using less stringent glycemic goals during this period will have insulin resistance, and studies have shown an increased
provide the body an opportunity to reset the counterregula­ incidence of metabolic syndrome, obesity, impaired glucose
tory response to hypoglycemia to a more appropriate blood tolerance, and frank type 2 diabetes mellitus. PCOS remains
glucose range, if possible. Although developing mutual hemo­ a diagnosis of exclusion that is made both clinically and with
globin A1e goals with the patient is important, hypoglycemia ancillary testing. A patient with PCOS with a positive urinary
in this scenario precludes reaching his goal safely. Once he no LH measurement result may interpret this as a true LH surge
longer has hypoglycemia, changes to the meal-time insulin and expect ovulation to be imminent. Attempts at conception
doses or to meal content and volume can be evaluated to fine­ with intercourse or insemination may be focused on this
tune his regimen while avoiding hypoglycemia. reading and may therefore be ineffective. A more accurate

131
Answers and Critiques

assessment of LH surge and anticipated ovulation in patients Denosumab is a receptor activator of nuclear factor KB
with PCOS would be through transvaginal identification of (RANK) ligand inhibitor FDA approved for the treatment of
ovarian folliculogenesis and confirmatory serum assessment osteoporosis in postmenopausal women who are at high risk
of reproductive hormones. of fracture. Since this patient has not failed bisphosphonate
Late-onset (nonclassic) congenital adrenal hyperplasia, therapy (shown a significant decrease in BMD while on
although a common cause of hirsutism and oligo-ovulation, bisphosphonate therapy) nor is she intolerant of the current
is typically associated with normal or low LH levels owing to therapy, there is no reason to change her therapy.
the negative feedback of elevated androgens of adrenal origin Teriparatide is appropriate as first-line therapy for
on the anterior pituitary. patients at high risk for fracture (T-score < -3.0) or who
Functional hypothalamic amenorrhea affects 3% of have experienced progressive osteoporotic disease while
women between the ages ofl8 and 40 years and is a diagno­ on bisphosphonate therapy. This change would be unnec­
sis of exclusion. Risk factors for this condition include a low essary since the patient's BMD has been maintained for the
body weight and fat percentage, rapid and substantial weight past 5 years.
loss, eating disorders, excessive exercise, severe emotional A drug holiday is indicated for patients who have been
stress, severe nutritional deficiencies, and chronic or acute on bisphosphonate therapy for 3 to 5 years, have had no
illness. FSH and LH levels are inappropriately low or normal progression of the disease, and have minimal risk factors
and cannot account for a positive urinary LH test. for additional fractures. TI1is patient has multiple risk fac­
Hypothyroidism and elevated serum prolactin levels tors for fractures; therefore, a drug holiday would not be
suppress rather than elevate serum LH levels and would not appropriate.
account for this patient's increased urinary LH measurement.
KEV POINT
KEV POINT • In patients at high risk for osteoporosis, it is appropri­
• Women with polycystic ovary syndrome typically ate to continue bisphosphonate therapy alone if ade­
have elevated resting luteinizing hormone (LH) levels, quate bone stability has been achieved.
which may be mistaken on home urinary LH kits for
ovulation. Bibliography
National Osteoporosis Foundation (www. my.nof.org/bone-source).
Clinician's Guide to Prevention and Treatment of Osteoporosis.
Bibliography Washington. DC: National Osteoporosis Foundation: 2014.
Rotterdam ESHRE/ASRM-Sponsored PCOS Consensus Workshop Group.
Revised 2003 consensus on diagnostic criteria and long-term health risks
related to polycystic ovary syndrome. Fertil Steril. 2004 Jan:81(1):19-25. Item 79 Answer: D
[PMID: 14711538]
Educational Objective: Manage primary
hyperaldosteronism.
Item 78 Answer: C
The most appropriate treatment for this patient is spironolac­
Educational Objective: Treat a high-risk patient with
tone. He has primary hyperaldosteronism (PA) due to a bilat­
osteoporosis.
eral adrenal source as evidenced by the Jack of lateralization
After counseling about smoking cessation. this patient on adrenal vein sampling (AVS). Bilateral adrenal hyperplasia
should continue her current alendronate therapy. She has is the most common etiology of PA. accounting for approxi­
documented osteoporosis and is at high risk for subsequent mately 60% of cases, and spironolactone is the treatment of
fractures due to multiple risk factors, including current choice. Spironolactone is a mineralocorticoid receptor (MR)
smoking and a previous fracture. Her bone mineral density antagonist that can improve blood pressure, normalize serum
(BMD) has been well maintained on an oral bisphosphonate potassium concentration, and reduce excess cardiovascular
for the last several years. The best way to evaluate a dual-en­ risk related to hyperaldosteronism. Eplerenone is an alterna­
ergy x-ray absorptiometry (DEXA) scan from measurement tive MR antagonist that is less likely to cause gynecomastia
to measurement is to compare the bone mineral density in men because of greater MR selectivity; however, use of
readings from year to year, not the T -score. A change in BMD eplerenone for this indication is off-label. Antagonists of the
that is less than about 4% (or the percentage noted by the aldosterone-sensitive sodium channel (amiloride) can be used
DEXA machine manufacturer) is not considered a statisti­ as second-line therapy.
cally significant change. This regimen should be considered Bilateral adrenalectomy is not appropriate for the rou­
successful therapy since the goal of bisphosphonates is not to tine management of PA, as this would risk primary adrenal
build bone mass but to stabilize bone loss. Since this patient failure, thus necessitating life-long glucocorticoid and min­
has had stable BMD while on alendronate. there is no indi­ eralocorticoid therapy.
cation to convert to a more invasive, expensive option at this Dexamethasone, a long-acting synthetic glucocorticoid,
time. It will be important, however, to continue to follow her has a role in treating only a small percentage of patients who
for atypical fractures of the long bone due to her prolonged have glucocorticoid-remedial hypertension, a very rare auto­
bisphosphonate treatment. lf leg pain or an atypical fracture somal dominant condition resulting from ectopic expres­
is noted, bisphosphonate therapy should be discontinued. sion of aldosterone synthase in the cortisol-producing zona

132
Answers and Critiques

fasciculata. Administration of dexamethasone will suppress TI1ere is no indication for pituitary MRI at this time
pituitary adrenocorticotropic hormone (ACTH) secretion in because her hyperprolactinemia is explained by hypothy­
these patients and therefore mineralocorticoid production. roidism. It is necessary to make sure that her hyperprolactin­
However, hyperaldosteronism in most patients is indepen­ emia normalizes after treatment with levothyroxine.
dent of ACTH secretion, and suppression with exogenous
KEY POINT
glucocorticoid is not an effective therapy.
Left adrenalectomy should not be performed because • In patients with hyperprolactinemfa and hypothy­
the patient has a bilateral cause of PA. The left adrenal roidism, the hypothyroidism should be treated first,
adenoma detected by CT scan is an incidental finding and then the patient should be reevaluated to ensure that
is likely not the cause of this patient's hyperaldosteronism. the hyperprolactinernia resolves.
Because nonsecreting adrenal adenomas are common, AVS
is needed in most patients with hyperaldosteronism to Bibliography
determine the source of aldosterone secretion when imag­ Melmed S. Casanueva FF. Hoffman AR, et al. Diagnosis and treatment of
ing studies show an adrenal adenoma to assess its contri­ hyperprolactinemia: An Endocrine Society clinical practice guideline.
J Clin Endocrinol Metab. 2011 Feb:96(2):273-88. [PMID: 21296991]
bution to excess mineralocorticoid production. AVS should
be done at a high-volume referral center due to a high risk
of complications when significant procedural experience
Item 81 Answer: B
is lacking.
Educational Objective: Diagnose cystic fibrosis as a
KEY POINT cause of congenital bilateral absence of the vas def­
• For patients with primary hyperaldosteronism due to erens and azoospermia.
bilateral adrenal hyperplasia, medical therapy with a Congenital bilateral absence of the vas deferens is a common
mineralocorticoid antagonist such as spironolactone is cause of obstructive azoospermia and is frequently associated
the treatment of choice because of its proven efficacy with cystic fibrosis (CF). It may also present with unilateral
to lower blood pressure, normalize serum potassium absence of the vas deferens. Many patients are unaware that
concentration, and reduce cardiovascular risk. they have CF because they may have a mild form that causes
only nonspecific symptoms such as chronic sinusitis. Partner
Bibliography testing for CF-carrier status should be encouraged to assess
Funder JW, Carey RM, Fardella C, et al. Case detection, diagnosis. and treat­ the likelihood of transmission to a subsequent generation.
ment of patients with primary aldosteronism: an endocrine society
clinical practice guideline. J Clin Endocrinol Metab. 2008 Sep;93(9):3266- Sperm production is often normal in these patients; however,
81. [PMID: 185522881 the absence of the vas deferens limits any observable sperm
in the ejaculate. Therefore, testicular biopsy is necessary to
retrieve sperm for use in advanced reproductive techniques
Item 80 Answer: B (ART) such as in vitro fertilization and intracytoplasmic sperm
injection. Utilization of donor sperm is an alternative for cou­
Educational Objective: Manage hyperprolactinemia
ples not interested in ART.
caused by hypothyroidism.
Androgen abuse is common among elite and profes­
TI1e most appropriate treatment for this patient is to begin sional athletes and in young men. Physical examination
levothyroxine. She has primary hypothyroidism with an findings may include excessive muscular bulk, acne, gyneco­
elevated thyroid-stimulating hormone level and a low free mastia, and decreased testicular volume. Low sperm counts
thyroxine (T4 ) level. Her symptoms are consistent with hypo­ also may be present with exogenous androgen use. Andro­
thyroidism. Hypothyroidism is a cause of hyperprolactin­ gen abuse can result in hypogonadism and infertility, which
emia. When patients present with hyperprolactinemia and occasionally are irreversible. TI1is patient's normal physical
hypothyroidism, the hypothyroidism should be treated and examination, normal testicular volume, and absence of the
then the patient should be reevaluated to ensure that the vas deferens argue against this diagnosis.
hyperprolactinemia resolves. The patient's pituitary gland is Primary hypogonadism is due to testicular failure and
normal. There is no tumor. is defined as a low testosterone level with elevated lutein­
The hypothyroidism should be treated first, and then izing hormone and follicle-stimulating hormone levels.
the patient's prolactin level should be retested. There is no Primary hypogonadism can have congenital or acquired
indication for cabergoline, a dopamine agonist. at this point. causes. TI1e most common congenital cause is Klinefelter
Cabergoline is an appropriate therapy for a patient with syndrome (47,XXY karyotype). 11,e extra sex chromo­
hyperprolactinemia caused by a prolactinoma. This patient's some results in malformation of the seminiferous tubules
hyperprolactinemia is explained by hypothyroidism. and typically of the Leydig cells. Physical examination is
Sertraline does not cause hyperprolactinemia or hypo­ likely to reveal small. firm testes and decreased virilization.
thyroidism. Antipsychotic agents are a common cause of Additional manifestations include oligospermia and infer­
hyperprolactinemia, but selective serotonin reuptake inhib­ tility. Klinefelter syndrome does not result in obstructive
itors such as sertraline are not. azoospermia due to absence of the vas deferens.

133
Answers and Critiques

A clinically palpable varicocele typically affects fertility early morning serum cortisol level is unequivocally low, as
by lowering sperm motility through a local heat effect. A in this patient.
scrotal bulge may be noted by the clinician, and the patient Measurement of the plasma ACTH level is used to dif­
may note pain that is worse with the Valsalva maneuver. ferentiate primary adrenal failure from other causes of low
Azoospermia would not be caused by varicocele alone. cortisol. In primary adrenal failure, the plasma ACTH level is
Y chromosome microdeletions can be associated with typically significantly elevated (200 pg/mL [44 pmol!L]) and
oligospermia or azoospermia and small testicular volume. would confirm the diagnosis. However, withholding ther­
This chromosomal abnormality is not associated with apy while awaiting diagnostic confirmation would not be
absence of the vas deferens and is therefore not a likely cause appropriate because of the potential life-threatening nature
of this patient's findings. of primary adrenal failure.
Although prednisone is an acceptable agent for glu­
KEY POINT
cocorticoid replacement, it has almost pure glucocorticoid
• Congenital bilateral absence of the vas deferens is a activity and would not be an appropriate single agent for
common cause of obstructive azoospermia and is fre­ treatment of primary adrenal failure in which replacement
quently associated with cystic fibrosis. of both glucocorticoid and mineralocorticoid is required.
KEY POINT
Bibliography
Stahl PJ, Schlegel PN. Genetic evaluation of the azoospermic or severely • Cortisol replacement therapy should be initiated
oligozoospermic male. Curr Opin Obstet Gynecol. 2012 Aug;24(4):221-8. immediately in persons with confirmed adrenal insuf­
[PMID: 227290881
ficiency, which is diagnosed by an early morning
serum cortisol level below 3 µg/dL (82.8 nmol/L) in the
setting of signs and symptoms of cortisol deficiency.
Item 82 Answer: B
Educational Objective: Manage primary adrenal failure.
Bibliography
Initiation of cortisol replacement therapy with a glucocor­ Neary N. Nieman L. Adrenal insufficiency: etiology, diagnosis and treatment.
Curr Opin Endocrinol Diabetes Obes. 2010 Jun;l7(3):217-23. [PMID:
ticoid (such as hydrocortisone) and mineralocorticoid (such 20375886]
as fludrocortisone) is the most appropriate next step in man­
agement. This patient has symptoms consistent with adre­
nal insufficiency (fatigue, unintentional weight loss, nausea, Item 83 Answer: A
and vomiting); her family history of autoimmune disorders,
Educational Objective: Manage the "honeymoon"
hyperpigmentation noted on physical examination, and
phase of type 1 diabetes mellitus.
hyperkalemfa on laboratory testing suggest primary adrenal
failure. Patients with primary adrenal failure frequently have The most appropriate management for this patient's hypo­
increased pigmentation over the extensor surfaces and buccal glycemia is to decrease insulin glargine and insulin aspart.
mucosa due to the excessive secretion of melanocyte-stim­ The glucose toxicity present at the time of diabetic ketoaci­
ulating hormone, which shares a common precursor with dosis has diminished with an intensive insulin regimen. Her
adrenocorticotropic hormone (ACTH). Hyperkalemia occurs remaining functional pancreatic beta cells have regained the
due to deficiency of aldosterone. 'TI1e diagnosis of adrenal ability to produce some insulin in the "honeymoon" phase,
insufficiency is made by documenting an inappropriately low which explains the hypoglycemia on previously well-toler­
serum cortisol level. An early morning serum cortisol level ated doses of insulin. The decreased need for insulin will not
that is less than 3 µg/dL (82.8 nmol/L) in the setting of signs be long term as pancreatic beta cells continue to be destroyed
and symptoms of cortisol deficiency is diagnostic of this disor­ over the course of type 1 diabetes. Continuing insulin, even at
der. Treatment should not be withheld while awaiting further low doses, is recommended during the "honeymoon" phase
diagnostic testing since adrenal insufficiency is a potentially in order to preserve beta cell function as long as possible
life-threatening condition that may result in hemodynamic by reducing the metabolic stress on these cells. The low­
instability. Treatment of primary adrenal failure requires both dose insulin regimen can consist of a basal and prandial
glucocorticoid and mineralocorticoid replacement. Hydrocor­ insulin combination or a basal insulin regimen. She is expe­
tisone or another long-acting agent with primarily glucocor­ riencing symptomatic postprandial and fasting hypoglyce­
ticoid activity should be given along with a mineralocorticoid mia. Decreasing the insulin aspart and insulin glargine doses
agent such as fludrocortisone. would address the prandial and fasting hypoglycemia, while
The synthetic ACTH (cosyntropin) stimulation test is also still providing low-dose insulin to protect the function­
extremely sensitive for detecting either primary or second­ ing beta cells. She will require close monitoring of her blood
ary adrenal insufficiency. In patients with nondiagnostic glucose levels to determine when insulin doses should be
basal cortisol values (4-12 µg/dL [110.4-331.2 nmol!L]), stim­ increased as she nears the end of the "honeymoon" phase. The
ulation testing with synthetic ACTH is indicated. A normal "honeymoon" period may persist for several weeks to months.
response is a peak serum cortisol greater than 20 µg/dL Meal-time insulin doses are generally reduced by 50%
(552 nmol/L). ACTH stimulation testing is not needed if the when pramlintide is initiated due to the risk of hypoglycemia.

134
Answers and Critiques

The addition of pramlintide would likely exacerbate the cur­ commonly demonstrate oligoasthenosperrnia (reduced sperm
rent issue of hypoglycemia even with reduction of meal-time motility) and at times complete azoospermia. Three months
insulin. of treatment are typically needed prior to return of improved
The use of sliding-scale insulin without basal insulin semen parameters. Prolactin is secreted by the pituitary lac­
is discouraged. When sliding-scale insulin is used without totroph cells under tonic inhibition by dopamine. Dopamine
basal insulin, the likelihood of wide swings from hypergly­ agonist therapy can normalize prolactin levels, reverse hypo­
cemia to hypoglycemia increases. Without a basal insulin gonadism, and shrink tumors by at least 50% in almost 90%
regimen, she may not consistently receive daily insulin to of patients. Evaluation of the infertile male with abnormal
decrease the metabolic stress on her functioning pancreatic findings on semen analysis should always include investi­
beta cells. gation of the hypothalamic-pituitary-testicular (HPT) axis.
Discontinuation of both insulin glargine and insulin Disturbances in this axis may result in failure of gonadotropin
aspart increases metabolic stress on the pancreatic beta cells release from the ante1ior pituitary and insufficient testosterone
and accelerates the loss of functional cells producing insulin. production as well as absent or diminished spermatogenesis.
As pancreatic beta cell function declines toward the end of Clomiphene citrate is effective only when the HPT axis
the "honeymoon" phase, the risk of diabetic ketoacidosis is intact, which does not apply for this patient. Although
increases without any exogenous insulin. the indication for clomiphene citrate in the infertile male
population remains controversial, some clinicians use it
KEY POINT
to increase endogenous follicle-stimulating hormone and
• Continuing insulin, even at low doses, is recom­ luteinizing hormone output from the anterior pituitary to
mended during the "honeymoon phase" of type 1 dia­ support testosterone production by Leydig cells.
betes mellitus to reduce metabolic stress on function­ Sildenafil may improve erectile dysfunction in this
ing beta cells and preserve any residual function for patient, but it will not increase endogenous testosterone
as long as possible. levels and will not improve his hyperprolactinemic state.
Testosterone replacement therapy would be helpful to
Bibliography alleviate the sexual side effects of this patient's hyperpro­
DeWitt DE, Hirsch 113. Outpatient insulin therapy in type land type 2 diabe­ lactinemic state; however, no restoration of spermatogenesis
tes mellitus: scientific review. JAMA. 2003 May 7;289(17):2254-64. [PMID: would occur, and therefore infertility would persist. Neither
12734137]
testosterone replacement nor sildenafil therapy will reduce
the size of the patient's prolactinoma.
Item 84 Answer: A
KEY POINT
Educational Objective: Treat hypogonadism second­
• Secondary hypogonadism in a male patient caused by
ary to hyperprolactinemia in a male patient.
hyperprolactinemia as a result of a prolactinoma is a
TI1e most appropriate treatment is a dopamine agonist such as possible cause of erectile dysfunction and should be
cabergoline. Hyperprolactinemia as a result of a prolactinoma treated with a dopamine agonist such as cabergoline.
is a possible cause of erectile dysfunction and decreased libido
and may be successfully treated with a dopamine agonist. In Bibliography
addition to the sexual dysfunction associated with hyperpro­ Mann WA. Treatment for prolactinomas and hyperprolactinaemia: a lifetime
lactinemia, semen parameters are often abnormal; patients approach. Eur J Clin Invest. 2011 Mar:41(3):334-42. [PMID: 20955213]

135
Index

Note: Page numbers followed by fand t denote figures and tables. respectively. Test Antiresorptive therapy, in Paget disease of bone, 71, Q40
questions are indicated by Q. Antithyroid drugs, in Graves disease, 44
Apathetic hyperthyroidism,42
A Asherman syndrome (AS},54, Q68
Abdominal CT scan, for androgen-producing adrenal tumor, Atenolol, in thyrotoxicosis, 43
Q52 Atherosclerotic cardiovascular disease (ASCVD}, 16
Acarbose, for diabetes mellitus, St, lOt Autoimmune adrenalitis, and primary adrenal failure, 36
ACE inhibitor, in diabetic nephropathy, 17 Autoimmune-mediated type 1 diabetes mellitus,1, 3
Acquired type 1 diabetes, 3 Autonomic neuropathy, 18
Acromegaly, 27
causes of, 27 B
clinical features and diagnosis in, 27 Bariatric surgery, 8, Q49
transsphenoidal surgery l'or, Q61 �-adrenoceptors blockers, for pheochromocytomas, 33-34
treatment of, 27-28 �-blockers
ACTH stimulation test, 22, 23 in thyroid storm treatment, 49
Adenoma in thyrotoxicosis. 43, QlO
adrenal, 32 Biguanides, for diabetes mellitus, lOt
pituitary,20 (see also Pituitary tumors} Bilateral adrenal hemorrhage, Q42
thyroid, toxic, 44 Bisphosphonates
Adrenal crisis, 36 in hypercalcemia,64, 65
Adrenal failure, bilateral adrenal hemorrhage and, Q42 in osteoporosis, 68-69
Adrenal fatigue, 38 Bone mineral density (BMD), 66-67. Ql2
Adrenal gland, disorders of, 29-40 decline in,67
adrenal hormone excess and, 30-36 and fracture risk, 67
adrenal insufficiency and,36-39 increase in. 66
adrenal masses, 39-40 measurement of,67, 681
related anatomy and physiology, 29, 291' Bromocriptine, for prolactinomas, 26
Adrenal insufficiency,36
during critical illness, 38-39 C
primary adrenal failure, 36-38 Cabergoline, Q84
Adrenal masses, 39-40 for Cushing disease,28
imaging characteristics of, 32t for prolactinomas, 26, Q19
incidentally noted, 39-40, 39f, QSO, Q60 Calcitonin, 42
Adrenal metastases, 39 for osteoporosis, 69
Adrenal vein sampling (AVS), 34 Calcitriol,62, 66
Adrenocortical carcinomas (ACCs), 32,32t, 39. 40 Calcium
surgical excision of,Q27 homeostasis and bone physiology, 61-62
Adrenocorticotropic hormone (ACTH), 19, J9t postsurgical hypoparathyroidism and supplementation intake,
and cortisol production, 29 Q34
deficiency of, 22-23, 23t Calcium carbonate, in hypocalcemia, 66
Aging male, androgen deficiency in, 57-58 Calcium citrate, in hypocalcemia, 66
Aldosterone, 29 Calcium-creatinine clearance ratio, 64-65
Aldosterone-producing adrenocortical adenomas, 34 Canagliflozin, for diabetes mellitus, 101
Alendronate Capsaicin cream, 18
for osteoporosis, 69, Q78 Cardiovascular disease (CVD}, diabetes mellitus and. 16-17,
for Paget disease of bone, 71 161
Alogliptin,for diabetes mellitus, IOt Catecholamines, 29, 29f
a-adrenoceptors blockers, for pheochromocytomas, 33 Celiac disease,autoimmune-mediated type l diabetes and, 3
a-glucosidase inhibitors, for type 2 diabetes. 4, St, lOt Central diabetes insipidus, 24-25
Amenorrhea, 54-55 Central hyperthyroidism, 45
evaluation of, 55 Central hypothyroidism,measurement of serum free thyroxine (T,,} level
primary, 54,Q9 in,Q36
secondary, 54-55 Chlorpropamide, for diabetes mellitus, IOt
American Diabetes Association (ADA), on screening for type 2 diabetes. Clomiphene citrate, 57, Q64
I, 2t Continuous glucose monitoring (CGM}, 7, Q53
Amiloride, in primary hyperaldosteronism,35 Continuous subcutaneous insulin infusion (CSII} therapy, 7-9
Amiodarone-induced thyrotoxicosis (AIT), 46 Corticotrophin-releasing hormone (CRH}, 19, 28
type l, 46 Cortisol,29
type 2, 46 Critical illness, adrenal function during, 38-39
Amitriptyline, in distal symmetric polyneuropathy, 18 Cushing disease, 28-29
Amylinomimetics, for diabetes mellitus, lOt Cushing syndrome (CS}, 28, 30-32
Anabolic steroid abuse, in men, 60 causes of, 30, 30t
Androgen abuse, in women, 56 clinical manifestations of'. 30,30f, 3lt
Androgen deficiency, in aging male, 57-58 CT of adrenal glands in, Q5
Androgen-producing adrenal tumors, 36, Q52 diagnosis of, 30-32, 31f,Q5, Q48
Angiotensin receptor blocker (ARB), in diabetic nephropathy,17 endogenous, 30
Anticonvulsants,in distal symmetric polyneuropathy, 18 iatrogenic, 30, Q23
Antidepressants, in distal symmetric polyneuropathy,18 imaging studies in, 32,321
Antidiuretic hormone (ADH}, 19 and pseudo-Cushing syndrome, 30
deficiency of, 24-25 treatment of, 32
excess of, 28 Cystic fibrosis, and infertility, QSI

137
Index

D Foot care, in diabetic patient,18


Dapagliflozin. for diabetes mellitus, lOt Foot ulcers, in diabetic patient, 18
Denosumab Fracture Risk Assessment Tool (FRAX) calculator, 67-68
in hypercalcemia, 65 Fractures
in osteoporosis, 69 osteoporosis and,67
Desmopressin, in central diabetes insipidus, 25 prediction of risk of, 67-68
Destructive thyroiditis,44 Framingham Risk Score,4
Dexamethasone
in primary adrenal failure,38t G
in secondary cortisol deficiency. 23 Gabapentin, in distal symmetric polyneuropathy, 18
1-mg dexamethasone suppression test,Q48 Gestational diabetes mellitus, S
DHEA,in primary adrenal failure, 37, 38t complications related to, S
Diabetes mellitus, I screening for, 5
and chronic kidney disease. QSI therapy for,S
classification of. 1, 3t Gigantism, 27
insulin deficiency, 1, 3. 3t Gliclazide, for diabetes mellitus, !Ot
insulin resistance, 3-5, 3t Glimepiride, for diabetes mellitus, lOt
uncommon types, 3t, S Glipizide,for diabetes mellitus,!Ot
complications of Glucagon-like peptide I (GLP-1) mimetics,for diabetes mellitus,
acute,14, !Sf,!St 9,101
chronic,16-18, 16t Glucocorticoids
diagnostic criteria for. 1, 21 in hypercalcemia, 65
elevated plasma glucose levels in. 1 in lymphocytic hypophysitis, 20
insulin therapy in in primary adrenal failure,37-38,38t, Q82
basal and prandial insulin, in hospitalized patient, Q47 in thyroid storm treatment, 49
timing issues with. Q38 Glucose metabolism, disorders of. See also specific disorder
management of. 5-6. 6t diabetes mellitus, 1-11
blood glucose monitoring, 7. 7t hyperglycemia, 11-12
nonpharmacologic approaches. 7-8 hypoglycemia. 12-14
patient education, 6-7 Glutamic acid decarboxylase (GAD65), 1
pharmacologic therapy, 8-11, tot Glyburide, for diabetes mellitus, !Ot
screening for, I, 2t Glycemic targets, and diabetic complications, Qi4
type I, 1, 3, 3t (see also Type 1 diabetes) Goiter, 41
type 2, 4,St (see also Type 2 diabetes mellitus) multinodular, 51-52
Diabetes self-management education (DSME), 6-7 simple, 52
Diabetes self-management support (DSMS),6-7 Gonadotropin-producing pituitary adenomas,28
Diabetic amyotrophy,18 Gonadotropin-releasing hormone (GnRH), 19,53
Diabetic complications, and glycemic targets, Ql4 Graves disease, 43-44
Diabetic foot ulcers. 18 Graves ophthalmopathy, 44, Q70
Diabetic ketoacidosis (OKA), 3. 14, !Sf, !St. Q24 Growth hormone (GH), 19
Diabetic nephropathy,16t, 17 deficiency of, 24
Diabetic neuropathy, 16t, 17-18. Q66 excess of, 27 (see also Acromegaly)
Diabetic retinopathy, 17 Gynecomastia, 60-61
Diet and exercise, for prevention/delay of type 2 diabetes,4,St causes of. 60-61
Dipeptidyl peptidase-4 (DPP-4) inhibitors, for diabetes mellitus. 9, tot examination in, 61
Distal symmetric polyneuropathy (DPN), 18
Dopamine agonises (DA), for prolactinomas. 26 H
Drug-induced thyroid dysfunction, 46, 47t Hashimoto thyroiditis, and hypothyroidism, 45
Dual-energy x-ray absorptiometry (DEXA) scan, 64, 67, Qi2,Q43 Hemochromatosis, diagnosis of, Q30
Duloxetine Hemodialysis, in hypercalcemia, 65, Q75
in diabetic polyneuropathy, Q66 Hemoglobin A,, testing,I. 2t,7,7t
in distal symmetric polyneuropathy, 18 disadvantages of,1
in patient with diabetes mellitus and chronic kidney disease,
E QSJ
Epinephrine,29 Hirsutism, 55-56
Eplerenone, in primary hyperaldosteronism, 35 Honeymoon phase, insulin therapy in, 3
Estrogen agonists and antagonists,for osteoporosis, 69 Human chorionic gonadotropin (HCG) testing, 55
Estrogen replacement, in hypergonadotropic hypogonadism, 54-55 Hydrocortisone
Euthyroicl sick syndrome (ESS), 48. QI3 after adrenalectomy in patient with Cushing syndrome, Q39
Exenatide. for diabetes mellitus. tot in primary adrenal failure, 38t, Q82
Exercise in secondary cortisol deficiency, 23
for diabetes management. 8 25-Hydroxyvitamin D, 61, 62
and hypoglycemia, 12, Q35 Hyperandrogenism syndromes,55-56
Hypercalcemia, 62-66
F causes of,62, 631
Familial hyperparathyroidism, and hypercalcemia, 65 clinical features of,62
Familial hypocalciuric hypercalcemia (FHH),64-65 diagnosis of,62,63f
urine calcium and creatinine levels in, measurement of, Q37 non-parathyroid hormone-mediated, 65, Q22
l'asting h yperglycemia, Q4 parathyroid hormone-n1ediated. 62-65
Fasting hypoglycemia, 13 treatment of, 65-66
Fasting plasma glucose (FPG). I. 2t, QS7 Hyperglycemia, management of. 11
Female reproductive disorders hospitalized patients with diabetes mellitus, 11-12
amenorrhea, 54-55 hospitalized patients without diabetes mellitus, 12
female infertility, 56-57 Hyperglycemic hyperosmolar syndrome (HHS),14, lSf, !St
female reproduction, physiology of, 53, S3f Hypergonadotropic hypogonadism, Klinefelter syndrome and,QS8
hyperandrogenism syndromes. 55-56 Hyperprolactinemia, 25-27, 2St. QSO, Q84
Fine-needle aspiration (FNA), 42 antipsychotics and, Qll
of thyroid nodules, 51, Slt, Q8 causes of, 25-26, 2St
Fludrocortisone, in primary adrenal failure, 381, Q44, Q82 clinical features and diagnosis, 26
Fluid resuscitation, in hypercalcemia,65 and secondary amenorrhea,54
Follicle-stimulating hormone (FSH), 19, l9t, 53 therapy for. 26

138
Index

Hyperthyroidism Ketoacidosis. diabetic. 3. 14, ISf. ISt. Q24


during pregnancy, QI Ketoconazole. in Cushing disease, 28
subclinical, 45 Klinefelter syndrome. Q58
triiodothyronine (T3) level, measurement of, Q21 and primary hypogonadism. 57
Hypocalcemia
clinical features of, 66 L
diagnosis and causes of, 66 Lactic acidosis. 9
hypoparathyroidism and, 66 Laparoscopic aclrenalectomy. 35
low magnesium level and, Q28 Laser photocoagulation. in diabetic retinopathy, 17
treatment of, 66 Late autoimmune diabetes in adults (LADA), 3
Hypoglycemia Late-night (LN) salivary cortisol, 30-32
and altered mental status. Q20 Letrozole, 57
definition of. 12 Levothyroxine
exercise-induced, Q35 for hypothyroidism, 45. Q16
in patients with diabetes mellitus. 12-13 hypothyroidism from pituitary dysfunction and dosage of. Q56
in patients without diabetes mellitus, 13 for hypothyroidism in pregnancy, Q41
differential diagnosis of. 13t Lifestyle modifications. in diabetes management. 4. 5
fasting hypoglycemia. 13 Linagliptin. for diabetes mellitus. lOt
postprandial hypoglycemia. 13 Lipase inhibitors. for prevention/delay of type 2 diabetes. 4. St
in patient without diabetes mellitus, Q67 Liraglutide, for diabetes mellitus. IOt
Hypoglycemic unawareness. 18 Lithium. and hypercalcemia. 65
treatment of, Q76 Loop diuretics, in hypercalcemia, 65
Hypogonadism. 57-59 Low-close clexamethasone suppression test (LOST). 30-31, 40
androgen deficiency in aging male, 57-58 Low triioclothyronine (T3) syndrome, 48
desire for fertility and therapy for, Q32 Luteinizing hormone (LH), 19. 19t. 53, Q77
evaluation of, 58, 59f Lymphocytic hypophysitis. 20
primary. 57 Lymphocytic thyroiclitis. 40
secondary, 57
testosterone replacement therapy for. 58-59. 60t M
Hypogonadotropic hypogonadism, 54 Male reproductive disorders
GnRH deficiency and, 24 anabolic steroid abuse, 60
hemochromatosis and, Q30 gynecomastia. 60-61
Hypoparathyroidism hypogonadism, 57-59
and hypocalcemia, 66 male infertility. 60
postsurgical, Q34 male reproduction. physiology of, 57, 57f
Hypopituitarism. 21-22 Malignancy-associated hypercalcemia, 65
adrenocorticotropic hormone deficiency. 22-23, 231 Maturity-onset diabetes of young (MODY). 5
causes of, 21, 221 Medical nutrition therapy. for diabetes management, 7-8
central diabetes insipidus. 24-25 Meclullary thyroid cancer (MTC). 53
gonadotropin deficiency. 24 plasma fractionated metanephrine levels in, Q74
growth hormone deficiency, 24 Meglitinides, for diabetes mellitus. IOt
panhypopituitarism, 25 Menstrual cycle, 53
Sheehan syndrome and, 22 Mental status changes
thyroid-stimulating hormone deficiency, 23-24 hypoglycemia and, Q20
Hypothalamic amenorrhea, and GnRH deficiency, 24 in myxeclema coma. 49
Hypothalamic-pituitary-ovarian axis, 53 Metabolic bone disease. 66
Hypothermia. in myxedema coma. 49 Metabolic syndrome. 3
Hypothyroid, 44 and cardiovascular disease (CVD). 3
Hypothyroidism definition of. 3. 4t
causes of. 45 Endocrine Society on screening for, 4
central, Q36 and type 2 diabetes mellitus. 3
evaluation of, 45 10-year CVD risk. calculation of, 4
management of, 45-46 Metformin, for type 2 diabetes. 4, St. 9, lOt. Q29
medication induced, 45 Methimazole
in pregnancy, Q41 in Graves ophthalmopathy. Q70
Hysterosalpingogram (HSG). for tubal patency. Q73 in thyrotoxicosis. 43
Metoprolol. in subacute thyroiditis. QIO
Metyrapone. in Cushing disease, 28
lbandronate, for osteoporosis. 69 Microprolactinomas. in asymptomatic patients. Q65
Idiopathic type 1 diabetes. 3 Miglitol, for diabetes mellitus, tOt
Incidentally noted adrenal mass, 39-40. 39f. QSO. Q60 Mineralocorticoicl antagonist, in primary hyperaldosteronism, 35
Infertility Mineralocorticoids. in primary adrenal failure. 37. 381
female, 56-57 Mitotane. 40
male, 60 Mixed meal tolerance test, 13
Insulin resistance, 3 Multinodular goiter, 44, 51-52
gestational diabetes. S Multiple daily injection (MDI) insulin therapy, 7, 8
metabolic syndrome. 3-4, 4t Multiple endocrine neoplasia syndrome (MEN). 65
obesity and, 3 Multiple endocrine neoplasia syndrome type 2 (MEN2), Q2
type 2 diabetes mellitus, 4. St Myxedema coma, 49
Insulin therapy, for type I diabetes. 8-9, 8t. Q63. Q83. See also Diabetes mellitus
lntrapetrosal sinus sampling ()PSS). 28 N
Iodine deficiency. 41 Nateglinide. for diabetes mellitus. tot
Iodine drops. in thyroid storm treatment. 49 National Osteoporosis Foundation. on osteoporosis evaluation. 67
Ionized calcium level, testing for, Q31 Nerve compression syndromes. t8
Nonthyroidal illness syndrome. 48
J Norepinephrine, 29
Jod-Basedow phenomenon. 44 Normocalcemic primary hyperparathyroidism, 64

K 0
Kallmann syndrome, and GnRH deficiency, 24 Oral glucose tolerance test (OGTT), I. 2t
Karyotype testing, Q58 Orlistat, for prevention/delay of type 2 diabetes. St

139
Index

Osteonecrosis of jaw. 68 Prednisone


Osteopenia. 66-68. See ct/so Osteoporosis in primary adrenal failure, 38l, Q44
Osteoporosis in secondary cortisol deliciency. 23
annual reassessment in. 69 Pregabalin, in distal symmetric polyneuropathy. 18
BMD testing and DEXA scan. 67-68. Qi 2. Q43 Pregnancy
bone mineral density loss and. 67. Q25 diabetes in. 5
delined. 67 thyroid function and disease during, 46. Ql
diagnosis of. 67-68 hypothyroidism in. Q41
hyperconisolism and risk or. Q43 prolactinoma in, 26-'27
National Osteoporosis JC'oundation and Endocrine Society and secondary amenorrhea. 54
recommendations. 70 thionamides use during, 48
physiology related to, 66-67 thyroid function in. 46-48. 48f
risk assessment and screening guidelines. 67. 67l. 68t visual field testing during. Q3
secondary caLtses of. 68. Q25 Premature ovarian insuf'liciency (POI). 54
treatment or. 68-69 Prelibial myxedema. 43
vertebral imaging, 68 Primary adrenal failure. 36-38, Q82
vitamin D and. 69-70 causes and clinical features. 36. 37l
Overnight blood glucose monitoring. Q4 diagnosis or. 36-37
prednisone and lludrocortisone in. Q44
p treatment of'. 37-38. 38t
Pagel disease of bone. 70-71. Q40 Primary amenorrhea. 54. Q9
clinical manifestation. 70. 7lf Primary hyperaldosteronism (PA). 34-36. 35l
diagnosis of'. 71 Primary hyperparathyroidism. 62-64. 64l. Q2
treatment for. 71 parathyroidectomy for, 63. Ql7
Pamidronate. for Paget disease of bone. 71 and vitamin D deliciency. Q46
Panhypopituilarism. 25 Primary hypogonadism. 57
hormone replacement therapy in. Q56 Primary hypothyroidism, and hyperprolactinemia. 26
Paragangliomas. 32-34 Primary thyroid lymphoma. 52, Q45
Parathyroid carcinoma. 64 Progesterone challenge test. 55
Parathyroidecwmy. for primary hyperparathyroidism. 63. Prolactin. 19
Qi7 Prolactinoma. 25
Parathyroid hormone (PTH). 61, 62. See also Hypercalcemia in pregnancy, 26-27. Q3
Paroxetine. in distal symmetric polyneuropathy. 18 therapy for, 26
Pasireotide. in Cushing disease. 28 Propranolol, in thyrotoxicosis. 43
Patient education. for diabetes management. 6-7 Propylthiouracil (PTU)
Pegvisomanl. 27 for hyperthyroidism during pregnancy. QI
Phenoxybenzamine, for pheochromocytomas, 33, Q55 for thyrotoxicosis, 43
Pheochromocytomas. 32-34. 32l. 34l. Q55, Q74 Pseudo-Cushing syndrome. 30
diagnosis of'. 33 Pseudogynecomastia, 61
multiple endocrine neoplasia (MEN) syndromes and. 33. 331 Pseudohypercalcemia. 61. Q31
radiographic localization of'. 33 Pseudohypocalcemia, 61
treatment of. 33-34
Physiologic insulin therapy. /'or type I diabetes. 8-9. 8l R
Pioglitazone. f'or diabetes mellilus. 5l. IOl Radioactive iodine ablation
Pituitury adenoma. '20. See also Pituitary tumors !'or thyrotoxicosis, 43
Pituitary apoplexy. 21-22. Q7 for toxic nodules, 44
Pituitary gland disorders Radioactive iodine uptake (RAlU). ,12. QJ8
anatomy and physiology related to. 18-20. 191'. 19t Raloxifine. /'or osteoporosis. 69
hypopituitarism. 21-25 Receptor activator of nuclear factor KB (RANK) ligand inhibitors. for
pituitary hormone deficiency and excess. testing for. l 9t osteoporosis. 69
pituitary hormone excess. 25-29 Repaglinicle. f' o r diabetes mellitus. IOl
pituitary tumors. 20-21. 20f Resistant hypertension. Q71
Pituitary tumors. 20 Risedronate
empty sella, 21 ror osteoporosis. 69
runctional. 25 ror Paget disease of bone. 71
incidentally noted. 20 -21 Risperidone, and hyperprolactinemia. Qi I
macroadenoma. 20. 2or Rosiglitazone
mass effects of. 21 f'or diabetes mellitus. 101
microadenoma. 20 for prevention/delay or type 2 diabetes. 5l
nonfunctioning, 21
sellar mass. approach to. 20 s
Plasma aldosterone concentration (PAC), 34. 35t Sarcoidosis, hypercalcemia in, Q22
Plasma aldosterone-plasma renin activity ratio. Q71 Saxagliptin, tor diabetes mellitus, IOt
Plasma free metanephrines. measurement of. Q50 Secondary amenorrhea. 54-55
Plasma renin activity (PRA). 34. 35t Secondary hypogonadism. 57
Polycystic ovary syndrome (PCOS). 54. 55-56. Q64 Secondary hypothyroidism. 23-24
elevated resting luteinizing hormone levels in. Q77 Self-monitoring of' blood glucose (SMBG), 7
estrogen-progestin oral contraceptive pills in. Q'26. Q54 Serum f'erritin level and transferrin saturation, measurement of, Q30
Polyuria. hypercalcemia and. 65 Sheehan syndrome. hypopituitarism and. 22
Pooled Cohort Equation. 4. 16 Silagliptin. f'or diabetes mellitus. IOl
Postmenopausal women Smoking cessation, for prevention/delay of type 2 diabetes. St
f'racture risk in. 67 Sodium-glucose transponer-2 (SGLT2) inhibitors. for diabetes mellitus. 9. lOt
osteoporosis treatment in. 68-69 Somatostatin analogues. in acromegaly. 27
Postpartum thyroidilis, 44 Spironolactone. in primary hyperaldosteronism. 35, Q79
Postprandial hyperglycemia. Q69 Stereotaclic radiosurgery. in acromegaly. 27
in pregnancy. 5 Subclinical hyperthyroidism. 45
Postprandial hypoglycemia, 13 Subclinical hypothyroidism. 46. Qi6
Potassium chloride. in diabetic ketoacidosis. Q24 Sulfonylureas
Pramlintide. f'or diabetes mellitus. 101 f'or diabetes mellitus. lOt
Prediabetes. I and hypoglycemia. Q20
Prednisolone. in primary adrenal failure. 381 Syndrome of inappropriate ADH secretion (SfADH), 28

140
Index

T Tissue transglutaminase antibody testing. QtS


Teriparatide, for osteoporosis, 69 Tolbutamide. for diabetes mellitus. tOl
Tertiary hyperparathyroidism, 64 Toxic adenoma. 44
Testicular failure. See Primary hypogonadism Transsphenoidal pituitary decompression. Q7
Testosterone replacement therapy. 58-59. 601 Transsphenoidal tumor resection
Thiazide diuretics, and hypercalcemia, 65 in acromegaly, 27
Thiazolidinediones in Cushing disease. 28
for diabetes mellitus, IOl Transvaginal ultrasound. for Ashennan syndrome, Q68
for prevention/delay of type 2 diabetes. 4. St Triiodothyronine (T). 41. Q2!
Thionamides Troglitazone, for preventionidelay of type 2 diabetes. St
in thyroid storm treatment, 49 T-score. 67
in thyrotoxicosis, 43 Turner syndrome, 54
Thyroglobulin (TgAb), 42 Type t diabetes. I. 3t, Q14. See olso Diabetes mellitus
Thyroid autoantibody measurement. 41 acquired. 3
Thyroid cancer. 52-53 autoimmune-mediated, I. 3
and adjuvant radioactive iodine therapy, Q62 idiopathic. 3
incidence of, 52 insulin deficiency and. I
medullary, 53 therapy for, 8-9. St. Q83
staging and prognosis of. 52 Type 2 diabetes mellitus. 4. Q29. See olso Diabetes mellitus
treatment of. 52-53 diagnosis of. Q57
types of. frequency of, 52, 52l epidemiology of. 4
Thyroidectomy, for substernal goiter, Q72 etiology of. 4
Thyroid emergencies, 48-49 ketosis-prone patients with. 4, Q6
Thyroid function, in pregnancy. 46-48, 48f metabolic syndrome and. 3
Thyroid function test. in elderly patient, Q59 obesity and, 4
Thyroid gland prevention/delay of, strategies for. 4. St
anatomy and physiology, 40-41 therapy for. 9-11. !Ot
disorders of
euthyroid sick syndrome. 48 u
functional. 42-46 Ulcers, foot, in diabetic patient, 18
in pregnancy, 46-48. 48f Urine albumin excretion. elevated. !6t, 17
structural. 50-52 24-hour urine free cortisol (UFC). 30. 31
thyroid cancers, 52-53, 52f U.S. Preventive Services Task Force (USPSTF)
thyroid emergencies, 48-49 BMD testing and vertebral imaging, recommendations for.
function of, evaluation of, 41-42 681
hormones of, 4 t on screening for type 2 diabetes. I. 21
Thyroid hormone replacement therapy. 45
Thyroid hormone therapy, in myxedema coma. 49 V
Thyroiditis. 44 Valproate. in distal symmetric polyneuropathy, 18
autoimmune-mediated type I diabetes and. 3 Venlafaxine, in distal symmetric polyneuropathy. 18
destructive, 44 Vertebral fractures, osteoporosis and. 67-69
Hashimoto, 45 Vildagliptin. for diabetes mellitus, tOt
painful, 44 Viral infections. and type t diabetes mellitus. I
painless. 44 Visual field testing, 21
postpartum, 44 during pregnancy, Q3
subacute. QIO Vitamin D. 61-62
Thyroid nodules. 50-51 hypercalcemia. 62-66
evaluation of, 50-51. sor. QIS hypocalcemia, 66
fine-needle aspiration of. SI, Sit. QB production of. 62f
history and physical examination. so sources of, 611
incidentally discovered. SO Vitamin D., (ergocalcif'erol). 61
types of, sot Vitamin o; (cholecalciferol), 61, Q46
Thyroid peroxidase (TPO), 41 Vitamin D deficiency, 69-70, QtS
Thyroid-stimulating hormone (TSH), 19, 19t. 41 Voglibose. for diabetes mellitus. St, tot
deficiency of. 23-24 Volume repletion. for hypercalcemia. 65
in elderly patient, Q59
Thyroid-stimulating hormone-secreting tumors. 28 w
Thyroid-stimulating immunoglobulins (TS!) autoantibodies. 42 Weight Joss, for prevention of type 2 diabetes. 4
Thyroid storm, 48-49, Q33 Whipple triad. 13
diagnosis of. 49 Wolff'-ChaikolT effect. 46
treatment of. 49
Thyrotoxicosis, 42 z
evaluation of. 42-43 Zoledronic acid
management of, 43 for hypercalcemia, 65
symptoms of. 42-43 for osteoporosis. 69
Thyrotropin receptor (TRAb), 41 for Paget disease of bone. 71
Thyroxine (T ,). 4t Z-score. 67

141

You might also like